1. Trang chủ
  2. » Địa lí lớp 7

Biến đổi lượng giác và hệ thức lượng - Võ Anh Khoa, Hoàng Bá Minh

211 5 0

Đang tải... (xem toàn văn)

Tài liệu hạn chế xem trước, để xem đầy đủ mời bạn chọn Tải xuống

THÔNG TIN TÀI LIỆU

Nội dung

[r]

(1)

LƯỢNG

GIÁC

(2)

TP H

CHÍ MINH

L

ƯỢ

NG GIÁC

(3)

Cuốn sách “LƯỢNG GIÁC – MỘT SỐCHUYÊN ĐỀ VÀ ỨNG DỤNG” biên soạn với mục đích cung cấp, bổ sung kiến thức cho học sinh THPT số bạn đọc

quan tâm đến mảng kiến thức trình học tập làm việc Ở sách này, việc đưa khái niệm dạng tập bản, chúng tơi sẽthêm vào lịch sử ứng dụng mơn học để bạn hiểu rõ “Nó xuất phát từđâu lại phải học nó?”

Ởcác chương chính, chúng tơi chia làm phần :

minh.9a1.dt@gmail.com

CÁC TÁC GIẢ

VÕ ANH KHOA – HOÀNG BÁ MINH

- Phần I : Nêu lý thuyết ví dụ minh họa sau đó, giúp bạn đọc hiểu biết cách trình bày Đồng thời đưa dạng toán bản, thường gặp trình làm lớp học sinh THPT Ở phần này, chúng tơi trình bày sốbài để bạn đọc nắm vững hơn, tránh sai sót

- Phần II : Trong trình tham khảo tổng hợp tài liệu, sẽđưa vào phần dạng tốn khó nhằm giúp cho học sinh bồi dưỡng, rèn luyện kĩ

giải LƯỢNG GIÁC thành thạo gặp phải dạng toán

- Phần III : Chúng sẽđưa lời giải gợi ý cho sốbài, qua bạn đọc kiểm tra lại đáp số, lời giải tham khảo thêm

Trong q trình biên soạn, cố gắng việc tham khảo lượng lớn tài liệu có sẵn tiếp thu có chọn lọc ý kiến từ bạn đồng nghiệp để dần hoàn thiện sách này, khó tránh khỏi thiếu sót tầm hiểu biết kinh nghiệm cịn hạn chế, chúng tơi mong nhận ý kiến đóng góp quý báu bạn đọc gần xa

(4)

L

I C

M

Ơ

N

Trong trình biên soạn, xin cám ơn đến bạn cung cấp tài liệu tham khảo vui lòng nhận kiểm tra lại phần thảo đánh máy, tạo điều kiện hoàn thành sách :

- Tô Nguyễn Nhật Minh (ĐH Quốc Tế Tp.HCM)

- Ngô Minh Nhựt (ĐH Kinh Tế Tp.HCM)

- Mai Ngọc Thắng (ĐH Kinh Tế Tp.HCM)

- Trần Lam Ngọc (THPT Chuyên Trần Đại Nghĩa Tp.HCM)

- Nguyễn Huy Hoàng (THPT Chuyên Lê Hồng Phong Tp.HCM)

- Nguyễn Hoài Anh (THPT Chuyên Phan Bội Châu Tp.Vinh)

(5)

T

P : BI

N

ĐỔ

I L

ƯỢ

NG GIÁC VÀ H

TH

C L

ƯỢ

NG

CHƯƠNG : SƠ LƯỢC VỀ KHÁI NIỆM VÀ LỊCH SỬ

CHƯƠNG : CÁC BIẾN ĐỔI LƯỢNG GIÁC

2.1 CHỨNG MINH MỘT ĐẲNG THỨC LƯỢNG GIÁC

2.2 2.3 2.4 BÀI TẬP TỰ LUYỆN 77

3.2 CHỨNG MINH BẤT ĐẲNG THỨC LƯỢNG GIÁC TRONG TAM GIÁC 81

BÀI TẬP TỰ LUYỆN 133

3.3 NHẬN DẠNG TAM GIÁC VÀ TÍNH CÁC GÓC TRONG TAM GIÁC 143

BÀI TẬP TỰ LUYỆN 191

BÀI TẬP TỰ LUYỆN 15

TÍNH GIÁ TRỊ CỦA BIỂU THỨC 21

BÀI TẬP TỰ LUYỆN 33

CHỨNG MINH ĐẲNG THỨC LƯỢNG GIÁC SUY TỪĐẲNG THỨC LƯỢNG GIÁC KHÁC CHO TRƯỚC 36

BÀI TẬP TỰ LUYỆN 45

CHỨNG MINH BIỂU THỨC LƯỢNG GIÁC KHÔNG PHỤ THUỘC VÀO BIẾN SỐ 46

BÀI TẬP TỰ LUYỆN 51

CHƯƠNG : HỆ THỨC LƯỢNG TRONG TAM GIÁC 52

(6)

TÓM LƯỢC TIỂU SỬ CÁC NHÀ KHOA HỌC

CÓ ẢNH HƯỚNG ĐẾN LƯỢNG GIÁC 199

(7)

CH

ƯƠ

NG 1

S

Ơ

L

ƯỢ

C V

KHÁI NI

M VÀ L

CH S

I. KHÁI NIỆM

Trong toán học nói chung lượng giác học nói riêng, hàm lượng giác hàm tốn học góc, dùng nghiên cứu tam giác tượng có tính chất tuần hồn Các hàm lượng giác góc thường định nghĩa tỷ lệ chiều dài hai cạnh tam giác vng chứa góc đó, tỷ lệ chiều dài đoạn thẳng nối

các điểm đặc biệt vòng tròn đơn vị Sâu xa hơn, khía cạnh đại hơn, định nghĩa hàm lượng giác chuỗi vô hạn nghiệm phương trình vi phân, điều cho

phép

phép hàm hàm llưượợnngg ggiiáácc cócó tthhểểcócó đđốốii ssốố llàà mmộộtt ssốố thực hay số phức

( Dạng đồ thị hàm sin )

Những nghiên cứu cách hệ thống việc lập bảng tính hàm lượng giác cho thực Hipparchus(1) (180-125 TCN), người lập bảng tính độ dài cung trịn chiều dài dây cung tương ứng Sau đó, Ptomely(2) tiếp tục

phát triển cơng trình, tìm cơng thức cộng trừ cho •‹ሺ ൅ ሻ …‘•ሺ ൅ ሻ,

Ptomely suy diễn cơng thức hạ bậc, cho phép ơng lập bảng tính với bất kỳđộ xác cần thiết Tuy nhiên, bảng tính bị thất truyền

Các phát triển diễn ởẤn Độ, công trình Surya Siddhanta(3) (thế kỷ 4-5) định nghĩa hàm sin theo nửa góc nửa dây cung Đến kỷ10, người Ả Rập

dùng cả6 hàm lượng giác với độchính xác đến chữ số thập phân

Các cơng trình vềcác hàm lượng giác phát triển nhằm phục vụtrong cơng trình thiên văn học, cụ thểlà dùng để tính tốn đồng hồ mặt trời

II.

(8)

ͳ

ʹሺͳͶǡͺ െ ͻǡʹሻ ൌ ʹǡͺ

Hệ số mà cần đểkéo căng đồ thị hình sin theo chiều ngang

chúng ta đo thời gian ݐ ngày? Bởi có 365 ngày/ năm, chu kỳ mơ hình nên 365

Nhưng mà giai đoạn ݕ ൌ •‹ ݐ ʹߨ, nên hệ số kéo căng theo chiều ngang :

Ngày nay, chúng dùng đểđo khoảng cách tới gần, mốc giới hạn hay hệ thống hoa tiêu vệ tinh Rộng nữa, chúng áp dụng vào nhiều lĩnh vực khác : quang học, phân tích thị trường tài chính, điện tử học, lý thuyết xác suất, thống kê, sinh học, dược khoa, hóa học, lý thuyết số, địa chấn học, khí tượng học, hải dương học…

Ta lấy ví dụ từ tốn sau trích từLucia C Hamson, Daylight, Twilight, Darkness and Time :

Việc mơ hình hóa số chiếu sáng mặt trời hàm thời gian năm nhiều vĩđộ khác Cho biết Philadelphia nằm vĩđộ ͶͲ୭ Bắc, tìm hàm biểu thị số

giờ chiếu sáng mặt trời Philadelphia

Chú ý đường cong tương tự với hàm số sin mà bị di chuyển kéo

căng Tại độ cao Philadelphia, thời gian chiếu sáng kéo dài 14,8 vào ngày 21 tháng 9,2 vào ngày 21 tháng 12, nên biên độ đường cong (hệ số kéo

(9)

ܿ ൌ ʹߨ ͵͸ͷ

Chúng ta để ý đường cong bắt đầu chu trình vào ngày 21 tháng 3, ngày thứ 80 năm nên phải phải dịch chuyển đường cong bên phải 80 đơn vị Ngoài ra, phải đưa lên 12 đơn vị Do mơ

hình hóa số chiếu sáng của mặt trời năm Philadelphia vào ngày thứݐ

năm hàm số :

ܮሺݐሻ ൌ ͳʹ ൅ ʹǡͺ •‹ ൤ ʹߨ

(10)

CH

ƯƠ

NG 2

CÁC BI

N

ĐỔ

I L

ƯỢ

NG GIÁC

I. BẢNG GIÁ TRỊ LƯỢNG GIÁC CỦA CÁC CUNG CÓ LIÊN QUAN ĐẶC BIỆT

Ta gọi cung có liên quan đặc biệt với cung ߙ cung :

- Đối với ߙ : െߙ

- Bù với ߙ : ߨ െ ߙ

- Hiệu ߨ với ߙ : ߨ ൅ ߙ

- Hơn kémగ

ଶ với ߙ : గ ૛േ ߙ

െߙ ߨ െ ߙ ߨ ʹെ ߙ

ߨ ʹ൅ ߙ

cos …‘• ߙ •‹ ߙ െ •‹ ߙ

sin െ •‹ ߙ െ •‹ ߙ …‘• ߙ …‘• ߙ

tan െ –ƒ ߙ –ƒ ߙ …‘– ߙ െ …‘– ߙ

cot െ …‘– ߙ …‘– ߙ –ƒ ߙ െ –ƒ ߙ െ•‡… ߙ ൌ ͳ െ…•… ߙ ൌ ͳ

1. CÔNG THỨC CƠ BẢN

•‹ଶݔ ൅ …‘•ଶݔ ൌ ͳ –ƒ ݔ …‘– ݔ ൌ ͳ ቀݔ ് ݇ߨ

ʹǡ ݇ א Ժቁ –ƒ ݔ ൌ •‹ ݔ

…‘• ݔ ͳ ൅ –ƒଶݔ ൌ ͳ

…‘•ଶݔቀݔ ്

ߨ

ʹ൅ ݇ߨǡ ݇ א Ժቁ …‘– ݔ ൌ…‘• ݔ

•‹ ݔͳ ൅ …‘–ଶݔ ൌ ͳ

•‹ଶݔሺݔ ് ݇ߨǡ ݇ א Ժሻ

ߨ ൅ ߙ െ …‘• ߙ െ …‘• ߙ

•‹ ߙ െ –ƒ ߙ െ …‘– ߙ

Ngồi ra, có sốhàm lượng giác khác :

…‘• ߙ •‹ ߙ

െ˜‡”•‹ߙ ൌ ͳ െ …‘• ߙ െ ‡š•‡…ߙ ൌ •‡… ߙ െ ͳ

(11)

Từ hình vẽ thực tiễn trên, ta rút số công thức vềhàm lượng giác :

2. CÔNG THỨC CỘNG

•‹ሺܽ േ ܾሻ ൌ •‹ ܽ …‘• ܾ േ •‹ ܾ …‘• ܽ …‘•ሺܽ ൅ ܾሻ ൌ …‘• ܽ …‘• ܾ ט •‹ ܽ •‹ ܾ –ƒሺܽ േ ܾሻ ൌ –ƒ ܽ േ –ƒ ܾ

ͳ ט –ƒ ܽ –ƒ ܾ

ߨ …‘–ሺܽ േ ܾሻ ൌ

…‘• ʹݔ ൌ ൝ –ƒ ʹݔ ൌ

ͳ െ –ƒଶݔቀݔǡ ʹݔ ്

ߨ

ʹ൅ ݇ߨǡ ݇ א Ժቁ

b CÔNG THỨC NHÂN

•‹ ͵ݔ ൌ ͵ •‹ ݔ െ Ͷ •‹ଷݔ ൌ Ͷ •‹ ݔ •‹ ቀߨ

͵െ ݔቁ •‹ ቀ ߨ ͵൅ ݔቁ …‘• ͵ݔ ൌ Ͷ …‘•ଷݔ െ ͵ …‘• ݔ ൌ Ͷ …‘• ݔ …‘• ቀߨ

͵െ ݔቁ …‘• ቀ ߨ ͵൅ ݔቁ –ƒ ͵ݔ ൌ͵ –ƒ ݔ െ –ƒ

ଷݔ

ͳ െ ͵ –ƒଶݔ ൌ –ƒ ݔ –ƒ ቀ

ߨ

͵െ ݔቁ –ƒ ቀ ߨ ͵൅ ݔቁ

Công thức tổng quát hàm tan :

–ƒሺܽ ൅ ܾ ൅ ܿሻ ൌ –ƒ ܽ ൅ –ƒ ܾ ൅ –ƒ ܿ െ –ƒ ܽ –ƒ ܾ –ƒ ܿ ቀܽǡ ܾǡ ܽ േ ܾ ്

ʹ൅ ݇ߨǡ ݇ א Ժቁ …‘– ܽ …‘– ܾ ט ͳ

…‘– ܽ േ …‘– ܾ ሺܽǡ ܾǡ ܽ േ ܾ ് ݇ߨǡ ݇ א Ժሻ

3. CÔNG THỨC NHÂN

a CÔNG THỨC NHÂN

•‹ ʹݔ ൌ ʹ •‹ ݔ …‘• ݔ …‘•ଶݔ െ •‹ଶݔ

ʹ …‘•ଶݔ െ ͳ

ͳ െ ʹ •‹ଶݔ

(12)

c CƠNG THỨC TÍNH THEO ݐ ൌ –ƒ ݔ •‹ ʹݔ ൌ ʹݐ

ͳ ൅ ݐଶ

…‘• ʹݔ ൌͳ െ ݐ

ͳ ൅ ݐଶቀݔ ്

ߨ

ʹ൅ ݇ߨǡ ݇ א Ժቁ –ƒ ʹݔ ൌ ʹݐ

ͳ െ ݐଶ

d CÔNG THỨC HẠ BẬC

•‹ଶݔ ൌͳ െ …‘• ʹݔ

ʹ …‘•ଶݔ ൌ

ͳ ൅ …‘• ʹݔ

ʹ –ƒଶݔ ൌ

ͳ െ …‘• ʹݔ ͳ ൅ …‘• ʹݔ •‹ଷݔ ൌെ •‹ ͵ݔ ൅ ͵ •‹ ݔ

Ͷ …‘•ଷݔ ൌ

…‘• ͵ݔ ൅ ͵ …‘• ݔ Ͷ

4. CÔNG THỨC BIẾN ĐỔI

a TÍCH THÀNH TỔNG

…‘• ܽ …‘• ܾ ൌͳ

ʹሾ…‘•ሺܽ ൅ ܾሻ ൅ …‘•ሺܽ െ ܾሻሿ •‹ ܽ •‹ ܾ ൌ െͳሾ…‘•ሺܽ ൅ ܾሻ െ …‘•ሺܽ െ ܾሻሿ •‹ ܽ …‘• ܾ ൌͳ

ʹ …‘• ܽ •‹ ܾ ൌͳ ʹ

b TỔNG THÀNH TÍCH

…‘•ܽ െ ܾ ʹ ܽ ൅ ܾ

ʹ •‹ ܽ െ ܾ

ʹ ܽ ൅ ܾ

ʹ …‘• ܽ െ ܾ

ʹ ܽ ൅ ܾ

ʹ •‹ ܽ െ ܾ

ʹ –ƒ ܽ േ –ƒ ܾ ൌ•‹ሺܽ േ ܾሻ

…‘• ܽ …‘• ܾቀܽǡ ܾ ് ߨ

ʹ൅ ݇ߨǡ ݇ א Ժቁ …‘– ܽ േ …‘– ܾ ൌ•‹ሺܾ േ ܽሻ

•‹ ܽ •‹ ܾሺܽǡ ܾ ് ݇ߨǡ ݇ א Ժሻ –ƒ ܽ ൅ …‘– ܾ ൌ •‹ሺܽ െ ܾሻ

…‘• ܽ •‹ ܾቀܽ ് ߨ

ʹ൅ ݇ߨǡ ܾ ് ݈ߨǡ ݇ǡ ݈ א Ժቁ …‘– ܽ െ –ƒ ܾ ൌ …‘•ሺܽ ൅ ܾሻ

•‹ ܽ …‘• ܾቀܽ ് ݇ߨǡ ܾ ് ߨ

ʹ ൅ ݈ߨǡ ݇ǡ ݈ א Ժቁ ʹ

ሾ•‹ሺܽ ൅ ܾሻ ൅ •‹ሺܽ െ ܾሻሿ ሾ•‹ሺܽ ൅ ܾሻ െ •‹ሺܽ െ ܾሻሿ

(13)

c CÔNG THỨC BỔ SUNG

•‹ ܽ േ …‘• ܽ ൌ ξʹ •‹ ቀܽ േߨ Ͷቁ …‘• ܽ േ •‹ ܽ ൌ ξʹ …‘• ቀܽ טߨ

Ͷቁ ξ͵ •‹ ܽ േ …‘• ܽ ൌ ʹ •‹ ቀܽ േߨ

͸ቁ ൌ ʹ …‘• ቀܽ ט ߨ ͵ቁ •‹ ܽ േ ξ͵ …‘• ܽ ൌ ʹ •‹ ቀܽ േߨ

͵ቁ ൌ ʹ …‘• ቀܽ ט ߨ ͸ቁ ݉ •‹ ܽ ൅ ݊ …‘• ܽ ൌ ඥ݉ଶ൅ ݊ଶ•‹ሺܽ ൅ ܾሻ

Trong

൝ ݉

ଶ൅ ݊ଶ ൐ Ͳ

…‘• ܾ ൌ ݉

ξ݉ଶ൅ ݊ଶǢ •‹ ܾ ൌ

III. CÁC LOẠI TOÁN VÀ PHƯƠNG PHÁP GIẢI

Giải:

a Ta có :

ൌ …‘• ܽ •‹ ܽ െ

•‹ ܽ …‘• ܽ ൌ

…‘•ଶܽ െ •‹ଶܽ

•‹ ܽ …‘• ܽ ൌ

ʹ …‘• ʹܽ •‹ ʹܽ ൌ

b Ta có :

ൌ ʹ •‹ ܽ …‘• ƒ ൬•‹ ܽ …‘• ܽ ൅

…‘• ܽ

•‹ ܽ൰ ൌ ʹሺ•‹ଶܽ ൅ …‘•ଶܽሻ ൌ ʹ ݊

ξ݉ଶ൅ ݊ଶ

1. CHỨNG MINH MỘT ĐẲNG THỨC LƯỢNG GIÁC

- Ta thường sử dụng phương pháp : biến đổi vế phức tạp nhiều số hạng thành vếđơn giản; biến đổi tương đương; xuất phát từđẳng thức đó, biến đổi vềđẳng thức cần chứng minh

- Trong biến đổi ta sử dụng cơng thức thích hợp hướng đến kết phải đạt

- Lưu ý số công thức phải chứng minh trước sử dụng

Bài 1: Chứng minh đẳng thức sau : a …‘– ܽ െ –ƒ ܽ ൌ ʹ …‘– ʹܽ

(14)

Giải:

a Ta có :

ቀ…‘• ݔ ൅ ͳቁ•‹ ݔ …‘•ଶݔ

ൌ•‹ ݔ ൅ …‘• ݔଷ ൌ

Ͷ

•‹ଶʹݔെ ʹ ൌ

Ͷ ͳ െ …‘• Ͷݔ

ʹ

െ ʹ ൌ͸ ൅ ʹ …‘• Ͷݔ ͳ െ …‘• Ͷݔ ൌ

d Ta có :

ൌ •‹ ݔ ቀ ͳ

…‘• ݔ െ ͳቁ •‹ଷݔ ൌ

ͳ െ …‘• ݔ

…‘• ݔ ሺͳ െ …‘• ݔሻሺͳ ൅ …‘• ݔሻൌ

ͳ

…‘• ݔ ሺͳ ൅ …‘• ݔሻൌ ƒǤ –ƒଷݔ ൅ –ƒଶݔ ൅ –ƒ ݔ ൅ ͳ ൌ•‹ ݔ ൅ …‘• ݔ

…‘•ଷݔ

„Ǥͳ ൅ …‘– ݔ ͳ െ …‘– ݔ ൌ

–ƒ ݔ ൅ ͳ –ƒ ݔ െ ͳ …Ǥ͸ ൅ ʹ …‘• Ͷݔ

ͳ െ …‘• Ͷݔ ൌ …‘–ଶݔ ൅ –ƒଶݔ †Ǥ–ƒ ݔ െ •‹ ݔ

•‹ଷݔ ൌ

ͳ

…‘• ݔ ሺͳ ൅ …‘• ݔሻ

Bài 2: Chứng minh đẳng thức sau :

ൌ –ƒଶݔ ሺ–ƒ ݔ ൅ ͳሻ ൅ –ƒ ݔ ൅ ͳ ൌ ሺ–ƒ ݔ ൅ ͳሻሺ–ƒଶݔ ൅ ͳሻ ൌ

…‘• ݔ

b Ta có điều cần chứng minh tương đương với

ሺͳ ൅ …‘– ݔሻሺ–ƒ ݔ െ ͳሻ ൌ ሺ–ƒ ݔ ൅ ͳሻሺͳ െ …‘– ݔሻ ฻ –ƒ ݔ െ ͳ ൅ –ƒ ݔ …‘– ݔ െ …‘– ݔ ൌ –ƒ ݔ െ –ƒ ݔ …‘– ݔ ൅ ͳ െ …‘– ݔ

Điều hiển nhiên nên ta có điều phải chứng minh c Ta có :

(15)

Giải:

a Ta có :

•‹଺ݔ ൅ …‘•଺ݔ ൌ ሺ•‹ଶݔ ൅ …‘•ଶݔሻሺ•‹ସݔ െ •‹ଶݔ …‘•ଶݔ ൅ …‘•ସݔሻ

ൌ ሺ•‹ଶݔ ൅ …‘•ଶݔሻଶെ ͵ •‹ଶݔ …‘•ଶݔ ൌ ͳ െ͵

Ͷ•‹ଶʹݔ ൌ ͳ െ ͵

ͺሺͳ െ …‘• Ͷݔሻ ൌͷ ൅ ͵ …‘•

ସݔ

ͺ

Vậy ta có điều phải chứng minh b Ta có :

ͳ െ …‘• ʹݔ

•‹ ʹݔ ൌ ൌ –ƒ ݔ

Nên

–ƒ ߨ ͳʹൌ

ͳ െ …‘•ߨ͸ •‹ߨ͸ ൌ

ͳ െ ξʹ͵ ͳ ʹ –ƒ͵ߨ

ͳʹ ൌ

ͳ െ …‘•͵ߨ͸ •‹͵ߨ͸ –ƒͷߨ

ͳʹ ൌ •‹ͷߨ͸ ͸

ൌ ʹ ൅ ξ͵

Vậy ൌ ൫ʹ െ ξ͵൯ଶ൅ ͳ ൅ ൫ʹ ൅ ξ͵൯ଶ ൌ ͳͷ •‹ସݔ ൌ͵

ͺെ ͳ

ʹ…‘• ʹݔ ൅ ͳ

ͺ…‘• Ͷݔ ൌ •‹ସ ߨ

ͳ͸൅ •‹ସ ͵ߨ

ͳ͸൅ •‹ସ ͷߨ

ͳ͸൅ •‹ସ ͹ߨ ͳ͸

Bài 4: Chứng minh

Áp dụng tính tổng sau :

ൌ –ƒଶ ߨ

ͳʹ൅ –ƒଶ ͵ߨ

ͳʹ൅ –ƒଶ ͷߨ ͳʹ

Bài 3: Chứng minh :

a ͷ ൅ ͵ …‘• Ͷݔ ൌ ͺሺ•‹଺ݔ ൅ …‘•଺ݔሻ

b •‹ ʹݔ –ƒ ݔ ൌ ͳ െ …‘• ʹݔ

Suy giá trị :

ʹ •‹ଶݔ

ʹ •‹ ݔ …‘• ݔ ൌ ʹ െ ξ͵

(16)

Giải:

Ta có :

•‹ସݔ ൌ ൬ͳ െ …‘• ʹݔ

ʹ ൰

ൌͳ ൅ …‘•ଶʹݔ െ ʹ …‘• ʹݔ Ͷ ൌ

ͳ ൅ͳ ൅ …‘• Ͷݔʹ െ ʹ …‘• ʹݔ Ͷ

ൌ͵ ͺ൅

…‘• Ͷݔ ͺ െ

ͳ

ʹ…‘• ʹݔ

Suy

•‹ସ ߨ

ͳ͸ൌ ͵ ͺെ

ͳ ʹ…‘•

ߨ ͺ൅

ͳ ͺ…‘•

ߨ Ͷ •‹ସ͵ߨ

ͳ͸ ൌ ͵ ͺെ

ͳ ʹ…‘•

͵ߨ ͺ ൅

ͳ ͺ…‘•

͵ߨ Ͷ •‹ସͷߨ

ͳ͸ ൌ ͵ ͺെ

ͳ ʹ…‘•

ͷߨ ͺ ൅

ͳ ͺ…‘•

ͷߨ Ͷ •‹ସ͹ߨ

ͳ͸ ൌ ͵ ͺെ

ͳ ʹ…‘•

͹ߨ ͺ ൅

ͳ ͺ…‘•

͹ߨ Ͷ

…‘•͵ߨ

ͺ ൅ …‘• ͷߨ

ͺ ൌ …‘• ߨ ͺ൅ …‘•

͹ߨ

ͺ ൌ …‘• ߨ

Ͷ൅ …‘• ͵ߨ

Ͷ ൌ …‘• ͷߨ

Ͷ ൅ …‘• ͹ߨ

Ͷ ൌ Ͳ

Nên

ൌ ͵ ʹ

Giải: Ta có :

…‘•ଶݔ ൅ …‘•ଶݕ ൌ ͳ ൅ …‘• ʹݔ

ʹ ൅

ͳ ൅ …‘• ʹݕ

ʹ ൌ ͳ ൅ …‘•ሺݔ ൅ ݕሻ …‘•ሺݔ െ ݕሻ …‘•ଶݖ ൌ …‘•ଶ൫݊ߨ െ ሺݔ ൅ ݕሻ൯ ൌ …‘•ଶሺݔ ൅ ݕሻ

…‘•ଶݔ ൅ …‘•ଶݕ ൅ …‘•ଶݖ ൌ ͳ ൅ ʹǤ ሺെͳሻ௡…‘• ݔ …‘• ݕ …‘• ݖ

Bài 5: Cho ݔǡ ݕǡ ݖ với ݔ ൅ ݕ ൅ ݖ ൌ ݊ߨ ሺ݊ א Գሻ

(17)

Nên

ൌ ͳ ൅ …‘•ሺݔ ൅ ݕሻ ሾ…‘•ሺݔ ൅ ݕሻ ൅ …‘•ሺݔ െ ݕሻሿ ൌ ͳ ൅ ʹ …‘•ሺݔ ൅ ݕሻ …‘• ݔ …‘• ݕ ൌ ͳ ൅ ʹ …‘•ሺ݊ߨ െ ݖሻ …‘• ݔ …‘• ݕ

Khi

- ݊ ൌ ʹ݉ …‘•ሺ݊ߨ െ ݖሻ ൌ …‘• ݖ

- ݊ ൌ ʹ݉ ൅ ͳ …‘•ሺ݊ߨ െ ݖሻ ൌ െ …‘• ݖ

Vậy ta có điều phải chứng minh

Giải: Đặt

ൌ …‘•ߨ ͹െ …‘•

ʹߨ

͹ ൅ …‘• ͵ߨ

͹

Ta có :

ʹ •‹ߨ

͹ ൌ •‹ ʹߨ

͹ െ ʹ •‹ ߨ ͹…‘•

ʹߨ

͹ ൅ ʹ •‹ ߨ ͹…‘•

͵ߨ ͹ ൌ •‹ʹߨ

͹ െ ൤•‹ ͵ߨ

͹ ൅ •‹ ቀെ ߨ

͹ቁ൨ ൅ ൤•‹ Ͷߨ

͹ ൅ •‹ ൬െ ʹߨ

͹ ൰൨ ൌ •‹ʹߨ

͹ െ •‹ ͵ߨ

͹ ൅ •‹ ߨ ͹൅ •‹

Ͷߨ ͹ െ •‹

ʹߨ

͹ ൌ •‹ ߨ ͹൬•‹

Ͷߨ

͹ ൌ •‹ ͵ߨ

͹ ൰

Do

ൌͳ ʹ

•‹଺ݔ …‘•ଶݔ ൅ •‹ଶݔ …‘•଺ݔ ൌ ͳ

ͺሺͳ െ …‘•ସʹݔሻ

Bài 7: Chứng minh

…‘•ߨ

͹െ …‘• ʹߨ

͹ ൅ …‘• ͵ߨ

͹ ൌ ͳ ʹ

Bài 6: Chứng minh

(18)

Giải: Ta có điều cần chứng minh tương đương với

•‹ଶݔ …‘•ଶݔ ሺ•‹ସݔ ൅ …‘•ସݔሻ ൌͳ

ͺሺͳ െ …‘•ଶʹݔሻሺͳ ൅ …‘•ଶʹݔሻ ฻ ͺ •‹ଶݔ …‘•ଶݔ െ ͳ͸ •‹ସݔ …‘•ସݔ ൌ •‹ଶʹݔ ሺͳ ൅ …‘•ଶʹݔሻ

฻ ʹ •‹ଶʹݔ െ •‹ସʹݔ ൌ •‹ଶʹݔ ൅ •‹ଶʹݔ …‘•ଶʹݔ

฻ •‹ଶʹݔ ൌ •‹ଶʹݔ ሺ•‹ଶʹݔ ൅ …‘•ଶʹݔሻ

Điều hiển nhiên nên ta có điều phải chứng minh

Giải: Ta có :

ൌ ቌ •‹ ܽ ൅ …‘– ܽ ͳ ൅ •‹ ܽ ͳ ቍ

ൌ …‘–௡ܽ

ൌ •‹

௡ܽ ൅ …‘–௡ܽ

…‘–௡ܽ ൅ •‹௡ܽൌ …‘–௡ܽ

Giải: Ta có :

ൌ •‹ଶܽ ൅ …‘•ଶܽ ൅ ʹ …‘•ଶܽ ሺͳ െ …‘•ଶܽሻ

ൌ ͳ

ͳ ൅ …‘– ܽሺ•‹ଶܽ ൅ …‘– ܽ …‘•ଶܽሻ ൅ •‹ ܽ …‘• ܽ ൅ ʹ •‹ଶܽ …‘•ଶܽ •‹ଶܽ െ ʹ …‘•ସܽ ൅ ͵ …‘•ଶܽ ൌ •‹ଶܽ

ͳ ൅ …‘– ܽ൅

ͳ െ •‹ଶܽ

ͳ ൅ –ƒ ܽ ൅ •‹ ܽ …‘• ܽ ൅ ʹ •‹ଶܽ …‘•ଶܽ

Bài 9: Chứng minh

൬ •‹ ܽ ൅ …‘– ܽ ͳ ൅ •‹ ܽ –ƒ ܽ൰

Bài 8: Chứng minh

ൌ ൬…‘– ܽ•‹ ܽ ൅ …‘– ܽ •‹ ܽ ൅ …‘– ܽ …‘– ܽ

•‹௡ܽ ൅ …‘–௡ܽ

ͳ ൅ •‹௡ܽ ͳ ൌ …‘– ௡ܽ

…‘–௡ܽ

Do đó, ta có điều phải chứng minh

•‹௡ܽ ൅ …‘–௡ܽ

(19)

ൌ •‹ ܽ •‹ ܽ ൅ …‘• ܽቆ

•‹ଷܽ ൅ …‘•ଷܽ

•‹ ܽ ቇ ൅ •‹ ܽ …‘• ܽ ൅ ʹ •‹ଶܽ …‘•ଶܽ ൌ ͳ െ •‹ ܽ …‘• ܽ ൅ •‹ ܽ …‘• ܽ ൅ ʹ •‹ଶܽ …‘•ଶܽ ൌ ͳ ൅ ʹ •‹ଶܽ …‘•ଶܽ

Do đó, ta có điều phải chứng minh

Giải:Đặt

͸Ͷ ൌ •‹ ͸୭•‹ ͷͶ୭•‹ ͸͸୭

฻ ͸Ͷ ൌͳ

Ͷ•‹ ͸୭•‹ሺ͸Ͳ୭൅ ͸୭ሻ •‹ሺ͸͸୭ െ ͸୭ሻ ൌ ͳ

Ͷ•‹ ͳͺ୭ ൌ

െͳ ൅ ξͷ ͳ͸

Vậy

ൌ ξͷ െ ͳ ͳͲʹͶ

•‹ ʹ୭•‹ ͳͺ୭•‹ ʹʹ୭•‹ ͵ͺ୭•‹ Ͷʹ୭•‹ ͷͺ୭•‹ ͸ʹ୭•‹ ͹ͺ୭•‹ ͺʹ୭ ൌξͷ െ ͳ

ͳͲʹͶ

Bài 10: Chứng minh

(ĐHSP Hải Phòng 2001)

ൌ •‹ ʹ୭•‹ ͳͺ୭•‹ ʹʹ୭•‹ ͵ͺ୭•‹ Ͷʹ୭•‹ ͷͺ୭•‹ ͸ʹ୭•‹ ͹ͺ୭•‹ ͺʹ୭

Ta có :

•‹ ͵ܽ ൌ Ͷ •‹ ܽ •‹ሺ͸Ͳ୭൅ ܽሻ •‹ሺ͸Ͳ୭െ ܽሻ

Áp dụng công thức trên, ta :

Ͷ •‹ ʹ୭•‹ሺ͸Ͳ୭൅ ʹ୭ሻ •‹ሺ͸Ͳ୭െ ʹ୭ሻ ൌ •‹ ͸୭

Ͷ •‹ ͳͺ୭•‹ሺ͸Ͳ୭൅ ͳͺ୭ሻ •‹ሺ͸Ͳ୭ െ ͳͺ୭ሻ ൌ •‹ ͷͶ୭

Ͷ •‹ ʹʹ୭•‹ሺ͸Ͳ୭൅ ʹʹ୭ሻ •‹ሺ͸Ͳ୭ െ ʹʹ୭ሻ ൌ •‹ ͸͸୭

(20)

Giải:

x Ta có : –ƒ ݔ ൌ …‘– ݔ െ ʹ …‘– ʹݔ

Sử dụng công thức này, ta :

–ƒ ܽ ൌ …‘– ܽ െ ʹ …‘– ʹܽ ͳ

ʹ–ƒ ܽ ʹൌ

ͳ ʹ…‘–

ܽ

ʹെ ʹ …‘– ܽ ͳ

Ͷ–ƒ ܽ Ͷൌ

ͳ ʹଶ…‘–

ܽ Ͷെ

ͳ ʹ…‘–

ܽ ʹ

………

ͳ ʹ௡–ƒ

ܽ ʹ௡ ൌ

ͳ ʹ௡…‘–

ܽ ʹ௡െ

ͳ ʹ௡ିଵ…‘–

ܽ ʹ௡ିଵ

–ƒ ܽ ൅ͳ ʹ–ƒ

ܽ

ʹ൅ǥ ൅ ͳ ʹ௡–ƒ

ܽ ʹ௡ ൌ

ͳ ʹ௡…‘–

ܽ

ʹ௡െ ʹ …‘– ʹܽ

…‘• ܽ ൅ …‘• ͵ܽ ൅ ǥ ൅ …‘•ሺʹ݊ െ ͳሻܽ ൌ •‹ ʹ݊ܽ

ʹ •‹ ܽǡ ׊ܽ א ቀͲǢ ߨ

ʹቁ ǡ ݊ א Գ •‹ ܽ ൅ •‹ ͵ܽ ൅ǥ ൅ •‹ሺʹ݊ െ ͳሻܽ ൌ •‹

ଶ݊ܽ

•‹ ܽ ǡ ׊ܽ א ቀͲǢ ߨ

ʹቁ ǡ ݊ א Գ

Bài 11: Chứng minh

Cộng lại, ta có điều phải chứng minh

x Ta sử dụng công thức ʹ •‹ ݔ …‘• ݕ ൌ •‹ሺݔ ൅ ݕሻ ൅ •‹ሺݔ െ ݕሻ

Ta có : ʹ •‹ ܽ ൌ •‹ ʹܽ ൅ ሺ•‹ Ͷܽ െ •‹ ʹܽሻ ൅ ሺ•‹ ͸ܽ െ •‹ Ͷܽሻ ൅ǥ ൅ ሾ•‹ ʹ݊ܽ െ •‹ሺʹ݊ െ ʹሻܽሿ ൌ •‹ ʹ݊ܽ

Vậy ta có điều phải chứng minh

x Ta sử dụng công thức ʹ •‹ ݔ •‹ ݕ ൌ …‘•ሺݔ െ ݕሻ െ …‘•ሺݔ ൅ ݕሻ

Ta có : ʹ •‹ ܽ ൌ ሺͳ െ …‘• ʹܽሻ ൅ ሺ…‘• ʹܽ െ …‘• Ͷܽሻ ൅ ሺ…‘• Ͷܽ െ …‘• ͸ܽሻ ൅ ǥ ൅ ሾ…‘•ሺʹ݊ െ ʹሻܽ െ …‘• ʹ݊ܽሿ ൌ ͳ െ …‘• ʹ݊ܽ ൌ ʹ •‹ଶ݊ܽ

(21)

- BÀI TẬP TỰ LUYỆN

2.1.1. Chứng minh đẳng thức sau

a •‹ ͵ݔ •‹ଷݔ ൅ …‘• ͵ݔ …‘•ଷݔ ൌ …‘•ଷʹݔ

b Ͷ …‘•ଷݔ •‹ ͵ݔ ൅ Ͷ •‹ଷݔ …‘• ͵ݔ ൌ ͵ •‹ Ͷݔ

c –ƒ ݔ ൅ ʹ …‘– ʹݔ ൌ …‘– ݔ

2.1.2. Chứng minh

ሺ•‹ଶݔ ൅ –ƒଶݔ ൅ ͳሻሺ…‘•ଶݔ െ …‘–ଶݔ ൅ ͳሻ

ሺ…‘•ଶݔ ൅ …‘–ଶݔ ൅ ͳሻሺ•‹ଶݔ ൅ –ƒଶݔ െ ͳሻ ൌ ͳ

2.1.3. Chứng minh

–ƒଶݔ ൅ –ƒଶቀߨ

͵െ ݔቁ ൅ –ƒଶቀ ߨ ͵

Áp dụng tính tổng :

2.1.4. Chứng minh

ƒሻ•‹ሺ݊ െ ͳሻܽ •‹ ܽ ൌ …ሻͳ ൅ ͳ

…‘• ʹ௡ܽൌ

†ሻ ͳ Ͷ௡…‘•ଶ ܽ

ʹ௡

ൌ ቎ ͳ •‹ଶ ܽ

ʹ௡െ ͳ

െ ͳ Ͷ •‹ଶ ܽ

ʹ௡

2.1.5. Chứng minh –ƒଶʹͲ୭,–ƒଶͶͲ୭,–ƒଶͺͲ୭ là nghiệm của phương trình ݔଷെ

͵͵ݔଶ൅ ʹ͹ݔ െ ͵ ൌ Ͳ

Từđó suy giá trị

ൌ –ƒଶʹͲ୭ ൅ –ƒଶͶͲ୭൅ –ƒଶͺͲ୭

ൌ –ƒଶʹͲ୭–ƒଶͶͲ୭൅ –ƒଶͶͲ୭–ƒଶͺͲ୭൅ –ƒଶͺͲ୭–ƒଶʹͲ୭

ൌ –ƒଶʹͲ୭–ƒଶͶͲ୭–ƒଶͺͲ୭

൅ ݔቁ ൌ ͻ –ƒଶ͵ݔ ൅ ͸

ൌ –ƒଶͷ୭ ൅ –ƒଶͳͲ୭൅ ڮ ൅ –ƒଶͺͷ୭

•‹ ܽ

…‘–ሺ݊ െ ͳሻܽ െ …‘– ݊ܽ

„ሻ–ƒሺ݊ െ ͳሻܽ –ƒ ݊ܽ ൌ …‘– ܽ ሾ–ƒ ݊ܽ െ –ƒሺ݊ െ ͳሻܽሿ െ ͳ –ƒ ʹ௡ܽ

–ƒ ʹ௡ିଵܽ

(22)

2.1.6. Cho góc ǡ ǡ thỏa ൅ ൅ ൌ Ͷͷ୭

Chứng minh

–ƒ ൅ –ƒ ൅ –ƒ െ –ƒ –ƒ –ƒ ൌ ͳ െ –ƒ –ƒ െ –ƒ –ƒ െ –ƒ –ƒ

2.1.7. Chứng minh

•‹଺ݔ ൅ …‘•଺ݔ ൌ ͷ

ͺ൅ ͵

ͺ…‘• Ͷݔ

2.1.8. Chứng minh

•‹ସܽ ൅ …‘•ସܽ െ ͳ

•‹଺ܽ ൅ …‘•଺ܽ െ ͳ ൌ

ʹ ͵

2.1.9. Chứng minh

ͳ ൅ …‘• ܽ ʹ •‹ ܽ ቈͳ െ

ሺͳ െ …‘• ܽሻଶ

•‹ଶܽ ቉ ൅ െ …‘–ଶܾ …‘–ଶܿ ൌ …‘– ܽ െ ͳ

2.1.10. Chứng minh

൅ ͳ ൅ ͳ

•‹ ͳ͸ݔ ൌ …‘– ݔ െ …‘– ͳ͸ݔ ݇ߨ

ʹ௟ ሺ݇ א Ժǡ ݈ א Գሻ

൅ǥ ൅ ͳ

•‹ ʹ௡ݔ ൌ …‘– ݔ െ …‘– ʹ௡ݔ

•‹଼ݔ ൅ …‘•଼ݔ ൌ͵ͷ

͸Ͷ൅ ͹

ͳ͸…‘• Ͷݔ ൅ ͳ

͸Ͷ…‘• ͺݔ

2.1.12. Chứng minh

…‘• ͳʹ୭൅ …‘• ͳͺ୭െ Ͷ …‘• ͳͷ୭…‘• ʹͳ୭…‘• ʹͶ୭ ൌ െξ͵ ൅ ͳ

ʹ

(ĐHQG Hà Nội 2001)

(ĐHQG Hà Nội 1996)

…‘•ଶܾ െ •‹ଶܿ

•‹ଶܾ •‹ଶܿ

ͳ ͳ

•‹ ʹݔ •‹ Ͷݔ൅•‹ ͺݔ ừ¯×ǡ …Šứ‰‹Š ׷ ˜ớ‹ọ‹݊ א Գǡ ݔ ്

൅ ͳ ͳ

•‹ Ͷݔ •‹ ʹݔ

(23)

2.1.13. Chứng minh Ͷ •‹ ͳͺ୭•‹ ͷͶ୭ ൌ ͳ

(ĐH Phòng Cháy Chữa Cháy 2001)

2.1.14. Chứng minh

–ƒ ͵Ͳ୭൅ –ƒ ͶͲ୭൅ –ƒ ͷͲ୭൅ –ƒ ͸Ͳ୭ ൌ ͺ

ξ͵…‘• ʹͲ

(ĐHQG Hà Nội 1995)

2.1.15. Chứng minh

ƒሻͳ͸ •‹ ͳͲ୭•‹ ͵Ͳ୭•‹ ͷͲ୭•‹ ͹Ͳ୭ ൌ ͳ

„ሻͺ ൅ Ͷ –ƒߨ

ͺ൅ ʹ –ƒ ߨ

ͳ͸൅ –ƒ ߨ

͵ʹൌ …‘– ߨ ͵ʹ

2.1.16. Chứng minh

ƒǤ …‘• ߨ ͳͷ…‘•

ʹߨ ͳͷ…‘•

͵ߨ ͳͷ…‘•

Ͷߨ ͳͷ…‘•

ͷߨ ͳͷ…‘•

ͳ

൅•‹ሺܾ െ ܿሻ …‘• ܾ …‘• ܿ ൅

•‹ሺܿ െ ܽሻ …‘• ܿ …‘• ܽ ൌ Ͳ

2.1.19. Chứng minh

ͳ െ ʹ •‹ଶܽ

ʹ …‘– ቀߨͶ ൅ ܽቁ …‘•ଶቀߨ

Ͷ െ ܽቁ ൌ ͳ

2.1.20. Chứng minh

ͳ ൅ …‘• ܽ ൅ …‘• ʹܽ ൅ …‘• ͵ܽ

ʹ …‘•ଶܽ ൅ …‘• ܽ െ ͳ ൌ ʹ …‘• ܽ

͸ߨ ͳͷ…‘•

͹ߨ ͳͷ ൌʹ଻

„Ǥ –ƒ ͷ୭–ƒ ͷͷ୭–ƒ ͸ͷ୭–ƒ ͹ͷ୭ ൌ ͳ

2.1.17. Chứng minh

–ƒ ͳͲ୭–ƒ ʹͲ୭–ƒ ͵Ͳ௢ǥ –ƒ ͹Ͳ୭–ƒ ͺͲ୭ ൌ ͳ

2.1.18. Chứng minh

(24)

2.1.21. Chứng minh

…‘– ܽ െ –ƒ ܽ െ ʹ –ƒ ʹܽ െ Ͷ –ƒ Ͷܽ െǥെ ʹ௡–ƒ ʹ௡ܽ ൌ ʹ௡ାଵ…‘– ʹ௡ାଵܽ

2.1.22. Chứng minh

•‹଼ܽ െ …‘•଼ܽ െ Ͷ •‹଺ܽ ൅ ͸ •‹ସܽ െ Ͷ •‹ଶܽ ൌ ͳ

- GỢI Ý GIẢI BÀI TẬP TỰ LUYỆN 2.1.1 ƒǡ „– Sử dụng công thức hạ bậc

2.1.3 Đặt

ݐ ൌ –ƒ ݔ

Khi

ൌ ݐଶ ൅ ቆξ͵ െ ݐ

ͳ ൅ ξ͵ݐቇ

൅ ቆξ͵ ൅ ݐ

Áp dụng tính tổng, viết lại thành

ൌ ሺ–ƒଶͷ୭൅ –ƒଶͷͷ୭൅ –ƒଶ͸ͷ୭ሻ ൅ ሺ–ƒଶͳͲ୭൅ –ƒଶͷͲ୭൅ –ƒଶ͹Ͳ୭ሻ

ൌ ൌ

ͳ

ʹ ሾ…‘• ܽ െ …‘•ሺʹ݊ െ ͳሻܽሿ …‘• ݊ܽ …‘•ሺ݊ െ ͳሻܽ

c) Ta có :

ൌ ͳ ൅ …‘• ʹ

௡ܽ

…‘• ʹ௡ܽ ൌ

ʹ …‘•ଶʹ௡ିଵܽ •‹ ʹ௡ିଵܽ

…‘• ʹ௡ܽ •‹ ʹ௡ିଵܽ ൌ

•‹ ʹ௡ܽ …‘• ʹ௡ିଵܽ

…‘• ʹ௡ܽ •‹ ʹ௡ିଵܽ ൌ

d) Ta có điều cần chứng minh tương đương với :

ͳ

Ͷ௡…‘•ଶ ܽ ൅

ͳ

Ͷ௡•‹ଶ ܽ ൌ

ͳ Ͷ௡ିଵ

ͳ •‹ଶ ܽ

ͳ െ ξ͵ݐቇ

൅ ሺ–ƒଶͳͷ୭൅ –ƒଶͶͷ୭൅ –ƒଶ͹ͷ୭ሻ ൅ ሺ–ƒଶʹͲ୭൅ –ƒଶͶͲ୭ ൅ –ƒଶͺͲ୭ሻ

൅ ሺ–ƒଶʹͷ୭൅ –ƒଶ͵ͷ୭൅ –ƒଶͺͷ୭ሻ ൅ –ƒଶ͵Ͳ୭൅ –ƒଶ͸Ͳ୭

Rồi sử dụng công thức chứng minh

2.1.4

a) Để ý

ൌ •‹ሺ݊ܽ െ ݊ܽ ൅ ܽሻ•‹ ܽ •‹ሺ݊ െ ͳሻܽ •‹ ݊ܽ

b) Để ý

(25)

2.1.5 Sử dụng công thức

–ƒ ͵ܽ ൌ ͵ –ƒ ܽ െ –ƒ

ଷܽ

ͳ െ ͵ –ƒଶܽ

Cho ܽ ൌ ʹͲ୭, ta có :

͵ –ƒ ʹͲ୭െ –ƒଷʹͲ୭

ͳ െ ͵ –ƒଶʹͲ୭ ൌ ξ͵

Suy

ሺ͵ –ƒ ʹͲ୭െ –ƒଷʹͲ୭ሻଶ ൌ ͵ሺͳ െ ͵ –ƒଶʹͲ୭ሻଶ

2.1.6 Áp dụng công thức :

–ƒሺܽ ൅ ܾ ൅ ܿሻ ൌ

2.1.9 Cần chứng minh

ͳ ൅ …‘• ܽ

ʹ •‹ ܽ ቈͳ െ ቉ ൌ …‘– ܽ െ …‘–ଶܾ …‘–ଶܿ ൌ െͳ

2.1.10 Để ý

…‘– ݔ െ …‘– ʹݔ ൌ Ǣ…‘– ʹݔ െ …‘– Ͷݔ ൌ ͳ •‹ Ͷݔ Ǣ…‘– ͺݔ െ …‘– ͳ͸ݔ ൌ ͳ

•‹ ͳ͸ݔ

ൌ ʹ …‘• ͳͷ୭…‘• ͵୭ െ ʹ …‘• ͳͷ୭ሺ…‘• Ͷͷ୭൅ …‘• ͵୭ሻ ൌ …‘• ͵Ͳ୭ െ …‘• ͸Ͳ୭

2.1.13 Nhân vế cho …‘• ͳͺ୭

2.1.14 Áp dụng công thức

–ƒ ܽ ൅ –ƒ ܾ ൌ •‹ሺܽ ൅ ܾሻ …‘• ܽ …‘• ܾ

–ƒ ܽ ൅ –ƒ ܾ ൅ –ƒ ܿ െ –ƒ ܽ –ƒ ܾ –ƒ ܿ ͳ െ –ƒ ܽ –ƒ ܾ െ –ƒ ܾ –ƒ ܿ െ –ƒ ܿ –ƒ ܽ

ሺͳ െ …‘• ܽሻଶ

•‹ଶܽ

…‘•ଶܾ െ •‹ଶܿ

•‹ଶܾ •‹ଶܿ

ͳ •‹ ʹݔ

ͳ •‹ ͺݔ …‘– Ͷݔ െ …‘– ͺݔ ൌ

(26)

Viết lại thành ሺ–ƒ ͷͲ୭ ൅ –ƒ ͶͲ୭ሻ ൅ ሺ–ƒ ͵Ͳ୭൅ –ƒ ͸Ͳ୭ሻ

2.1.15

a) Để ý

…‘• ͳͲ୭ ൌ ͺ •‹ ʹͲ୭ͳ

ʹ…‘• ͶͲ୭…‘• ʹͲ୭

b) Sử dụng công thức

…‘– ܽ െ –ƒ ܽ ൌ ʹ …‘– ʹܽ

Ta có điều phải chứng minh tương đương với

ቂ…‘– ߨ

͵ʹെ –ƒ ߨ

͵ʹቃ െ ʹ –ƒ ߨ

ͳ͸െ Ͷ –ƒ ߨ ͺ ൌ ͺ

2.1.16

a Cần chứng minh

ʹ଻•‹ ߨ

ͳͷ ൌ ʹଷ•‹ ߨ ͳͷ…‘•

͵ߨ ͳͷ…‘•

ͷߨ ͳͷ…‘•

͸ߨ ͳͷ

Suy

ʹ଻•‹͵ߨ

ͳͷ ൌ ʹଶ•‹ ͸ߨ ͳͷ…‘•

͸ߨ ͳͷ…‘•

ͷߨ ͳͷ

–ƒ ݔ െ –ƒ ݕ ൌ •‹ሺݔ െ ݕሻ …‘• ݔ …‘• ݕ

2.1.19 Ta cần chứng minh

ʹ …‘– ቀߨ൅ ܽቁ …‘•ଶቀߨെ ܽቁ ൌ ʹ •‹ ቀߨെ ܽቁ …‘• ቀߨെ ܽቁ

b Ta có điều cần chứng minh tương đương với

ሺ•‹ ͷ୭•‹ ͹ͷ୭ሻሺ•‹ ͷͷ୭•‹ ͸ͷ୭ሻ ൌ ሺ…‘• ͷ୭…‘• ͹ͷ୭ሻሺ…‘• ͷͷ୭…‘• ͸ͷ୭ሻ

2.1.17

Để ý ଶ ൌ ሺ–ƒ ͳͲ୭…‘– ͳͲ୭ሻሺ–ƒ ʹͲ୭…‘– ʹͲ୭ሻ ǥ ሺ–ƒ ͺͲ୭…‘– ͺͲ୭ሻ ൌ ͳ

(27)

2.1.21 Sử dụng công thức sau :

…‘– ܽ െ –ƒ ܽ ൌ ʹ …‘– ʹܽ

2. TÍNH GIÁ TRỊ CỦA BIỂU THỨC

- Ở loại tập này, ngồi cơng thức biến đổi bản, ta cần ý thêm công thức sau :

…‘• ܽ ൌ •‹ ቀߨ

ʹെ ܽቁ Ǣ •‹ ܽ ൌ …‘• ቀ ߨ

ʹെ ܽቁ Ǣ –ƒ ܽ ൌ …‘– ቀ ߨ ʹെ ܽቁ

- Nhờ cung liên kết ta có thểđưa cung lớn ͻͲ୭ hay cung âm về cung trong

khoảng ሺͲ୭ǡ ͻͲ୭ሻ.

- Khi cần rút gọn biểu thức

ൌ …‘• ܽ …‘• ʹܽ …‘• Ͷܽ ǥ …‘• ʹ௡ܽ

Ta dùng công thức

…‘• ܽ ൌ •‹ ʹܽ ʹ •‹ ܽ

- Khi cần rút gọn biểu thức

ൌ …‘• ܽ ൅ …‘• ʹܽ ൅ …‘• Ͷܽ ൅ ڮ ൅ …‘• ʹ݊ܽ

Ta viết

ൌ ʹ •‹ ܽ ʹ ʹ •‹ܽ

- Ngoài ra, để tính giá trị biểu thức ta chứng tỏ số hạng biểu thức nghiệm phương trình, từđó ta dùng cơng thức Viète(4)để tính tổng tích lượng phải tìm

- Cần nhớ lại công thức Viète bậc sau:

Gọi ݔଵǡ ݔଶǡ ݔଷ nghiệm phương trình ܽݔଷ൅ ܾݔଶ൅ ܿݔ ൅ ݀ ൌ Ͳ thì

ʹ

(28)

ە ۖ ۔ ۖ

ۓ ݔଵ൅ ݔଶ൅ ݔଷ ൌ െܾ ܽ ݔଵݔଶ൅ ݔଶݔଷ൅ ݔଷݔଵ ൌ

ܿ ܽ ݔଵݔଶݔଷ ൌ െ݀

ܽ

Từđó suy

ݔଵଶ൅ ݔ

ଶଶ൅ ݔଷଶ ൌ ሺݔଵ൅ ݔଶ൅ ݔଷሻଶെ ʹሺݔଵݔଶ൅ ݔଶݔଷ൅ ݔଷݔଵሻ ൌ

ܾଶ

ܽଶ െ

ʹܿ ܽ ͳ

ݔଵ൅

ͳ ݔଶ൅

ͳ ݔଷ ൌ

ݔଵݔଶ൅ ݔଶݔଷ൅ ݔଷݔଵ ݔଵݔଶݔଷ ൌ െ

ܿ ݀

Giải: Ta có :

ൌͳ

ʹሺͳ െ …‘• ͳͲͲ୭ሻ ൅

ͳ

ʹሺ…‘• ͳʹͲ୭൅ …‘• ʹͲ୭ሻ ͳ

ʹ൬െ ͳ

ʹ൅ …‘• ʹͲ୭൰ ͳ

Ͷെ ͳ

ʹ…‘• ʹͲ୭ ൌ ͷ Ͷ ൅ʹ •‹ሺെ͵Ͳ

୭ሻ …‘• ͺ୭

ʹ •‹ ͺ୭െ •‹ ͺ୭ ൌ …‘– ͺ୭െ

…‘• ͺ୭

•‹ ͺ୭ ൌ Ͳ

ൌ ͳ ൅ …‘• ݔ •‹ ݔ ቈͳ ൅

ሺͳ െ …‘• ݔሻଶ

•‹ଶݔ ቉

…‘• ݔ ൌ െͳ

ʹǡ ݔ א ቀ ߨ ʹǡ ߨቁ

Bài 2: Rút gọn biểu thức

Tính giá trị

ൌ ͳ –ƒ ͵͸ͺ୭൅

Bài 1: Tính

ͳ

ʹሺͳ െ …‘• ͳͶͲ୭ሻ െ ͳ

ൌ ͳ െ

ʹሺ…‘• ͳͲͲ୭൅ …‘• ͳͶͲ୭ሻ െ ൌ ͳ െ ሺ…‘• ͳʹͲ୭…‘• ʹͲ୭ሻ ൅

ൌ ͳ –ƒ ͺ୭

ൌ •‹ଶͷͲ୭൅ •‹ଶ͹Ͳ୭ െ …‘• ͷͲ୭…‘• ͹Ͳ୭

ʹ •‹ ʹͷͷͲ୭…‘•ሺെͳͺͺ୭ሻ

(29)

Giải:

Ta có :

ൌͳ ൅ …‘• ݔ •‹ ݔ Ǥ

ʹሺͳ െ …‘• ݔሻ •‹ଶݔ ൌ

ʹሺͳ െ …‘•ଶݔሻ

•‹ଷݔ ൌ

ʹ •‹ଶݔ

•‹ଷݔ ൌ

ʹ •‹ ݔ

Mặt khác

•‹ଶݔ ൌ ͳ െ …‘•ଶݔ ൌ ͳ െͳ

Ͷൌ ͵

Ͷฺ •‹ ݔ ൌ ξ͵

ʹ ฺ ൌ ʹ •‹ ݔ ൌ

Ͷ ξ͵

ൌ ሺʹ …‘• ͸Ͳ୭…‘• ͳͺ୭ሻଶെͳ

ʹሺ…‘• ͳʹͲ୭൅ …‘• ͵͸୭ሻ ൌ …‘•ଶͳͺ୭െͳ

ʹ൬െ ͳ

ʹ൅ …‘• ͵͸୭൰ ൌ

ͳ ൅ …‘• ͵͸୭

ʹ ൅ ͳ Ͷെ

ͳ

ʹ…‘• ͵͸୭ ൌ ͵ Ͷ ൌ •‹ ͸୭•‹ Ͷʹ୭•‹ ͸͸୭•‹ ͹ͺ୭ ൌ •‹ ͸୭…‘• Ͷͺ୭…‘• ʹͶ୭…‘• ͳʹ୭

ൌ •‹ ͳʹ

ʹ …‘• ͸୭Ǥ

•‹ ʹͶ୭

ʹ •‹ ͳʹ୭Ǥ

•‹ Ͷͺ୭

ʹ •‹ ʹͶ୭Ǥ

•‹ ͻ͸୭

ʹ •‹ Ͷͺ୭ ൌ

•‹ ͻ͸୭

ͳ͸ …‘• ͸୭ ൌ

•‹ሺͻͲ୭൅ ͸୭ሻ

ͳ͸ …‘• ͸୭

ൌ ͳ ͳ͸

ൌ …‘•ଶ͹͵୭൅ …‘•ଶͶ͹୭൅ …‘• ͹͵୭…‘• Ͷ͹୭

ൌ •‹ ͸୭•‹ Ͷʹ୭•‹ ͸͸୭•‹ ͹ͺ୭

ൌ …‘•ߨ ͹…‘•

Ͷߨ ͹ …‘•

ͷߨ ͹ ൌ …‘•ʹߨ

͹ ൅ …‘• Ͷߨ

͹ ൅ …‘• ͸ߨ

͹ ൌ ͳ

•‹ ͳͲ୭െ Ͷ •‹ ͹Ͳ୭

Bài 3: Tính giá trị biểu thức sau

Giải: Ta có :

(30)

ൌ …‘•ߨ ͹…‘•

Ͷߨ ͹ …‘•

ͷߨ

͹ ൌ െ …‘• ߨ ͹…‘•

Ͷߨ ͹ …‘•

ʹߨ ͹ ൌ െ

•‹ʹߨ͹ ʹ •‹ߨ͹Ǥ

•‹Ͷߨ͹ ʹ •‹ʹߨ͹ Ǥ

•‹ͺߨ͹ ʹ •‹Ͷߨ͹ ൌ െ•‹

ͺߨ ͹ ͺ •‹ߨ͹ ൌ െ

•‹ ቀߨ ൅ߨ͹ቁ ͺ •‹ߨ͹ ൌ

ͳ ͺ ൌʹ •‹

ߨ

͹ ቀ…‘•ʹߨ͹ ൅ …‘•Ͷߨ͹ ൅ …‘•͸ߨ͹ ቁ ʹ •‹ߨ͹

ൌʹ •‹ ߨ

͹ …‘•ʹߨ͹ ൅ ʹ •‹ߨ͹ …‘•Ͷߨ͹ ൅ ʹ •‹ߨ͹ …‘•͸ߨ͹ ʹ •‹ߨ͹

ൌ•‹ ͵ߨ

͹ ቁ

ൌെ •‹ ߨ ͹ ʹ •‹ߨ͹ ൌ െ

ͳ ൌ ͳ െ Ͷ •‹ ͹Ͳ

୭•‹ ͳͲ୭

•‹ ͳͲ୭ ൌ ൌ ʹ

Giải: Ta có :

ͳ ξܽ•‹ ݔ ටͳ ൅ ܾ െ ܽܽ •‹ଶݔ

ඨܽ ൅ܾ •‹ଶݔ …‘•ଶݔ ൌ

•‹ ݔ

ξܾ •‹ଶݔ ൅ ܽ …‘•ଶݔඨ

ܽ …‘•ଶݔ ൅ ܾ •‹ଶݔ

…‘•ଶݔ

ൌ •‹ ݔ ȁ…‘• ݔȁൌ ൞

–ƒ ݔǡ ݔ א ቀെߨ ʹǡ

ߨ ʹቁ െ –ƒ ݔ ǡ ݔ א ൬ߨ

ʹǡ ͵ߨ

ʹ ൰ ൌ ͳ

ξܾ െ ܽ

ටܾ െ ܽ ܽ •‹ ݔ ටͳ ൅ ܾ െ ܽܽ •‹ଶݔ

ඥܽ ൅ ܾ –ƒଶݔ

Bài 4: Rút gọn biểu thức sau với ܾ ൐ ܽ ൐ Ͳ

൅ •‹ ቀെ ͹ߨቁ ൅ •‹ ͹ͷߨ ൅ •‹ ቀെ ͹͵ߨቁ ൅ •‹ ߨ ൅ •‹ ቀെ ͹ͷߨ ʹ •‹ ͹ߨ

ʹ

ͳ ൅ ʹሺ…‘• ͺͲ୭െ …‘• ͸Ͳ୭ሻ

(31)

Giải: Ta có :

ͻͲ୭ ൌ ͵Ǥͳͺ୭൅ ʹǤͳͺ୭

Nên …‘• ʹǤͳͺ୭ ൌ •‹ ͵Ǥͳͺ୭

Suy ͳ െ ʹ •‹ଶͳͺ୭ ൌ ͵ •‹ ͳͺ୭െ Ͷ •‹ଷͳͺ୭

Đặt ൌ •‹ ͳͺ୭ ൐ Ͳ ; ݐ nghiệm của phương trình

Ͷݐଷെ ʹݐଶെ ͵ݐ ൅ ͳ ൌ Ͳ

Hay

ሺݐ െ ͳሻሺͶݐଶ൅ ʹݐ െ ͳሻ ൌ Ͳ

Vì •‹ ͳͺ୭ ് ͳ nên

ݐ ൌ െͳ േ ξͷ Ͷ

Vì ݐ ൐ Ͳ nên

•‹ ͳͺ୭ ൌെͳ ൅ ξͷ

Ͷ

•‹ ͳͺ୭ ൌെͳ ൅ ξͷ

Ͷ

Nên ξͷ số hữu tỷ (vơ lý) Vậy ta có điều phải chứng minh

Bài 5: Tính •‹ ͳͺ୭ Từđó chứng minh •‹ ͳ୭ số vô tỷ

Giả sử•‹ ͳ୭ số hữu tỷ, suy •‹ ͵୭ ൌ ͵ •‹ ͳ୭െ Ͷ •‹ଷͳ୭ cũng số hữu tỷ

Như ta có •‹ ͻ୭ ൌ ͵ •‹ ͵୭െ Ͷ •‹ଷ͵୭ ; •‹ ʹ͹୭ ൌ ͵ •‹ ͻ୭െ Ͷ •‹ଷͻ୭ ;

•‹ ͺͳ୭ ൌ ͵ •‹ ʹ͹୭െ Ͷ •‹ଷʹ͹୭ cũng những số hữu tỷ

Do đó, •‹ ͳͺ୭ ൌ ʹ •‹ ͻ୭…‘• ͻ୭ ൌ ʹ •‹ ͻ୭•‹ ͺͳ୭ cũng số hữu tỷ

(32)

Giải:Ta xét trường hợp sau * Nếu …‘•ሺݑ ൅ ݒሻ ൌ Ͳ ൌ ܽ * Nếu …‘•ሺݑ ൅ ݒሻ ് Ͳ

ൌ …‘•ଶሺݑ ൅ ݒሻ ሾܽ –ƒଶሺݑ ൅ ݒሻ ൅ ܾ –ƒሺݑ ൅ ݒሻ ൅ ܿሿ

ൌ ͳ

ͳ ൅ –ƒଶሺݑ ൅ ݒሻሾܽ –ƒଶሺݑ ൅ ݒሻ ൅ ܾ –ƒሺݑ ൅ ݒሻ ൅ ܿሿ

–ƒሺݑ ൅ ݒሻ ൌ –ƒ ݑ ൅ –ƒ ݒ ͳ െ –ƒ ݑ –ƒ ݒ ൌ

ݔଵ൅ ݔଶ

ͳ െ ݔଵݔଶ ൌ െܾܽ ͳ െܿܽൌ

ܾ ܿ െ ܽ

Vậy

ൌ ͳ ͳ ൅ ቀܿ െ ܽቁܾ ଶ

ቈܽ ൬ ܾ ܿ െ ܽ൰

൅ ܾ

ܿ െ ܽ൅ ܿ቉ ൌ ܿ

Giải: Nếu ta có

ݔଵ൅ ݔଶ൅ ݔଷ ൌ ܽ ݔଵݔଶ൅ ݔଶݔଷ൅ ݔଷݔଵ ൌ ܾ

ݔଵݔଶݔଷ ൌ ܿ

ݔଵ ൌ …‘•ߨ

͹ǡ ݔଶ ൌ …‘• ͵ߨ

͹ ǡ ݔଷ ൌ …‘• ͷߨ

͹ ൌ ͳ

…‘•ߨ͹ ൅ ͳ …‘•͵ߨ͹ ൅

ͳ …‘•ͷߨ͹

Bài 7: Tìm phương trình bậc có nghiệm

Từđó, tính tổng

ൌ ܽ•‹ଶሺݑ ൅ ݒሻ ൅ ܾ •‹ሺݑ ൅ ݒሻ …‘•ሺݑ ൅ ݒሻ ൅ ܿ …‘•ଶሺݑ ൅ ݒሻ

Bài 6: Cho phương trình ܽݔଶ൅ ܾݔ ൅ ܿ ൌ Ͳ có nghiệm ݔ

ଵ ൌ –ƒ ݑ ǡ ݔଶ ൌ –ƒ ݒ Hãy

(33)

Thì ݔଵǡ ݔଶǡ ݔଷ nghiệm phương trình bậc

ݔଷെ ܽݔଶ൅ ܾݔ െ ܿ ൌ Ͳ

Ta có :

ݔଵ൅ ݔଶ൅ ݔଷ ൌ …‘• ߨ ͹൅ …‘• ͵ߨ ͹ ൅ …‘• ͷߨ ͹ ൌʹ •‹ ߨ ͹ …‘•ߨ͹ ൅ ʹ •‹ߨ͹ …‘•͵ߨ͹ ൅ ʹ •‹ߨ͹ …‘•ͷߨ͹ ʹ •‹ߨ͹ ൌ •‹͸ߨ͹ ʹ •‹ߨ͹ൌ ͳ ʹ ݔଵݔଶ൅ ݔଶݔଷ൅ ݔଷݔଵ ൌ …‘•ߨ ͹…‘• ͵ߨ ͹ ൅ …‘• ͵ߨ ͹ …‘• ͷߨ ͹ ൅ …‘• ͷߨ ͹ …‘• ߨ ͹ ൌ …‘•ʹߨ ͹ ൅ …‘• Ͷߨ ͹ ൅ …‘• ͸ߨ ͹ ൌ െ ͳ ʹ ݔଵݔଶݔଷ ൌ …‘•ߨ ͹…‘• ͵ߨ ͹ …‘• ͷߨ ͹ ൌ ͳ ʹ൬…‘• ͺߨ ͹ ൅ …‘• ʹߨ ͹ ൰ …‘• ߨ ͹ ൌͳ Ͷ൬…‘• ͻߨ ͹ ൅ …‘• ߨ ൅ …‘• ͵ߨ ͹ ൅ …‘• ߨ ͹൰ ൌͳ Ͷ൬…‘• ߨ ͹൅ …‘• ͵ߨ ͹ ൅ …‘• ͷߨ ͹ െ ͳ൰ ൌ ͳ Ͷ൬ ͳ ʹെ ͳ൰ ൌ െ ͳ ͺ

Vậy phương trình cần tìm

ݔଷ൅ͳ ʹݔଶെ ͳ ʹݔ ൅ ͳ ͺൌ Ͳ

Suy ൌ Ͷ

Giải:

0ểý”ằ‰ʹߨ ͹ Ǣ Ͷߨ ͹ Ǣ ͺߨ ͹ Ž‰Š‹ệ…ủƒ’Šươ‰–”¿Š͹ݔ ൌ ʹߨ ൅ ݇ʹߨሺ݇ א Ժሻ ඨ…‘•ʹߨ ͹ య ൅ ඨ…‘•Ͷߨ ͹ య ൅ ඨ…‘•ͺߨ ͹ య ൌ ඨͷ െ ͵ξ͹ య ʹ య

Bài 8: Chứng minh

(34)

Hay …‘• ͵ݔ ൌ …‘• Ͷݔሺכሻ

Từሺכሻ ta có …‘• ݔ ൌ ͳ ฻ ݔ ൌ ݇ʹߨ (loại khơng thỏa nghiệm trên)

Như

ʹߨ ͹ Ǣ

Ͷߨ ͹ Ǣ

ͺߨ

͹ Ž‰Š‹ệ…ủƒ’Šươ‰–”¿Šݐଷ൅ ݐଶെ ʹݐ െ ͳ ൌ Ͳሺݐ ൌ ʹ …‘• ݔሻ Š‡‘¯ịnh lý Viète, ta có

ݐଵ൅ ݐଶ൅ ݐଷ ൌ െͳ

ݐଵݐଶ൅ ݐଶݐଷ൅ ݐଷݐଵ ൌ െʹ ݐଵݐଶݐଷ ൌ ͳ

Đặt

ቊ ൌ ඥݐଵ

య ൅ ඥݐ

య ൅ ඥݐ

ൌ ඥݐయ ଵݐଶ൅ ඥݐయ ଶݐଷ൅ ඥݐయ ଷݐଵ

Khi

ଷൌ ሺݐ

ଵ ൅ ݐଶ ൅ ݐଷሻ ൅ ͵ െ ͵ඥݐయ ଵݐଶݐଷ ൌ ͵ െ Ͷ

ଷ ൌ ሺݐ

ଵݐଶ൅ ݐଶݐଷ൅ ݐଷݐଵሻ ൅ ͵ െ ͵ඥሺݐయ ଵݐଶݐଷሻଶ ൌ ͵ െ ͷ

Suy

ሺሻଷ ൌ ሺ͵ െ Ͷሻሺ͵ െ ͷሻ ฻ ሺ െ ͵ሻଷ ൌ െ͹

Do ൌ ͵ െ ξ͹య

Nên ൌ ඥͷ െ ͵ξ͹య య

Vậy

ඨ…‘•ʹߨ ͹

൅ ඨ…‘•Ͷߨ ͹

൅ ඨ…‘•ͺߨ ͹

ൌ ξʹ

య ൌ ඨ

ͷ െ ͵ξ͹య

ʹ

(35)

Giải: Từ hệ ta có :

൜ ଶ൅ ଵ ൌ ݊

ଶെ ଵ ൌ …‘• ʹܽ ൅ …‘• Ͷܽ ൅ ǥ ൅ …‘• ʹ݊ܽ

Suy

Do

ଶെ ଵൌ

ฺ ൞

ʹ •‹ ܽ

Giải: Từ giả thuyết, ta có :

ʹ •‹ܽ ൅ ܾ ʹ …‘•

ܽ െ ܾ

ʹ ൌ Ͷ •‹ ܽ ൅ ܾ

ʹ …‘• ܽ ൅ ܾ

ʹ

Vì ܽ ൅ ܾ ് ʹ݇ߨ nên

൜ଵ ൌ •‹ଶܽ ൅ •‹ଶʹܽ ൅ ǥ൅ •‹ଶ݊ܽ ଶ ൌ …‘•ଶܽ ൅ …‘•ଶʹܽ ൅ ǥ ൅ …‘•ଶ݊ܽ

Bài 9: Tính tổng

Với ݊ א Գǡ ܽ ് ݇ߨǡ ݇ א Ժ

ൌ –ƒܽ ʹ–ƒ

ܾ ʹ

Bài 10: Cho •‹ ܽ ൅ •‹ ܾ ൌ ʹ •‹ሺܽ ൅ ܾሻ Ǣ ܽ ൅ ܾ ് ʹ݇ߨǡ ݇ א Ժ

Hãy tìm

ʹ •‹ ܽ ሺଶെ ଵሻ ൌ ʹ •‹ ܽ …‘• ʹܽ ൅ ʹ •‹ ܽ …‘• Ͷܽ ൅ ǥ ൅ ʹ •‹ ܽ …‘• ʹ݊ܽ

ൌ •‹ ͵ܽ െ •‹ ܽ ൅ •‹ ͷܽ െ •‹ ͵ܽ ൅ǥ ൅ •‹ሺʹ݊ ൅ ͳሻܽ െ •‹ሺʹ݊ െ ͳሻܽ ൌ •‹ሺʹ݊ ൅ ͳሻܽ െ •‹ ܽ ൌ ʹ …‘•ሺ݊ ൅ ͳሻܽ •‹ ݊ܽ

ଶ൅ ଵ ൌ ݊ •‹ ܽ

…‘•ሺ݊ ൅ ͳሻܽ •‹ ݊ܽ ଶ ൌ…‘•ሺ݊ ൅ ͳሻܽ •‹ ݊ܽ ൅ ݊ •‹ ܽ ଵ ൌ

ʹ •‹ ܽ

(36)

ܽ ൅ ܾ

ʹ ് ݇ߨ ֜ •‹ ܽ ൅ ܾ

ʹ ് Ͳ —›”ƒǡ …‘•ܽ െ ܾ

ʹ ൌ ʹ …‘• ܽ ൅ ܾ

ʹ ฺ …‘•ܽ

ʹ…‘• ܾ ʹ൅ •‹

ܽ ʹ•‹

ܾ

ʹൌ ʹ ൬…‘• ܽ ʹ…‘•

ܾ ʹെ •‹

ܽ ʹ•‹

ܾ ʹ൰ ฺ ͵ •‹ܽ

ʹ•‹ ܾ

ʹ ൌ …‘• ܽ ʹ…‘•

ܾ ʹ ฺ ൌͳ

͵

Giải: Ta có :

Giải: Ta có :

–ƒ ቀߨ

Ͷെ ߙቁ ൌ

–ƒߨͶ െ –ƒ ߙ ͳ ൅ –ƒߨͶ –ƒ ߙ ൌ

ͳ െ –ƒ ߙ ͳ ൅ –ƒ ߙ Š‘ •‹ ߙ ൌ͵

ͷ˜à

π

ʹ൏ ߙ ൏ ߨǤ퐊 –ƒ ቀ ߨ Ͷ െ ߙቁǤ

Bài 12:

(ĐH Huế 1996)

Bài 11: Rút gọn biểu thức sau

ൌ ඥͳ ൅ Ͷ •‹ଶܽ …‘•ଶܽ ൅ Ͷ •‹ ܽ …‘• ܽ ሺ•‹ଶܽ ൅ …‘•ଶܽሻ

൅ ඥ…‘•ସܽ െ ͷሺͳ െ …‘•ଶܽሻଶെ Ͷ •‹ ܽ …‘• ܽ ሺ•‹ଶܽ ൅ …‘•ଶܽሻ ൅ ͸ •‹ଶܽ

ൌ ඥ•‹ସܽ ൅ …‘•ସܽ ൅ ͸ •‹ଶܽ …‘•ଶܽ ൅ Ͷ •‹ଷܽ …‘• ܽ ൅ Ͷ •‹ ܽ …‘•ଷܽ

൅ ඥ…‘•ସܽ ൅ •‹ସܽ ൅ ͸ •‹ଶܽ …‘•ଶܽ െ Ͷ •‹ଷܽ …‘• ܽ െ Ͷ •‹ ܽ …‘•ଷܽ

ൌ ඥሺ•‹ ܽ ൅ …‘• ܽሻସ൅ ඥሺ•‹ ܽ െ …‘• ܽሻସ

ൌ ሺ•‹ ܽ ൅ …‘• ܽሻଶ൅ ሺ•‹ ܽ െ …‘• ܽሻଶ

ൌ •‹ଶܽ ൅ …‘•ଶܽ ൅ ʹ •‹ ܽ …‘• ܽ ൅ •‹ଶܽ ൅ …‘•ଶܽ െ ʹ •‹ ܽ …‘• ܽ ൌ ʹ

ൌ ඥͳ ൅ Ͷ •‹ଶܽ …‘•ଶܽ ൅ Ͷ •‹ ܽ …‘• ܽ

(37)

Mặt khác :

…‘•ଶߙ ൌ ͳ െ •‹ଶߙ ൌͳ͸

ʹͷ

Do …‘• ߙ ൏ Ͳ nên

…‘• ߙ ൌ െͶ ͷ

Suy

–ƒ ߙ ൌ •‹ ߙ …‘• ߙ ൌ

͵ ͷ െͶͷ ൌ െ

͵ Ͷ ฺ –ƒ ቀߨ

Ͷെ ߙቁ ൌ ͳ ൅͵Ͷ ͳ െ͵Ͷൌ ͹

Giải: Ta áp dụng vào toán đẳng thức

ܽଷ ൅ ܾଷ൅ ܿଷെ ͵ܾܽܿ ൌ ሺܽ ൅ ܾ ൅ ܿሻଷെ ͵ሺܽ ൅ ܾ ൅ ܿሻሺܾܽ ൅ ܾܿ ൅ ܿܽሻ

Dễ thấy

–ƒଶ͵ ቀߨ

ͳͺቁ ൌ –ƒଶ͵ ൬ ͷߨ

ͳͺ൰ ൌ –ƒଶ͵ ൬ ͹ߨ ͳͺ൰ ൌ

ͳ ͵ —›”ƒ ߨ

ͳͺǢ ͷߨ ͳͺǢ

͹ߨ

ͳͺŽ‰Š‹ệ…ủƒ’Šươ‰–”¿Š –ƒଶ͵ݔ ൌ ͳ ͵ ฻ ቆ͵ –ƒ ݔ െ –ƒ

ଷݔ

ͳ െ ͵ –ƒଶݔ ቇ ଶ

ൌͳ ͵

฻ ͵ –ƒ଺ݔ െ ʹ͹ –ƒସݔ ൅ ͵͵ –ƒଶݔ െ ͳ ൌ Ͳ

Šư :

–ƒଶ ߨ

ͳͺǢ –ƒଶ ͷߨ

ͳͺǢ –ƒଶ ͹ߨ

ͳͺŽ‰Š‹ệ…ủƒ’Šươ‰–”¿Š͵ݕଷെ ʹ͹ݕଶ൅ ͵͵ݕ െ ͳ ൌ Ͳ Š‡‘¯ịnh lý Viète, ta có :

ݕଵ൅ ݕଶ൅ ݕଷ ൌ ͻ ݕଵݕଶ൅ ݕଶݕଷ൅ ݕଷݕଵ ൌ ͳͳ

ݕଵݕଶݕଷ ൌ

ͳ ͵

Suy ൌ ݕଵଷ൅ ݕ

ଶଷ൅ ݕଷଷ ൌ ሺݕଵ൅ ݕଶ൅ ݕଷሻଷെ ͵ሺݕଵ൅ ݕଶ൅ ݕଷሻሺݕଵݕଶ൅ ݕଶݕଷ൅ ݕଷݕଵሻ ൅

͵ݕ ݕ ݕ ൌ Ͷ͵͵

ൌ –ƒ଺ ߨ

ͳͺ൅ –ƒ଺ ͷߨ

ͳͺ൅ –ƒ଺ ͹ߨ ͳͺ

(38)

Giải:Ở toán này, ta thấy –ƒ ݔ Ǣ –ƒ ݕ Ǣ –ƒ ݖ ൐ Ͳ

Do đó, theo định lý Viète, ta có :

൞ –ƒ ݔ ൅ –ƒ ݕ ൅ –ƒ ݖ ൌ െ ܾ ܽ

–ƒ ݔ –ƒ ݕ ൅ –ƒ ݕ –ƒ ݖ ൅ –ƒ ݖ –ƒ ݔ ൌ ܿ ܽ

Mặt khác :

ൌ െ ͳ ͺͳ൬

ܾ ܽ൰

ቀܿ ܽቁ

ൌ ͳ

ͺͳሺ–ƒ ݔ ൅ –ƒ ݕ ൅ –ƒ ݖሻଷሺ–ƒ ݔ –ƒ ݕ ൅ –ƒ ݕ –ƒ ݖ ൅ –ƒ ݖ –ƒ ݔሻଶ

Áp dụng bất đẳng thức :

Ta :

ͳ ͺͳ

൑ ͳ

ͺͳǤͻሺ–ƒ ݔ ൅ –ƒ ݕ ൅ –ƒ ݖሻ଻

Cần chứng minh bất đẳng thức :

ͳ

ͺͳǤͻሺ–ƒ ݔ ൅ –ƒ ݕ ൅ –ƒ ݖሻ଻ ൑

Thật vậy, với –ƒ ݔ ൅ –ƒ ݕ ൅ –ƒ ݖ ൐ Ͳ, ta có :

ሺ–ƒ ݔ ൅ –ƒ ݕ ൅ –ƒ ݖሻ ൒ ቀξ–ƒ ݔ Ǥ ඥ–ƒ଻ݔ ൅ ඥ–ƒ ݕ Ǥ ඥ–ƒ଻ݕ ൅ ξ–ƒ ݖ Ǥ ඥ–ƒ଻ݖቁଶ

൒ͳ

ͻǤ ͻሺ–ƒସݔ ൅ –ƒସݕ ൅ –ƒସݖሻଶ ൒ ͳ

ͺͳǤͻሺ–ƒଶݔ ൅ –ƒଶݕ ൅ –ƒଶݖሻସǤ ͵ସ ൒ ͳ

ͺͳǤͻሺ–ƒ ݔ ൅ –ƒ ݕ ൅ –ƒ ݖሻ଼

ܽݐଷ൅ ܾݐଶ൅ ܿݐ ൅ ݀ ൌ Ͳሺܽ ് ͲሻǢ Ͳ ൏ ݔǡ ݕǡ ݖ ൏ߨ

ʹ –ƒ଻ݔ ൅ –ƒ଻ݕ ൅ –ƒ଻ݖ ൒ െܾଷܿଶ

ͺͳܽହ

Bài 14: Cho –ƒ ݔ Ǣ –ƒ ݕ Ǣ –ƒ ݖ nghiệm phương trình :

Chứng minh

݉ଶ൅ ݊ଶ൅ ݇ଶ ൒ ݉݊ ൅ ݊݇ ൅ ݇݉

ฺ ሺ݉ ൅ ݊ ൅ ݇ሻଶ ൒ ͵ሺ݉݊ ൅ ݊݇ ൅ ݇݉ሻ

ͳ

ͻሺ݉ ൅ ݊ ൅ ݇ሻସ ൒ ሺ݉݊ ൅ ݊݇ ൅ ݇݉ሻଶ

(39)

Do đó,

–ƒ଻ݔ ൅ –ƒ଻ݕ ൅ –ƒ଻ݖ ൒ െܾଷܿଶ

ͺͳܽହ

Vậy ta có điều phải chứng minh

- BÀI TẬP TỰ LUYỆN

2.2.1 Tính giá trị biểu thức sau:

ƒǤ –ƒ ͻ୭െ –ƒ ʹ͹୭െ –ƒ ͸͵୭൅ –ƒ ͺͳ୭

„Ǥሺ…‘– ͶͶ

୭൅ –ƒ ʹʹ͸୭ሻ …‘• ͶͲ͸୭

…‘• ͵ͳ͸୭ െ …‘– ͹ʹ୭…‘– ͳͺ୭

…Ǥ …‘•ሺെʹͲ

୭ሻ •‹ ͹Ͳ୭

•‹ ͳ͸Ͳ୭…‘• ͵ͶͲ୭–ƒ ʹͷͲ୭

2.2.2 Tìm phương trình bậc có nghiệm

ݔଵ ൌ …‘•

ʹߨ

ͻ Ǣ ݔଶ ൌ …‘• Ͷߨ

ͻ Ǣ ݔଷ ൌ …‘• ͺߨ

ͻ

Từđó, tính tổng

ൌ …‘•ଶʹߨ

ͻ ൅ …‘•ଶ Ͷߨ

ͻ ൅ …‘•ଶ ͺߨ

ͻ

2.2.3 Cho

͵ •‹ସݔ ൅ ʹ …‘•ସݔ ൌ ͻͺ

ͺͳ

Tính ൌ ʹ •‹ସݔ ൅ ͵ …‘•ସݔ

2.2.4. Tính –ƒሺݔ െ Ͷͷ୭ሻ , biết

…‘• ݔ ൌ െ ͻ

Ͷͳǡ ݔ א ൬ߨǡ ͵ߨ

ʹ ൰

2.2.5. Rút gọn biểu thức sau :

ൌ ͳ ൅ …‘• ݔ ͳ െ …‘• ݔ–ƒଶ

ݔ

ʹെ …‘•ଶݔ ൌ •‹

ଶʹݔ െ Ͷ •‹ଶݔ

(40)

ൌ •‹ሺ͸Ͳ

୭൅ ݔሻ

Ͷ •‹ ቀʹͷ୭൅ݔ

Ͷቁ •‹ ቀ͹ͷ୭െݔͶቁ

ൌ •‹଺ሺߨ ൅ ݔሻ ൅ …‘•଺ሺݔ െ ߨሻ െ ʹ •‹ସሺʹߨ ൅ ݔሻ െ •‹ସ൬͵ߨ

ʹ ൅ ݔ൰ ൅ …‘•ଶቀ ߨ ʹെ ݔቁ ൌ –ƒ ቀݔ െ

ߨ

ʹቁ …‘• ቀ͵ߨʹ ൅ ݔቁ െ •‹ଷቀ͹ߨʹ െ ݔቁ …‘• ቀݔ െߨʹቁ –ƒ ቀ͵ߨʹ ൅ ݔቁ

ൌ •‹ ݔ ൅ ݉ •‹ ͵ݔ ൅ •‹ ͷݔ •‹ ͵ݔ ൅ ݉ •‹ ͷݔ ൅ •‹ ͹ݔ ൌ ξʹ െ •‹ ݔ െ …‘• ݔ

•‹ ݔ െ …‘• ݔ

௡ୢấ୳ୡ£୬

ߨ ൌ

ൌ ൌ

ൌ ʹ …‘• Ͷܽ ൅ ͷ …‘• ͸ܽ ൅ ʹ …‘• ʹܽ ൅ …‘• ͺܽ

2.2.6 Tính

ൌ ͳ ൅…‘• ܽ …‘• ܽ ൅

…‘• ʹܽ …‘•ଶʹܽ൅

…‘• ͵ܽ

…‘•ଷܽ൅ ǥ ൅

…‘• ݊ܽ …‘•௡ܽ

2.2.7. Tính ൌ ݉ •‹ ʹܽ ൅ ݊ …‘• ʹܽ biết

–ƒ ܽ ൌ݉

݊ǡ ݊ ് Ͳ ൌ ඪʹ ൅ ඩʹ ൅ ඨʹ ൅ ටǥ ൅ ξʹ ൅ ʹ …‘• ݔ

ᇣᇧᇧᇧᇧᇧᇧᇧᇧᇧᇧᇧᇤᇧᇧᇧᇧᇧᇧᇧᇧᇧᇧᇧᇥ

ቆݔ א ቀͲǡ ʹቁቇ •‹ଶሺܽ ൅ ܾሻ െ •‹ଶܽ െ •‹ଶܾ

•‹ଶሺܽ ൅ ܾሻ െ …‘•ଶܽ െ …‘•ଶܾ

•‹଺ܽ ൅ …‘•଺ܽ

Ͷ …‘•ଶʹܽ ൅ •‹ଶʹܽ

(41)

2.2.8. Tính –ƒ ܽ –ƒ ܾ theo ݉ǡ ݊ biết

…‘•ሺܽ ൅ ܾሻ …‘•ሺܽ െ ܾሻൌ

݉ ݊

2.2.9 Cho –ƒ ݔ ൌ Ͷ Tính giá trị biểu thức sau

ൌ ͹ …‘•

ହݔ െ ͵ …‘•ସݔ •‹ ݔ ൅ ʹ …‘•ଶݔ •‹ଷݔ െ ͷ …‘• ݔ •‹ସݔ െ ͳ͵ •‹ହݔ

•‹ସݔ …‘• ݔ െ ͳͳ •‹ଷݔ …‘•ଶݔ ൅ ʹ •‹ଶݔ …‘•ଷݔ െ ͻ •‹ ݔ …‘•ସݔ ൅ ͷ …‘•ହݔ

ൌ ͵ •‹

ସݔ െ ͺ •‹ଷݔ …‘• ݔ ൅ ͹ •‹ଶݔ …‘•ଶݔ െ Ͷ •‹ ݔ …‘•ଷݔ െ ʹ …‘•ସݔ

͹ •‹ସݔ ൅ ͷ •‹ଷݔ …‘• ݔ െ Ͷ •‹ଶݔ …‘•ଶݔ ൅ ͸ •‹ ݔ …‘•ଷݔ െ ͵ …‘•ସݔ

ሺܽǢ ܾ א ሾͲǡͳሿሻ ൝͵ܽଶ൅ ʹܾଶ ൌͻͺͺͳ

ܽ ൅ ܾ ൌ ͳ

2.2.4 Để ý

–ƒሺݔ െ Ͷͷ୭ሻ ൌ –ƒ ݔ െ –ƒ Ͷͷ୭

ͳ െ –ƒ ݔ –ƒ Ͷͷ୭

2.2.6 Để ý

…‘• ݇ܽ …‘•௞ܽ ൌ

•‹ ܽ …‘• ݇ܽ •‹ ܽ …‘•௞ܽ ൌ

ͳ

ʹ ሾ•‹ሺ݇ ൅ ͳሻܽ െ •‹ሺ݇ െ ͳሻܽሿ •‹ ܽ …‘•௞ܽ ൌ

•‹ሺ݇ ൅ ͳሻܽ •‹ ܽ …‘•௞ܽ െ

•‹ ݇ܽ •‹ ܽ …‘•௞ିଵܽ

2.2.7 Để ý

•‹ ʹܽ ൌ ʹ –ƒ ܽ

ͳ ൅ –ƒଶܽǢ …‘• ʹܽ ൌ

ͳ െ –ƒଶܽ

ͳ ൅ –ƒଶܽ

2.2.10 Cho –ƒ ݔ ൅ …‘– ݔ ൌ ݉ Tính

ൌ –ƒଶݔ െ …‘–ଶݔǢ ൌ –ƒଷݔ െ …‘–ଷݔǢ ൌ –ƒହݔ െ …‘–ହݔ

2.2.11 Cho –ƒ ݔ െ …‘– ݔ ൌ ݊ Tính

ൌ –ƒଶݔ ൅ …‘–ଶݔǢ ൌ –ƒଷݔ െ …‘–ଷݔǢ ൌ –ƒ଻ݔ െ …‘–଻ݔ

2.2.12. Cho –ƒሺܽ ൅ ܾሻ ൌ ݉ –ƒሺܽ െ ܾሻ ൌ ݊ Tính –ƒ ʹܽ

- GỢI Ý GIẢI BÀI TẬP TỰ LUYỆN 2.2.3 Đặt

ቄܽ ൌ •‹ଶݔ ܾ ൌ …‘•ଶݔ

(42)

2.2.8 Từ hệ thức

݉ ݊ ൌ

ͳ െ –ƒ ܽ –ƒ ܾ ͳ ൅ –ƒ ܽ –ƒ ܾ

Giải: Ta có :

•‹ସݔ

ܽ ൅

…‘•ସݔ

ܾ ൌ ͳ ܽ ൅ ܾ •‹ଶ଴ଵଶݔ

ܽଵ଴଴ହ ൅

…‘•ଶ଴ଵଶݔ

ܾଵ଴଴ହ ൌ

ͳ ሺܽ ൅ ܾሻଵ଴଴ହ

Bài 2: Chứng minh ܽǡ ܾ ൐ Ͳ Thì

Bài 1: Cho ൝

Ta biến đổi –ƒ ܽ –ƒ ܾ theo ݉ǡ ݊

2.2.9 Để ý bậc tử bậc mẫu, –ƒ ݔ có giá trị thực nên …‘• ݔ ് Ͳ, từđó ta

lần lượt chia tử mẫu cho …‘•ହݔ đối với cho …‘•ସݔ cho

2.2.10 Để ý

ൌ ൅ ݉

2.2.11.Để ý

ൌ ሺ–ƒସݔ ൅ …‘–ସݔሻ െ ݊

3. CHỨNG MINH ĐẲNG THỨC LƯỢNG GIÁC SUY TỪ ĐẲNG THỨC LƯỢNG GIÁC KHÁC CHO TRƯỚC

- Đây loại tập chứng minh đẳng thức lượng giác có điều kiện từđiều kiện kết hợp với công thức lượng giác phù hợp để suy điều cần phải chứng minh

ܽଶ൅ ܾଶ൅ ܿଶ ൌ •‹ଶݔ ൅ …‘•ଶݔ ሺͳ െ …‘•ଶݕሻ ൅ …‘•ଶݔ …‘•ଶݕ

ൌ •‹ଶݔ ൅ …‘•ଶݔ െ …‘•ଶݔ …‘•ଶݕ ൅ …‘•ଶݔ …‘•ଶݕ ൌ ͳ

Vậy ta có điều phải chứng minh

ܽ ൌ •‹ ݔ ܾ ൌ …‘• ݔ •‹ ݕ

ܿ ൌ …‘• ݔ …‘• ݕ Chứng minh : ܽ

(43)

Giải: Ta có :

ሺܽ ൅ ܾሻ ቆ•‹ସݔ ܽ ൅

…‘•ସݔ

ܾ ቇ ൌ ሺ•‹ଶݔ ൅ …‘•ଶݔሻଶ ฻ ܽ

ܾ…‘•ସݔ ൅ ܾ

ܽ•‹ସݔ െ ʹ •‹ଶݔ …‘•ଶݔ ൌ Ͳ ฻ •‹

ଶݔ

ܽ ൌ

…‘•ଶݔ

ܾ ൌ ͳ

ܽ ൅ ܾ൬ݐ ൌ ͳ

ܽ ൅ ܾ ൐ Ͳ൰

Suy

•‹ଶ଴ଵଶݔ

ܽଵ଴଴ହ ൅

…‘•ଶ଴ଵଶݔ

ܾଵ଴଴ହ ൌ

ͳ

ܽଵ଴଴ହሺܽݐሻଵ଴଴଺൅

ͳ

ܾଵ଴଴ହሺܾݐሻଵ଴଴଺ ൌ ൅

ܾ ሺܽ ൅ ܾሻଵ଴଴଺

ൌ ͳ ሺܽ ൅ ܾሻଵ଴଴ହ

ฺ ʹ ฺ ʹ –ƒ ݔ ൌ ͵ …‘– ݕ ฺ –ƒ ݕ ൌ ͵ ʹ –ƒ ݔ

Khi :

ͳ …‘•ଶݔ

ʹ •‹ଶݔ ൅ ͵ …‘•ଶݔ

…‘•ଶݔ

ͳ …‘•ଶݕ

ʹ •‹ଶݕ ൅ ͵ …‘•ଶݕ

…‘•ଶݕ

ൌ ͳ ൅ –ƒ

ଶݔ

ʹ –ƒଶݔ ൅ ͵൅

ͳ ൅ –ƒଶݕ

ʹ –ƒଶݕ ൅ ͵ ൌ

ͳ ൅ –ƒଶݔ

ʹ –ƒଶݔ ൅ ͵൅

Ͷ –ƒଶݔ ൅ ͻ

͸ሺ͵ ൅ –ƒଶݔሻ

ൌ͸ሺͳ ൅ –ƒ

ଶݔሻ ൅ Ͷ –ƒଶݔ ൅ ͻ

͸ሺ͵ ൅ ʹ –ƒଶݔሻ ൌ

ͷሺ͵ ൅ ʹ –ƒଶݔሻ

͸ሺ͵ ൅ ʹ –ƒଶݔሻൌ

ͷ ͸

ൌͷ ͸ ܽ ሺܽ ൅ ܾሻଵ଴଴଺

Giải: Ta có : ʹ •‹ ݔ •‹ ݕ െ ͵ …‘• ݔ …‘• ݕ ൌ Ͳ •‹ ݔ

…‘• ݔൌ ͵ …‘• ݕ •‹ ݕ

ͳ

ʹ •‹ଶݔ ൅ ͵ …‘•ଶݔ൅

ͳ

ʹ •‹ଶݕ ൅ ͵ …‘•ଶݕ

Bài 3: Cho ʹ •‹ ݔ •‹ ݕ െ ͵ …‘• ݔ …‘• ݕ ൌ Ͳ

(44)

Giải: Ta có :

…‘• …‘• ൌ ͳ

ʹሾ…‘•ሺ െ ሻ ൅ …‘•ሺ ൅ ሻሿ ൏ ͳ

ʹሺͳ െ …‘• ሻ ൌ •‹ଶ Ͳ ൏ ൏ ܥ ൏ ߨ

ʹ ฺ ʹ ൅ ൏ ߨ ฺ Ͳ ൏ ʹ ൏

ߨ

ʹെ ൏ ߨ ʹ 0ặ–ݑ ൌ …‘• Ǣ ݒ ൌ •‹

ʹǢ ݓ ൌ …‘• ฺ ൝ Ͳ ൏ ݒ

൏ ͳ

Và ݂ሺݔሻ ൌ ݉ݔଶ൅ ݊ݔ ൅ ݌

Ta có trường hợp sau :

ế—݌ ൌ Ͳ–Š¿݉ ݑ ൅

݊ ݒ ൌ Ͳ െ ݊

݉ ൌ

ݑ ݊

݉ቁ ൌ Ͳ…א‰Š‹ệ–Š—ộ…ሺͲǢ ͳሻ ế—݌ ് Ͳ–Š¿

ݑ ൅ ݊ ݒ൅

݌

ݓ ൌ Ͳ ฻ ݑ ݒଶቆ݉

ݒଶ

ݑଶ൅ ݊

ݒ

ݑ൅ ݌ቇ ൌ

ݑݓ െ ݒଶ

ݒଶݓ ݌

฻ ݂ ቀݒ ݑቁ ൌ

ݑݓ െ ݒଶ

ݑݓ ݂ሺͲሻ

Ởđây, ta sử dụng định lý : Nếu hàm số݂ liên tục đoạn ሾܽǢ ܾሿ ݂ሺܽሻǤ ݂ሺܾሻ ൏ Ͳ

thì tồn điểm ܿ א ሺܽǢ ܾሻ cho ݂ሺܿሻ ൌ Ͳ

Như vậy, ta thấy

݉ …‘• ൅

݊ •‹ʹ൅

݌

…‘• ൌ Ͳǡ ൏ ܥ ൏ ߨ ʹ

Bài 4: Cho ȟ ݉ǡ ݊ǡ ݌ thỏa

Chứng minh : ݉ݔଶ൅ ݊ݔ ൅ ݌ ൌ Ͳ có nghiệm ݔ א ሺͲǢ ͳሻ

(Đề nghị Olympic 30-4, 2006)

ʹ ฺ •‹

ʹ ൏ …‘• ݑ

Ͳ ൏ ݑݓ ൏ ݒଶ

- Nếu ݉ ൌ Ͳ ฺ ݂ሺݔሻ ൌ Ͳ có vơ số nghiệm thuộc ሺͲǢ ͳሻ

- Nếu ݉ ് Ͳ

(45)

݂Ž‹²–ụ…–”²Թ ฺ ݂Ž‹²–ụ…–”² ቂͲǢݒ ݑቃ ݂ ቀݒ

ݑቁ ݂ሺͲሻ ൌ

ݑݓ െ ݒଶ

ݑݓ ݂ଶሺͲሻ ൏ Ͳ

Do đó, ݂ሺݔሻ ൌ Ͳ có nghiệm thuộc ሺͲǢ ͳሻ Vậy ta có điều phải chứng minh

ቐ ͵

ʹ•‹ ʹݔ ൌ ͵ •‹ ݔ …‘• ݔ

൝ݔ ൅ ݕ ്•‹ ݔ ൌ ʹ •‹ሺݔ ൅ ݕሻߨ ʹ൅ ݇ߨǡ ݇ א Ժ –ƒሺݔ ൅ ݕሻ ൌ •‹ ݕ

…‘• ݕ െ ʹ

Bài 6: Chứng minh

Thì

(ĐH Thương Mại Hà Nội 1998)

Šứ‰‹Šݔ ൅ ʹݕ ൌߨ ʹ

Bài 5: Cho ݔǡ ݕ góc nhọn thỏa hệ൜͵ •‹ ʹݔ െ ʹ •‹ ʹݕ ൌ Ͳ͵ •‹ଶݔ ൅ ʹ •‹ଶݕ ൌ ͳ

Giải: Ta cần chứng minh …‘•ሺݔ ൅ ʹݕሻ ൌ Ͳ

Thật vậy, ta có …‘•ሺݔ ൅ ʹݕሻ ൌ …‘• ݔ …‘• ʹݕ െ •‹ ݔ •‹ ʹݕ

͵ •‹ଶݔ ൅ ʹ •‹ଶݕ ൌ ͳ ฺ ͵ •‹ଶݔ ൌ …‘• ʹݕ

͵ •‹ ʹݔ െ ʹ •‹ ʹݕ ൌ Ͳ ฺ •‹ ʹݕ ൌ

Suy

…‘•ሺݔ ൅ ʹݕሻ ൌ …‘• ݔ Ǥ ͵ •‹ଶݔ െ •‹ ݔ Ǥ ͵ •‹ ݔ …‘• ݔ ൌ Ͳ

(46)

Giải:

‘ݔ ൅ ݕ ് ߨ

ʹ൅ ݇ߨǡ ݇ א Ժ˜ …‘• ݕ െ ʹ ് Ͳ²¯‹ề—‹ệ¯ượ…šž…¯ịŠǤ ƒ…×ǣ•‹ ݔ ൌ •‹ሾሺݔ ൅ ݕሻ െ ݕሿ ൌ •‹ሺݔ ൅ ݕሻ …‘• ݕ െ …‘•ሺݔ ൅ ݕሻ •‹ ݕ ‘¯×

•‹ሺݔ ൅ ݕሻ …‘• ݕ െ …‘•ሺݔ ൅ ݕሻ •‹ ݕ ൌ ʹ •‹ሺݔ ൅ ݕሻ ฺ ሺ…‘• ݕ െ ʹሻ •‹ሺݔ ൅ ݕሻ ൌ …‘•ሺݔ ൅ ݕሻ •‹ ݕ ฺ –ƒሺݔ ൅ ݕሻ ൌ •‹ ݕ

…‘• ݕ െ ʹ

‹ải: Ta có :

…‘•ሺݔ െ ߙሻ

…‘•ሺݔ െ ߚሻ ൅•‹ሺݔ െ ߚሻ•‹ሺݔ െ ߙሻ ͳ ൅•‹ሺݔ െ ߚሻ•‹ሺݔ െ ߙሻ Ǥ…‘•ሺݔ െ ߙሻ…‘•ሺݔ െ ߚሻ

ͳ

ʹ ሾ•‹ ʹሺݔ െ ߙሻ ൅ •‹ ʹሺݔ െ ߚሻሿ

•‹ሺݔ െ ߙሻ …‘•ሺݔ െ ߚሻ ൅ •‹ሺݔ െ ߚሻ …‘•ሺݔ െ ߙሻ ൌ

•‹ሺʹݔ െ ߙ െ ߚሻ …‘•ሺߙ െ ߚሻ •‹ሺʹݔ െ ߙ െ ߚሻ ൌ …‘•ሺߙ െ ߚሻ

Vậy ta có điều phải chứng minh

•‹ሺݔ െ ߙሻ •‹ሺݔ െ ߚሻൌ

ܽ ܾ Ǣ

Ǣ ܽ ൅ ܾ ് Ͳ

Bài 7: Cho

Chứng minh

ܽ ൅ ܾ

ܽ ൅ ܾ ൌ ܽ ቀ

൅ ܾܽቁ ܽ ቀͳ ൅ ܾܽǤ ቁ

…‘•ሺݔ െ ߙሻ …‘•ሺݔ െ ߚሻൌ

…‘•ሺߙ െ ߚሻ ൌ ܽ ൅ ܾ

(47)

Giải: Lấy ሺͳሻ ൅ ሺʹሻ suy :

݉ ൅ ݊ ൌ ܽሺ…‘•ଷݔ ൅ •‹ଷݔሻ ൅ ͵ܽ •‹ ݔ …‘• ݔ ሺ•‹ ݔ ൅ …‘• ݔሻ

ൌ ܽሺ…‘• ݔ ൅ •‹ ݔሻሾሺ…‘•ଶݔ ൅ •‹ଶݔ െ …‘• ݔ •‹ ݔሻ ൅ ͵ …‘• ݔ •‹ ݔሿ

ൌ ܽሺ…‘• ݔ ൅ •‹ ݔሻሺ…‘• ݔ ൅ •‹ ݔሻଶ ൌ ܽሺ…‘• ݔ ൅ •‹ ݔሻଷ

Vậy యඥሺ݉ ൅ ݊ሻଶ

ൌ ሺ…‘• ݔ ൅ •‹ ݔሻଶǤ ξܽయ ଶ

Lấy ሺͳሻ െ ሺʹሻ suy :

Giải:Ở toán này, ta sử dụng công thức

ͳ െ …‘• ݔ

ͳ ൅ …‘• ݔ ൌ –ƒଶ ݔ ʹ –ƒܿ ൅ ܽ

ʹ –ƒ ܿ െ ܽ

ʹ ൌ

…‘• ܽ െ …‘• ܿ …‘• ܽ ൅ …‘• ܿ ൌ

…‘• ܽ െ …‘• ܽ …‘• ܾ …‘• ܽ ൅ …‘• ܽ …‘• ܾ ൌ

ͳ െ …‘• ܾ

ͳ ൅ …‘• ܾ ൌ –ƒଶ ܾ ʹ –ƒܿ ൅ ܽ

ʹ –ƒ ܿ െ ܽ

ʹ ൌ –ƒଶ ܾ ʹ ൜ܽ…‘•ଷݔ ൅ ͵ܽ …‘• ݔ •‹ଶݔ ൌ ݉ሺͳሻ

ܽ •‹ଷݔ ൅ ͵ܽ …‘•ଶݔ •‹ ݔ ൌ ݊ሺʹሻ

Bài 8: Cho

Chứng minh : యඥሺ݉ ൅ ݊ሻଶ൅ ඥሺ݉ െ ݊ሻయ ଶ ൌ ʹξܽయ ଶ

݉ െ ݊ ൌ ܽሺ…‘•ଷݔ െ •‹ଷݔሻ െ ͵ܽ …‘• ݔ •‹ ݔ ሺ…‘• ݔ െ •‹ ݔሻ

ൌ ܽሺ…‘• ݔ െ •‹ ݔሻሾሺ…‘•ଶݔ ൅ •‹ଶݔ ൅ …‘• ݔ •‹ ݔሻ െ ͵ …‘• ݔ •‹ ݔሿ

ൌ ܽሺ…‘• ݔ െ •‹ ݔሻଷ

Vậy యඥሺ݉ െ ݊ሻଶ ൌ ሺ…‘• ݔ െ •‹ ݔሻଶ య

Ǥ ξܽଶ

Do đó, ta :

ൌ ሾሺ…‘• ݔ ൅ •‹ ݔሻଶ൅ ሺ…‘• ݔ െ •‹ ݔሻଶሿǤయඥܽଶ ൌ ʹయඥܽଶ

(48)

Giải:Đặt

ݔ ൌ …‘• ܽ ݕ ൌ …‘• ܾ

ݖ ൌ …‘• ܿ ฺ ݔ ൅ ݕ ൅ ݖ ൌ Ͳ

Ta cần chứng minh: ͳʹݔݕݖ ൌ Ͷݔଷ൅ Ͷݕଷ൅ Ͷݖଷെ ͵ሺݔ ൅ ݕ ൅ ݖሻ

Hay ͵ݔݕݖ ൌ ݔଷ൅ ݕଷ൅ ݖଷ

Thật vậy, ta có :

Vậy ta có điều phải chứng minh

Giải: Áp dụng tính chất tỷ lệ thức, ta có :

ܽ ൅ ܾ

–ƒሺݔ ൅ ݕሻ ൅ –ƒሺݔ ൅ ݖሻ ൌ

ܽ െ ܾ

–ƒሺݔ ൅ ݕሻ െ –ƒሺݔ ൅ ݖሻ ฺ ܽ ൅ ܾ

ܽ െ ܾൌ

–ƒሺݔ ൅ ݕሻ ൅ –ƒሺݔ ൅ ݖሻ –ƒሺݔ ൅ ݕሻ െ –ƒሺݔ ൅ ݖሻൌ

•‹ሺʹݔ ൅ ݕ ൅ ݖሻ •‹ሺݕ െ ݖሻ

Do đó,

ൌ ܿ –ƒሺݔ ൅ ݐሻ ܾ ൅ ܿ

ܾ െ ܿ•‹ଶሺݖ െ ݐሻ ൅ ܿ ൅ ܽ

ܿ െ ܽ•‹ଶሺݐ െ ݕሻ ൌ Ͳ

Bài 10: Cho …‘• ܽ ൅ …‘• ܾ ൅ …‘• ܿ ൌ Ͳ

Chứng minh : ͳʹ …‘• ܽ …‘• ܾ …‘• ܿ ൌ …‘• ͵ܽ ൅ …‘• ͵ܾ ൅ …‘• ͵ܿ

Ͳ ൌ ሺݔ ൅ ݕ ൅ ݖሻଷ ൌ ݔଷ൅ ݕଷ൅ ݖଷ൅ ͵ሾݔݕሺݔ ൅ ݕሻ ൅ ݕݖሺݕ ൅ ݖሻ ൅ ݖݔሺݖ ൅ ݔሻሿ ൅ ͸ݔݕݖ

ൌ ݔଷ൅ ݕଷ൅ ݖଷ൅ ͵ሾݔݕሺെݖሻ ൅ ݕݖሺെݔሻ ൅ ݖݔሺെݕሻሿ ൅ ͸ݔݕݖ

ൌ ݔଷ൅ ݕଷ൅ ݖଷെ ͵ݔݕݖ

ܽ

–ƒሺݔ ൅ ݕሻ ൌ

ܾ –ƒሺݔ ൅ ݖሻ ܽ ൅ ܾ

ܽ െ ܾ•‹ଶሺݕ െ ݖሻ ൅

Bài 11: Cho sốܽǡ ܾǡ ܿđơi khác góc ݔǡ ݕǡ ݖǡ ݐđược liên hệ với hệ thức :

(49)

ܽ ൅ ܾ

ܽ െ ܾ•‹ଶሺݕ െ ݖሻ ൌ •‹ሺʹݔ ൅ ݕ ൅ ݖሻ •‹ሺݕ െ ݖሻ ൌ ͳ

ʹሾ…‘•ሺʹݔ ൅ ʹݖሻ െ …‘•ሺʹݔ ൅ ʹݕሻሿ

Tương tự, ta :

ܾ ൅ ܿ

ܾ െ ܿ•‹ଶሺݖ െ ݐሻ ൌ ͳ

ʹሾ…‘•ሺʹݔ ൅ ʹݐሻ െ …‘•ሺʹݔ ൅ ʹݖሻሿ ܿ ൅ ܽ

ܿ െ ܽ•‹ଶሺݐ െ ݕሻ ൌ ͳ

ʹሾ…‘•ሺʹݔ ൅ ʹݕሻ െ …‘•ሺʹݔ ൅ ʹݐሻሿ

Cộng đẳng thức lại, ta có điều phải chứng minh

ൌ ሺ…‘• ݔ ൅ …‘• ݕ ൅ …‘• ݖሻ …‘•ሺݔ ൅ ݕ ൅ ݖሻ ൅ ሺ•‹ ݔ ൅ •‹ ݕ ൅ •‹ ݖሻ •‹ሺݔ ൅ ݕ ൅ ݖሻ ൌ ܽ …‘•ଶሺݔ ൅ ݕ ൅ ݖሻ ൅ ܽ •‹ଶሺݔ ൅ ݕ ൅ ݖሻ ൌ ܽ

…‘• ݔ ൅ …‘• ݕ ൅ …‘• ݖ …‘•ሺݔ ൅ ݕ ൅ ݖሻ ൌ

Bài 12: Cho

Giải:Để ý :

…‘•ሺݔ ൅ ݕሻ ൌ …‘•ሾሺݔ ൅ ݕ ൅ ݖሻ െ ݖሿ ൌ …‘•ሺݔ ൅ ݕ ൅ ݖሻ …‘• ݖ ൅ •‹ሺݔ ൅ ݕ ൅ ݖሻ •‹ ݖ

Tương tự vậy, ta có :

…‘•ሺݕ ൅ ݖሻ ൌ …‘•ሺݔ ൅ ݕ ൅ ݖሻ …‘• ݔ ൅ •‹ሺݔ ൅ ݕ ൅ ݖሻ •‹ ݔ …‘•ሺݖ ൅ ݔሻ ൌ …‘•ሺݔ ൅ ݕ ൅ ݖሻ …‘• ݕ ൅ •‹ሺݔ ൅ ݕ ൅ ݖሻ •‹ ݕ

Cộng đẳng thức lại, ta :

•‹ ݔ ൅ •‹ ݕ ൅ •‹ ݖ •‹ሺݔ ൅ ݕ ൅ ݖሻ ൌ ܽ

Chứng minh : …‘•ሺݔ ൅ ݕሻ ൅ …‘•ሺݕ ൅ ݖሻ ൅ …‘•ሺݖ ൅ ݔሻ ൌ ܽ

(50)

Giải: Từ giả thuyết, ta có :

฻ …‘• ݔ ൌ …‘• ݕ ൅ …‘• ݖ ͳ ൅ …‘• ݕ …‘• ݖ ฺ –ƒଶݔ

ʹൌ

ͳ െ …‘• ݔ

ͳ ൅ …‘• ݔ ൌ ൌ

ሺͳ െ …‘• ݕሻሺͳ െ …‘• ݖሻ ሺͳ ൅ …‘• ݕሻሺͳ ൅ …‘• ݖሻ ‘Ͳ ൏ ݔǡ ݕǡ ݖ ൏

ʹǡ ݔ ് ݕǡ ݕ ് ݖ ฺ –ƒ ݔ ʹǡ –ƒ

ݕ ʹǡ –ƒ

ݖ ʹ൐ Ͳ

Khi đó,

–ƒݔ

ʹ ൌ –ƒ ݕ ʹ–ƒ

ݖ ʹ

Vậy ta có điều phải chứng minh

Ͳ ൏ ݔǡ ݕǡ ݖ ൏ߨ

ʹǡ ݔ ് ݕǡ ݕ ് ݖ …‘• ݔ െ …‘• ݕ

…‘• ݔ െ …‘• ݖ ൌ

•‹ଶݕ …‘• ݖ

•‹ଶݖ …‘• ݕ

–ƒݔ

ʹൌ –ƒ ݕ ʹ–ƒ

ݖ ʹ

Bài 13: Cho

Chứng minh

…‘• ݔ …‘• ݕ •‹ଶݖ െ …‘• ݔ •‹ଶݕ …‘• ݖ ൌ •‹ଶݖ …‘•ଶݕ െ •‹ଶݕ …‘•ଶݖ

Hay

…‘• ݔ ሺ…‘• ݕ •‹ଶݖ െ •‹ଶݕ …‘• ݖሻ ൌ ሺ•‹ ݖ …‘• ݕ െ •‹ ݕ …‘• ݖሻሺ•‹ ݖ …‘• ݕ ൅ •‹ ݕ …‘• ݖሻ

฻ …‘• ݔ ሾ…‘• ݕ ሺͳ െ …‘•ଶݖሻ െ ͳሺͳ െ …‘•ଶݕሻ …‘• ݖሿ ൌ •‹ሺݖ െ ݕሻ •‹ሺݖ ൅ ݕሻ

฻ …‘• ݔ ሺ…‘• ݕ െ …‘• ݖሻሺͳ ൅ …‘• ݕ …‘• ݖሻ ൌ …‘•ଶݕ െ …‘•ଶݖ

ͳ ൅ …‘• ݕ …‘• ݖ െ …‘• ݕ െ …‘• ݖ ͳ ൅ …‘• ݕ …‘• ݖ ൅ …‘• ݕ ൅ …‘• ݖ ൌ –ƒଶݕ

(51)

- BÀI TẬP TỰ LUYỆN

2.3.1 Cho góc ܽǡ ܾǡ ܿ thỏa điều kiện

–ƒଶܽ –ƒଶܾ ൅ –ƒଶܾ –ƒଶܿ ൅ –ƒଶܿ –ƒଶܽ ൅ ʹ –ƒଶܽ –ƒଶܾ –ƒଶܿ ൌ ͳ

Chứng minh : •‹ଶܽ ൅ •‹ଶܾ ൌ …‘•ଶܿ

2.3.2 Cho …‘•ሺܽ ൅ ܾሻ ൌ ݇ …‘•ሺܽ െ ܾሻ Ǣ ݇ ് െͳ

Chứng minh

–ƒ ܽ –ƒ ܾ ൌͳ െ ݇ ͳ ൅ ݇

2.3.3 Cho ܽ ൅ ܾ ൅ ܿ ൌ ͻͲ୭ Chứng minh :

ܽ ൅ ܾ ൅ ܿ ൌߨ ʹ

2.3.5 Chứng minh

…‘• ܽ ൌ •‹

•‹ Ǣ…‘• ܾ ൌ •‹

Thì –ƒଶ ൌ –ƒଶ ൅ –ƒଶ

2.3.6 Cho

(Đề nghị Olympic 30-4, 2004)

ݔ

ͳ െ ݔǤͳ െ ݕ൅ ݕ ͳ െ ݕǤ

ݖ ͳ െ ݖ൅

ݖ ͳ െ ݖǤ

ݔ

ͳ െ ݔ൅ ʹ ݔ ͳ െ ݔǤ

ݕ ͳ െ ݕǤ

ݖ

ͳ െ ݖൌ ͳ

2.3.2 Từ giả thuyết, ta rút

ሺͳ െ ݇ሻ …‘• ܽ …‘• ܾ ൌ ሺͳ ൅ ݇ሻ •‹ ܽ •‹ ܾ

Chỉ cần chứng minh …‘• ܽ …‘• ܾ ് Ͳ ݇ ് െͳthì ta có điều phải chứng minh

2.3.3.Để ý, từ giả thuyết, ta :

൜•‹ ܽ ൌ …‘•ሺܾ ൅ ܿሻ…‘• ܽ ൌ •‹ሺܾ ൅ ܿሻ

2.3.4 Điều cần chứng minh tương đương với …‘–ሺܽ ൅ ܾ ൅ ܿሻ ൌ Ͳ hay

Ͳ ൌ …‘–ሺܽ ൅ ܾ ൅ ܿሻ ൌ …‘– ܽ …‘– ܾ …‘– ܿ െ …‘– ܿ െ …‘– ܽ െ …‘– ܾ …‘– ܽ …‘– ܾ െ ͳ ൅ …‘– ܿ ሺ…‘– ܽ ൅ …‘– ܾሻ •‹ଶܽ ൅ •‹ଶܾ ൅ •‹ଶܿ ൌ ͳ െ ʹ •‹ ܽ •‹ ܾ •‹ ܿ

2.3.4 Cho …‘– ܽ ൅ …‘– ܾ ൅ …‘– ܿ ൌ …‘– ܽ …‘– ܾ …‘– ܿ Chứng minh

൅ ݇ߨǡ ݇ א Ժ •‹

Ǣ…‘•ሺܽ ൅ ܾሻ ൌ •‹ •‹ ሺ–ƒ଺ʹͲ୭െ ͵͵ –ƒସʹͲ୭൅ ʹ͹ –ƒଶʹͲ୭ሻ௡െ ͳ ڭ ʹଶ଴଴ସǡ ݊ א Գ

Chứng minh : ݊ ൒ ʹଶ଴଴ଶ

- GỢI Ý GIẢI BÀI TẬP TỰ LUYỆN 2.3.1 Đặt ݔ ൌ •‹ଶܽǢ ݕ ൌ •‹ଶܾǢ ݖ ൌ •‹ଶܿ

Ta chứng minh ݔ ൅ ݕ ൅ ݖ ൌ ͳ

Biến đổi từ giả thuyết sau :

(52)

2.3.5 Từ giả thuyết, ta biến đổi sau :

…‘• ܽ …‘• ܾ െ •‹ ܽ •‹ ܾ ൌ •‹ •‹ ฺ ሺͳ െ …‘•ଶܽሻሺͳ െ …‘•ଶܾሻ ൌ ሺ…‘• ܽ …‘• ܾ െ •‹ •‹ ሻଶ

ฺ ቆͳ െ•‹

•‹ଶቇ ቆͳ െ

•‹ଶ

•‹ଶቇ ൌ •‹ଶ •‹ଶ ൬

ͳ

•‹ଶെ ͳ൰ ଶ

ൌ •‹ଶ •‹ଶ …‘–ସ

Chia vế đẳng thức cho •‹ଶ •‹ଶ, ta được :

൬ ͳ •‹ଶെ

ͳ •‹ଶ൰ ൬

ͳ •‹ଶെ

ͳ

•‹ଶ൰ ൌ …‘–ସ

ฺ ሺ…‘–ଶ െ …‘–ଶሻሺ…‘–ଶ െ …‘–ଶሻ ൌ …‘–ସ

ฺ …‘–ଶ …‘–ଶ ൌ …‘–ଶ ሺ…‘–ଶ ൅ …‘–ଶሻ

ฺ ͳ …‘–ଶ ൌ

ͳ …‘–ଶ൅

ͳ …‘–ଶ

ቇ ൅ ݒଶሺ݊ሻ

x Đặt ẩn phụܽ ൌ …‘•ଶݔǢ ܾ ൌ •‹ଶݔ với kết quả sau : ܽ ൅ ܾ ൌ ͳǢ ܽଶ൅ ܾଶ ൌ ͳ െ

ʹܾܽǢ ܽଷ൅ ܾଷ ൌ ͳ െ ͵ܾܽǢ ܽସ൅ ܾସ ൌ ͳ ൅ ʹܽଶܾଶെ Ͷܾܽ ǥ

x Dùng công thức hạ bậc

x Đặt ẩn phụ theo ݐ ൌ –ƒଶݔ

- Chú ý : Đối với bạn đọc biết khái niệm đạo hàm hàm số

lượng giác, ta dùng kiến thức

Nếu ݂ᇱሺݔሻ ൌ Ͳ với mọi ݔ א ݂ሺݔሻ hàm hằng với mọi ݔ א .

Từđó, ta có điều phải chứng minh

2.3.6 Để ý 2.1.5 ta chứng minh –ƒଶʹͲ୭ nghiệm của phương trình ݔଷെ

͵͵ݔଶ൅ ʹ͹ݔ െ ͵ ൌ Ͳ, nên –ƒ଺ʹͲ୭െ ͵͵ –ƒସʹͲ୭൅ ʹ͹ –ƒଶʹͲ୭ ൌ ͵

Do đó, giả thuyết tương đương với : ͵௡െ ͳ ڭ ʹଶ଴଴ସ

Lưu ý mệnh đề : ܽȁܾ ฻ ݒ௣ሺܽሻ ൑ ݒ௣ሺܾሻǡ ݌ nguyên tố Suy : ͵௡െ ͳ ڭ ʹଶ଴଴ସ ฻ ݒ

ଶሺ͵௡െ ͳሻ ൒ ݒଶሺʹଶ଴଴ସሻ ൌ ʹͲͲͶ

Mặt khác, ta có bổđề :

ݒଶሺܽ௡െ ܾ௡ሻ ൌ ݒଶቆ

ܽଶെ ܾଶ

ʹ

Nên ݒଶሺ͵௡െ ͳሻ ൌ ݒ

ଶሺͶሻ ൅ ݒଶሺ݊ሻ ൒ ʹͲͲͶ

Do đó, ݒଶሺ݊ሻ ൒ ʹͲͲʹ ฺ ݊ ൒ ʹଶ଴଴ଶ

Trong đó, ݒ௣ሺݔሻ ൌ ܽ hiểu ܽ௣ȁݔnhưng ܽ௣ାଵץ ݔ

4. CHỨNG MINH BIỂU THỨC LƯỢNG GIÁC KHÔNG PHỤ THUỘC VÀO BIẾN SỐ

- Khi gặp biểu thức ݂ሺݔሻ có chứa•‹ଶݔǢ …‘•ଶݔ, ta thường sử dụng phương

(53)

Giải:

ൌ ͵ሺ•‹ସݔ ൅ …‘•ସݔሻ െ ʹሺ•‹଺ݔ ൅ …‘•଺ݔሻ

ൌ ͵ ൬ͳ െͳ

ʹ•‹ଶʹݔ൰ െ ʹ ൬ͳ െ ͵ Ͷ

ͳ ʹ൅

ͳ

ʹ…‘• ʹݔ െ ͳ

ʹሺ…‘• ʹܽ ൅ …‘• ʹݔሻ ൌ •‹ଶሺܽ െ ݔሻ ൅ •‹ሺܽ ൅ ݔሻ ሾ•‹ሺܽ ൅ ݔሻ ൅ ʹ •‹ሺܽ െ ݔሻ …‘• ʹܽሿ

ൌ •‹ଶሺܽ െ ݔሻ ൅ •‹ሺܽ ൅ ݔሻ ሾ•‹ሺܽ ൅ ݔሻ ൅ •‹ሺ͵ܽ െ ݔሻ ൅ •‹ሺെܽ െ ݔሻሿ

ൌ •‹ଶሺܽ െ ݔሻ ൅ •‹ሺܽ ൅ ݔሻ •‹ሺ͵ܽ െ ݔሻ

ൌͳ െ …‘•ሺʹܽ െ ʹݔሻ ʹ ൅

ͳ

ʹ…‘•ሺെʹܽ ൅ ʹݔሻ െ ͳ

ʹ…‘• Ͷܽ ൌ ͳ

ʹሺͳ െ …‘• Ͷܽሻ ൌ •‹ଶʹܽ

ൌ •‹଼ݔ ൅ …‘•଼ݔ ൅ ͸ •‹ସݔ …‘•ସݔ ൅ Ͷ •‹ଶݔ …‘•ଶݔ ሺ•‹ସݔ ൅ …‘•ସݔሻ

Đặt ݐ ൌ –ƒଶݔ

ൌ •‹ସݔ ሺ͵ െ ʹ •‹ଶݔሻ ൅ …‘•ସݔ ሺ͵ െ ʹ …‘•ଶݔሻ

ൌ ͵ሺ•‹଼ݔ െ …‘•଼ݔሻ ൅ Ͷሺ…‘•଺ݔ െ ʹ •‹଺ݔሻ ൅ ͸ •‹ସݔ

ൌ …‘•ଶݔ ൅ …‘•ଶሺܽ ൅ ݔሻ െ ʹ …‘• ܽ …‘• ݔ …‘•ሺܽ ൅ ݔሻ

ൌ •‹ଶሺܽ ൅ ݔሻ ൅ •‹ଶሺܽ െ ݔሻ ൅ ʹ •‹ሺܽ ൅ ݔሻ •‹ሺܽ െ ݔሻ …‘• ʹܽ

ൌ •‹଼ݔ ൅ …‘•଼ݔ ൅ ͸ •‹ସݔ …‘•ସݔ ൅ Ͷ •‹ଶݔ …‘•ଶݔ ሺ•‹ସݔ ൅ …‘•ସݔሻ

Bài 1: Chứng minh biểu thức sau không phụ thuộc vào ݔ

ൌ ͵ሾሺ•‹ଶݔ ൅ …‘•ଶݔሻଶെ ʹ •‹ଶݔ …‘•ଶݔሿ

െ ʹሺ•‹ଶݔ ൅ …‘•ଶݔሻሺ•‹ସݔ െ •‹ଶݔ …‘•ଶݔ ൅ …‘•ସݔሻ

•‹ଶʹݔ൰ ൌ ͳ

ൌ ͵ሾܽସെ ሺͳ െ ܽሻସሿ ൅ Ͷሾሺͳ െ ܽሻଷെ ʹܽଷሿ ൅ ͸ܽଶǢ ܽ ൌ •‹ଶݔ

ൌ ͵ሺͶܽଷെ ͸ܽଶ൅ Ͷܽ െ ͳሻ ൅ Ͷሺെ͵ܽଷ൅ ͵ܽଶെ ͵ܽ ൅ ͳሻ ൅ ͸ܽଶ

ൌ ͳʹܽଷെ ͳͺܽଶ൅ ͳʹܽ െ ͵ െ ͳʹܽଷ൅ ͳʹܽଶെ ͳʹܽ ൅ Ͷ ൅ ͸ܽଶ ൌ ͳ

ൌ …‘•ଶݔ ൅ …‘•ሺܽ ൅ ݔሻ ሾ…‘•ሺܽ ൅ ݔሻ െ ʹ …‘• ܽ …‘• ݔሿ

ൌ …‘•ଶݔ ൅ …‘•ሺܽ ൅ ݔሻ ሾെ …‘• ܽ …‘• ݔ െ •‹ ܽ •‹ ݔሿ

ൌ …‘•ଶݔ െ …‘•ሺܽ ൅ ݔሻ …‘•ሺܽ െ ݔሻ ൌ

(54)

ฺ •‹ଶݔ ൌ ݐ

ݐ ൅ ͳǢ …‘•ଶݔ ൌ ͳ ݐ ൅ ͳ ฺ ൌ ݐ

ሺݐ ൅ ͳሻସ൅

ͳ

ሺݐ ൅ ͳሻସ൅ ͸

ݐଶ

ሺݐ ൅ ͳሻସ൅ Ͷ

ݐ ሺݐ ൅ ͳሻଶቈ

ݐଶ

ሺݐ ൅ ͳሻଶ൅

ͳ ሺݐ ൅ ͳሻଶ቉

ൌݐ

ସ൅ Ͷݐଷ ൅ ͸ݐଶ൅ Ͷݐ ൅ ͳ

ሺݐ ൅ ͳሻସ ൌ ͳ

Giải: Ta có :

ൌ ൌ

ሾʹ …‘•ଶሺܽ െ ܾሻ െ ʹ ൅ ʹ •‹ଶሺܽ ൅ ܾሻሿ

ൌ ʹሾͳ െ ݉

ଶ•‹ଶሺܽ ൅ ܾሻሿ

ͳ െ ʹ݉ଶ•‹ଶሺܽ ൅ ܾሻ ൅ ݉ସ•‹ଶሺܽ ൅ ܾሻ െ ݉ଶ൅ ݉ଶ•‹ଶሺܽ ൅ ܾሻ

ൌ ʹሾͳ െ ݉

ଶ•‹ଶሺܽ ൅ ܾሻሿ

ͳ െ ݉ଶ•‹ଶሺܽ ൅ ܾሻ ൅ ݉ଶሾ݉ଶ•‹ଶሺܽ ൅ ܾሻ െ ͳሿ

ൌ ʹሾͳ െ ݉ଶ•‹ଶሺܽ ൅ ܾሻሿ

ሾͳ െ ݉ଶ•‹ଶሺܽ ൅ ܾሻሿሺͳ െ ݉ଶሻൌ

ʹ ͳ െ ݉ଶ

Do ܽ െ ܾ ് ݇ߨ nên ݉ •‹ሺܽ ൅ ܾሻ ൌ …‘•ሺܽ െ ܾሻ ് േͳ ฺ ݉ଶ•‹ଶሺܽ ൅ ܾሻ ് ͳ

ൌ ͳ

ͳ െ ݉ •‹ ʹܽ ൅

ͳ ͳ െ ݉ •‹ ʹܾ

Bài 2: Chứng minh : Nếu ݉ •‹ሺܽ ൅ ܾሻ ൌ …‘•ሺܽ െ ܾሻ ǡ ܽ െ ܾ ് ݇ߨǡ ݇ א Ժǡ ݉ ് േͳ

thì biểu thức sau khơng phụ thuộc vào ܽǡ ܾ

ʹ െ ݉ሺ•‹ ʹܽ ൅ •‹ ʹܾሻ

ͳ െ ݉ሺ•‹ ʹܽ ൅ •‹ ʹܾሻ ൅ ݉ଶ•‹ ʹܽ •‹ ʹܾ

ʹ െ ʹ݉ •‹ሺܽ ൅ ܾሻ …‘•ሺܽ െ ܾሻ

ͳ െ ʹ݉ •‹ሺܽ ൅ ܾሻ …‘•ሺܽ െ ܾሻ ൅݉ʹଶሾ…‘• ʹሺܽ െ ܾሻ െ …‘• ʹሺܽ ൅ ܾሻሿ

Mà ݉ •‹ሺܽ ൅ ܾሻ ൌ …‘•ሺܽ െ ܾሻ nên …‘•ଶሺܽ െ ܾሻ ൌ ݉ଶ•‹ଶሺܽ ൅ ܾሻ

Do đó,

ʹሾͳ െ ݉ଶ•‹ଶሺܽ ൅ ܾሻሿ

(55)

Giải:Ở này, ta có cách chứng minh

Cách 1: Ta chứng minh ݂ሺݔሻ hàm ׊ݔ א Թ

Thật vậy, ta có :

݂ሺݔሻ ൌ •‹଺ݔ ൅͵

Ͷ

͵

Ͷ•‹ଶʹݔ ൌ ͳ െ ͵ •‹ଶݔ …‘•ଶݔ ൅͵

Ͷ ฺ ݂ ቀ ߨ ቁ ൌ ͳ

͵

ʹ•‹ Ͷݔ

ʹ•‹ Ͷݔ ൌ െ͵ •‹ ʹݔ …‘• ʹݔ ൅ ͵

Ͷ•‹ Ͷݔ ൌ Ͳ

Như vậy,

݂ ቀ ߨ

ͳͻͻͺቁ ൌ ݂ ቀ ߨ ʹቁ ൌ ͳ

݂ሺݔሻ ൌ •‹଺ݔ ൅ …‘•଺ݔ ൅ ݉ሺ•‹ସݔ ൅ …‘•ସݔሻ ൅ ʹ •‹ଶʹݔ

Bài 4: Tìm ݉ để giá trị hàm số sau không phụ thuộc vào biến số

݂ሺݔሻ ൌ •‹଺ݔ ൅͵

Ͷ•‹ଶʹݔ ൅ …‘•଺ݔ ݂ ቀ ߨ

ͳͻͻͺቁ

Bài 3: Cho

a Chứng minh : ݂ᇱሺݔሻ ൌ Ͳǡ ׊ݔ א Թ

b Tính giá trị

(ĐH Hồng Đức 1998)

•‹ଶʹݔ ൅ …‘•଺ݔ ൌ •‹ସݔ ൅ …‘•ସݔ െ •‹ଶݔ …‘•ଶݔ ൅

•‹ଶʹݔ ൌ ͳ

ͳͻͻͺ

Cách 2: Ta có :

݂ᇱሺݔሻ ൌ ͸ •‹ହݔ …‘• ݔ െ ͸ …‘•ହݔ •‹ ݔ ൅ ͵ •‹ ʹݔ …‘• ʹݔ

ൌ ͸ •‹ ݔ …‘• ݔ ሺ•‹ସݔ െ …‘•ସݔሻ ൅

(56)

Giải: Ta có :

•‹଺ݔ ൅ …‘•଺ݔ ൌ •‹ସݔ ൅ …‘•ସݔ െ •‹ଶݔ …‘•ଶݔ ൌ ͳ െ͵

Ͷ•‹ଶʹݔ ݉ሺ•‹ସݔ ൅ …‘•ସݔሻ ൌ ݉ሺͳ െ •‹ଶݔ …‘•ଶݔሻ ൌ ݉ ൬ͳ െͳ

Ͷ•‹ଶʹݔ൰

Vậy

݂ሺݔሻ ൌ ͳ െ͵

Ͷ•‹ଶʹݔ ൅ ݉ ൬ͳ െ ͳ

Ͷ•‹ଶʹݔ൰ ൅ ʹ •‹ଶʹݔ ൌ ͳ ൅ ݉ ൅ ൬ ͷ Ͷെ

݉

Ͷ൰ •‹ଶʹݔ

(57)

- BÀI TẬP TỰ LUYỆN

2.4.1 Chứng minh biểu thức sau không phụ thuộc vào biến số

ൌ ʹሺ•‹ସݔ ൅ …‘•ସݔ ൅ •‹ଶݔ …‘•ଶݔሻଶെ ሺ•‹଼ݔ ൅ …‘•଼ݔሻ

ൌ …‘•ଶݔ ൅ …‘•ଶቀߨ

͵൅ ݔቁ ൅ …‘•ଶቀ ߨ ͵െ ݔቁ ൌ …‘•ସݔ ൅ …‘•ସቀݔ ൅ߨ

Ͷቁ ൅ …‘•ସቀݔ ൅ ߨ

ʹቁ ൅ …‘•ସ൬ݔ ൅ ͵ߨ

Ͷ ൰ ൌ ʹ

–ƒ ݔ െ ͳ൅

…‘– ݔ ൅ ͳ …‘– ݔ െ ͳ ൌ •‹ ݔ

•‹ ݔ െ ξ…‘–ଶݔ െ …‘•ଶݔെ

…‘•ଶݔ

ʹ •‹ଶݔ െ ͳǡ Ͳ ൏ ݔ ൏

ߨ ʹ ൌ ඥ•‹ସݔ ൅ Ͷ …‘•ଶݔ ൅ ඥ…‘•ସݔ ൅ Ͷ •‹ଶݔ

ൌ ඨ•‹ସݔ െͳ

Ͷ…‘•ଶݔ ൬ ͵

ʹെ ʹ …‘• ʹݔ ൅ ͳ

ʹ…‘• Ͷݔ൰

൅ ඨ•‹ସݔ …‘•ଶݔ ൅ͳ

ʹ

…‘• ݉ ൌ െͳ ʹ …‘•ଶʹݔ െ •‹ଶݔ ൅ͳ

ʹ

2.4.2 Tìm ݉để giá trị hàm số sau khơng phụ thuộc vào biến số

݂ሺݔሻ ൌ •‹ሺݔ ൅ ݉ሻ ൅ •‹ሺݔ ൅ ʹ݉ሻ ൅ •‹ ݔ ൅ ܽ

- GỢI Ý GIẢI BÀI TẬP TỰ LUYỆN 2.4.2 Để ý

݂ሺݔሻ ൌ ʹ •‹ሺݔ ൅ ݉ሻ …‘• ݉ ൅ •‹ሺݔ ൅ ݉ሻ ൅ ܽ ൌ ሺͳ ൅ ʹ …‘• ݉ሻ •‹ሺݔ ൅ ݉ሻ ൅ ܽ

(58)

CH

ƯƠ

NG 3

HÊ TH

C L

ƯỢ

NG TRONG TAM GIÁC

1. ĐỊNH LÝ HÀM SỐ SIN

Trong tam giác , ta ln có :

ൌ ܾ •‹ ൌ

ܿ

•‹ ൌ ʹܴ

ܽଶ ൌ ܾଶ൅ ܿଶെ ʹܾܿ …‘• ൌ ሺܾ െ ܿሻଶ൅ Ͷܾܿ •‹ଶ

ʹ ܾଶ ൌ ܿଶ൅ ܽଶെ ʹܿܽ …‘• ൌ ሺܿ െ ܽሻଶ൅ Ͷܿܽ •‹ଶ

ʹ

I. CÁC KÝ HIỆU CƠ BẢN

െǡ ǡ : góc đỉnh ǡ ǡ

െܽǡ ܾǡ ܿ: độ dài cạnh đối diện với đỉnh ǡ ǡ െ݄௔ǡ ݄௕ǡ ݄௖ : độdài đường cao hạ từđỉnh ǡ ǡ

െ݉௔ǡ ݉௕ǡ ݉௖ : độdài đường trung tuyển kẻ từđỉnh ǡ ǡ െ݈௔ǡ ݈௕ǡ ݈௖ : độ dài đường phân giác kẻ từđỉnh ǡ ǡ െܴ: bán kính đường trịn ngoại tiếp tam giác

െݎ: bán kính đường trịn nội tiếp tam giác

െݎ௔ǡ ݎ௕ǡ ݎ௖ : bán kính đường tròn bàng tiếp tam giác đỉnh ǡ ǡ

െ݌ : nửa chu vi tam giác

െܵ : diện tích tam giác

II. CÁC ĐỊNH LÝ VÀ CƠNG THỨC CƠ BẢN

ܽ •‹

Từđó, ta có hệ sau :

ܽ ൌ ʹܴ •‹ Ǣ ܾ ൌ ʹܴ •‹ Ǣ ܿ ൌ ʹܴ •‹

2. ĐỊNH LÝ HÀM SỐ COS

(59)

ܿଶ ൌ ܽଶ ൅ ܾଶെ ʹܾܽ …‘• ൌ ሺܽ െ ܾሻଶ൅ Ͷܾܽ •‹ଶ

ʹ

Từđó, ta có hệ sau để tính sốđo góc tam giác :

…‘• ൌܾ

ଶ൅ ܿଶെ ܽଶ

ʹܾܿ Ǣ…‘• ൌ

ܿଶ൅ ܽଶെ ܾଶ

ʹܿܽ Ǣ…‘• ൌ

ܽଶ൅ ܾଶെ ܿଶ

ʹܾܽ

Từ hệ quảtrên, ta có thêm kết sau :

൐ߨ

ʹ ฻ ܾଶ൅ ܿଶ ൏ ܽଶǢ ൐ ߨ

ʹ ฻ ܿଶ൅ ܽଶ ൏ ܾଶǢ ൐ ߨ

ʹ ฻ ܽଶ൅ ܾଶ ൏ ܿଶ

3. ĐỊNH LÝ HÀM SỐ TAN

Trong tam giác , ta có :

ܽ െ ܾ ܽ ൅ ܾ ൌ

–ƒ െ ʹ –ƒ ൅ ʹ Ǣ

ܾ െ ܿ ܾ ൅ ܿ ൌ

–ƒ െ ʹ

–ƒ ൅ ʹ Ǣ ൌ

–ƒ െ ʹ –ƒ ൅ ʹ ܽଶ൅ ܾଶ൅ ܿଶ

Ͷܵ

݉௔ଶ ൌʹܾଶ൅ ʹܿଶെ ܽଶ

Ͷ ฺ ܽଶ

ʹ ൅ ʹ݉௔ଶ ൌ ܾଶ൅ ܿଶ ݉௕ଶ ൌʹܿଶ൅ ʹܽଶെ ܾଶ

Ͷ ฺ ܾଶ

ʹ ൅ ʹ݉௕ଶ ൌ ܿଶ൅ ܽଶ ݉௖ଶ ൌ

ʹܽଶ൅ ʹܾଶെ ܿଶ

Ͷ ฺ ܿଶ

ʹ ൅ ʹ݉௖ଶ ൌ ܽଶ൅ ܾଶ

Từđó, ta có cơng thức tổng bình phương đường trung tuyến tam giác :

݉ଶ൅ ݉ଶ൅ ݉ଶ ൌ͵ሺܽଶ൅ ܾଶ൅ ܿଶሻ

ܿ െ ܽ ܿ ൅ ܽ

4. ĐỊNH LÝ HÀM SỐ COT

Trong tam giác , ta ln có :

…‘– ൅ …‘– ൅ …‘– ൌ

5. ĐỊNH LÝ CÁC HÌNH CHIẾU

Trong tam giác , ta ln có :

ܽ …‘• ൅ ܿ …‘• ൌ ܾǢ ܽ …‘• ൅ ܾ …‘• ൌ ܿǢ ܾ …‘• ൅ ܿ …‘• ൌ ܽ

6. CÔNG THỨC VỀĐỘ DÀI TRUNG TUYẾN

(60)

7. CÔNG THỨC VỀĐỘ DÀI PHÂN GIÁC TRONG

Trong tam giác , độ dài đường phân giác xác định công thức :

݈௔ ൌ

ʹܾܿ ܾ ൅ ܿ…‘•

ʹ ൌ

ʹܾܿ ܾ ൅ ܿඨ

݌ሺ݌ െ ܽሻ ܾܿ ݈௕ ൌ ʹܿܽ

ܿ ൅ ܽ…‘• ʹ ൌ

ʹܿܽ ܿ ൅ ܽඨ

݌ሺ݌ െ ܾሻ ܿܽ ݈௖ ൌ

ʹܾܽ ܽ ൅ ܾ…‘•

ʹൌ

ʹܾܽ ܽ ൅ ܾඨ

݌ሺ݌ െ ܿሻ ܾܽ ݄௔ ൌ

ʹܵ

ܽ Ǣ݄௕ ൌ ʹܵ ݎ ൌ ܵ

݌

ʹ ൌ ሺ݌ െ ܿሻ –ƒ ʹ ܴ ൌ ൌ ܽ

ʹ •‹ ൌ ܾ ʹ •‹ ൌ

ܿ ʹ •‹ ݎ௔ ൌ ݌ –ƒ

ʹ ൌ

ܵ ݌ െ ܽ ݎ௕ ൌ ݌ –ƒ

ʹ ൌ

ܵ ݌ െ ܾ ݎ௖ ൌ ݌ –ƒ

ʹൌ

ܵ ݌ െ ܿ

8. CÔNG THỨC VỀĐỘ DÀI ĐƯỜNG CAO

Trong tam giác , độ dài đường cao xác định công thức :

ܾ Ǣ݄௖ ൌ ʹܵ

ܿ

9. CƠNG THỨC VỀĐỘ DÀI BÁN KÍNH

a BÁN KÍNH ĐƯỜNG TRỊN NỘI TIẾP

ൌ ሺ݌ െ ܽሻ –ƒ

ʹ ൌ ሺ݌ െ ܾሻ –ƒ

b BÁN KÍNH ĐƯỜNG TRỊN NGOẠI TIẾP

ܾܽܿ Ͷܵ

(61)

10. CƠNG THỨC VỀ DIỆN TÍCH TAM GIÁC

Ta có cơng thức tính diện tích tam giác nhiều công thức khác :

ܵ ൌ ە ۖ ۖ ۖ ۔ ۖ ۖ ۖ

ۓ ͳ

ʹ݄ܽ௔ ൌ ͳ

ʹܾ݄௕ ൌ ͳ ʹ݄ܿ௖ ͳ

ʹܾܿ •‹ ൌ ͳ

ʹܿܽ •‹ ൌ ͳ

ʹܾܽ •‹ ݎ௔ሺ݌ െ ܽሻ ൌ ݎ௕ሺ݌ െ ܾሻ ൌ ݎ௖ሺ݌ െ ܿሻ

ඥ݌ሺ݌ െ ܽሻሺ݌ െ ܾሻሺ݌ െ ܿሻ ܾܽܿ

Ͷܴ ݌ݎ

ʹ ൰ ൌ …‘• ʹǢ …‘• ൬

ʹ

ʹǢ–ƒሺ ൅ ሻ ൌ െ –ƒ Ǣ…‘–ሺ ൅ ሻ ൌ െ …‘– Ǣ –ƒ ൬ ൅

ʹ ൰ ൌ …‘–

ʹǢ…‘– ൬ ൅

ʹ ൰ ൌ –ƒ ʹ

x Sử dụng định lý hàm số sin, hàm sốcos : Ta thường dùng định lý để biến đổi hệ thức phải chứng minh thành hệ thức có hàm sốlượng giác dùng công thức biến đổi lượng giác để chứng minh

x Sử dụng cơng thức tính diện tích : dùng để tìm mối quan hệ cạnh,

góc, bán kính đường trịn ngoại tiếp, nội tiếp, bàng tiếp

Lưu ý: Công thức ܵ ൌ ඥ݌ሺ݌ െ ܽሻሺ݌ െ ܾሻሺ݌ െ ܿሻđược nhà toán học vật lý Heron(5) phát nên thường gọi “Cơng thức Heron”

III. CÁC LOẠI TỐN VÀ PHƯƠNG PHÁP GIẢI

1. CHỨNG MINH ĐẲNG THỨC LƯỢNG GIÁC TRONG TAM GIÁC - Để chứng minh loại tốn này, có nhiều phương pháp giải khác nhau,

chẳng hạn : biến đổi vế thành vế kia, xuất phát từ hệ thức

biết đểsuy đẳng thức cần chứng minh, chứng minh tương đương…

- Trong lúc chứng minh, ta ý số kỹ thuật sau :

x Sử dụng biến đổi lượng giác : sử dụng cơng thức biến đổi tích thành tổng ngược lại, cơng thức hạ bậc, cơng thức cung có liên quan đặc biệt :

•‹ሺ ൅ ሻ ൌ •‹ Ǣ…‘•ሺ ൅ ሻ ൌ െ …‘• Ǣ•‹ ൬ ൅

(62)

Trước hết, ta nên nhớ sốđẳng thức tam giác nhằm giúp cho sử dụng thành thạo kỹ thuật chứng minh dạng toán này, đồng thời làm tăng “độ nhạy” gặp toán phức tạp khác

Giải:

a Ta có :

ൌ ʹ •‹ …‘• െ ʹ ൅ ʹ •‹ ʹ…‘• ʹൌ ʹ …‘• ʹ൬…‘• െ ʹ ൅ •‹ ʹ൰ ൌ ʹ …‘• ʹ൬…‘• െ ʹ ൅ …‘• ൅ ʹ ൰ ൌ Ͷ …‘• ʹ…‘• ʹ…‘• ʹ

b Ta có :

ൌ ʹ …‘• ൅ ʹ …‘• െ ʹ ൅ ͳ െ ʹ •‹ଶ ʹ ൌ ͳ ൅ ʹ •‹ ʹ൬…‘• െ ʹ െ •‹ ʹ൰ ൌ ͳ ൅ ʹ •‹ ʹ൬…‘• െ ʹ െ …‘• ൅ ʹ ൰ ൌ ͳ ൅ Ͷ •‹ ʹ•‹ ʹ•‹ ʹ

c Ta có :

ൌ ʹ •‹ሺ ൅ ሻ …‘•ሺ െ ሻ ൅ ʹ •‹ …‘• ൌ ʹ •‹ ሾ…‘•ሺ െ ሻ ൅ …‘• ሿ ൌ ʹ •‹ ሾ…‘•ሺ െ ሻ െ …‘•ሺ ൅ ሻሿ ൌ Ͷ •‹ •‹ •‹ ƒǤ •‹ ൅ •‹ ൅ •‹ ൌ Ͷ …‘• ʹ…‘• ʹ…‘• ʹ „Ǥ …‘• ൅ …‘• ൅ …‘• ൌ ͳ ൅ Ͷ •‹ ʹ•‹ ʹ•‹ ʹ ‰Ǥ •‹ଶ ʹ൅ •‹ଶ ʹ൅ •‹ଶ ʹൌ ͳ െ ʹ •‹ ʹ•‹ ʹ ʹ ŠǤ …‘•ଶ ʹ൅ …‘•ଶ ʹ൅ …‘•ଶ ʹൌ ʹ ൅ ʹ •‹ ʹ•‹ ʹ ʹ ŒǤ –ƒ ʹ–ƒ ʹ൅ –ƒ ʹ–ƒ ʹ൅ –ƒ ʹ ൌ ͳ

Bài 1: Chứng minh đẳng thức tam giác :

൅ ʹ …Ǥ •‹ ʹ ൅ •‹ ʹ ൅ •‹ ʹ ൌ Ͷ •‹ •‹ •‹ †Ǥ …‘• ʹ ൅ …‘• ʹ ൅ …‘• ʹ ൌ െͳ െ Ͷ …‘• …‘• …‘• ‡Ǥ •‹ଶ ൅ •‹ଶ ൅ •‹ଶ ൌ ʹ ൅ ʹ …‘• …‘• …‘• ˆǤ …‘•ଶ ൅ …‘•ଶ ൅ …‘•ଶ ൌ ͳ െ ʹ …‘• …‘• …‘• •‹ •‹ –ƒ ʹ

(63)

d Ta có :

ൌ ʹ …‘•ሺ ൅ ሻ …‘•ሺ െ ሻ െ ͳ ൅ ʹ …‘•ଶ ൌ െʹ …‘• ሾ…‘•ሺ െ ሻ െ …‘• ሿ െ ͳ

ൌ െʹ …‘• ሾ…‘•ሺ െ ሻ ൅ …‘•ሺ ൅ ሻሿ െ ͳ ൌ െͳ െ Ͷ …‘• …‘• …‘•

e Ta có :

ൌ ͳ െ …‘• ʹ ʹ ൅ ͳ െ …‘• ʹ ʹ ൅ ͳ െ …‘•ଶ ൌ ʹ െ ͳ ʹሺ…‘• ʹ ൅ …‘• ʹሻ െ …‘•ଶ ൌ ʹ െ …‘•ሺ ൅ ሻ …‘•ሺ െ ሻ െ …‘•ଶ ൌ ʹ ൅ …‘• ሾ…‘•ሺ െ ሻ െ …‘• ሿ ൌ ʹ ൅ …‘• ሾ…‘•ሺ െ ሻ ൅ …‘•ሺ ൅ ሻሿ ൌ ʹ ൅ ʹ …‘• …‘• …‘•

f Ta có :

ൌ ͳ ൅ …‘• ʹ ʹ ൅ ͳ ൅ …‘• ʹ ʹ ൅ …‘•ଶ ൌ ͳ ൅ ͳ ʹ

g Ta có :

ൌͳ െ …‘• ʹ ൅ ͳ െ …‘• ʹ ൅ •‹ଶ ʹൌ ͳ െ …‘• ʹ …‘• െ ʹ ൅ •‹ଶ ʹ ൌ ͳ െ •‹ ʹ൬…‘• െ ʹ െ •‹ ʹ൰ ൌ ͳ െ •‹ ൬…‘• ʹ െ …‘• ൅ ʹ ൰ ൌ ͳ െ ʹ •‹ ʹ•‹

h Ta có :

ൌ ͳ ൅ …‘• ʹ ൅ ͳ ൅ …‘• ʹ ʹ ൌ ͳ ൅ …‘• ൅ ʹ …‘• െ ʹ ൅ ͳ െ •‹ଶ ʹ െ ʹ െ …‘• ൅ ʹ ൰ ൌ ʹ ൅ ʹ •‹ ʹ•‹ ʹ•‹ ʹ ฺ ൌ െ –ƒ ฺ –ƒ ൅ –ƒ ൅ –ƒ ൌ –ƒ –ƒ –ƒ

j Ta có :

ʹ൅ ʹ ൌ ߨ ʹെ ʹ ฺ –ƒ ൬ ʹ൅ ʹ൰ ൌ –ƒ ൬ ߨ ʹെ ʹ൰ ฺ –ƒʹ ൅ –ƒʹ ͳ െ –ƒʹ –ƒʹ ൌ …‘– ʹൌ ͳ –ƒʹ ฺ –ƒ ʹ–ƒ ʹ൅ –ƒ ʹ–ƒ ʹ൅ –ƒ ʹ–ƒ ʹ ൌ ͳ ሺ…‘• ʹ ൅ …‘• ʹሻ ൅ …‘•ଶ ൌ ͳ ൅ …‘•ሺ ൅ ሻ …‘•ሺ െ ሻ ൅ …‘•ଶ ൌ ͳ െ …‘• ሾ…‘•ሺ െ ሻ െ …‘• ሿ ൌ ͳ െ …‘• ሾ…‘•ሺ െ ሻ ൅ …‘•ሺ ൅ ሻሿ ൌ ͳ െ ʹ …‘• …‘• …‘• ൅ ʹ െ •‹ ʹ ʹ ൅ …‘•ଶ ൌ ʹ ൅ •‹ ʹ൬…‘•

i Ta có : ൅ ൅ ൌ ߨ ฺ ൅ ൌ ߨ െ ฺ –ƒሺ ൅ ሻ ൌ –ƒሺߨ െ ሻ ൌ െ –ƒ

(64)

Giải: Ta có cách chứng minh tốn

Cách 1: Ta có :

•‹ •‹ …‘• ൅ •‹ •‹ …‘• ൌ •‹ ሺ•‹ …‘• ൅ •‹ …‘• ሻ ൌ •‹ •‹ሺ ൅ ሻ ൌ •‹ଶ

Tương tự :

൝ܽ

ଶ ൌ ܾଶ൅ ܿଶെ ʹܾܿ …‘•

ܾଶ ൌ ܿଶ൅ ܽଶ െ ʹܿܽ …‘•

Giải: Ta có : ݊ ൅ ݊ ൅ ݊ ൌ ݊ߨ ฺ ݊ ൅ ݊ ൌ ݊ߨ െ ݊ ฺ –ƒሺ݊ ൅ ݊ሻ ൌ –ƒሺ݊ߨ െ ݊ሻ ൌ െ –ƒ ݊

ฺ –ƒ ݊ ൅ –ƒ ݊

ͳ െ –ƒ ݊ –ƒ ݊ ൌ െ –ƒ ݊

ฺ –ƒ ݊ ൅ –ƒ ݊ ൅ –ƒ ݊ ൌ –ƒ ݊ –ƒ ݊ –ƒ ݊

–ƒ ݊ ൅ –ƒ ݊ ൅ –ƒ ݊ ൌ –ƒ ݊ –ƒ ݊ –ƒ ݊ǡ ݊ א Գ

Bài 3: Trong tam giác , chứng minh đẳng thức

(ĐH Y Hải Phòng 1998)

•‹ଶ ൅ •‹ଶ ൅ •‹ଶ ൌ ʹሺ•‹ •‹ …‘• ൅ •‹ •‹ …‘• ൅ •‹ •‹ …‘• ሻ

Bài 2: Chứng minh tam giác , ta ln có

(ĐH Giao Thông Vận Tải 1995)

•‹ •‹ …‘• ൅ •‹ •‹ …‘• ൌ •‹ଶ

•‹ •‹ …‘• ൅ •‹ •‹ …‘• ൌ •‹ଶ

Cộng đẳng thức trên, ta có điều phải chứng minh

Cách 2:Theo định lý hàm số cos, ta có :

ฺ ܽଶ൅ ܾଶ൅ ܿଶ ൌ ʹሺܾܽ …‘• ൅ ܾܿ …‘• ൅ ܿܽ …‘• ሻ

൝ܽ ൌ ʹܴ •‹ ܾ ൌ ʹܴ •‹ ܿ ൌ ʹܴ •‹ ܿଶ ൌ ܽଶ൅ ܾଶെ ʹܾܽ …‘•

Theo định lý hàm số sin, ta có :

Suy :

Ͷܴଶሺ•‹ଶ ൅ •‹ଶ ൅ •‹ଶሻ

ൌ ͺܴଶሺ•‹ •‹ …‘• ൅ •‹ •‹ …‘• ൅ •‹ •‹ …‘• ሻ

ฺ •‹ଶ ൅ •‹ଶ ൅ •‹ଶ ൌ ʹሺ•‹ •‹ …‘• ൅ •‹ •‹ …‘• ൅ •‹ •‹ …‘• ሻ

(65)

Giải:

ʹ൅ …‘– ʹ…‘–

ʹ ൌ …‘– ʹ…‘–

ʹ…‘–

ʹ

Mặt khác, ta lại có :

–ƒ ʹ൅ …‘–

ʹ ൌ

ʹ

•‹ Ǣ–ƒ ʹ൅ …‘–

ʹ ൌ

ʹ

•‹ Ǣ–ƒ ʹ൅ …‘–

ʹൌ

ʹ •‹

Cộng đẳng thức thêm hệ thức sẵn có, ta có điều phải chứng minh b Ta có :

ൌ•‹

ʹ …‘•ʹ …‘•ʹ ൅ •‹ʹ …‘•ʹ …‘• ʹ ൅ •‹ʹ …‘•ʹ …‘•ʹ …‘•ʹ …‘•ʹ …‘•ʹ

ƒǤ ͳ •‹ ൅

ͳ •‹ ൅

ͳ •‹ ൌ

ͳ ʹ൬–ƒ

ʹ൅ –ƒ

ʹ൅ –ƒ

ʹ൅ …‘–

ʹ…‘–

ʹ…‘–

ʹ൰ „Ǥ –ƒ

ʹ൅ –ƒ

ʹ൅ –ƒ ʹൌ

͵ ൅ …‘• ൅ …‘• ൅ …‘• •‹ ൅ •‹ ൅ •‹ …Ǥ•‹ሺ െ ሻ

•‹ ൌ

ܽଶെ ܾଶ

ܿଶ

†Ǥ •‹ ʹ …‘•ʹ …‘•ʹ൅

•‹ʹ …‘•ʹ …‘•ʹ൅ ‡Ǥ …‘– ൅ …‘– ൅ …‘– ൌ

Bài 4: Chứng minh tam giác ta ln có

(ĐH Ngoại Thương Hà Nội 1998)

(ĐH Ngoại Thương Tp.HCM 2001)

(ĐHQG Hà Nội 1998)

(ĐH Dược Hà Nội 1998) a Trong tam giác , ta ln có :

…‘–

•‹ ʹ

…‘•ʹ …‘• ʹ ൌ ʹ ܽଶ൅ ܾଶ൅ ܿଶ

Ͷܵ

(66)

Mặt khác : •‹ ʹ…‘• ʹ…‘• ʹൌ ͳ ʹ•‹ ʹ൬…‘• ൅ ʹ ൅ …‘• െ ʹ ൰ ൌ ͳ ʹ•‹ ʹ൬•‹ ʹ൅ …‘• െ ʹ ൰ ൌͳ ʹ൬•‹ଶ ʹ൅ …‘• ൅ ʹ …‘• െ ʹ ൰ ൌ ͳ ʹ൤ ͳ െ …‘• ʹ ൅ ͳ ʹሺ…‘• ൅ …‘• ሻ൨ ൌͳ Ͷሺͳ െ …‘• ൅ …‘• ൅ …‘• ሻ

Tương tự, ta có :

•‹ ʹ…‘• ʹ…‘• ʹ ൌ ͳ Ͷሺͳ െ …‘• ൅ …‘• ൅ …‘• ሻ •‹ ʹ…‘• ʹ…‘• ʹ ൌ ͳ Ͷሺͳ െ …‘• ൅ …‘• ൅ …‘• ሻ Suy •‹ ʹ…‘• ʹ…‘• ʹ൅ •‹ ʹ…‘• ʹ…‘• ʹ൅ •‹ ʹ…‘• ʹ…‘• ʹ ൌ ͳ

Ta xét :

…‘• ʹ…‘• ʹ…‘• ʹൌ ͳ ʹ…‘• ʹ൬•‹ ʹ൅ …‘• െ ʹ ൰ ൌ ͳ ʹ …‘• ʹ൅ •‹ ൅ ʹ …‘• െ ʹ ൰ ൌͳ ʹ൤ ͳ ʹ•‹ ൅ ͳ ʹ ሺ•‹ ൅ •‹ ൅ •‹ ሻ ൌ ൌ ܽ ʹܴ Ǥܿ ଶ൅ ܽଶെ ܾଶ ʹܽܿ െʹܴ Ǥܾ ܾ ଶ൅ ܿଶെ ܽଶ ʹܾܿ ܿ ʹܴ ൌ ൌܽ ଶെ ܾଶ ܿଶ

d Ta có :

•‹ʹ …‘•ʹ …‘•ʹ ൌ …‘• ൅ ʹ …‘•ʹ …‘•ʹ ൌ …‘•ʹ …‘•ʹ െ •‹ʹ •‹ʹ …‘•ʹ …‘•ʹ ൌ ͳ െ –ƒ ʹ–ƒ ʹ

Tương tự, ta có :

•‹ʹ …‘•ʹ …‘•ʹ ൌ ͳ െ –ƒ ʹ–ƒ ʹ •‹ʹ …‘•ʹ …‘•ʹ ൌ ͳ െ –ƒ ʹ–ƒ ʹ Ͷሺ͵ ൅ …‘• ൅ …‘• ൅ …‘• ሻ ൬•‹ ʹ ሺ•‹ ൅ •‹ ሻ൨ ൌͳ Ͷ

Vậy ta đãcó điều phải chứng minh c Ta có :

•‹ …‘• െ •‹ …‘• •‹

ሺܿଶ൅ ܽଶെ ܾଶሻ െ ሺܾଶ൅ ܿଶെ ܽଶሻ

(67)

Cộng đẳng thức lại, ta có : ൌ ͵ െ ൬–ƒ ʹ–ƒ ʹ൅ –ƒ ʹ–ƒ ʹ൅ –ƒ ʹ–ƒ ʹ൰ Mà –ƒ ʹ–ƒ ʹ൅ –ƒ ʹ–ƒ ʹ൅ –ƒ ʹ–ƒ ʹ ൌ ͳ

Nên ൌ ʹ

e Theo định lý cos, ta có :

ܽଶ ൌ ܾଶ൅ ܿଶെ ʹܾܿ …‘• ൌ ܾଶ൅ ܿଶെ ʹܾܿ •‹ …‘– ൌ ܾଶ൅ ܿଶെ Ͷܵ …‘–

Tương tự, ta có :

ܾଶ ൌ ܿଶ൅ ܽଶ െ Ͷܵ …‘–

ܿଶ ൌ ܽଶ൅ ܾଶെ Ͷܵ …‘–

Cộng đẳng thức ta :

ฺ …‘– ൅ …‘– ൅ …‘– ൌ

Giải:

a Ta có :

•‹ ൅ •‹ ൅ •‹ ൌ Ͷ …‘• ʹ…‘• ʹ…‘• ʹ ƒǤ•‹ ൅ •‹ ൅ •‹ •‹ ൅ •‹ െ •‹ ʹ „Ǥ •‹ ʹ…‘• ʹ…‘• ʹ …‘• ʹ൅ •‹ ʹ…‘• ʹ…‘• ʹ •‹ ʹ൅ –ƒ ʹ–ƒ ʹ൅ –ƒ ʹ–ƒ ʹ൅ –ƒ ʹ–ƒ ʹ …Ǥ •‹ ൅ •‹ െ •‹ …‘• ൅ …‘• െ …‘• ൅ ͳ ൌ –ƒ ʹ–ƒ ʹ…‘– ʹ

(Học Viện Quan Hệ Quốc Tế 2000)

(Học Viện Ngân Hàng 2000)

ܽଶ൅ ܾଶ൅ ܿଶ ൌ ʹሺܽଶ൅ ܾଶ൅ ܿଶሻ െ Ͷܵሺ…‘– ൅ …‘– ൅ …‘– ሻ

ܽଶ൅ ܾଶ൅ ܿଶ

Ͷܵ

Vậy ta có điều phải chứng minh

ൌ …‘– …‘– ʹ ൅ •‹ ʹ…‘• ʹ ൌ •‹ ʹ•‹ ʹ

Bài 5: Chứng minh tam giác ta ln có

(68)

Mặt khác : •‹ ൅ •‹ െ •‹ ൌ ʹ •‹ ൅ ʹ …‘• െ ʹ െ ʹ •‹ ʹ…‘• ʹ ൌ ʹ …‘• ʹ൬…‘• െ ʹ െ …‘• ൅ ʹ ൰ ൌ െͶ …‘• ʹ•‹ ʹ•‹ ൬െ ʹ൰ ൌ Ͷ …‘• ʹ•‹ ʹ•‹ ʹ

Vậy

ൌ Ͷ …‘• ʹ …‘•ʹ …‘•ʹ Ͷ …‘•ʹ •‹ʹ •‹ʹ ൌ …‘– ʹ…‘– ʹ

b Ta có :

–ƒ ʹ–ƒ ʹ൅ –ƒ ʹ–ƒ ʹ൅ –ƒ ʹ

Do đó, điều cần chứng minh tương đương với :

•‹ ʹ…‘• ʹ…‘• ʹ൅ •‹ ʹ…‘• ʹ…‘• ʹ൅ •‹ ʹ…‘• ʹ ʹ ൌ •‹ ʹ•‹ ʹ•‹ ʹ൅ ͳ ฻ •‹ ʹ൬…‘• ʹ…‘• ʹെ •‹ ʹ•‹ ʹ൰ ൅ …‘• ʹ ൅ •‹ ʹ…‘• ʹ൰ ൌ ͳ ฻ •‹ ʹ…‘• ൅ ʹ ൅ …‘• ʹ•‹ ฻ •‹ ൬ ൅ ൅ ൰ ൌ ͳ ʹ•‹ ʹ•‹ ʹ

Ta xét :

…‘• ൅ …‘• െ …‘• ൅ ͳ ൌ ʹ …‘• ൅ ʹ …‘• െ ʹ ൅ ʹ •‹ଶ ʹ ൌ ʹ •‹ ʹ൬…‘• െ ʹ ൅ …‘• ൅ ʹ ൰ ൌ Ͷ …‘• ʹ…‘• ʹ•‹ ʹ

Do đó,

ൌ Ͷ …‘• ʹ •‹ʹ •‹ʹ Ͷ …‘•ʹ …‘•ʹ •‹ʹൌ –ƒ ʹ–ƒ ʹ…‘– ʹ –ƒ ʹ ൌ ͳ …‘• ൬•‹ ʹ…‘• ʹ ൅ ʹ ൌ ͳ ʹ

Điều hiển nhiên đúng, ta có điều phải chứng minh c Ởcâu a, ta chứng minh :

(69)

Giải:

a Ta có :

•‹ଷ •‹ሺ െ ሻ ൌ •‹ଶ •‹ሺ ൅ ሻ •‹ሺ െ ሻ ൌ െͳ

Ͷ ൌͳ

Ͷ

Tương tự, ta có :

•‹ଷ •‹ሺ െ ሻ ൌ ͳ

Ͷ •‹ଷ •‹ሺ െ ሻ ൌ ͳ

ൌ ൌʹሺ•‹

ଶ ൅ •‹ଶ ൅ •‹ଶሻ

•‹ ʹ ൅ •‹ ʹ ൅ •‹ ʹ ൌ•‹

ଶ ൅ •‹ଶ ൅ •‹ଶ

ʹ •‹ •‹ •‹ ൌ

•‹ ʹ •‹ •‹ ൅

•‹ ʹ •‹ •‹ ൅

•‹ ʹ •‹ •‹ ൌ •‹ሺ ൅ ሻ

ʹ •‹ •‹ ൅

•‹ሺ ൅ ሻ ʹ •‹ •‹ ൅

•‹ሺ ൅ ሻ ʹ •‹ •‹ ൌͳ

ʹሺ…‘– ൅ …‘– ሻ ൅ ͳ

ʹሺ…‘– ൅ …‘– ሻ ൅ ͳ

ʹሺ…‘– ൅ …‘– ሻ ൌ …‘– ൅ …‘– ൅ …‘–

c Ta có :

ൌ ܽ െ ܾ –ƒ ൅

ܾ െ ܿ –ƒ ൅

ܿ െ ܽ –ƒ ƒǤܽଷ•‹ሺ െ ሻ ൅ ܾଷ•‹ሺ െ ሻ ൅ ܿଷ•‹ሺ െ ሻ ൌ Ͳ

„Ǥ ܽ •‹ ൅ ܾ •‹ ൅ ܿ •‹

ܽ …‘• ൅ ܾ …‘• ൅ ܿ …‘• ൌ …‘– ൅ …‘– ൅ …‘– …Ǥሺܽ െ ܾሻ –ƒ

ʹ–ƒ

ʹ൅ ሺܾ െ ܿሻ –ƒ ʹ–ƒ

ʹ൅ ሺܿ െ ܽሻ –ƒ ʹ–ƒ

ʹ ൌ Ͳ

Bài 6: Cho tam giác Chứng minh :

ሺͳ െ …‘• ʹሻሺ…‘• ʹ െ …‘• ʹሻ ሺ…‘• ʹ െ …‘• ʹ ൅ …‘• ʹ …‘• ʹ െ …‘• ʹ …‘• ʹሻ

ሺ…‘• ʹ െ …‘• ʹ ൅ …‘• ʹ …‘• ʹ െ …‘• ʹ …‘• ʹሻ ሺ…‘• ʹ െ …‘• ʹ ൅ …‘• ʹ …‘• ʹ െ …‘• ʹ …‘• ʹሻ Ͷ

Cộng đẳng thức trên, ta :

•‹ଷ •‹ሺ െ ሻ ൅ •‹ଷ •‹ሺ െ ሻ ൅ •‹ଷ •‹ሺ െ ሻ ൌ Ͳ

Vậy theo định lý hàm sốsin, ta có điều phải chứng minh b Ta có :

•‹ ʹ ൅ •‹ ʹ ൅ •‹ ʹ ൌ Ͷ •‹ •‹ •‹

Do đó, theo định lý hàm số sin, ta có :

ʹܴሺ•‹ଶ ൅ •‹ଶ ൅ •‹ଶሻ

(70)

Mặt khác, ta có :

ܽ ൌ ݌ െ ܾ ൅ ݌ െ ܿ ൌ ݎ ൬…‘– ʹ൅ …‘–

ʹ൰Ǣ ܾ ൌ ݎ ൬…‘– ʹ൅ …‘–

ʹ൰ ฺ ܽ െ ܾ ൌ ݎ ൬…‘–

ʹെ …‘–

ʹ൰ ฺ ሺܽ െ ܾሻ …‘–

ʹ ൌ ݎ …‘– ʹ൬…‘–

ʹ െ …‘–

ʹ൰

Tương tự :

ሺܾ െ ܿሻ …‘–

ʹ ൌ ݎ …‘– ʹ൬…‘–

ʹെ …‘–

ʹ൰ ሺܿ െ ܽሻ …‘–

ʹ ൌ ݎ …‘– ʹ൬…‘–

ʹെ …‘–

ʹ൰

Cộng đẳng thức trên, ta có điều phải chứng minh

Giải:

a Ta có :

ൌ ʹ •‹ሺʹ݊ ൅ ͳሻ ൅ ሺʹ݊ ൅ ͳሻ ʹ …‘•

ሺʹ݊ ൅ ͳሻ െ ሺʹ݊ ൅ ͳሻ ʹ

൅ ʹ •‹ሺʹ݊ ൅ ͳሻ

ʹ …‘•ሺʹ݊ ൅ ͳሻ ʹ ൌ ʹ •‹ሺʹ݊ ൅ ͳሻ ൅

ʹ …‘•ሺʹ݊ ൅ ͳሻ

ʹ ൅ ʹ •‹ሺʹ݊ ൅ ͳሻ

ʹ …‘•ሺʹ݊ ൅ ͳሻ ʹ

Ta xét :

൅ ʹ ൌ

ߨ ʹെ

ʹฺ ሺʹ݊ ൅ ͳሻ

ʹ ൌ ሺʹ݊ ൅ ͳሻ ൬ ߨ ʹെ

ʹ൰ ൌ ሺെͳሻ௡Ͷ …‘•ሺʹ݊ ൅ ͳሻ

ʹ…‘•ሺʹ݊ ൅ ͳሻ •‹ሺʹ݊ ൅ ͳሻ

ʹ•‹ሺʹ݊ ൅ ͳሻ ʹ ƒǤ •‹ሺʹ݊ ൅ ͳሻ ൅ •‹ሺʹ݊ ൅ ͳሻ ൅ •‹ሺʹ݊ ൅ ͳሻ

ʹ…‘•ሺʹ݊ ൅ ͳሻ ʹ „Ǥ •‹ ʹ݊ ൅ •‹ ʹ݊ ൅ •‹ ʹ݊ ൌ ሺെͳሻ௡ାଵͶ •‹ ݊ •‹ ݊ •‹ ݊

…Ǥ •‹ଶ݊ ൅ •‹ଶ݊ ൅ •‹ଶ݊ ൌ ʹ ൅ ሺെͳሻ௡ାଵʹ …‘• ݊ …‘• ݊ …‘• ݊

†Ǥ …‘•ሺʹ݊ ൅ ͳሻ ൅ …‘•ሺʹ݊ ൅ ͳሻ ൅ …‘•ሺʹ݊ ൅ ͳሻ ൌ ͳ ൅ ሺെͳሻ௡Ͷ •‹ሺʹ݊ ൅ ͳሻ

ʹ

‡Ǥ …‘• ʹ݊ ൅ …‘• ʹ݊ ൅ …‘• ʹ݊ ൌ െͳ ൅ ሺെͳሻ௡Ͷ …‘• ݊ …‘• ݊ …‘• ݊

ˆǤ …‘•ଶ݊ ൅ …‘•ଶ݊ ൅ …‘•ଶ݊ ൌ ͳ ൅ ሺെͳሻ௡ʹ …‘• ݊ …‘• ݊ …‘• ݊

(71)

ฺ •‹ሺʹ݊ ൅ ͳሻ ൅

ʹ ൌ •‹ሺʹ݊ ൅ ͳሻ ൬ ߨ ʹെ

ʹ൰ ൌ •‹ ൬݊ߨ ൅ ߨ

ʹെ ሺʹ݊ ൅ ͳሻ ʹ൰ ൌ ሺെͳሻ௡…‘•ሺʹ݊ ൅ ͳሻ

ʹ

Tương tự vậy, ta có :

ሺʹ݊ ൅ ͳሻ

ʹ ൌ ሺʹ݊ ൅ ͳሻ ൬ ߨ ʹെ

൅ ʹ ൰ ฺ •‹ሺʹ݊ ൅ ͳሻ

ʹ ൌ •‹ ൬݊ߨ ൅ ߨ

ʹെ ሺʹ݊ ൅ ͳሻ

ʹ ൰ ൌ ሺെͳሻ௡…‘•ሺʹ݊ ൅ ͳሻ ʹ

Suy

ൌ ʹǤ ሺെͳሻ௡…‘•ሺʹ݊ ൅ ͳሻ

ʹ ൤…‘•ሺʹ݊ ൅ ͳሻ ൬ െ

ʹ

൅ ʹ ൰൨ ൌ ሺെͳሻ௡Ͷ …‘•ሺʹ݊ ൅ ͳሻ

ʹ…‘•ሺʹ݊ ൅ ͳሻ

ʹ

b Ta có : Ta thấy :

c Ta có :

൅ͳ െ …‘• ʹ݊

ʹ ൅ ͳ െ …‘•ଶ݊ ൌ ʹ െͳ

ʹሺ…‘• ʹ݊ ൅ …‘• ʹ݊ሻ െ …‘•ଶ݊ ൌ ʹ െ …‘• ݊ሺ ൅ ሻ …‘• ݊ሺ െ ሻ െ …‘•ଶ݊

ൌ ʹ െ ሺെͳሻ௡…‘• ݊ ሾ…‘• ݊ሺ െ ሻ ൅ …‘• ݊ሺ ൅ ሻሿ

ൌ ʹ ൅ ሺെͳሻ௡ାଵʹ …‘• ݊ …‘• ݊ …‘• ݊

d Ta có :

ൌ ʹ …‘•ሺʹ݊ ൅ ͳሻ ൅

ʹ …‘•ሺʹ݊ ൅ ͳሻ

ʹ ൅ ͳ െ ʹ •‹ଶሺʹ݊ ൅ ͳሻ ʹ

ሺʹ݊ ൅ ͳሻ ൅ ൌ ሺʹ݊ ൅ ͳሻ ൬ߨെ൰

൰ ൅ …‘•ሺʹ݊ ൅ ͳሻ ൬

…‘•ሺʹ݊ ൅ ͳሻ ʹ

ൌ ʹ •‹ ݊ሺ ൅ ሻ …‘• ݊ሺ െ ሻ ൅ ʹ •‹ ݊ …‘• ݊ ൅ ൌ ߨ െ ฺ ݊ሺ ൅ ሻ ൌ ݊ߨ െ ݊ ฺ •‹ ݊ሺ ൅ ሻ ൌ •‹ሺ݊ߨ െ ݊ሻ ൌ ሺെͳሻ௡ାଵ•‹ ݊

݊ ൌ ݊ߨ െ ݊ሺ ൅ ሻ ฺ …‘• ݊ ൌ …‘•൫݊ߨ െ ݊ሺ ൅ ሻ൯ ൌ ሺെͳሻ௡…‘• ݊ሺ ൅ ሻ

Suy

ൌ ʹሺെͳሻ௡ାଵ•‹ ݊ …‘• ݊ሺ െ ሻ ൅ ʹሺെͳሻ௡…‘• ݊ሺ ൅ ሻ •‹ ݊

ൌ ʹሺെͳሻ௡•‹ ݊ ሾ…‘• ݊ሺ ൅ ሻ െ …‘• ݊ሺ െ ሻሿ

ൌ ሺെͳሻ௡ାଵͶ •‹ ݊ •‹ ݊ •‹ ݊

(72)

ฺ …‘•ሺʹ݊ ൅ ͳሻ ൅

ʹ ൌ …‘•ሺʹ݊ ൅ ͳሻ ൬ ߨ ʹെ

ʹ൰ ൌ …‘• ൬݊ߨ ൅ ߨ

ʹെ ሺʹ݊ ൅ ͳሻ ʹ൰ ൌ െͳǤ ሺെͳሻ௡ିଵ•‹ሺʹ݊ ൅ ͳሻ

ʹൌ ሺെͳሻ௡•‹ሺʹ݊ ൅ ͳሻ ʹ

ሺʹ݊ ൅ ͳሻ

ʹ ൌ ሺʹ݊ ൅ ͳሻ ൬ ߨ ʹെ

൅ ʹ ൰ ฺ •‹ሺʹ݊ ൅ ͳሻ

ʹൌ •‹ሺʹ݊ ൅ ͳሻ ൬ ߨ ʹെ

ʹ ൰ ൌ •‹ ൬݊ߨ ൅ ߨ

ʹെ ሺʹ݊ ൅ ͳሻ

൅ ʹ ൰ ൌ ሺെͳሻ௡…‘•ሺʹ݊ ൅ ͳሻ ൅

ʹ

Suy

ൌ ͳ ൅ ʹሺെͳሻ௡•‹ሺʹ݊ ൅ ͳሻ

ʹ൤…‘•ሺʹ݊ ൅ ͳሻ

െ ʹ

൅ ʹ ൨ ൌ ͳ ൅ ሺെͳሻ௡Ͷ •‹ሺʹ݊ ൅ ͳሻ

ʹ•‹ሺʹ݊ ൅ ͳሻ

ʹ

f Ta có :

ൌ ͳ ൅ …‘• ʹ݊

ʹ ൅ ൅ …‘•ଶ݊ ൌ ͳ ൅ ͳ

ʹሺ…‘• ʹ݊ ൅ …‘• ʹ݊ሻ ൅ …‘•ଶ݊

•‹ ൅ •‹ ൅ •‹ ൌ Ͷ •‹ ݔ •‹ ݕ •‹ ݖ

Bài 8: Gọi tâm đường tròn nội tiếp tam giác Đặt ݔ ൌ ෢ ǡ ݕ ൌ ෢ ǡ ݖ ൌ ෢ Chứng minh

െ …‘•ሺʹ݊ ൅ ͳሻ

ʹ•‹ሺʹ݊ ൅ ͳሻ

e Ta có :

ൌ ʹ …‘• ݊ሺ ൅ ሻ …‘• ݊ሺ െ ሻ ൅ ʹ …‘•ଶ݊ െ ͳ

ൌ ʹ …‘•ሺ݊ߨ െ ݊ሻ …‘• ݊ሺ െ ሻ ൅ ʹ …‘•ଶ݊ െ ͳ

ൌ െͳ ൅ ʹሺെͳሻ௡…‘• ݊ ሾ…‘• ݊ሺ െ ሻ ൅ …‘• ݊ሺ ൅ ሻሿ

ൌ െͳ ൅ ሺെͳሻ௡Ͷ …‘• ݊ …‘• ݊ …‘• ݊

ͳ ൅ …‘• ʹ݊ ʹ

ൌ ͳ ൅ …‘• ݊ሺ ൅ ሻ …‘• ݊ሺ െ ሻ ൅ …‘•ଶ݊

ൌ ͳ ൅ …‘•ሺ݊ߨ െ ݊ሻ …‘• ݊ሺ െ ሻ ൅ …‘•ଶ݊

ൌ ͳ ൅ ሺെͳሻ௡…‘• ݊ …‘• ݊ሺ െ ሻ ൅ ሺെͳሻ௡…‘• ݊ …‘• ݊ሺ ൅ ሻ

ൌ ͳ ൅ ሺെͳሻ௡…‘• ݊ ሾ…‘• ݊ሺ െ ሻ ൅ …‘• ݊ሺ ൅ ሻሿ

(73)

Giải:

Ta có :

ݔ ൌ ͳͺͲ୭െ ൅

ʹ ൌ ͻͲ୭൅

ʹǡ ݕ ൌ ͻͲ୭ ൅

ʹ

Suy

Ͷ •‹ ݔ •‹ ݕ •‹ ݖ ൌ Ͷ •‹ ൬ͻͲ୭൅

ʹ

ʹ൰ ൌ Ͷ …‘• ʹ…‘•

ʹ…‘•

ʹ ൌ •‹ ൅ •‹ ൅ •‹

Giải:

Từ giả thuyết, ta suy

ە ۖ ۔ ۖ

ۓ ൌ ߨ͹ ൌʹߨ

͹ ൌ Ͷߨ

͹

a Ta có :

ͳ •‹ଶ൅

ͳ •‹ଶ൅

ͳ

•‹ଶൌ …‘–ଶ ൅ …‘–ଶʹ ൅ …‘–ଶͶ ൅ ͵

ƒǤ ͳ •‹ଶ൅

ͳ •‹ଶ൅

ͳ ͺ †Ǥͳ

ܽ ൌ ͳ ܾ ൅

ͳ ܿ

ʹǡ ݖ ൌ ͻͲ୭൅ ൰ •‹ ൬ͻͲ୭൅

ʹ൰ •‹ ൬ͻͲ୭൅ ͳ

•‹ଶൌ ͺ „Ǥ …‘• …‘• …‘• ൌ െ

…Ǥ …‘•ଶ ൅ …‘•ଶ ൅ …‘•ଶ ൌͷ

Ͷ

Bài 9: Cho tam giác có góc ǡ ǡ theo thứ tự tạo thành cấp số nhân công bội

(74)

ൌ ሺ…‘–ଶ െ ͳሻ ൅ ሺ…‘–ଶʹ െ ͳሻ ൅ ሺ…‘–ଶͶ െ ͳሻ ൅ ͸

ൌ…‘• ʹ •‹ଶ ൅

…‘• Ͷ •‹ଶʹ൅

…‘• ͺ •‹ଶͶ൅ ͸

ൌʹ …‘• ʹ …‘• ʹ •‹ଶ …‘• ൅

ʹ …‘• Ͷ …‘• ʹ ʹ •‹ଶʹ …‘• ʹ൅

ʹ …‘• ͺ …‘• Ͷ ʹ •‹ଶͶ …‘• Ͷ൅ ͸

ൌ ʹ …‘– ʹ …‘– ൅ ʹ …‘– Ͷ …‘– ʹ ൅ ʹ …‘– ͺ …‘– Ͷ ൅ ͸ ൌ ʹሺ…‘– …‘– ൅ …‘– …‘– ൅ …‘– …‘– ሻ ൅ ͸

(vì …‘– ͺ ൌ …‘–ሺߨ ൅ ሻ ൌ …‘– )

Mặt khác, tam giác ta ln có :

…‘– …‘– ൅ …‘– …‘– ൅ …‘– …‘– ൌ ͳ

Nên ൌ ͺ

Do đó, ta có điều phải chứng minh b Ta có :

…‘• …‘• …‘• ൌ …‘• …‘• ʹ …‘• Ͷ ൌ •‹ ʹ

ʹ •‹ Ǥ Ǥ ൌ

•‹ ͺ ͺ •‹ ൌ െ

ͳ ͺ

(vì •‹ ͺ ൌ •‹ሺߨ ൅ ሻ ൌ െ •‹ ) Vậy ta có điều phải chứng minh c Trong tam giác , ta ln có :

ͳ ͺ൰ ൌ

ͷ Ͷ ൌ ͳ

•‹ʹߨ͹ ൅ ͳ •‹Ͷߨ͹

Ta có :

ͳ

•‹ʹߨ͹ ൅ ͹ ൌ

൅ •‹Ͷߨ͹ •‹ʹߨ͹ •‹Ͷߨ͹ ൌ

ʹ •‹͵ߨ͹ …‘•ߨ͹ •‹ʹߨ͹ •‹͵ߨ͹ ൌ

ʹ …‘•ߨ͹ ʹ •‹ߨ͹ …‘•ߨ͹ ൌ

ͳ •‹ߨ͹

Vậy ta có điều phải chứng minh

•‹ Ͷ ʹ •‹ ʹ

•‹ ͺ ʹ •‹ Ͷ …‘•ଶ ൅ …‘•ଶ ൅ …‘•ଶ ൌ ͳ െ ʹ …‘• …‘• …‘• ൌ ͳ െ ʹ ൬െ

Vậy ta có điều phải chứng minh

d Theo định lý hàm sốsin, điều cần chứng minh tương đương với

ͳ •‹ ͹ߨ ͳ

•‹Ͷߨ

(75)

Giải:

…‘• ʹ…‘•

ʹൌ ͵ •‹ ʹ ฻ ʹ ൬…‘•

ʹ…‘• ʹ

ʹ

ʹ൅ •‹

ʹ•‹

ʹ ฻ ʹ …‘• ൅

ʹ ൌ …‘• ฻ ʹ …‘• ൅

ʹ

െ ʹ …‘•

ʹ ฻ Ͷ •‹

ʹ…‘•

ʹ ൌ ʹ •‹ ൅

ʹ …‘• െ

ʹ ฻ ʹ •‹ ൌ •‹ ൅ •‹

Theo định lý hàm sốsin, ta có điều phải chứng minh

b …‘– ǡ …‘– ǡ …‘– lập thành cấp số cộng ฻ …‘– ൅ …‘– ൌ ʹ …‘– ฻ •‹ሺ ൅ ሻ

•‹ •‹ ൌ

ʹ …‘• •‹

฻ •‹ଶ ൌ ʹ •‹ •‹ …‘•

฻ •‹ଶ ൌ …‘• ሾ…‘•ሺ െ ሻ ൅ …‘• ሿ

฻ •‹ଶ ൌ െ …‘•ሺ ൅ ሻ …‘•ሺ െ ሻ ൅ …‘•ଶ

ʹܾ ൌ ܽ ൅ ܿ ฻ …‘– ʹ…‘–

ʹ ൌ ͵

–ƒ –ƒ ൅ ͳ –ƒ –ƒ െ ͳ

Bài 10:

ƒǤ Cho tam giác , ൌ ܽǡ ൌ ܾǡ ൌ ܿ Chứng minh

a Ta có giả thuyết tương đương với

•‹ ʹ െ •‹ •‹

ʹ൰ ൌ …‘• ʹ…‘• െ

ʹ …‘•

ʹ ൌ …‘•

ൌ –ƒଶ

(ĐH Cần Thơ 1998)

b Chứng minh : tam giác …‘– ǡ …‘– ǡ …‘– theo thứ tự tạo thành cấp số cộng ܽଶǡ ܾଶǡ ܿଶcũng tạo thành cấp số cộng.

(ĐH Thương Mại Hà Nội 2000) c Cho tam giác có ܽଶ൅ ܾଶെ ܿଶ ൌ Ͷܴଶ Chứng minh rằng

(76)

฻ •‹ଶ ൌ ͳ െ •‹ଶ െͳ

ʹሺ…‘• ʹ ൅ …‘• ʹሻ ฻ ʹ •‹ଶ ൌ ͳ െͳ

ʹሺͳ െ ʹ •‹ଶ ൅ ͳ െ ʹ •‹ଶሻ ฻ ʹ •‹ଶ ൌ •‹ଶ ൅ •‹ଶ

Theo định lý hàm sốsin, ta có điều phải chứng minh c Theo định lý hàm số sin, ta suy

ܽଶ

•‹ଶ ൌ

ܾଶ

•‹ଶ ൌ

ܿଶ

•‹ଶ ൌ Ͷܴଶ

Áp dụng tính chất tỷ lệ thức, ta có :

ܽଶ൅ ܾଶെ ܿଶ

•‹ଶ ൅ •‹ଶ െ •‹ଶൌ Ͷܴଶ

ฺ Ͷܴ

•‹ଶ ൅ •‹ଶ െ •‹ଶൌ Ͷܴଶ

ฺ •‹ଶ ൅ •‹ଶ ൌ ͳ ൅ •‹ଶ

฻ െͳሺ…‘• ʹ ൅ …‘• ʹሻ ൌ •‹ଶ

ൌ •‹

…‘• ฻ ൌ –ƒ –ƒ ൌ ฻ ͳ ൅ –ƒ –ƒ

ͳ െ –ƒ –ƒ ൌ

–ƒ ሺ–ƒ ൅ –ƒ ሻ ͳ െ –ƒ –ƒ

(vì •‹ •‹ ് …‘• …‘• ฺ –ƒ –ƒ ് ͳ)

฻ ͳ ൅ –ƒ –ƒ

ͳ െ –ƒ –ƒ ൌ –ƒ –ƒሺ ൅ ሻ ൌ –ƒଶ ƒǤͳ ൅ݎ

ܴ ൌ …‘• ൅ …‘• ൅ …‘•

„Ǥ݉௔ଶ ൅ ݉௕ଶ൅ ݉௖ଶ ൌ ͵ܴଶሺʹ ൅ ʹ …‘• …‘• …‘• ሻ

Bài 11: Cho tam giác có tâm đường trịn nội tiếp Chứng minh đẳng thức sau :

ʹ

฻ …‘• …‘•ሺ െ ሻ ൌ •‹ଶ

Ởđẳng thức ta thấy …‘• ǡ …‘•ሺ െ ሻ ് Ͳ nên

…‘•ሺ െ ሻ …‘• …‘• ൅ •‹ •‹ •‹ሺ ൅ ሻ •‹ …‘• ൅ •‹ …‘•

Giả sử…‘• …‘• ൌ Ͳ …‘•ሺ ൅ ሻ ൅ …‘•ሺ െ ሻ ൌ Ͳ hay …‘•ሺ െ ሻ ൌ …‘• Khi ൌ ͻͲ୭ǡ ൌ ൌ Ͷͷ୭

Mặt khác, …‘• ് Ͳ nên •‹ •‹ ് …‘• …‘• Đến đây, ta có mâu thuẫn Do :

(77)

Giải:

a Ta cần chứng minh :

ݎ

ܴ ൌ Ͷ •‹ ʹ•‹

ʹ•‹

ʹ

Thật vậy, ta có :

ܵ ൌ ͳ

ʹܾܽ •‹ ൌ ݌ݎ ฺ ݎ ൌ

Mà theo định lý hàm sốsin, ta :

ݎ ൌ ܾܽ •‹ ܽ ൅ ܾ ൅ ܿ ൌ

Suy

ݎ ܴ ൌ

Mặt khác, ta lại có :

ʹ…‘•

ʹʹ •‹

ʹ…‘•

ʹʹ •‹

ʹ…‘•

ʹ

ʹ…‘• ʹ…‘•

ʹ

Do đó,

ݎ

ܴ ൌ Ͷ •‹ ʹ•‹

ʹ•‹

ʹ ฺ ͳ ൅ ݎ

ܴ ൌ ͳ ൅ Ͷ •‹ ʹ•‹

ʹ•‹

ʹൌ …‘• ൅ …‘• ൅ …‘•

b Ta có :

ൌʹܾ

ଶ൅ ʹܿଶെ ܽଶ

Ͷ ൅

ʹܽଶ൅ ʹܿଶെ ܾଶ

Ͷ ൅

ʹܽଶ൅ ʹܾଶെ ܿଶ

Ͷ ൌ ͵

Ͷሺܽଶ൅ ܾଶ൅ ܿଶሻ ൌ ͵ܴଶሺ•‹ଶ ൅ •‹ଶ ൅ •‹ଶሻ ൌ ͵ܴଶሺʹ ൅ ʹ …‘• …‘• …‘• ሻ

…Ǥ ൬ͳ ܽ൅

ͳ

ܾ൰ ݈௖൅ ൬ ͳ ܾ൅

ͳ

ܿ൰ ݈௔൅ ൬ ͳ ܿ൅

ͳ

ܽ൰ ݈௕ ൌ ʹ ൬…‘•

ʹ൅ …‘• ʹ൅ …‘•

ʹ൰ †Ǥܽ ൌ ݌ •‹

ʹ …‘•ʹ …‘•ʹ ‡ǤǤ Ǥ ൌ Ͷܴݎଶ

ܾܽ •‹ ܽ ൅ ܾ ൅ ܿ ʹܴ •‹ ʹܴ •‹ •‹ ʹܴሺ•‹ ൅ •‹ ൅ •‹ ሻ Ͷ •‹ •‹ •‹

•‹ ൅ •‹ ൅ •‹ •‹ •‹ •‹ ൌ ʹ •‹

(78)

c Ta có :

ൌ ܽ ൅ ܾ ܾܽ ݈௖൅

ܾ ൅ ܿ ܾܿ ݈௔൅

ܿ ൅ ܽ ܿܽ ݈௕ ൌܽ ൅ ܾ

ܾܽ Ǥ ʹܾܽ ܽ ൅ ܾ…‘•

ʹ൅

ܾ ൅ ܿ ܾܿ Ǥ

ʹܾܿ ܾ ൅ ܿ…‘•

ʹ൅

ܿ ൅ ܽ ܿܽ Ǥ

ʹܿܽ ܿ ൅ ܽ…‘•

ʹ ൌ

d Theo định lý hàm số sin, ta có :

ܽ ݌ ൌ ʹǤ

ʹܴ •‹ ܽ ൅ ܾ ൅ ܿൌ

Ͷܴ •‹

ʹܴሺ•‹ ൅ •‹ ൅ •‹ ሻൌ

ͺܴ •‹ʹ …‘•ʹ ͺܴ …‘•ʹ …‘•ʹ …‘•ʹൌ

•‹ʹ …‘•ʹ …‘•ʹ

Vậy ta có điều phải chứng minh e

•‹ʹ ൌ

ݎ •‹ʹ ݎ

•‹ʹǢ ൌ ݎ •‹ʹ ฺ ൌ ݎ

•‹ʹ •‹ʹ •‹ʹ

Mặt khác, ta lại có :

ݎ

Ͷܴ ൌ •‹ ʹ•‹

ʹ•‹

ʹ

Nên

ൌ ݎ

ݎ ൌ Ͷܴݎଶ

Ta thấy tam giác vuông nên

Tương tự, ta có :

(79)

Giải:

Trước hết ta chứng minh : ʹܴሺ݄௔൅ ݄௕ ൅ ݄௖ െ ʹݎሻ ൌ ܾܽ ൅ ܾܿ ൅ ܿܽ െ Ͷܴݎ

Thật ta có :

ʹܴሺ݄௔൅ ݄௕ ൅ ݄௖ሻ ൌ ʹܴ ൬ʹܵ ܽ ൅

ʹܵ ܾ ൅

ʹܵ ܿ ൰ ൌ •‹ ൌ ʹ ֜ ܽ

ʹܴ ൌ ݌ଶ൅ ݎଶ ൌ ʹܴሺ݄

௔൅ ݄௕ ൅ ݄௖െ ʹݎሻ

Bài 12: Cho tam giác Chứng minh ta ln có :

(Đề nghị Olympic 30-4, 2007)

Ͷܴܵሺܾܽ ൅ ܾܿ ൅ ܿܽሻ

ܾܽܿ ൌ ܽ ൅ ܾ ൅ ܿ ֜ ʹܴሺ݄௔൅ ݄௕ ൅ ݄௖ െ ʹݎሻ ൌ ܾܽ ൅ ܾܿ ൅ ܿܽ െ Ͷܴݎ ֜ ݌ଶ൅ ݎଶ൅ Ͷܴݎ ൌ ܾܽ ൅ ܾܿ ൅ ܿܽ

Lại có :

ʹݎ ʹ –ƒ ݌ െ ܽ ͳ ൅ –ƒଶ

ʹ ͳ ൅ ሺ݌ െݎ

ܽሻଶ

֜ ܽଷ െ ʹ݌ܽଶ൅ ሺ݌ଶ൅ ݎଶ൅ Ͷܴݎሻܽ െ Ͷ݌ܴݎ ൌ Ͳ

Tương tựthì ta có :

ܾଷെ ʹ݌ܾଶ൅ ሺ݌ଶ൅ ݎଶ ൅ Ͷܴݎሻܾ െ Ͷ݌ܴݎ ൌ Ͳ

ܿଷെ ʹ݌ܿଶ൅ ሺ݌ଶ൅ ݎଶ൅ Ͷܴݎሻܿ െ Ͷ݌ܴݎ ൌ Ͳ

Vậy ܽǡ ܾǡ ܿ nghiệm phương trình sau :

ݐଷെ ʹ݌ݐଶ൅ ሺ݌ଶ൅ ݎଶ൅ Ͷܴݎሻݐ െ Ͷ݌ܴݎ ൌ Ͳ

Theo định lý Viète :

ܾܽ ൅ ܾܿ ൅ ܿܽ ൌ ݌ଶ൅ ݎଶ൅ Ͷܴݎ

(80)

Giải:

a Ta có :

ݎ௔൅ ݎ௕൅ ݎ௖ െ ݎ ൌ ܵ ൬ ͳ ݌ െ ܽ൅

ͳ ݌ െ ܾ൅

ͳ ݌ െ ܿെ

ͳ

݌൰ ൌ ܵ ൤ ൅

݌ െ ݌ ൅ ܿ ݌ሺ݌ െ ܿሻ൨ ൌ ܵܿ ൤ ͳ

ሺ݌ െ ܽሻሺ݌ െ ܾሻ൅ ͳ

݌ሺ݌ െ ܿሻ൨ ൌ ܵܿ ൌܿ

ܵሾʹ݌ଶെ ݌ሺܽ ൅ ܾ ൅ ܿሻ ൅ ܾܽሿ ൌ

b Ta có :

ݎ௔൅ ݎ௕൅ ݎ െ ݎ௖ ൌ ܵܿ ൤ ͳ

ሺ݌ െ ܽሻሺ݌ െ ܾሻെ ܵሾ݌ሺܽ ൅ ܾ ൅ ܿሻ െ ܾܽሿ ൌܿ

ܵ൤ ͳ ʹ ൌ ʹܴ

ଶܿ

ܾܽܿ Ͷܴ

ൌ …‘• ʹ…‘•

ʹͶ …‘•

ʹ•‹

ʹ•‹

ʹെ ʹ •‹ •‹ ൰ ൌ •‹ •‹ െ ʹ •‹ •‹ ൰ ൌͺܴ

Ͷܴଶ൬Ͷ …‘•ଶ

ʹെ ʹ൰ ൌ Ͷܴ …‘•

c Ta có :

ൌ ܵ

ሺ݌ െ ܽሻሺ݌ െ ܾሻሺ݌ െ ܿሻ ൌ ܵଶ݌ݎ

ܵଶ

݌

ൌ ݎ݌ଶ

d Ta có :

ൌ ܵଶݎଶ൤ ͳ

ሺ݌ െ ܽሻሺ݌ െ ܾሻ൅

ͳ

ሺ݌ െ ܾሻሺ݌ െ ܿሻ൅

ͳ

ሺ݌ െ ܿሻሺ݌ െ ܽሻ൨

Bài 13: Chứng minh tam giác ta ln có : a ݎ௔൅ ݎ௕൅ ݎ௖ ൌ Ͷܴ ൅ ݎ

b ݎ௔൅ ݎ௕൅ ݎ ൌ Ͷܴ …‘• ൅ ݎ௖

c ݎ௔ݎ௕ݎ௖ ൌ ݎ݌ଶ

d ݎଶሺݎ

௔ݎ௕൅ ݎ௕ݎ௖൅ ݎ௖ݎ௔ሻ ൌ ܵଶ

e ሺܾ ൅ ܿሻସെ ʹሺܽଶ൅ ʹ݈ ௔

ଶሻሺܾ ൅ ܿሻଶ൅ ܽଶሺܽଶ൅ Ͷ݄

௔ଶሻ ൌ Ͳ

݌ െ ܽ ൅ ݌ െ ܾ ሺ݌ െ ܽሻሺ݌ െ ܾሻ

݌ሺ݌ െ ܿሻ ൅ ሺ݌ െ ܽሻሺ݌ െ ܾሻ ݌ሺ݌ െ ܽሻሺ݌ െ ܾሻሺ݌ െ ܿሻ ܾܽܿ

ܵ ൌ Ͷܴ ͳ

൨ ൌ ݌ሺ݌ െ ܿሻ

ܿ ሺܽ ൅ ܾ ൅ ܿሻሺܽ ൅ ܾ െ ܿሻ െ ܾܽ൨

ሾሺ•‹ ൅ •‹ ൅ •‹ ሻሺ•‹ ൅ •‹ െ •‹ ሻ െ ʹ •‹ •‹ ሿ ͺܴଷ

ܾܽ ൬Ͷ …‘• ʹ ͺܴଷ

(81)

ൌ ܵଶݎଶ݌ െ ܽ ൅ ݌ െ ܾ ൅ ݌ െ ܿ

ሺ݌ െ ܽሻሺ݌ െ ܾሻሺ݌ െ ܿሻ ൌ

ܵଶݎଶ݌

ܵଶ

݌

ൌ ݌ଶݎଶൌ ܵଶ

e Ta có :

ൌ ሺܾ ൅ ܿሻସെ ʹܽଶሺܾ ൅ ܿሻଶ൅ ܽସെ Ͷ݈

௔ଶሺܾ ൅ ܿሻଶ൅ Ͷܽଶ݄௔ଶ

ൌ ሾሺܾ ൅ ܿሻଶെ ܽଶሿଶെ Ͷሺܾ ൅ ܿሻଶǤ Ͷܾଶܿଶ

ሺܾ ൅ ܿሻଶ…‘•ଶ

ʹ൅ ͳ͸ܵଶ ൌ ሺܾ ൅ ܿ െ ܽሻଶሺܾ ൅ ܿ ൅ ܽሻଶെ Ͷሺܾ ൅ ܿሻଶǤ Ͷܾଶܿଶ

ሺܾ ൅ ܿሻଶǤ

ͳ ൅ …‘•

ʹ ൅ ͳ͸ܵଶ

Mặt khác, theo cơng thức Heron, ta có :

ܵଶൌ ݌ሺ݌ െ ܽሻሺ݌ െ ܾሻሺ݌ െ ܿሻ ൌ Ǥ Ǥܽ ൅ ܿ െ ܾ

ʹ Ǥ

ܽ ൅ ܾ െ ܿ ʹ

Giải:

a Ta có :

ͳ െʹݎ

݄௔ ൌ ͳ െ

ʹܵ ݌ Ǥ

ܽ

ʹܵ ൌ ͳ െ

ʹܽ

ܽ ൅ ܾ ൅ ܿ ൌ ͳ െ

Ͷܴ •‹

ʹܴሺ•‹ ൅ •‹ ൅ •‹ ሻ ൌ ͳ െ ͺܴ •‹

ʹ …‘•ʹ

ͺܴ …‘•ʹ …‘•ʹ …‘•ʹൌ ͳ െ

…‘• ൅ ʹ …‘•ʹ …‘•ʹ ƒǤ –ƒ

ʹ–ƒ

ʹൌ ͳ െ ʹݎ௔

ൌ ݎሺʹܴ െ ݎሻሺ•‹ ൅ •‹ ൅ •‹ ሻ

ܾଶ൅ ܿଶെ ܽଶ

ൌ ሺܾ ൅ ܿ െ ܽሻଶሺܾ ൅ ܿ ൅ ܽሻଶെ ͺܾଶܿଶቆͳ ൅ ቇ ൅ ͳ͸ܵଶ

ʹܾܿ

ൌ ሺܾ ൅ ܿ െ ܽሻଶሺܾ ൅ ܿ ൅ ܽሻଶെ Ͷܾܿሾሺܾ ൅ ܿሻଶെ ܽଶሿ ൅ ͳ͸ܵଶ

ൌ ሺܾ ൅ ܿ െ ܽሻଶሺܾ ൅ ܿ ൅ ܽሻଶെ Ͷܾܿሺܾ ൅ ܿ െ ܽሻሺܾ ൅ ܿ ൅ ܽሻ ൅ ͳ͸ܵଶ

ൌ ሺܾ ൅ ܿ െ ܽሻሺܾ ൅ ܿ ൅ ܽሻሺܾ െ ܿ െ ܽሻሺܾ െ ܿ ൅ ܽሻ ൅ ͳ͸ܵଶ

ܽ ൅ ܾ ൅ ܿ ʹ

ܾ ൅ ܿ െ ܽ ʹ

Suy ͳ͸ܵଶ ൌ ሺܽ ൅ ܾ ൅ ܿሻሺܾ ൅ ܿ െ ܽሻሺܽ ൅ ܿ െ ܾሻሺܽ ൅ ܾ െ ܿሻ

Vậy ൌ Ͳ

ʹݎ ݄௔ ൌ

ͳ ݄௔ ൅ ͳ

„Ǥሺ݌ െ ܽሻଶ•‹ ൅ ሺ݌ െ ܾሻଶ•‹ ൅ ሺ݌ െ ܿሻଶ•‹

(82)

ൌ ͳ െ…‘•

ʹ …‘•ʹ െ •‹ʹ •‹ʹ

…‘•ʹ …‘•ʹ ൌ –ƒ ʹ–ƒ

ʹ

Ta lại có :

ʹݎ௔

݄௔ ൌ ʹܵ ݌ െ ܽǤ

ܽ ʹܵ ൌ

ܽ ݌ െ ܽ ൌ

ʹܽ ܾ ൅ ܿ െ ܽ ൌ

Ͷܴ •‹

ʹܴሺ•‹ ൅ •‹ െ •‹ ሻൌ

•‹ʹ …‘•ʹ …‘•ʹ •‹ʹ •‹ʹ ൌ …‘•

൅ ʹ •‹ʹ •‹ʹ ൌ

…‘•ʹ …‘•ʹ െ •‹ʹ •‹ʹ •‹ʹ •‹ʹ ൌ …‘–

ʹ…‘–

ʹെ ͳ

Suy

ͳ ʹݎ௔

݄௔ ൅ ͳ

ൌ ͳ

…‘–ʹ …‘–ʹ െ ͳ ൅ ͳ ൌ –ƒ ʹ

b Ta có :

ʹ൰ ൌ

ʹ •‹ʹ …‘•ʹ …‘•ଶ

ʹ

ሺ݌ െ ܽሻଶ

ሺ݌ െ ܽሻǤ ܵ

݌ –ƒʹ ൌ ʹܵǤ ݌ െ ܽ

݌ ൌ ʹܵǤ

ܾ ൅ ܿ െ ܽ ܾ ൅ ܿ ൅ ܽ ൌ ʹܵǤ•‹ ൅ •‹ െ •‹

•‹ ൅ •‹ ൅ •‹ ൌ ʹܵǤ

Ͷ …‘•ʹ •‹ʹ •‹ʹ

Ͷ …‘•ʹ …‘•ʹ …‘•ʹൌ ʹܵ –ƒ ʹ–ƒ

ʹ

Tương tự vậy, ta có :

ሾሺ݌ െ ܾሻଶ൅ ݎଶሿ •‹ ൌ ʹܵ –ƒ

ʹ–ƒ ʹ ሾሺ݌ െ ܿሻଶ൅ ݎଶሿ •‹ ൌ ʹܵ –ƒ

ʹ–ƒ ʹ –ƒ

ʹ ൌ ݎሺʹܴ •‹ ൅ ʹܴ •‹ ൅ ʹܴ •‹ ሻ െ ݎଶሺ•‹ ൅ •‹ ൅ •‹ ሻ

ൌ ݎሺܽ ൅ ܾ ൅ ܿሻ െ ݎଶሺ•‹ ൅ •‹ ൅ •‹ ሻ

ൌ ʹ݌ݎ െ ݎଶሺ•‹ ൅ •‹ ൅ •‹ ሻ ൌ ʹܵ െ ݎଶሺ•‹ ൅ •‹ ൅ •‹ ሻ

Do đó, điều cần chứng minh tương đương với

ሺ݌ െ ܽሻଶ•‹ ൅ ሺ݌ െ ܾሻଶ•‹ ൅ ሺ݌ െ ܿሻଶ•‹ ൌ ʹܵ െ ݎଶሺ•‹ ൅ •‹ ൅ •‹ ሻ

฻ ሾሺ݌ െ ܽሻଶ൅ ݎଶሿ •‹ ൅ ሾሺ݌ െ ܾሻଶ൅ ݎଶሿ •‹ ൅ ሾሺ݌ െ ܿሻଶ൅ ݎଶሿ •‹ ൌ ʹܵ

Mặt khác, ta thấy :

ሾሺ݌ െ ܽሻଶ൅ ݎଶሿ •‹ ൌ ሺ݌ െ ܽሻଶ•‹ ൬ͳ ൅ –ƒଶ

(83)

Mà ta lại có :

–ƒ ʹ–ƒ

ʹ൅ –ƒ

ʹ–ƒ

ʹ൅ –ƒ

ʹ–ƒ

ʹ ൌ ͳ

Vậy cộng đẳng thức trên, ta có điều phải chứng minh

- BÀI TẬP TỰ LUYỆN

3.1.1. Cho tam giác Chứng minh

ƒǤܵ ൌ ͳ

Ͷሺܽଶ•‹ ʹ ൅ ܾଶ•‹ ʹሻ „Ǥ –ƒ

Ͷ൅ –ƒ Ͷ൅ –ƒ

Ͷ൅ –ƒ

Ͷ–ƒ

Ͷ൅ –ƒ

Ͷ–ƒ

Ͷ ൌ ͳ ൅ –ƒ

Ͷ–ƒ Ͷ–ƒ

Ͷ

•‹ ʹെ •‹

͵ ʹ •‹

͵ ʹ •‹

͵ ʹ ʹ–ƒ

ʹ ൌ ͳ

(ĐH Cần Thơ 2000)

ܿ ܾ ൌ

݉௕

݉௖ ് ͳ

Chứng minh : ʹ …‘– ൌ …‘– ൅ …‘–

(ĐH Tổng Hợp 1995)

3.1.4. Cho tam giác có ݈௖ ൌ ݈௕ Chứng minh ܽ ൌ ܾ

(Định lý Steiner(6) – Lehmus(7))

3.1.5. Cho tam giác thỏa hệ thức :

•‹ ʹ ൌ •‹

ʹ•‹

ʹ

Chứng minh :

൅ –ƒ Ͷ–ƒ

Ͷ …Ǥሺܽ ൅ ܾሻ …‘• ൅ ሺܾ ൅ ܿሻ …‘• ൅ ሺܿ ൅ ܽሻ …‘• ൌ ʹ݌

†Ǥܾܽሺܽ ൅ ܾሻ …‘• ൅ ܾܿሺܾ ൅ ܿሻ …‘• ൅ ܿܽሺܿ ൅ ܽሻ …‘• ൌ ܽଷ൅ ܾଷ൅ ܿଷ

‡Ǥ …‘•ଷ ൅ …‘•ଷ ൅ …‘•ଷ ൌ ͳ ൅ ͵ •‹

ʹ•‹ ʹ

3.1.2. Cho tam giác , ൌ ܽǡ ൌ ܾǡ ൌ ܿ

ͷ –ƒ

Chứng minh ͵ܿ ൌ ʹሺܽ ൅ ܾሻ

(84)

ƒǤ –ƒ ʹ–ƒ

ʹ ൌ

ͳ ʹ „Ǥ –ƒ

ʹ–ƒ ʹ൅ –ƒ

ʹ–ƒ

ʹ ൌ

ͳ ʹ

(ĐH Dược Hà Nội 1998)

3.1.6. Cho tam giác có ܽସ ൌ ܾସ൅ ܿସ Chứng minh rằng tam giác nhọn

–ƒ –ƒ ൌ ʹ •‹ଶ.

3.1.7. Trong tam giác , chứng minh :

ƒǤݎ௔ൌ ݎ ൅ Ͷܴ •‹ଶ

ʹ „Ǥ •‹

ʹ ൌ

ݎ௔

ඥሺݎ௔൅ ݎ௕ሻሺݎ௔൅ ݎ௖ሻ …Ǥ …‘• ൌ ʹܴ ൅ ݎ െ ݎ௔

ʹܴ

†Ǥ …‘• ൅ …‘• ൅ ൌ …‘– ൅ …‘– ൅ …‘– ͳ

Ͷ

ൌͳ ʹ

–ƒͶ ൅ –ƒͶ ͳ െ –ƒͶ –ƒͶ ൌ

ͳ െ –ƒͶ ͳ ൅ –ƒͶ

c Áp dụng định lý hình chiếu d Áp dụng định lý hàm số cos e Sử dụng công thức

…‘• ൅ …‘• ൅ …‘• ൌ ͳ ൅ Ͷ •‹ ʹ•‹

ʹ•‹

ʹ …‘•

ܾ …‘• ൅ ܿ …‘• ܽ …‘• ൅ ܿ …‘• ܾ …‘• ൅ ܽ …‘• ʹܴ ‡Ǥሺܾ െ ܿሻሺ݌ െ ܽሻ …‘• ൅ ሺܿ െ ܽሻሺ݌ െ ܾሻ …‘• ൅ ሺܽ െ ܾሻሺ݌ െ ܿሻ …‘• ൌ Ͳ

- GỢI Ý GIẢI BÀI TẬP TỰ LUYỆN 3.1.1.

a Theo định lý hàm số sin, ta có :

ሺܽଶ•‹ ʹ ൅ ܾଶ•‹ ʹሻ ൌ ܴଶሺ•‹ଶ •‹ ʹ ൅ •‹ଶ •‹ ʹሻ

ൌ ʹܴଶ•‹ •‹ ሺ•‹ …‘• ൅ •‹ …‘• ሻ ൌ ʹܴଶ•‹ •‹ •‹

ܾܽ •‹ ൌ ܵ

(85)

3.1.2 Để ý, từ giả thuyết ta có :

Ͷ •‹ ʹ•‹

ʹ ൌ …‘• ʹ…‘•

ʹെ •‹

ʹ•‹

ʹ ฻ ʹ ൬…‘• െ

ʹ െ …‘• ൅

ʹ ൰ ൌ …‘•

൅ ʹ ฻ ͵ ൬ʹ •‹ ൅

ʹ …‘•

ʹ ൰ ൌ Ͷ •‹ ൅

ʹ …‘•

െ ʹ ฻ ͵ •‹ ൌ ʹሺ•‹ ൅ •‹ ሻ

3.1.3 Để ý :

ܿଶ

ܾଶ ൌ

݉௕ଶ

݉௖ଶ ൌ

ʹሺܽଶ൅ ܿଶሻ െ ܾଶ

ʹሺܽଶ൅ ܾଶሻ െ ܿଶ

݈௕ ൌ ݈௖ ฻

ʹܿܽ

ܿ ൅ ܽඨ ඨ

݌ሺ݌ െ ܿሻ ܾܽ

3.1.5

a Để ý :

ൌ •‹ ൅

ʹ ൌ …‘• ʹ…‘•

ʹെ •‹

ʹ•‹

ʹ –ƒ

ʹ–ƒ ʹ൅ –ƒ

ʹ–ƒ

ʹ൅ –ƒ

ʹ–ƒ

ʹ ൌ ͳ

3.1.6 Từ giả thuyết, ta có ܽ ൌ ƒšሼܽǡ ܾǡ ܿሽ Do

ቄܾଶ ൏ ܽଶ

ܿଶ ൏ ܽଶ ฺ ቄܾ

ସ ൏ ܽଶܾଶ

ܿସ ൏ ܽଶܿଶ ฺ ܽସ ൌ ܾସ൅ ܿସ ൏ ܽଶሺܾଶ൅ ܿଶሻ

ฺ ܽଶ ൏ ܾଶ൅ ܿଶ ฺ …‘• ൐ Ͳ

฻ ʹܽଶ ൌ ܾଶ൅ ܿଶ

3.1.4 Ta sử dụng công thức vềđộ dài phân giác :

݌ሺ݌ െ ܾሻ ܿܽ ൌ

ʹܾܽ ܽ ൅ ܾ

฻ ܽሺܽ ൅ ܾ ൅ ܿሻሾሺܽ ൅ ܾ ൅ ܿሻሺܽଶ൅ ʹܾܿሻ ൅ ʹܾܽܿሿሺܾ െ ܿሻ ൌ Ͳ ฻ ܾ ൌ ܿ

•‹ ʹ

(86)

–ƒ –ƒ ൌ Ͷܽ

ଶܾܿ •‹ •‹

ܽସെ ሺܾଶെ ܿଶሻଶ ൌ ʹܽଶ

•‹ ܾ Ǥ

•‹

ܿ ൌ ʹ •‹ଶ

3.1.7

a Ta có :

ݎ௔ െ ݎ ൌ ݌ –ƒ

ʹെ ሺ݌ െ ܽሻ –ƒ

ʹ ൌ ܽ –ƒ

ʹ ൌ Ͷܴ •‹ ʹ…‘•

ʹ–ƒ

ʹ ൌ Ͷܴ •‹ଶ ʹ

b Để ý :

ൌ ݌ –ƒ ʹ

݌ටቀ–ƒʹ ൅ –ƒʹቁ ቀ–ƒʹ ൅ –ƒʹቁ ൌ

ൌ •‹ ʹ

c Để ý :

ൌ ͳ ൅ ൌ …‘• ݎ

–ƒʹ…‘• ൌ ʹܴݎǤ ʹ …‘• ൅

ʹ •‹ െ

ʹ Ǥ …‘•ʹ

•‹ʹǤ …‘• ൌ Ͷܴݎ •‹ െ

ʹ •‹ ൅

ʹ …‘• ൌ ʹܴݎሺ…‘• …‘• െ …‘• …‘• ሻ

Tương tự vậy, ta có :

ሺܿ െ ܽሻሺ݌ െ ܾሻ …‘• ൌ ʹܴݎሺ…‘• …‘• െ …‘• …‘• ሻ ሺܽ െ ܾሻሺ݌ െ ܿሻ …‘• ൌ ʹܴݎሺ…‘• …‘• െ …‘• …‘• ሻ

–ƒʹ

•‹ ʹ ൅ •‹ ൅ ʹ …‘•ଶ

ʹ …‘• ʹ…‘• ʹ ݎ െ ݎ௔

ʹܴ ൌ ͳ െ

Ͷܴ •‹ଶ

ʹ ʹܴ

d Áp dụng định lý hình chiếu e Ta có :

(87)

2. CHỨNG MINH BẤT ĐẲNG THỨC LƯỢNG GIÁC TRONG TAM GIÁC - Ngoài việc nhớ đẳng thức áp dụng kỹ thuật biến đổi để chứng

minh đẳng thức lượng giác vào dạng tốn này, ta nên nắm số kỹ thuật chứng minh bất đẳng thức, chẳng hạn :

x Dùng quan hệ cạnh góc : Trong tam giác , ta có :

ܽ ൏ ܾ ൏ ܿ ฻ ൏ ܤ ൏ ܥ ฻ ቄͲ ൏ •‹ ൏ •‹ ൏ •‹ …‘• ൐ …‘• ൐ …‘•

Từ tính chất trên, ta có kết sau :

ܽଵ ܾଵ ൌ

ܽଶ

ܾଶ ൌ ǥ ൌ

ܽ௡ ܾ௡

iii Bất đẳng thức Chebyshev(10) :

Cho hai dãy số thực tăng : ܽଵ ൑ ܽଶ ൑ ǥ ൑ ܽ௡ ܾଵ ൑ ܾଶ ൑ ǥ ൑ ܾ௡ :

ܽଵ൅ ܽଶ൅ ǥ൅ ܽ௡ ݊ Ǥ

ܾଵ൅ ܾଶ ൅ǥ൅ ܾ௡ ݊ ൑

ܽଵܾଵ൅ ܽଶܾଶ൅ ǥ൅ ܽ௡ܾ௡ ݊

ሺܽ െ ܾሻሺ•‹ െ •‹ ሻ ൒ Ͳ ฻ ܽ •‹ ൅ ܾ •‹ ൒ ܽ •‹ ൅ ܾ •‹ ሺܽ െ ܾሻሺ…‘• െ …‘• ሻ ൑ Ͳ ฻ ܽ …‘• ൅ ܾ …‘• ൑ ܽ …‘• ൅ ܾ …‘•

x Dùng bất đẳng thức cổđiển : i Bất đẳng thức Cauchy(8) :

Cho ݊ số không âm : ܽଵǡ ܽଶǡ ǥ ǡ ܽ௡ሺ݊ א Ժǡ ݊ ൒ ʹሻ :

ܽଵ൅ ܽଶ൅ǥ൅ ܽ௡ ൒ ݊ ඥܽଵܽଶǥ ܽ௡

Dấu ̶ ൌ ̶ xảy ܽଵ ൌ ܽଶ ൌ ǥ ൌ ܽ௡

ii Bất đẳng thức Bunyakovsky(9) :

Cho hai dãy số thực : ܽଵǡ ܽଶǡ ǥ ǡ ܽ௡ሺ݊ א Ժǡ ݊ ൒ ʹሻ ܾଵǡ ܾଶǡ ǥ ǡ ܾ௡ሺ݊ א Ժǡ ݊ ൒ ʹሻ :

ȁܽଵܾଵ൅ ܽଶܾଶ൅ ǥ൅ ܽ௡ܾ௡ȁ ൑ ටሺܽଵଶ൅ ܽଶଶ൅ ǥ൅ ܽ௡ሻሺܾଵଶ൅ ܾଶଶ൅ǥ൅ ܾ௡ଶሻ

(88)

Cho dãy số thực tăng : ܽଵ ൑ ܽଶ ൑ ǥ ൑ ܽ௡ dãy số thực giảm ܾଵ ൒ ܾଶ ൒ ǥ ൒ ܾ௡ :

ܽଵ൅ ܽଶ൅ ǥ൅ ܽ௡ ݊ Ǥ

ܾଵ൅ ܾଶ ൅ǥ൅ ܾ௡ ݊ ൒

ܽଵܾଵ൅ ܽଶܾଶ൅ ǥ൅ ܽ௡ܾ௡ ݊

Dấu ̶ ൌ ̶ xảy ቄܾܽଵ ൌ ܽଶ ൌ ǥ ൌ ܽ௡

ଵ ൌ ܾଶ ൌ ǥ ൌ ܾ௡

iv Bất đẳng thức Bernoulli(11) : Với ܽ ൒ െͳ với א Գ :

ሺͳ ൅ ܽሻ௡ ൒ ͳ ൅ ݊ܽ

Dấu ̶ ൌ ̶ xảy ൥ܽ ൌ Ͳ݊ ൌ Ͳ ݊ ൌ ͳ

൒ ݂ ൬ݔଵ൅ ݔଶ൅ ǥ൅ ݔ௡ ݊ ൰ ൑ ݂ ൬ݔଵ൅ ݔଶ൅ ǥ൅ ݔ௡

݊ ൰

v Bất đẳng thức Jensen(12) :

Cho hàm số݂ሺݔሻcó đạo hàm cấp khoảng ൌ ሺܽǡ ܾሻ

Nếu với ݔ א ǡ ݂ᇱᇱሺݔሻ ൐ Ͳ ݔ

ଵǡ ݔଶǡ ǥ ǡ ݔ௡ א :

݂ሺݔଵሻ ൅ ݂ሺݔଶሻ ൅ǥ൅ ݂ሺݔ௡ሻ

݊

Nếu với ݔ א ǡ ݂ᇱᇱሺݔሻ ൏ Ͳ ݔ

ଵǡ ݔଶǡ ǥ ǡ ݔ௡ א :

݂ሺݔଵሻ ൅ ݂ሺݔଶሻ ൅ǥ൅ ݂ሺݔ௡ሻ ݊

Dấu ̶ ൌ ̶ xảy ݔଵ ൌ ݔଶ ൌ ǥ ൌ ݔ௡

x Dùng đạo hàm để áp dụng tính chất đồng biến, nghịch biến hàm số

(89)

Giải:

a Ta có :

ʹ ‘Ͳ ൑ ൅

ʹ ൑ ߨ˜ െ ߨ ʹ൑

െ ʹ ൑

ߨ ʹ

൅ ʹ

െ ʹ ൑ ͳ

Vậy ta chứng minh

൅ ʹ

Tương tự, ta có :

ߨ

͵൑ ʹ •‹ ቌ ൅ߨ͵

ʹ ቍ

Suy

•‹ ൅ •‹ ൅ •‹ ൅ •‹ߨ

͵ ൑ ʹ ቎•‹ ൅

ʹ ൅ •‹ ቌ ൅ߨ͵

ʹ ቍ቏ ൑ Ͷ •‹ ൬ ൅

Ͷ ൅ Ͷ൅

ߨ ͳʹ൰ ฺ •‹ ൅ •‹ ൅ •‹ ൅ •‹ߨ

͵ ൑ Ͷ •‹ ߨ ͵

Do đó,

•‹ ൅ •‹ ൅ •‹ ൑ ͵ •‹ߨ ͵ ൌ

͵ξ͵ ʹ

b Ta có :

…‘• ൅ …‘• ൌ ʹ …‘• ൅ ʹ …‘•

െ ʹ ƒǤ •‹ ൅ •‹ ൅ •‹ ൑͵ξ͵

ʹ „Ǥ …‘• ൅ …‘• ൅ …‘• ൑ ͵ ʹ …Ǥ •‹ •‹ •‹ ൑͵ξ͵

ͺ †Ǥ …‘• …‘• …‘• ൑ ͳ ͺ ‡Ǥ •‹

ʹ൅ •‹ ʹ൅ •‹

ʹ൑

͵

ʹ ˆǤ …‘• ʹ൅ …‘•

ʹ൅ …‘• ʹ൑

͵ξ͵ ʹ ‰Ǥ •‹

ʹ•‹ ʹ•‹

ʹ൑

ͳ

ͺŠǤ …‘• ʹ…‘•

ʹ…‘•

ʹ ൑

͵ξ͵ ͺ

Bài 1: Cho tam giác , chứng minh :

•‹ ൅ •‹ ൌ ʹ •‹ ൅ …‘• െ ฺ •‹

ʹ

൒ Ͳ˜Ͳ ൑ …‘• •‹ ൅ •‹ ൑ ʹ •‹

(90)

 …‘• ൅ ʹ ൌ •‹

ʹ ൒ Ͳ˜Ͳ ൑ …‘•

െ ʹ ൑ ͳ

Suy

…‘• ൅ …‘• ൑ ʹ …‘• ൅ ʹ

Tương tự, ta có :

…‘• ൅ …‘•ߨ

͵ ൑ ʹ …‘• ቌ ൅ߨ͵

ʹ ቍ

Do đó,

…‘• ൅ …‘• ൅ …‘• ൅ …‘•ߨ

͵ ൑ ʹ ቎…‘• ൅

ʹ ቍ቏ ൑ Ͷ …‘• ൬ ൅

Ͷ ൅ Ͷ൅

ߨ ͳʹ൰

Suy Hay

ߨ ͵ ൌ

͵ ʹ ൰

൑ ቆ͵ξ͵ ͸ ቇ

ൌ͵ξ͵ ͺ …‘• …‘• …‘• ൑ ൬…‘• ൅ …‘• ൅ …‘•

͵ ൰

ൌͳ ͺ

e Áp dụng bất đẳng thức chứng minh câu a, ta :

•‹ ʹ൅ •‹

ʹ൅ •‹

ʹ൅ •‹

ߨ

͸ ൑ ʹ ൤•‹

Ͷ ൅ •‹ ൬ Ͷ൅

ߨ

ͳʹ൰൨ ൑ Ͷ •‹ ൬ ൅

ͺ ൅ ͺ൅

ߨ ʹͶ൰

Suy

•‹ ʹ൅ •‹

ʹ൅ •‹

ʹ൅ •‹

ߨ

͸ ൑ Ͷ •‹ ߨ ͸

൅ …‘• ቌ ൅ ͵ ߨ ʹ …‘• ൅ …‘• ൅ …‘• ൅ …‘•ߨ

͵ ൑ Ͷ …‘• ߨ ͵ …‘• ൅ …‘• ൅ …‘• ൑ ͵ …‘•

c Theo bất đẳng thức Cauchy, ta có :

•‹ •‹ •‹ ൑ ൬•‹ ൅ •‹ ൅ •‹ ͵

d Ta thấy :

- Nếu tam giác có góc tù bất đẳng thức hiển nhiên

(91)

Vậy ta : •‹ ʹ൅ •‹ ʹ൅ •‹ ʹ൑ ͵ •‹ ߨ ͸ ൌ ͵ ʹ

f Áp dụng bất đẳng thức chứng minh câu b, ta :

…‘• ʹ൅ …‘• ʹ൅ …‘• ʹ൅ …‘• ߨ ͸ ൑ ʹ ൤…‘• ൅ Ͷ ൅ …‘• ൬ Ͷ൅ ߨ ͳʹ൰൨ ൑ Ͷ •‹ ൬ ൅ ͺ ൅ ͺ൅ ߨ ʹͶ൰ ൌ Ͷ …‘• ߨ ͸ Suy …‘• ʹ൅ …‘• ʹ൅ …‘• ʹ൑ ͵ …‘• ߨ ͸ൌ ͵ξ͵ ʹ

g Theo bất đẳng thức Cauchy, ta có :

•‹ ʹ•‹ ʹ•‹ ʹ൑ ቌ •‹ʹ ൅ •‹ʹ ൅ •‹ʹ ͵ ቍ ଷ ൌͳ ͺ

h Theo bất đẳng thức Cauchy, ta có :

…‘• ʹ…‘• ʹ…‘• ʹ൑ ቌ …‘•ʹ ൅ …‘•ʹ ൅ …‘•ʹ ͵ ቍ ଷ ൌ ቆξ͵ ʹ ቇ ଷ ൌ͵ξ͵ ͺ ƒǤ •‹ଶ ൅ •‹ଶ ൅ •‹ଶ ൑ͻ Ͷ „Ǥ …‘•ଶ ൅ …‘•ଶ ൅ …‘•ଶ ൒ ͵ Ͷ …Ǥ •‹ଶ ʹ൅ •‹ଶ ʹ൅ •‹ଶ ʹ ൒ ͵ Ͷ †Ǥʹ ൏ …‘•ଶ ʹ൅ …‘•ଶ ʹ൅ …‘•ଶ ʹ൑ ͻ Ͷ ‡Ǥ •‹ •‹ ൅ •‹ •‹ ൅ •‹ •‹ ൑ͻ Ͷ ˆǤ …‘• …‘• ൅ …‘• …‘• ൅ …‘• …‘• ൑͵ Ͷ ‰Ǥ •‹ ʹ•‹ ʹ൅ •‹ ʹ•‹ ʹ൅ •‹ ʹ•‹ ʹ ൑ ͵ Ͷ

(92)

Giải:

a Ta có :

•‹ଶ ൅ •‹ଶ ൅ •‹ଶ ൌͳ െ …‘• ʹ

ʹ ൅

ͳ െ …‘• ʹ

ʹ ൅ ͳ െ …‘•ଶ ൌ ʹ െͳ

ʹሺ…‘• ʹ ൅ …‘• ʹሻ െ …‘•ଶ

ൌ ʹ െ …‘•ሺ ൅ ሻ …‘•ሺ െ ሻ െ …‘•ଶ ൌ ʹ ൅ …‘• ሾ…‘•ሺ െ ሻ െ …‘• ሿ

- Nếu góc tù …‘• ሾ…‘•ሺ െ ሻ െ …‘• ሿ ൏ Ͳ Suy

•‹ଶ ൅ •‹ଶ ൅ •‹ଶ ൏ ʹ ൏ͻ

Ͷ

- Nếu góc khơng tù

•‹ଶ ൅ •‹ଶ ൅ •‹ଶ ൑ ʹ …‘• ሺͳ െ …‘• ሻ ൑ ʹ ൅ ቈ…‘• ൅ ሺͳ െ …‘• ሻ

ʹ ቉

ൌͻ Ͷ

b Ta có :

…‘•ଶ ൅ …‘•ଶ ൅ …‘•ଶ ൌͳ ൅ …‘• ʹ

ʹ ൅

ͳ ൅ …‘• ʹ

ʹ ൅ …‘•ଶ

ൌ ͳ ൅ …‘•ሺ ൅ ሻ …‘•ሺ െ ሻ ൅ …‘•ଶ ൌ ͳ െ …‘• ሾ…‘•ሺ െ ሻ െ …‘• ሿ

- Nếu góc tù െ …‘• ሾ…‘•ሺ െ ሻ െ …‘• ሿ ൐ Ͳ Suy

…‘•ଶ ൅ …‘•ଶ ൅ …‘•ଶ ൐ ͳ ൐ ͵

Ͷ

- Nếu góc khơng tù

…‘•ଶ ൅ …‘•ଶ ൅ …‘•ଶ ൒ ͳ െ …‘• ሺͳ െ …‘• ሻ ൒ ͳ െ ൬…‘• ൅ ͳ െ …‘•

ʹ ൰

ൌ͵ Ͷ

c Ta có :

•‹ଶ

ʹ൅ •‹ଶ

ʹ൅ •‹ଶ ʹൌ

ͳ െ …‘• ʹ ൅

ͳ െ …‘• ʹ ൅

ͳ െ …‘• ʹ ൌ ͵

ʹെ ͳ

ʹሺ…‘• ൅ …‘• ൅ …‘• ሻ ൒ ͵ ʹെ

ͳ ʹǤ

͵ ʹൌ

͵ Ͷ

d Ta có :

…‘•ଶ

ʹ൅ …‘•ଶ

ʹ൅ …‘•ଶ ʹ ൌ

ͳ ൅ …‘• ʹ ൅

ͳ ൅ …‘• ʹ ൅

ͳ ൅ …‘• ʹ ൌ͵

ʹ൅ ͳ

(93)

Mặt khác : …‘•ଶ ʹ൅ …‘•ଶ ʹ൅ …‘•ଶ ʹ ൌ ͵ ʹ൅ ͳ ʹሺ…‘• ൅ …‘• ൅ …‘• ሻ ൌ͵ ʹ൅ ͳ ʹ൬ͳ ൅ Ͷ •‹ ʹ•‹ ʹ•‹ ʹ൰ ൌ ʹ ൅ ʹ •‹ ʹ•‹ ʹ•‹ ʹ൐ ʹ ‘ •‹ ʹǢ•‹ ʹǢ•‹ ʹ൐ Ͳ

e Theo bất đẳng thức Bunyakovsky, ta có :

•‹ •‹ ൅ •‹ •‹ ൅ •‹ •‹ ൑ •‹ଶ ൅ •‹ଶ ൅ •‹ଶ ൑ ͻ

Ͷ

f Ta có :

ሺ…‘• ൅ …‘• ൅ …‘• ሻଶ ൒͵ Ͷ Suy ͵ Ͷ ͻ Ͷ

Do đó,

͵ Ͷ

g Ta có :

൬•‹ ʹ൅ •‹ ʹ൅ •‹ ʹ൰ ଶ ൌ •‹ଶ ʹ ൅ •‹ଶ ʹ൅ ʹ ൬•‹ ʹ•‹ ʹ൅ •‹ ʹ•‹ ʹ൅ •‹ ʹ•‹ ʹ൰ ൒ •‹ ʹ൅ •‹ ʹ•‹ ʹ൅ •‹ ʹ•‹ ʹ൰ Suy ͵ Ͷ൅ ʹ ൬•‹ ʹ•‹ ʹ൅ •‹ ʹ•‹ ʹ൅ •‹ ʹ•‹ ʹ൰ ൑ ͻ Ͷ

Do đó,

(94)

Giải:

a Theo bất đẳng thức Cauchy, ta có :

•‹ ൅•‹ ൅•‹ ൒ ൒ ͻ ͵ξ͵ ʹ ൒ ʹξ͵ …‘• ൅…‘• ൅…‘• ൒ ൒ ͻ …‘• ൅ …‘• ൅ …‘• ൒ ͻ ͵ ʹ ൌ ͸ ͳ •‹ʹ൅ ൒ ͵ ට•‹ ʹ•‹ʹ •‹ʹ య ൒ ͻ •‹ʹ ൅ •‹ʹ ൅ •‹ʹ൒ ͸

d Theo bất đẳng thức Cauchy, ta có :

ͳ …‘•ʹ൅ ͳ …‘•ʹ ൅ ͳ …‘•ʹ ൒ ͵ ට…‘• ʹ…‘•ʹ …‘•ʹ య ൒ ͻ …‘•ʹ ൅ …‘•ʹ ൅ …‘•ʹ൒ ʹξ͵

e Theo bất đẳng thức Cauchy, ta có :

ͳ •‹ଶ൅ ͳ •‹ଶ൅ ͳ •‹ଶ൒ ͵ ξ•‹ଶ •‹ଶ •‹ଶ య ൒ ͻ •‹ଶ ൅ •‹ଶ ൅ •‹ଶ൒ ͻ ͻ Ͷ ൌ Ͷ

f Theo bất đẳng thức Cauchy, ta có :

ͳ …‘•ଶ൅ ͳ …‘•ଶ൅ ͳ …‘•ଶ൒ ͵ ξ…‘•ଶ …‘•ଶ …‘•ଶ య ൒ ͵ ͳ ൌ ͳʹ ƒǤ ͳ •‹ ൅ ͳ •‹ ൅ ͳ •‹ ൒ ʹξ͵„Ǥ ͳ …‘• ൅ ͳ …‘• ൅ ͳ

…‘• ൒ ͸ሺŠọnሻ …Ǥ ͳ •‹ʹ൅ ͳ •‹ʹ൅ ͳ •‹ʹ൒ ͸†Ǥ ͳ …‘•ʹ൅ ͳ …‘•ʹ൅ ͳ …‘•ʹ ൒ ʹξ͵ ‡Ǥ ͳ •‹ଶ൅ ͳ •‹ଶ൅ ͳ •‹ଶ൒ Ͷ ˆǤ ͳ …‘•ଶ൅ ͳ …‘•ଶ൅ ͳ …‘•ଶ ൒ ͳʹ ‰Ǥ ͳ •‹ଶ ʹ ൅ ͳ •‹ଶ ʹ ൅ ͳ •‹ଶ ʹ ൒ ͳʹŠǤ ͳ …‘•ଶ ʹ ൅ ͳ …‘•ଶ ʹ ൅ ͳ ൒ Ͷ

Bài 3: Cho tam giác , chứng minh :

ͳ ͳ ͳ ͵

ξ•‹ •‹ •‹ ൒

ͻ

•‹ ൅ •‹ ൅ •‹

b Theo bất đẳng thức Cauchy, ta có :

ͳ ͳ ͳ ͵

ξ…‘• …‘• …‘•

c Theo bất đẳng thức Cauchy, ta có :

(95)

g Theo bất đẳng thức Cauchy, ta có :

ͳ •‹ଶ

ʹ

൅ ͳ •‹ଶ

ʹ

൅ ͳ •‹ଶ

ʹ

൒ ͵ ට•‹ଶ

ʹ •‹ଶʹ •‹ଶʹ

య ൒

ͻǤͶ ͵ ൌ ͳʹ

h Theo bất đẳng thức Cauchy, ta có :

ͳ …‘•ଶ

ʹ

൅ ͳ …‘•ଶ

ʹ

൅ ͳ …‘•ଶ

ʹ

൒ ͵ ට…‘•ଶ

ʹ …‘•ଶʹ …‘•ଶʹ

య ൒

͵ǤͶ ͵ ൌ Ͷ

Chú ý : Từ câu e, f, g, h ta rút kết sau :

Giải:

ሺ–ƒ ൅ –ƒ ൅ –ƒ ሻଷ ൒ ʹ͹ –ƒ –ƒ –ƒ ൌ ʹ͹ሺ–ƒ ൅ –ƒ ൅ –ƒ ሻ

Suy

–ƒ ൅ –ƒ ൅ –ƒ ൒ ͵ξ͵

Cách 2: Ta có

–ƒ –ƒ ൌ •‹ሺ ൅ ሻ …‘• …‘•

Mặt khác :

Ͳ ൏ ʹ …‘• …‘• ൌ …‘•ሺ ൅ ሻ ൅ …‘•ሺ െ ሻ ൑ ͳ ൅ …‘•ሺ ൅ ሻ ൌ ʹ …‘•ଶ ൅

ʹ …Ǥ –ƒ

ʹ൅ –ƒ ʹ

ʹ൅ …‘–

ʹ൅ …‘–

ʹ൒ ͵ξ͵ ‰Ǥ …‘–ଶ

ʹ൅ …‘–ଶ

ʹ൅ …‘–ଶ

ʹ൒ ͻŠǤ –ƒଶ

ʹ ൒ ͳ

a Ta có cách chứng minh :

Cách 1: Sử dụng đẳng thức –ƒ ൅ –ƒ ൅ –ƒ ൌ –ƒ –ƒ –ƒ

Theo bất đẳng thức Cauchy, ta có :

Bài 4: Cho tam giác , chứng minh :

ƒǤ –ƒ ൅ –ƒ ൅ –ƒ ൒ ͵ξ͵ሺŠọሻ „Ǥ …‘– ൅ …‘– ൅ …‘– ൒ ξ͵ሺŠọሻ

൅ –ƒ

ʹ൒ ξ͵†Ǥ …‘–

‡Ǥ …‘–ଶ ൅ …‘–ଶ ൅ …‘–ଶ ൒ ͳˆǤ –ƒଶ ൅ –ƒଶ ൅ –ƒଶ ൒ ͻ

൅ –ƒଶ

(96)

Nên –ƒ ൅ –ƒ ൒ʹ •‹ ൅ ʹ …‘• ൅ ʹ …‘•ଶ ൅ ʹ ൌ ʹ –ƒ ൅ ʹ

Tương tự, ta :

–ƒ ൅ –ƒߨ ͵ ൒ ʹ –ƒ ൬ ʹ൅ ߨ ͸൰

Do đó,

–ƒ ൅ –ƒ ൅ –ƒ ൅ –ƒߨ ͵ ൒ ʹ ൤–ƒ ൅ ʹ ൅ –ƒ ൬ ʹ൅ ߨ ͸൰൨ ൒ Ͷ –ƒ ߨ ͵ Suy –ƒ ൅ –ƒ ൅ –ƒ ൒ ͵ –ƒߨ ͵

b Ta có :

ሺ…‘– ൅ …‘– ൅ …‘– ሻଶ

Do đó,

c Ta có :

൬–ƒ ʹ൅ –ƒ ʹ൅ –ƒ ʹ൰ ଶ ൌ –ƒଶ ʹ ൅ –ƒଶ ʹ൅ ʹ ൬–ƒ ʹ–ƒ ʹ൅ –ƒ ʹ–ƒ ʹ൅ –ƒ ʹ–ƒ ʹ൰ ‘¯×ǡ –ƒ ʹ൅ –ƒ ʹ൅ –ƒ ʹ൒ ξ͵

d Ta sử dụng đẳng thức

…‘– ʹ൅ …‘– ʹ൅ …‘– ʹ ൌ …‘– ʹ…‘– ʹ…‘– ʹ

Theo bất đẳng thức Cauchy, ta có :

(97)

Giải:

a Áp dụng định lý hình chiếu, ta có :

݉௔൅ ݉௕ ൅ ݉௖ ൑ ൬

݉௔൅ ݉௕ ൅ ݉௖ ͵ ൰

ൌ ͳ

ʹ͹ሺ݉௔൅ ݉௕ ൅ ݉௖ሻଷ

Mặt khác, theo bất đẳng thức Bunyakovsky định lý hàm số sin :

ሺ݉௔൅ ݉௕ ൅ ݉௖ሻଶ ൑ ͵ሺ݉

௔ଶ൅ ݉௕ଶ൅ ݉௖ଶሻ ൌ

ͻ

Ͷሺܽଶ ൅ ܾଶ൅ ܿଶሻ ൌ ͻܴଶሺ•‹ଶ ൅ •‹ଶ ൅ •‹ଶሻ

Mà ta có bất đẳng thức :

•‹ଶ ൅ •‹ଶ ൅ •‹ଶ ൑ͻ

Ͷ

Bài 5: Chứng minh tam giác , ta ln có : a ʹሺܽ …‘• ൅ ܾ …‘• ൅ ܿ …‘• ሻ ൑ ܽ ൅ ܾ ൅ ܿ

b ʹሺܽ •‹ ൅ ܾ •‹ ൅ ܿ •‹ ሻ ൒ ሺܽ ൅ ܾሻ •‹ ൅ ሺܾ ൅ ܿሻ •‹ ൅ ሺܿ ൅ ܽሻ •‹

c ͺ݉௔݉௕݉௖ ൑ ʹ͹ܴଷ

(ĐH Ngoại Thương 1996)

d ܴ ൒ ʹݎ

e ݌ଶ ൑ ͸ܴଶ൅ ͵ݎଶ

(Đề nghị Olympic 30-4, 2007)

ܿ ൌ ܽ …‘• ൅ ܾ …‘•

ሺܽ െ ܾሻሺ…‘• െ …‘• ሻ ൑ Ͳ ฻ ܽ …‘• ൅ ܾ …‘• ൑ ܽ …‘• ൅ ܾ …‘•

Suy ܽ …‘• ൅ ܾ …‘• ൑ ܿ Tương tự, ta có :

ܾ …‘• ൅ ܿ …‘• ൑ ܽ ܿ …‘• ൅ ܽ …‘• ൑ ܾ

Cộng bất đẳng thức trên, ta suy điều phải chứng minh Dấu ̶ ൌ ̶ xảy tam giác

b Ta có :

ሺܽ െ ܾሻሺ•‹ െ •‹ ሻ ൒ Ͳ ฻ ܽ •‹ ൅ ܾ •‹ ൒ ܽ •‹ ൅ ܾ •‹

Tương tự, ta có :

ܾ •‹ ൅ ܿ •‹ ൒ ܾ •‹ ൅ ܿ •‹ ܿ •‹ ൅ ܽ •‹ ൒ ܿ •‹ ൅ ܽ •‹

Cộng bất đẳng thức trên, ta suy điều phải chứng minh Dấu ̶ ൌ ̶ xảy tam giác

(98)

Do đó,

ሺ݉௔൅ ݉௕ ൅ ݉௖ሻଶ ൑

ͺͳ Ͷ ܴଶ

Suy

݉௔݉௕݉௖ ൑ ͳ ʹ͹Ǥ ൬

ͻܴ ʹ ൰

ൌʹ͹ ͺ ܴଷ

Từđó ta có điều phải chứng minh

Dấu ̶ ൌ ̶ xảy tam giác d Ta có :

ܵ ൌ ݌ݎ ൌ ܾܽܿ

Ͷܴ ฺ ݎ ൌ ܾܽܿ Ͷ݌ܴ ൌ

ͺܴଷ•‹ •‹ •‹

ͶܴǤܽ ൅ ܾ ൅ ܿʹ ฺ ݎ ൌ Ͷܴ

ଶ•‹ •‹ •‹

ʹܴሺ•‹ ൅ •‹ ൅ •‹ ሻ ൌ ʹܴǤ

ʹ•‹

ʹ•‹

ʹ ൑ ͶܴǤͳ

ͺൌ ܴ ʹ

ܽଶ൅ ܾଶ൅ ܿଶ ൌ Ͷܴଶሺ•‹ଶ ൅ •‹ଶ ൅ •‹ଶሻ ൑ ͶܴଶǤͻ

Ͷൌ ͻܴଶሺככሻ

Từ(*) (**) ta :

Ͷ݌ଶ ൑ ͳͺܴଶ൅ ͳ͸ܴݎ ൅ Ͷݎଶ ֜ ʹ݌ଶ ൑ ͳʹܴଶ൅ ͸ݎଶെ ሺܴ െ ʹݎሻሺ͵ܴ െ ʹݎሻ ൑ ͳʹܴଶ൅ ͸ݎଶ

•‹ •‹ •‹

Ͷ …‘•ʹ …‘• ʹ…‘• ʹ ൌ Ͷܴ •‹

Từđó, ta có điều phải chứng minh

Dấu ̶ ൌ ̶ xảy tam giác e Ta có :

ܵ ൌ ݌ݎ ൌ ඥ݌ሺ݌ െ ܽሻሺ݌ െ ܾሻሺ݌ െ ܿሻ

֜ ݌ଶݎଶ ൌ ݌ሺ݌ െ ܽሻሺ݌ െ ܾሻሺ݌ െ ܿሻ ൌ ݌ሾ݌ଷെ ሺܽ ൅ ܾ ൅ ܿሻ݌ଶ൅ ሺܾܽ ൅ ܾܿ ൅ ܿܽሻ݌ െ ܾܽܿሿ

ൌ ݌ଶሾ݌ଶെ ሺܽ ൅ ܾ ൅ ܿሻ݌ ൅ ሺܾܽ ൅ ܾܿ ൅ ܿܽሻ െ Ͷܴݎሿ

֜ ݎଶ ൌ ݌ଶെ ሺܽ ൅ ܾ ൅ ܿሻ݌ ൅ ሺܾܽ ൅ ܾܽ ൅ ܿܽሻ െ Ͷܴݎ

֜ ݌ଶ ൌ ሺܾܽ ൅ ܾܿ ൅ ܿܽሻ െ ݎሺͶܴ ൅ ݎሻ

Do đó,

Ͷ݌ଶ ൌ ሺܽ ൅ ܾ ൅ ܿሻଶ ൌ ʹሺܽଶ൅ ܾଶ൅ ܿଶሻ ൅ ͶݎሺͶܴ ൅ ݎሻሺכሻ

(99)

ሺ¿ܴ ൒ ʹݎ ֜ ሺܴ െ ʹݎሻሺ͵ܴ െ ʹݎሻ ൒ Ͳሻ

Vậy ta có :

݌ଶ ൑ ͸ܴଶ൅ ͵ݎଶ

Dấu ̶ ൌ ̶ xảy tam giác

Giải:

a Ta có :

ʹ –ƒ ʹ

ݎ ܴ ൌ

Ͷܴ •‹ʹ •‹ʹ •‹ʹ

ܴ ൌ …‘• ൅ …‘• ൅ …‘• െ ͳ

Theo bất đẳng thức Cauchy, ta có :

–ƒ

ʹ•‹ ൅ –ƒ

ʹ•‹ ൒ ʹඨ–ƒ

ʹ•‹ –ƒ ʹ•‹ ƒǤ •‹

ʹ•‹ ʹ൅ •‹

ʹ•‹

ʹ൅ •‹

ʹ•‹

ʹ ൑

ͷ ͺ൅

ݎ Ͷܴ „Ǥݎ௔ଶ൅ ݎ௕ଶ൅ ݎ௖ଶ ൒ ݉௔ଶ ൅ ݉௕ଶ ൅ ݉௖ଶ

…Ǥܽଶ ൑ ܾଶ൅ ܿଶ൅ ܴଶ

†Ǥݎሺݎ௔൅ ݎ௕ ൅ ݎ௖ሻ ൑

Bài 6: Chứng minh tam giác ta có :

(Đề nghị Olympic 30-4, 2010)

–ƒ ʹ –ƒ

ሺ•‹ ൅ •‹ ሻ ൌ …‘• ൅ …‘• ሺ•‹ ൅ •‹ ሻ ൌ …‘• ൅ …‘• ሺ•‹ ൅ •‹ ሻ ൌ …‘• ൅ …‘• ܽଷ൅ ܾଷ൅ ܿଷ

͵ ൅

…‘•ଷܣ ൅ …‘•ଷܤ ൅ …‘•ଷܥ

͸

(Đề nghị Olympic 30-4, 2006)

ሺ0ề nghị Olympic 30-4, 2006ሻ

(100)

Hay •‹ ʹ•‹ ʹ ൑ ͳ Ͷ൬–ƒ ʹ•‹ ൅ –ƒ ʹ•‹ ൰

Tương tự, ta :

•‹ ʹ•‹ ʹ൑ ͳ Ͷ൬–ƒ ʹ•‹ ൅ –ƒ ʹ•‹ ൰ •‹ ʹ•‹ ʹ ൑ ͳ Ͷ൬–ƒ ʹ•‹ ൅ –ƒ ʹ•‹ ൰

Cộng bất đẳng thức trên, ta có :

•‹ ʹ•‹ ʹ൅ •‹ ʹ•‹ ʹ൅ •‹ ʹ•‹ ʹ ൑ …‘• ൅ …‘• ൅ …‘• ʹ ൌ…‘• ൅ …‘• ൅ …‘• Ͷ ൅ ͳ Ͷ൅ ݎ Ͷܴ

Ta lại có bất đẳng thức :

b Ta có :

ݎ௔ଶ൅ ݎ௕ଶ൅ ݎ௖ଶ ൌ ܵଶ ሺ݌ െ ܽሻଶ൅ ൌ ݌ ቈ ൅ሺ݌ െ ܾሻሺ݌ െ ܽሻ ݌ െ ܿ ቉ ݉௔ଶ ൅ ݉௕ଶ൅ ݉௖ଶ ൌ ͵ Ͷ

Ta đặt :

൞ ݔ ൌ ݌ െ ܽ ݕ ൌ ݌ െ ܾ ݖ ൌ ݌ െ ܿ ฺ ൞ ݔ ൅ ݕ ൌ ܿ ݔ ൅ ݖ ൌ ܾ ݕ ൅ ݖ ൌ ܽ ݔ ൅ ݕ ൅ ݖ ൌ ݌

Ta đưa điều cần chứng minh tương đương với

ሺݔ ൅ ݕ ൅ ݖሻ ൬ݕݖ ݔ ൅ ݔݖ ݕ ൅ ݔݕ ݖ ൰ ൒ ͵ Ͷሾሺݔ ൅ ݕሻଶ൅ ሺݕ ൅ ݖሻଶ൅ ሺݖ ൅ ݔሻଶሿ

Thật vậy, ta có :

ݔଶ൬ݕ ݖ൅ ݖ ݕ൰ ൅ ݕଶቀ ݔ ݖ ൅ ݖ ݔቁ ൅ ݖଶ൬ ݔ ݕ൅ ݕ ݔ൰ ൒ ʹሺݔଶ൅ ݕଶ൅ ݖଶሻ Suy ሺݔ ൅ ݕ ൅ ݖሻ ൬ݕݖ ݔ ൅ ݔݖ ݕ ൅ ݔݕ ݖ ൰ ൒ ʹሺݔଶ൅ ݕଶ൅ ݖଶሻ ൅ ݔݕ ൅ ݕݖ ൅ ݖݔ ͵ ʹ …‘• ൅ …‘• ൅ …‘• ൑

Do đó, ta có điều phải chứng minh

Dấu ̶ ൌ ̶ xảy tam giác

(101)

Theo bất đẳng thức Cauchy, ta có :

ͳ

ʹሺݔଶ൅ ݕଶ൅ ݖଶሻ ൒ ͳ

ʹሺݔݕ ൅ ݕݖ ൅ ݖݔሻ

Do đó,

ሺݔ ൅ ݕ ൅ ݖሻ ൬ݕݖ ݔ ൅

ݔݖ ݕ ൅

ݔݕ ݖ ൰ ൒

͵

ʹሺݔଶ൅ ݕଶ൅ ݖଶ൅ ݔݕ ൅ ݕݖ ൅ ݖݔሻ ൌ͵ሾሺݔ ൅ ݕሻଶ൅ ሺݕ ൅ ݖሻଶ൅ ሺݖ ൅ ݔሻଶሿ

ە ۖ ۔ ۖ

ۓܿ ൌ ݎ ൬–ƒ ʹ൅ –ƒ

ʹ൰ ݎ

–ƒʹ൅ ݎ –ƒʹ

–ƒʹ –ƒʹ൅

–ƒʹ

–ƒʹቍ ൒ Ͷݎݎ௖

Tương tự ta có

ܽଶ ൒ Ͷݎݎ

௔˜„ଶ ൒ Ͷݎݎ௕

ฺ ݎሺݎ௔൅ ݎ௕൅ ݎ௖ሻ ൑

ܽଶ൅ ܾଶ൅ ܿଶ

Ͷ

Mặt khác:

Ͷ

Dấu ̶ ൌ ̶ xảy tam giác c Bất đẳng thức tương đương với

Ͷ •‹ଶ ൑ Ͷ •‹ଶ ൅ Ͷ •‹ଶ ൅ ͳ

฻ Ͷሺͳ െ …‘•ଶሻ ൑ ʹሺͳ െ …‘• ʹሻ ൅ ʹሺͳ െ …‘• ʹሻ ൅ ͳ

฻ Ͷ …‘•ଶ െ ʹሺ…‘• ʹ ൅ …‘• ʹሻ ൅ ͳ ൒ Ͳ

฻ Ͷ …‘•ଶ ൅ Ͷ …‘• …‘•ሺ െ ሻ ൅ ͳ ൒ Ͳ

฻ ሾʹ …‘• ൅ …‘•ሺ െ ሻሿଶ൅ •‹ଶሺ െ ሻ ൒ Ͳ

Điều hiển nhiên

Dấu ̶ ൌ ̶ xảy •‹ሺ െ ሻ ൌ ʹ …‘• ൅ …‘•ሺ െ ሻ ൌ Ͳ hay tam giác

cân có góc ͳʹͲ୭

d Ta có :

(102)

ܽଶ൅ ܾଶ൅ ܿଶ

Ͷ ൌ ͳ

Ͷሾሺܽଶ൅ ܾଶെ ܿଶሻ ൅ ሺܾଶ൅ ܿଶെ ܽଶሻ ൅ ሺܿଶ൅ ܽଶെ ܾଶሻሿ ൌͳ

ʹሺܾܽ …‘• ൅ ܾܿ …‘• ൅ ܿܽ …‘• ሻ

Theo bất đẳng thức Cauchy, ta có :

ͳ

ʹሺܾܽ …‘• ൅ ܾܿ …‘• ൅ ܿܽ …‘• ሻ ൑ ͳ

͸ሾʹሺܽଷ൅ ܾଷ൅ ܿଷሻ ൅ …‘•ଷ ൅ …‘•ଷ ൅ …‘•ଷሿ ฺ ܽ

ଶ൅ ܾଶ൅ ܿଶ

Ͷ ൑

ܽଷ൅ ܾଷ൅ ܿଷ

͵ ൅

…‘•ଷܣ ൅ …‘•ଷܤ ൅ …‘•ଷܥ

͸ ฺ ݎሺݎ௔ ൅ ݎ௕ ൅ ݎ௖ሻ ൑

ܽଷ൅ ܾଷ൅ ܿଷ

͵ ൅

…‘•ଷܣ ൅ …‘•ଷܤ ൅ …‘•ଷܥ

͸

Dấu ̶ ൌ ̶ xảy tam giác

ƒǤ …‘•ଷ ൅ …‘•ଷ ൅ …‘•ଷ ൑ ͻ

ͺ൅ ͳ Ͷ „Ǥ •‹ •‹ ൑ …‘•ଶ

ʹ …Ǥ ξ•‹

య య య

ට…‘• ʹ

య య

൅ ට…‘•య ʹ ൑ ͳ †Ǥ Ǥ

…‘•ଶ

Ͷ …‘•ଶͶ

൅ Ǥ …‘•ଶ

Ͷ …‘•ଶͶ

൅ Ǥ …‘•ଶ

Ͷ …‘•ଶͶ ൏ Ͷ ‡Ǥ •‹

ʹ•‹ ʹඨ•‹

ʹ൑

ͳ ͵ξ͵

(ĐH An Ninh Hà Nội 1997)

(ĐHQG Hà Nội 1997)

(ĐH Bách Khoa Hà Nội 2000)

(Đề nghị Olympic 30-4, 2008)

ሺ…‘• ͵ ൅ …‘• ͵ ൅ …‘• ͵ሻ

൅ ξ•‹ ൅ ξ•‹ ൅ ට…‘• ʹ

(103)

Giải:

a Điều cần chứng minh tương đương với :

…‘• ͵ ൅ ͵ …‘• Ͷ ൅

…‘• ͵ ൅ ͵ …‘• Ͷ ൅

…‘• ͵ ൅ ͵ …‘• Ͷ

൑ͻ ͺ൅

ͳ

Ͷሺ…‘• ͵ ൅ …‘• ͵ ൅ …‘• ͵ሻ

Khi ta đưa tốn dạng bất đẳng thức :

…‘• ൅ …‘• ൅ …‘• ൑͵ ʹ

Dấu ̶ ൌ ̶ xảy tam giác b Ta có :

ൌͳ

ʹሾ…‘•ሺ െ ሻ െ …‘•ሺ ൅ ሻሿ ൑ ͳ

ʹ ʹ ൌ …‘•ଶ ʹ ͳ

ʹ ൅ …‘•ଶ

ʹ൅ …‘•ଶ ʹ൰ ൰

య య

൑•‹ ൅ •‹ ʹ ൌ •‹

൅ ʹ …‘•

ʹ ൑ …‘• ʹ

Suy

ξ•‹

൅ ξ•‹ య

ʹ ൑ ඨ…‘• ʹ

Tương tự, ta :

ξ•‹

൅ ξ•‹ య

ʹ ൑ ඨ…‘• ʹ

ሾͳ െ …‘•ሺ ൅ ሻሿ ൌ •‹ଶ ൅

Dấu ̶ ൌ ̶ xảy ൌ

Chú ý: Từbài toán này, ta rút kết sau cách chứng minh tương tự :

൬…‘•ଶ

ʹ •‹ •‹ ൅ •‹ •‹ ൅ •‹ •‹ ൑

Dấu ̶ ൌ ̶ xảy tam giác

c Ta chứng minh bất đẳng thức sau : Với ܽǡ ܾ א Թ ܽ ൅ ܾ ൒ Ͳ,

ܽଷ൅ ܾଷ

൒ ൬ܽ ൅ ܾ ʹ ʹ

Thật vậy, bất đẳng thức tương đương với :

Ͷሺܽଷ൅ ܾଷሻ ൒ ܽଷ൅ ܾଷ൅ ͵ܾܽଶ൅ ͵ܽଶܾ

฻ ሺܽ ൅ ܾሻሺܽଶ൅ ܾଶെ ʹܾܽሻ ൒ Ͳ

฻ ሺܽ ൅ ܾሻሺܽ െ ܾሻଶ ൒ Ͳ

Điều hiển nhiên

Áp dụng bất đẳng thức trên, ta có :

(104)

ξ•‹

൅ ξ•‹ య

ʹ ൑ ඨ…‘• ʹ

Cộng bất đẳng thức trên, ta có điều phải chứng minh Dấu ̶ ൌ ̶ xảy tam giác

d Ta chứng minh

…‘•ଶ

Ͷ

൏ ʹ –ƒ ʹ

Thật vậy, xét hàm số

݂ሺݔሻ ൌ –ƒ ݔ ൅ •‹ ݔ െ ʹݔǡ ݔ א ቀͲǢߨ ʹቁ ฺ ݂ᇱሺݔሻ ൌ ͳ

…‘•ଶݔ൅ …‘• ݔ െ ʹ ൒

ͳ

ξ…‘• ݔ ൐ Ͳ ‘¯×ǡ ݂ሺݔሻ¯ồ‰„‹ế–”² ቀͲǢߨቁ

Hay

൏ ʹ –ƒ ʹ

…‘•ଶ

Ͷ

൏ ʹ –ƒ ʹ

…‘•ଶ

Ͷ

൏ ʹ –ƒ ʹ

Như vậy, ta :

൏ Ͷ ൬–ƒ ʹ–ƒ

ʹ൅ –ƒ

ʹ–ƒ

ʹ൅ –ƒ

ʹ–ƒ

ʹ൰ ൌ Ͷ

e Bất đẳng thức tương đương với

•‹ଶ

ʹ•‹ଶ ʹ•‹

ʹ൑

ͳ ʹ͹

Ta có :

ʹ

Với ݔ ൐ Ͳ ฺ ݂ሺݔሻ ൐ ݂ሺͲሻ ฺ –ƒ ݔ ൅ •‹ ݔ െ ʹݔ ൐ Ͳ

Suy

–ƒ ʹ൅ •‹

ʹെ ൐ Ͳ

…‘•ଶ

Ͷ

(105)

•‹ଶ

ʹ•‹ଶ ʹ•‹

ʹ ൌ

ͳ Ͷ൬…‘•

െ ʹ െ •‹

ʹ൰

•‹ ʹ൑

ͳ

Ͷ൬ͳ െ •‹ ʹ൰

•‹ ʹ ൌͳ

ͺ൬ͳ െ •‹

ʹ൰ ൬ͳ െ •‹

ʹ൰ ʹǤ •‹ ʹ

Theo bất đẳng thức Cauchy, ta có :

൬ͳ െ •‹

ʹ൰ ൬ͳ െ •‹

ʹ൰ ʹǤ •‹ ʹ൑ ቌ

ͳ െ •‹ʹ ൅ ͳ െ •‹ʹ ൅ ʹ •‹ʹ ͵ ቍ

ൌ ͺ ʹ͹

Do đó,

•‹ଶ

ʹ•‹ଶ ʹ•‹

ʹ൑

ͳ ʹ͹

ʹൌ

ͳ ͵Ǥ

Chú ý:Ở tốn này, ta có kết tổng quát sau :

ƒǤξͳ ൅ ʹ …‘•ଶ

•‹ ൅

ξͳ ൅ ʹ …‘•ଶ

•‹ ൒ ͵ξʹ „Ǥͳ ൅ …‘•

ʹ

൅ ൅

ͳ ൅ …‘•ʹ

൐ ͵ξ͵ …Ǥ ͳ

•‹ʹ൅ ͳ •‹ʹ൅

ͳ

•‹ʹെ ൬–ƒ Ͷ൅ –ƒ

Ͷ൅ –ƒ

Ͷ൰ ൒ ͵ξ͵ †Ǥሺͳ െ …‘• ሻሺͳ െ …‘• ሻሺͳ െ …‘• ሻ ൒ …‘• …‘• …‘•

(ĐH Bách Khoa Hà Nội 1999)

(ĐHQG Hà Nội 2000)

•‹ ʹ•‹

ʹ ඨ•‹

ʹ

ấ—̶ ൌ ̶šả›”ƒŠ‹˜…ŠỉŠ‹–ƒ‰‹ž……Ÿ–ạ‹…׉ׅ–Šỏƒ •‹

൅ξͳ ൅ ʹ …‘•ଶ •‹ ͳ ൅ …‘• ʹ

Bài 8: Trong tam giác , chứng minh :

൑ ݊ ʹሺ݊ ൅ ͳሻ೙

(106)

Giải:

a Theo bất đẳng thức Bunyakovsky, ta có :

ͳ ൅ ʹ …‘•ଶ ൌ ͳ

ʹ൅ ͳ

ʹ൅ ʹ …‘•ଶ ൒ ͳ ͵൬

ͳ ξʹ൅

ͳ

ξʹ൅ ξʹ …‘• ൰

ฺ ͳ ൅ ʹ …‘•ଶ ൒ʹ

͵ሺͳ ൅ …‘• ሻଶ ൌ ͺ ͵…‘•ସ

ʹ

Tương tự, ta có :

ͳ ൅ ʹ …‘•ଶ ൒ͺ

͵…‘•ସ ʹ ͳ ൅ ʹ …‘•ଶ ൒ͺ

͵…‘•ସ ʹ

Theo bất đẳng thức Cauchy, ta có :

൒ ͵ඨξͳ ൅ ʹ …‘•ଶ

•‹ Ǥ Ǥ

൒ ͵ඪ

ͳ͸ξʹ

ൌ ͵ඨʹξʹ ͵ξ͵…‘–

ʹ…‘–

ʹ…‘–

ʹ

Mặt khác :

…‘–

ʹ ൌ …‘– ʹ…‘–

ʹ…‘–

ʹ൒ ͵ξ͵

Do đó,

ൌ ͵ξʹ …‘• ݔ ൐ ͳ െݔ

ʹ ǡ ׊ݔ ൐ Ͳ

Thật vậy, xét hàm số

݂ሺݔሻ ൌ …‘• ݔ ൅ݔ

ʹ െ ͳ ݂ᇱሺݔሻ ൌ െ •‹ ݔ ൅ ݔ

Đặt ݃ሺݔሻ ൌ ݂ᇱሺݔሻ,

݃ᇱሺݔሻ ൌ െ …‘• ݔ ൅ ͳ ൐ Ͳ

Do đó, ݃ሺݔሻ đồng biến Suy

݃ሺݔሻ ൐ ݃ሺͲሻ ൌ Ͳ ξͳ ൅ ʹ …‘•ଶ

•‹

ξͳ ൅ ʹ …‘•ଶ

•‹ ͵ξ͵ …‘•ଶ

ʹ …‘•ଶʹ…‘•ଶʹ ͺ •‹ʹ …‘•ʹ •‹ ʹ…‘• ʹ•‹ ʹ…‘• ʹ

൅ …‘– ʹ൅ …‘–

ʹ ൒ ͵ටʹξʹ

(107)

Ta có ݂ሺݔሻđồng biến Suy

݂ሺݔሻ ൐ ݂ሺͲሻ ൌ Ͳ

Vậy bất đẳng thức

Áp dụng vào toán, ta :

…‘•

ʹ ൐ ͳ െ ଶ

ͺ ฺ

ͳ ൅ …‘•ʹ ʹ ൐

ʹ െ

ͺ

Tương tự, ta có :

ͳ ൅ …‘•ʹ ʹ ൐

ʹ െ

ͺ ͳ ൅ …‘•ʹ

ʹ ൐ ʹ െ

ͺ

Do đó,

൐ ʹ ൬ͳ ൅

ͳ ൅

ͳ ൰ െ

Ta có bất đẳng thức :

ሺ ൅ ൅ ሻ ൬ͳ ൅

ͳ

ͳ ൅

ͳ ൅

ͳ ൒

ͻ ߨ

Vậy

െߨ

ͺ ൐ ͵ξ͵ ͳ

•‹ʹെ …‘–

ʹ ൌ –ƒ Ͷ

Do đó,

ͳ

•‹ʹെ …‘–

ʹ ൌ –ƒ Ͷ ͳ

•‹ʹെ …‘–

ʹൌ –ƒ Ͷ

Suy

ൌ …‘– ʹ൅ …‘–

ʹ൅ …‘–

ʹ൒ ͵ξ͵

Dấu ̶ ൌ ̶ xảy tam giác

൅ ൅ ͺ ൅ͳ

൰ ൒ ͻ ฺ ൐ͳͺ

ߨ

(108)

d Ta có trường hợp :

- Nếu tam giác vuông tù bất đẳng thức hiển nhiên

- Nếu tam giác nhọn :

Điều cần chứng minh tương đương với :

ቌͳ െͳ െ –ƒ ଶ ʹ ͳ ൅ –ƒଶ ʹ ቍ ቌͳ െͳ െ –ƒ ଶ ʹ ͳ ൅ –ƒଶ ʹ ቍ ቌͳ െͳ െ –ƒ ଶ ʹ ͳ ൅ –ƒଶ ʹ ቍ ൒ͳ െ –ƒ ଶ ʹ ͳ ൅ –ƒଶ ʹ Ǥͳ െ –ƒ ଶ ʹ ͳ ൅ –ƒଶ ʹ Ǥ Hay ͺ –ƒଶ ʹ–ƒଶ ʹ–ƒଶ ʹ൒ ൬ͳ െ –ƒଶ ʹ ʹ൰ ‘ Ͳ ൏ ʹǡ ʹǡ ʹ ൏ ߨ ʹ ฺ –ƒ ʹǡ –ƒ ʹ

Áp dụng công thức

Ta đưa toán trở thành :

ʹ ͳ –ƒ Ǥ ͳ –ƒ ʹ…‘– ʹ…‘– ʹ

Mặt khác :

–ƒ ൅ –ƒ ൌ •‹ሺ ൅ ሻ …‘• …‘• ൌ ʹ •‹ …‘•ሺ ൅ ሻ ൅ …‘•ሺ െ ሻ ൒ ʹ •‹ ͳ െ …‘• ൌ ʹ …‘– ʹ

Tương tự, ta có :

(109)

ʹሺ–ƒ ൅ –ƒ ൅ –ƒ ሻ ൒ ʹ ൬…‘– ʹ൅ …‘–

ʹ൅ …‘–

ʹ൰ ฻ –ƒ ൅ –ƒ ൅ –ƒ ൒ …‘–

ʹ൅ …‘– ʹ൅ …‘–

ʹൌ …‘–

ʹ…‘–

ʹ…‘–

ʹ

Dấu ̶ ൌ ̶ xảy tam giác

Giải:

a Ta có :

ൌ ʹ ൅ •‹ ʹ ൅ …‘•

ൌʹ …‘•

ʹ …‘• െ ʹ ൅ •‹ ʹ •‹ʹ …‘• െ ʹ ൅ …‘•

Ta chứng minh

ʹ …‘•ʹ …‘• െ ʹ ൅ •‹ ʹ •‹ʹ …‘• െ ʹ ൅ …‘• ൒

ʹ …‘•ʹ ൅ •‹ ʹ •‹ʹ ൅ …‘•

Thật vậy, điều tương đương với

൬ʹ …‘• ʹ…‘•

ʹ ൅ •‹ ൰ ൬ʹ •‹

ʹ൅ …‘• ൰ ൒ ൬ʹ •‹

ʹ…‘• െ

ʹ ൅ …‘• ൰ ൬ʹ …‘•

ʹ൅ •‹ ൰ ฻ ʹ …‘•

ʹ…‘• …‘•

ʹ ൅ ʹ •‹ •‹

ʹ൒ ʹ …‘•

ʹ…‘• ൅ ʹ •‹ •‹ ʹ…‘•

െ ʹ ƒǤ •‹ ൅ •‹ ൅ •‹

…‘• ൅ …‘• ൅ …‘• ൒ ͳ ൅ ξʹ

ʹ „Ǥ –ƒ௡ ൅ –ƒ௡ ൅ –ƒ௡ ൒ ͵ ൅͵݊

ʹ ǡ ݊ א Գ …Ǥ͵ •‹ସ

ʹ൅ ͻ –ƒ଼

ʹ൅ ͺ •‹଺

ʹ ൅ ʹ –ƒସ ʹ൒

͹͸͹ ͺ͸Ͷ

Bài 9: Cho tam giác nhọn, chứng minh :

(Đề nghị Olympic 30-4, 2008)

ʹ •‹ ൅ʹ …‘• െ ʹ …‘• ൅ʹ …‘• െ

൅ Ͷ –ƒ଺

ʹ൅ ʹͶ •‹଼ ʹ †ǤయඥͶୱ୧୬ ୅ାୱ୧୬୆ାୱ୧୬ େ൅ యඥʹ୲ୟ୬ ୅ା୲ୟ୬ ୆ା୲ୟ୬େ ൐ ʹଵାగଶ

(110)

฻ ൬…‘• െ

ʹ െ ͳ൰ ൬…‘• …‘•

ʹെ •‹ •‹ ʹ൰ ൒ Ͳ ฻ ൬…‘• െ

ʹ െ ͳ൰ …‘• ͵

ʹ ൒ Ͳ

Giả sử ൌ ƒšሼǡ ǡ ሽ

ฺ א ቂߨ ͵ǡ

ߨ ʹቃ ฺ

͵ ʹ א ൤

ߨ ʹǡ

͵ߨ

Ͷ ൨ ฺ …‘• ͵

ʹ ൑ Ͳ

Do đó, bất đẳng thức hiển nhiên

Ta xét hàm số :

݂ሺݔሻ ൌ ʹ …‘• ݔ

ʹ ൅ •‹ ݔ

ʹ •‹ݔʹ ൅ …‘• ݔǡ ݔ א ቂ ߨ ͵ǡ

ߨ ʹቃ ݂ᇱሺݔሻ ൌ •‹

͵ݔ ʹ െ ͳ ቀʹ •‹ݔʹ ൅ …‘• ݔቁଶ

ǡ

Suy ݂ሺݔሻ nghịch biến Do đó,

݂ሺݔሻ ൒ ݂ ቀߨ ʹ

Từđó, ta có :

൒ ͳ ൅ξʹ ʹ

൒ ͵ ൬ͳ ൅ͳ ʹ൰

͵ ൬ͳ ൅ͳ ʹ൰

൒ ͵ ቀͳ ൅݊ ʹቁ

Do đó,

–ƒ௡ ൅ –ƒ௡ ൅ –ƒ௡ ൒ ͵ ൅͵݊

ʹ

Dấu ̶ ൌ ̶ xảy ݊ ൌ Ͳ c Theo bất đẳng thức Cauchy, ta có :

͵ •‹ସ

ʹ൅ ͵ ൬ ͳ ʹ൰

൒͵ ʹ•‹ଶ

ʹ ͺ •‹଺

ʹ൅ ͺ ൬ ͳ ʹ൰

൅ ͺ ൬ͳ ʹ൰

൒͵ ʹ•‹ଶ

ʹ

൑ Ͳǡ ׊ݔ א ቂߨ ͵

ߨ ʹቃ ቁ ൌ ͳ ൅ξʹ

ʹ •‹ ൅ •‹ ൅ •‹ …‘• ൅ …‘• ൅ …‘•

Dấu ̶ ൌ ̶ xảy tam giác vuông cân b Theo bất đẳng thức Cauchy bất đẳng thức bản, ta có :

–ƒ௡ ൅ –ƒ௡ ൅ –ƒ௡ ൒ ͵య

ඥሺ–ƒ –ƒ –ƒ ሻ௡ ൒ ͵ට൫͵ξ͵൯

(111)

ʹͶ •‹଼ ʹ൅ ʹͶ ൬ ͳ ʹ൰ ଼ ൅ ʹͶ ൬ͳ ʹ൰ ଼ ൅ ʹͶ ൬ͳ ʹ൰ ଼ ൒ ͵ ʹ•‹ଶ ʹ ͻ –ƒ଼ ʹ൅ ͻ ൬ ͳ ξ͵൰ ଼ ൅ ͻ ൬ ͳ ξ͵൰ ଼ ൅ ͻ ൬ ͳ ξ͵൰ ଼ ൒Ͷ ͵–ƒଶ ʹ Ͷ –ƒ଺ ʹ ൅ Ͷ ൬ ͳ ξ͵൰ ଺ ൅ Ͷ ൬ ͳ ξ͵൰ ଺ ൒ Ͷ ͵–ƒଶ ʹ ʹ –ƒସ ʹ൅ ʹ ൬ ͳ ξ͵൰ ସ ൒ Ͷ ͵–ƒଶ ʹ

Mặt khác, theo bất đẳng thức bản, ta có :

͵ ʹ൬•‹ଶ ʹ൅ •‹ଶ ʹ൅ •‹ଶ ʹ൰ ൒ ͻ ͺ Ͷ ͵൬–ƒଶ ʹ൅ –ƒଶ ʹ൅ –ƒଶ ʹ ଶ ଷ ଵଷ ଶ ଷ ฻ ʹ ͵ሺ•‹ ൅ •‹ ൅ •‹ ሻ ൅ ฻ ൬ʹ ͵•‹ ൅ ͳ ͵ ͵ ʹ ͵•‹ ൅ ͳ ͵–ƒ െ ൰ ൐ Ͳ

Xét hàm số

݂ሺݔሻ ൌ ʹ ͵•‹ ݔ ൅ ͳ ͵–ƒ ݔ െ ݔǡ ݔ א ቀͲǡ ߨ ʹቁ ݂ᇱሺݔሻ ൌʹ ͵…‘• ݔ ൅ ͳ ͵Ǥ ͳ …‘•ଶݔെ ͳ ൌ ͳ ͵൬…‘• ݔ ൅ …‘• ݔ ൅ ͳ …‘•ଶݔ൰ െ ͳ ൒ ͳ ͵Ǥ ͵ െ ͳ ൌ Ͳ

Ta thấy dấu ̶ ൌ ̶ bất đẳng thức

…‘• ݔ ൅ …‘• ݔ ൅ ͳ

…‘•ଶݔ ൒ ͵

khơng thể xảy

Do đó, ݂ሺݔሻ đồng biến Nên ݂ሺݔሻ ൐ ݂ሺͲሻ Suy

ʹ

•‹ ݔ ൅ͳ–ƒ ݔ െ ݔ ൐ Ͳ ൰ ൒ Ͷ

͵

Cộng bất đẳng thức trên, ta có điều phải chứng minh Dấu ̶ ൌ ̶ xảy tam giác

d Theo bất đẳng thức Cauchy, ta có :

ଶ ଷ

ൌ ʹ ሺୱ୧୬ ୅ାୱ୧୬ ୆ାୱ୧୬ େሻ൅ ʹ ሺ୲ୟ୬ ୅ା୲ୟ୬ ୆ା୲ୟ୬ େሻ൒ ʹටʹ ሺୱ୧୬ ୅ାୱ୧୬ ୆ାୱ୧୬ େሻାଵଷሺ୲ୟ୬୅ା୲ୟ୬୆ା୲ୟ୬େሻ

Ta cần chứng minh :

(112)

Lần lượt thay ݔ ൌ ሼǡ ǡ ሽ Ta có điều phải chứng minh

൜ •‹ ൌ ݖ ݕ ൌ ݔ •‹

•‹

฻ ൜ݔ •‹ ൌ ݕ •‹ ݔ •‹ ൌ ݖ •‹ ƒǤܽ ൅ ܾ ൅ ܿ ൑ ൅ܿ

൰ „Ǥ ܽ

݉௔൅

ܾ ݉௕൅

ܿ

݉௖ ൒ ʹξ͵

…Ǥ ൬ ܽ …‘• ൅

ܾ

…‘• െ ܿ൰ ൬ ܾ …‘• ൅

ܿ

…‘• െ ܽ൰ ቀ ܿ …‘• ൅

ܽ

…‘• െ ܾቁ ൒ ʹ͹ܾܽܿ †Ǥ …‘• …‘•

…‘• ቀ െ ʹ ቁ൅

…‘• …‘• …‘• ቀ െ ʹ ቁ൅

…‘• …‘• …‘• ቀ െ ʹ ቁ ൑

͵ Ͷ

(Đề nghị Olympic 30-4, 2006)

…‘• ݔ ൅

…‘• ݕ ൅

…‘• ݖ ൑

ݔ ʹݕݖ൅

ݕ ʹݖݔ൅

ݖ ʹݔݕ

Bài 10: Cho tam giác số thực ݔǡ ݕǡ ݖ ൐ Ͳ

Hãy chứng minh

Giải:

Bất đẳng thức cần chứng minh tương đương với

ʹݕݖ …‘• ൅ ʹݔݖ …‘• ൅ ʹݔݕ …‘• ൑ ݔଶ൅ ݕଶ൅ ݖଶ

฻ ʹݕݖ …‘• ൅ ʹݔݖ …‘• ൅ ʹݔݕ …‘•

൑ ݔଶሺ…‘•ଶ ൅ •‹ଶሻ ൅ ݕଶሺ…‘•ଶ ൅ •‹ଶሻ ൅ ݖଶ

฻ ʹݕݖ …‘• ൅ ʹݔݖ …‘• െ ʹݔݕሺ…‘• …‘• െ •‹ •‹ ሻ ൑ ݔଶሺ…‘•ଶ ൅ •‹ଶሻ ൅ ݕଶሺ…‘•ଶ ൅ •‹ଶሻ ൅ ݖଶ

฻ ሺݔଶ•‹ଶ െ ʹݔݕ •‹ •‹ ൅ ݕଶ•‹ଶሻ

൅ሺݔଶ…‘•ଶ ൅ ݕଶ…‘•ଶ ൅ ݖଶ൅ ʹݔݕ …‘• …‘• െ ʹݔݖ …‘• െ ʹݕݖ …‘• ሻ ൒ Ͳ

฻ ሺݔ •‹ െ ݕ •‹ ሻଶ൅ ሺݔ …‘• ൅ ݕ …‘• െ ݖሻଶ ൒ Ͳ

Điều hiển nhiên Do đó, ta có điều phải chứng minh Dấu ̶ ൌ ̶ xảy

ݔ •‹ െ ݕ •‹ ൌ Ͳ

ݔ …‘• ൅ ݕ …‘• െ ݖ ൌ Ͳ ฻ ൞

ݔ …‘• ൅ݔ •‹ ฻ ݔǣ ݕǣ ݖ ൌ ܽǣ ܾǣ ܿ ߨ

͵൬ ܽ ൅

ܾ

(113)

Giải:

a Giả sửܽ ൑ ܾ ൑ ܿ ฺ ൑ ൑ ൑ ͻͲ୭.

Ta xét hàm số

݂ሺݔሻ ൌ •‹ ݔ

ݔ ǡ ݔ א ቀͲǢ ߨ ʹቁ ݂ᇱሺݔሻ ൌ ݔ െ –ƒ ݔ

ݔଶ…‘• ݔ

Lại xét hàm số

݃ሺݔሻ ൌ –ƒ ݔ െ ݔǡ ݔ א ቀͲǢߨ ʹቁ ݃ᇱሺݔሻ ൌ –ƒଶݔ ൐ Ͳ

ܽ ൒

ܾ

ܿ

Theo bất đẳng thức Chebyshev cho dãy

൝ܽ ൒

ܾ

ܿ ൅ ൅

͵ Ǥ ܽ

൅ ൅ܾ ܿ ͵ ൒

ܽ

͵ ൌ

ܽ ൅ ܾ ൅ ܿ ͵

b Ta có :

ൌ ܴଶሾͳ െ ሺ…‘• ʹ ൅ …‘• ʹሻ ൅ …‘•ଶሿ

ൌ ܴଶሾͳ ൅ ʹ …‘• …‘•ሺ െ ሻ ൅ …‘•ଶሿ

Suy

݉௔ଶ ൑ ܴଶሺͳ ൅ ʹ …‘• ൅ …‘•ଶሻ ൌ ܴଶሺͳ ൅ …‘• ሻଶ ൌ Ͷܴଶ…‘•ସ

ʹ ฺ ݉௔ଶ ൑ ʹܴ …‘•ଶ

ʹ

Theo định lý hàm số sin, ta có :

ܽ ݉௔ ൒

ܽ ʹܴ …‘•ଶ

ʹ

ൌ •‹ …‘•ଶ

ʹ

ൌ ʹ –ƒ ʹ

Do đó, ݃ሺݔሻ đồng biến Suy

݃ሺݔሻ ൐ ݃ሺͲሻ ฻ –ƒ ݔ ൐ ݔ ฻ ݂ᇱሺݔሻ ൏ Ͳ

Ta có ݂ሺݔሻ nghịch biến Suy

൒ ൑ ൑

Ǥ ൅ܾ Ǥ ൅ Ǥ ܿ

Vậy ta có điều phải chứng minh

Dấu ̶ ൌ ̶ xảy tam giác

݉௔ ൌͳ

(114)

ە ۔ ۓ ܾ

݉௕ ൒ ʹ –ƒ

ʹ ܿ

݉௖ ൒ ʹ –ƒ

ʹ

Cộng bất đẳng thức trên, ta có :

൒ ʹ ൬–ƒ ʹ൅ –ƒ

ʹ൅ –ƒ

ʹ൰

Mặt khác, theo bất đẳng thức bản, ta có :

–ƒ ʹ൅ –ƒ

ʹ൅ –ƒ ʹ൒ ξ͵

Do đó, ta có điều phải chứng minh

Dấu ̶ ൌ ̶ xảy tam giác

൬ʹܴ •‹ …‘• ൅

ʹܴ •‹

…‘• െ ʹܴ •‹ ൰ ൬

ʹܴ •‹

…‘• ൅ െ ʹܴ •‹ ൰ ൬

ʹܴ •‹ …‘• ൅ʹܴ •‹

…‘• ฻ ൬ •‹

…‘• …‘• െ •‹ ൰ ൬ െ •‹ ൰ ฻ ͳ െ …‘• …‘•

…‘• …‘• Ǥ Ǥ ൒ ʹ͹

Ta cần chứng minh

ൌ ʹ ቀ–ƒ

ʹ ൅ –ƒଶʹቁ ቀͳ െ –ƒଶ

ʹቁ ቀͳ െ –ƒଶʹቁ

Thật vậy,

ʹ ቀ–ƒଶ

ʹ ൅ –ƒଶʹቁ ቀͳ െ –ƒଶ

ʹቁ ቀͳ െ –ƒଶʹቁ

ൌʹ ቀ–ƒ

ʹ ൅ –ƒଶʹቁ …‘•ଶʹ …‘•ଶʹ …‘• …‘•

ൌʹ ቀ•‹

ʹ …‘•ଶʹ ൅ •‹ଶʹ …‘•ଶʹቁ …‘• …‘•

ൌሺͳ െ …‘• ሻሺͳ ൅ …‘• ሻ ൅ ሺͳ െ …‘• ሻሺͳ ൅ …‘• ሻ ʹ …‘• …‘• ൌ

ͳ െ …‘• …‘• …‘• …‘•

Theo bất đẳng thức Cauchy, ta có :

c Theo định lý hàm sốsin, điều cần chứng minh tương đương với

ʹܴ •‹ …‘• …‘• …‘• െ •‹ ൰ ൬

െ ʹܴ •‹ ൰ ൒ ʹ͹Ǥʹܴ •‹ Ǥ ʹܴ •‹ Ǥ ʹܴ •‹ •‹ •‹

…‘• …‘• ൒ ʹ͹ •‹ •‹ •‹ ͳ െ …‘• …‘•

…‘• …‘•

ͳ െ …‘• …‘• …‘• …‘• ͳ െ …‘• …‘•

(115)

ͳ െ …‘• …‘• …‘• …‘• ൌ

ʹ ቀ–ƒଶ

ʹ ൅ –ƒଶʹቁ ቀͳ െ –ƒଶ

ʹቁ ቀͳ െ –ƒଶʹቁ

൒ ʹ –ƒ

ʹ ͳ െ –ƒଶ

ʹ

Ǥ ʹ –ƒ

ʹ ͳ െ –ƒଶ

ʹ

ൌ –ƒ –ƒ

Tương tự, ta có :

ͳ െ …‘• …‘•

…‘• …‘• ൒ –ƒ –ƒ ͳ െ …‘• …‘•

…‘• …‘• ൒ –ƒ –ƒ

Như

ͳ െ …‘• …‘• …‘• …‘• Ǥ

ͳ െ …‘• …‘• …‘• …‘• Ǥ

ͳ െ …‘• …‘•

…‘• …‘• ൒ –ƒଶ –ƒଶ –ƒଶ

Mà theo bất đẳng thức bản, ta có :

d Theo bất đẳng thức Cauchy, ta có :

ʹ …‘• …‘• …‘• ቀ െ ʹ ቁ ൌ Ͷ

…‘• …‘• …‘• ൅ …‘• Ǥ •‹

ʹ

ʹ ൑ …‘• …‘• ൅ •‹ଶ ʹ

Tương tự, ta có :

ʹ

ʹ ʹ െ

ʹ ቁ

൑ …‘• …‘• ൅ •‹ଶ

ʹ

ʹ

ʹ൅ …‘• …‘• ൅ …‘• …‘• ൅ …‘• …‘• ൑ͳ

ʹሾ͵ െ ሺ…‘• ൅ …‘• ൅ …‘• ሻሿ ൅ ͳ

͵ሺ…‘• ൅ …‘• ൅ …‘• ሻଶ

Áp dụng bất đẳng thức bản, ta có :

ሺ…‘• ൅ …‘• ൅ …‘• ሻଶ ൑͵

ʹሺ…‘• ൅ …‘• ൅ …‘• ሻ

Do đó,

ʹ ൑ ͳ

ʹሺ…‘• ൅ …‘• ൅ …‘• ሻ ൅ ͳ

ʹሾ͵ െ ሺ…‘• ൅ …‘• ൅ …‘• ሻሿ ൌ ͵ ʹ

Vậy ta có điều phải chứng minh

Dấu ̶ ൌ ̶ xảy tam giác

–ƒ –ƒ –ƒ ൒ ͵ξ͵ ฺ –ƒଶ –ƒଶ –ƒଶ ൒ ʹ͹

Vậy ta có điều phải chứng minh

Dấu ̶ ൌ ̶ xảy tam giác

൑ ʹξ…‘• …‘• •‹ …‘• …‘•

…‘• ቀെ ʹ ቁ ൑ …‘• …‘• ൅ •‹

…‘• …‘• …‘• ቀ

Cộng bất đẳng thức trên, ta :

ʹ൅ •‹ଶ

(116)

Giải: Ta có :

ʹ ൅ ͵ ൌ ߨ ฺ ൌ ߨ ʹെ

͵

ʹ ฺ ൌ ߨ ʹ൅

ʹ

Suy

•‹ ൌ •‹ ൬ߨ ʹെ

͵

ʹ ൰ ൌ …‘• ͵

ʹ •‹ ൌ •‹ ൬ߨ

ʹ൅ ʹ

Theo định lý hàm số sin, ta có :

ܽ •‹ ൌ

ܾ •‹ ൌ

ܿ •‹ ฻ ܽ

…‘•ʹ ቀͶ …‘•ଶ

ʹ െ ͵ቁ

ൌ ܾ ൌ ฻ ܽ ൌ ܾ ൌ ܿ

ܽ ൅ ܾ Ͷ …‘•ଶ

ʹ െ ͵ ൅ ʹ •‹ʹ

Suy

ܽ ൅ ܾ ൌ ܿ ൬െͶ •‹ଶ

ʹ൅ ʹ •‹ ʹ൅ ͳ൰

Mặt khác

ʹ ൅ ͵ ൌ ߨ ฺ ʹ ൌ

ߨ ͸െ

͵ ฺ Ͳ ൏ ʹ ൏

ߨ

͸ ฺ Ͳ ൏ •‹ ʹ ൏

ͳ ʹ 0ặ–ݐ ൌ •‹

ʹǡ –ƒš±–Š•ố

݂ሺݐሻ ൌ െͶݐଶ൅ ʹݐ ൅ ͳǡ ݐ א ൬ͲǢͳ

ʹ൰ ݂ᇱሺݐሻ ൌ െͺݐ ൅ ʹ

ܽ ൅ ܾ ൏ͷ Ͷܿ

Bài 13: Chứng minh góc tam giác nhọn thỏa điều kiện ʹ ൅ ͵ ൌ ߨ cạnh thỏa mãn

(Đề nghị Olympic 30-4, 2008)

ൌ …‘•

ʹ൬Ͷ …‘•ଶ ʹെ ͵൰ ൰ ൌ …‘•

ʹ ʹ •‹ ʹ…‘• ʹ

ܿ …‘• ʹ Ͷ …‘•ଶ

ʹ െ ͵

ʹ •‹ ʹ

Theo tính chất tỷ lệ thức, ta có :

(117)

݂ᇱሺݐሻ ൌ Ͳ ฻ ݐ ൌͳ

Ͷ ݐ Ͳ ଵ

ଵ ଶ ݂Ԣሺݐሻ ൅ Ͳ െ

݂ሺݐሻ

ହ ସ

ͳ ͳ

Dựa vào bảng biến thiên, ta rút kết

ʹ

ʹ Ͷ

ߨ

Ͷ ʹ൏ •‹ ฺ Ͳ ൏ •‹

ʹ൏ •‹ ൏ …‘• ฺ ͳ •‹ʹ ൐

ͳ …‘• ¿ …‘• ൏ Ͳǡ …‘• ൐ Ͳ˜ …‘• െ

ʹ ൐ Ͳ ൬Ͳ ൏

െ ʹ ൏

ߨ ʹ൰ ² ͳ

…‘• ൅ ͳ …‘• ൌ

ʹ •‹ʹ …‘• െ ʹ

…‘• …‘• ൏ Ͳ ൏ ͳ •‹ʹ൅

ͳ •‹ʹ

Do đó, chiều thuận

Chiều nghịch: Giả sử tam giác nhọn Ta suy …‘• ǡ …‘• ǡ …‘• ൐ Ͳ

Theo bất đẳng thức Cauchy, ta có :

ͳ …‘• ൅

ͳ

…‘• ൅ ൅ ͳ •‹ʹ൅

ͳ •‹ʹ

khi

(Đề nghị Olympic 30-4, 2009)

ͷ ܽ ൅ ܾ ൌ ܿ ൬െͶ •‹ଶ ൅ ʹ •‹ ൅ ͳ൰ ൑ ܿ

Tuy nhiên, dấu ̶ ൌ ̶ khơng xảy ෠ ൐ ͻͲ୭ Do đó, ta cóđiều phải chứng minh

Giải:

Chiều thuận: Giả sử tam giác có góc tù ൐ ܤ ൒ ܥ Khi

Ͳ ൏ –ƒ ൏ ͳ Ͳ ൏ ൏ ฺ ൝Ͳ ൏ •‹

ͳ …‘• ൏

ͳ •‹ʹ

(118)

ͳ …‘• ൅

ͳ …‘• ൒

ʹ

ξ…‘• …‘• ൒

Ͷ

…‘• ൅ …‘• ൌ

ʹ

•‹ʹ …‘• െ ʹ ൒ ʹ •‹ʹ

Tương tự, ta có ;

ͳ …‘• ൅

ͳ …‘• ൒

ʹ •‹ʹ ͳ

…‘• ൅ ͳ …‘• ൒

ʹ •‹ʹ

Do đó,

ͳ …‘• ൅

ͳ …‘• ൅

ͳ …‘• ൒

ͳ •‹ʹ൅

ͳ •‹ʹ൅

ͳ •‹ʹ

Điều vơ lí Vậy chiều nghịch

Vậy ta có điều phải chứng minh

•‹ ൑ξ͵ ʹ ܽ

Áp dụng hàm số sin

ܽ •‹ ൌ

ܾ

•‹ ֜ ܽ ൌ

ܾ •‹ •‹ ൌ

ξ͵ •‹ •‹

Áp dụng hàm số cos

ଶ ଶ ଶ

ܿ ܾ ൌ

ͳ

ξ͵ ฺ ݄௔ ൑ ξ͵

ʹ ܽ

Bài 15: Cho tam giác chứng minh

(Đề nghị Olympic 30-4, 2010)

Giải:

Khơng tính tổng quát nên ta giả sửܿ ൌ ͳǢ ܾ ൌ ξ͵

Ta có: ݄௔ ൌ ܿ •‹ ൌ •‹

(119)

Ta có :

•‹ ൅ ξ͵ …‘• ൌ ʹ •‹ ቀ ൅ߨ

͵ቁ ൑ ʹ ֜ •‹ ൑ ʹ െ ξ͵ …‘• ֜ ʹ •‹ ൑ ܽଶ ֜ ʹ •‹ ൑ ܽξ͵ •‹

•‹ ֜ •‹ ൑ ξ͵

ʹ ܽ

Vậy ta có điều phải chứng minh

Giải: Ta có

ܵ ൌ ඥ݌ሺ݌ െ ܽሻሺ݌ െ ܾሻሺ݌ െ ܿሻ ൌ ඨ ͳ

ʹെ ܾ൰ ൬ ͳ ʹെ ܿ൰

ͳ͸ܵଶ ൌ െͳ െ ʹሺܽଶ൅ ܾଶ൅ ܿଶሻ െ ͺܾܽܿ ൐ Ͳ

Vậy

ܽଶ ൅ ܾଶ൅ ܿଶ൅ Ͷܾܽܿ ൏ͳ

ʹ ܽଶ൅ ܾଶ൅ ܿଶ൅ Ͷܾܽܿ ൏ͳ

ʹ

Bài 16: Cho tam giác có chu vi ͳ Chứng minh :

ͳ ʹ൬

ͳ

ʹെ ܽ൰ ൬

֜ ͳ͸ܵଶ ൌ ሺͳ െ ʹܽሻሺͳ െ ʹܾሻሺͳ െ ʹܿሻ ൌ ͳ െ ʹሺܽ ൅ ܾ ൅ ܿሻ ൅ Ͷሺܾܽ ൅ ܾܿ ൅ ܿܽሻ െ ͺܾܽܿ

ൌ െͳ ൅ Ͷሺܾܽ ൅ ܾܿ ൅ ܿܽሻ െ ͺܾܽܿ

Mặt khác

ʹሺܾܽ ൅ ܾܿ ൅ ܿܽሻ ൌ ሺܽ ൅ ܾ ൅ ܿሻଶെ ሺܽଶ൅ ܾଶ൅ ܿଶሻ

֜ Ͷሺܾܽ ൅ ܾܿ ൅ ܿܽሻ ൌ ʹ െ ʹሺܽଶ൅ ܾଶ൅ ܿଶሻ

Suy

(120)

Giải:Theo định lý hàm sốsin, ta có điều cần chứng minh tương đương với

͵ܽଶ ൅ ͵ܾଶ൅ ͵ܿଶ൅ Ͷܾܽܿ ൒ ͳ͵

Giả sửܽ ൑ ܾ ൑ ܿ Ta có : ܽ ൅ ܾ ൅ ܿ ൌ ͵ nên

൜͵ܿ ൒ ܽ ൅ ܾ ൅ ܿ ൌ ͵ܽ ൅ ܾ ൐ ܿ ฻ ቄ͵ െ ܿ ൐ ܿܿ ൒ ͳ ฻ ͳ ൑ ܿ ൏͵ ʹ

Mặt khác

ە ۔

ۓܾܽ ൏ ൬ܽ ൅ ܾ ʹ ൰

ൌ ൬͵ െ ܿ ʹ ൰

ܿ ൏ ͵ ʹ

͵ െ ʹܿ ൐ Ͳ

Nên

͵ െ ܿ ʹ ൰

ሺ͵ െ ʹܿሻ

Hay

͵ ʹܿଶ൅

ʹ͹ ʹ

Xét hàm số

݂ሺܿሻ ൌ ܿଷെ͵

ʹܿଶ൅ ʹ͹

ʹ ǡ ܿ א ൤ͳǢ ͵ ʹ൰ ݂ᇱሺܿሻ ൌ ͵ܿଶെ ͵ܿ

݂ᇱሺܿሻ ൌ Ͳ ฻ ܿ ൌ ͳ

ܿ ͳ ଷ

ଶ ݂Ԣሺܿሻ Ͳ ൅

݂ሺܿሻ

ଶ଻ ଶ ͳ͵

͵ሺ•‹ଶ ൅ •‹ଶ ൅ •‹ଶሻ ൅ ͺܴ •‹ •‹ •‹ ൒ ͳ͵

Ͷܴଶ

Bài 17: Cho tam giác có chu vi ͵ Chứng minh

(ĐH Sư Phạm Vinh 2001)

͵ܽଶ൅ ͵ܾଶ൅ ͵ܿଶ൅ Ͷܾܽܿ ൌ ͵ሾሺܽ ൅ ܾሻଶെ ʹܾܽሿ ൅ ͵ܿଶ൅ Ͷܾܽܿ

ൌ ͵ሺ͵ െ ܿሻଶ൅ ͵ܿଶ൅ Ͷܾܽܿ െ ͸ܾܽ ൌ ͵ሺ͵ െ ܿሻଶ൅ ͵ܿଶെ ʹܾܽሺ͵ െ ʹܿሻ

ฺ ൝െʹܾܽ ൒ െʹ ൬͵ െʹ ܿ ͵ܽଶ൅ ͵ܾଶ൅ ͵ܿଶ൅ Ͷܾܽܿ ൒ ͵ሺ͵ െ ܿሻଶ൅ ͵ܿଶെ ʹ ൬

(121)

Từ bảng biến thiên, ta có

͵ܽଶ ൅ ͵ܾଶ൅ ͵ܿଶ൅ Ͷܾܽܿ ൒ ͳ͵

Dấu ̶ ൌ ̶ xảy tam giác

Giải:

Do ܽ ൑ ܾ ൑ ܿ nên ܽ ൅ ܾ ൅ ܿ ൑ ʹܾ ൅ ܿ Suy Mặt khác ܾ ൑ ܿ nên

Vậy

Vì ܽ ൏ ܾ ൏ ܿ nên ሺܾ െ ܿሻሺܽ െ ܾሻሺܽ െ ܿሻ ൏ Ͳ Vậy ta có điều phải chứng minh

ͷʹ

ʹ͹൑ ܽଶ൅ ܾଶ൅ ܿଶ൅ ʹܾܽܿ ൏ ʹ

Bài 19: Cho tam giác có chu vi ʹ Chứng minh

ሺܽ ൅ ܾ ൅ ܿሻଶ ൑ ͻܾܿ

ܽଷሺܾଶെ ܿଶሻ ൅ ܾଷሺܿଶെ ܽଶሻ ൅ ܿଷሺܽଶെ ܾଶሻ ൏ Ͳ

Bài 18: Cho tam giác ܽ ൑ ܾ ൑ ܿ Chứng minh

Hơn nữa, ܽ ൏ ܾ ൏ ܿ Chứng minh

ሺܽ ൅ ܾ ൅ ܿሻଶ ൑ ሺʹܾ ൅ ܿሻଶ

ቄʹܾ െ ܿ ൑ ʹܾ െ ܾ ൌ ܾʹܾ െ ܿ ൑ ʹܿ െ ܿ ൌ ܿ ฺ ሺʹܾ െ ܿሻଶ ൑ ܾܿ

ሺܽ ൅ ܾ ൅ ܿሻଶ ൑ ͻܾܿ

Dấu ̶ ൌ ̶ xảy tam giác Ta lại có :

ܽଷሺܾଶെ ܿଶሻ ൅ ܾଷሺܿଶെ ܽଶሻ ൅ ܿଷሺܽଶെ ܾଶሻ

ൌ ܽଷሺܾଶെ ܿଶሻ ൅ ܾଶܿଶሺܾ െ ܿሻ െ ܽଶሺܾଷെ ܿଷሻ

ൌ ሺܾ െ ܿሻሾܽଷሺܾ ൅ ܿሻ ൅ ܾଶܿଶെ ܽଶሺܾଶ൅ ܿଶ൅ ܾܿሻሿ

ൌ ሺܾ െ ܿሻሺܽଷܾ ൅ ܽଷܿ ൅ ܾଶܿଶെ ܽଶܾଶെ ܽଶܿଶെ ܽଶܾܿሻ

ൌ ሺܾ െ ܿሻሾܽଶܾሺܽ െ ܾሻ ൅ ܽଶܿሺܽ െ ܾሻ ൅ ܿଶሺܾଶെ ܽଶሻሿ

ൌ ሺܾ െ ܿሻሺܽ െ ܾሻሾܽଶܾ ൅ ܽଶܿ െ ܿଶሺܽ ൅ ܾሻሿ

ൌ ሺܾ െ ܿሻሺܽ െ ܾሻሾܽܿሺܽ െ ܿሻ ൅ ܾሺܽଶ െ ܿଶሻሿ

(122)

Giải:

Giả sửܽ ൑ ܾ ൑ ܿ Ta suy

ʹܿ ൏ ܽ ൅ ܾ ൅ ܿ ൌ ʹ ฺ ܿ ൏ ͳ

Do :

ܽଶ ൅ ܾଶ൅ ܿଶ൅ ʹܾܽܿ ൌ ሺܽ ൅ ܾ ൅ ܿሻଶെ ʹሺܾܽ ൅ ܾܿ ൅ ܿܽሻ ൅ ʹܾܽܿ

ൌ ʹ ൅ ʹሺܽ ൅ ܾ ൅ ܿ െ ܾܽ െ ܾܿ െ ܿܽ ൅ ܾܽܿሻ ൌ ʹ ൅ ʹሺͳ െ ܿሻሺܽ െ ͳሻሺͳ െ ܾሻ ൏ ʹ

Mặt khác, theo bất đẳng thức Cauchy, ta có :

ሺͳ െ ܽሻ ൅ ሺͳ െ ܾሻ ൅ ሺͳ െ ܿሻ

͵ ൒ ඥሺͳ െ ܽሻሺͳ െ ܾሻሺͳ െ ܿሻ

Suy

ሺͳ െ ܽሻሺͳ െ ܾሻሺͳ െ ܿሻ ൑ ͳ ʹ͹

Hay

ͳ ʹ͹ ܽ ൅ ܾ ൅ ܿ ൌ ʹ ฺ ܾܽ ൅ ܾܿ ൅ ܿܽ ൌ ʹ െͳ

ʹ

Nên

ͳ െ ʹ ൅ ʹ െͳ ʹ

ͳ ʹ͹ ͷʹ

ʹ͹

Giải: Ta có :

ͷ •‹ଶ ൅ …‘•ଶ ൅ …‘•ଶ ൌ ʹ

฻ ͷሺͳ െ …‘•ଶሻ ൅ͳ ൅ …‘• ʹ

ʹ ൅

ͳ ൅ …‘• ʹ ʹ ൌ ʹ ฻ െͷ …‘•ଶ ൅ ͸ ൅ͳ

ʹሺ…‘• ʹ ൅ …‘• ʹሻ ൌ ʹ ฻ െͷ …‘•ଶ ൅ Ͷ ൅ …‘•ሺ ൅ ሻ …‘•ሺ െ ሻ ൌ Ͳ

•‹ ൑ ͵ ͷ

ͳ െ ܽ െ ܾ െ ܿ ൅ ܾܽ ൅ ܾܿ ൅ ܿܽ െ ܾܽܿ ൑ ሺܽଶ൅ ܾଶ൅ ܿଶሻ

ሺܽଶ൅ ܾଶ൅ ܿଶሻ െ ܾܽܿ ൑

Do

ܽଶ൅ ܾଶ൅ ܿଶ൅ ʹܾܽܿ ൒

Dấu ̶ ൌ ̶ xảy tam giác

Bài 20: Cho tam giác không vuông thỏa ͷ •‹ଶ ൅ …‘•ଶ ൅ …‘•ଶ ൌ ʹ Chứng

(123)

฻ െͷ …‘•ଶ ൅ Ͷ ൌ …‘• …‘•ሺ െ ሻ

Ta xét trường hợp :

- Nếu góc nhọn …‘• ൐ Ͳ Do …‘•ሺ െ ሻ ൑ ͳ, ta suy

െͷ …‘•ଶ ൅ Ͷ ൌ …‘• …‘•ሺ െ ሻ ൑ …‘•

Suy ͷ …‘•ଶ ൅ …‘• െ Ͷ ൒ Ͳ Do đó

…‘• ൒ Ͷ ͷ

Vì góc nhọn nên ta có •‹ ൐ Ͳ, suy

ʹͷ

ฺ •‹ ൑ ͵ ͷ ậ›–ƒŽ—ؐ…× •‹ ൑͵

ͷǤ

Giải: Ta có :

Ͷ݌ሺ݌ െ ܽሻ ൑ ܾܿ ฻ ʹሺܽ ൅ ܾ ൅ ܿሻ ൬ܽ ൅ ܾ ൅ ܿ

ʹ െ ܽ൰ ൑ ܾܿ ฻ ሺܾ ൅ ܿ ൅ ܽሻሺܾ ൅ ܿ െ ܽሻ ൑ ܾܿ ฻ ሺܾ ൅ ܿሻଶെ ܽଶ ൑ ܾܿ

฻ ܾଶ൅ ܿଶെ ܽଶ൅ ʹܾܿ ൑ ܾܿ

฻ ʹܾܿ …‘• ൑ െܾܿ ฻ …‘• ൑ െͳ

ʹ฻ ͳʹͲ୭ ൑ ෡ ൏ ͳͺͲ୭

ʹ•‹ ʹ•‹

ʹ ൑

ʹξ͵ െ ͵ ͺ

•‹ ൌ ඥͳ െ …‘•ଶ ൑ ඨͳ െͳ͸ൌ͵

ͷ

- Tương tự, góc tù െͷ …‘•ଶ ൅ Ͷ ൌ …‘• …‘•ሺ െ ሻ ൒ …‘•

Ͷ ͷ ฺ ͷ …‘•ଶ ൅ …‘• െ Ͷ ൑ Ͳ ฻ …‘• ൑

Do …‘• ൏ Ͳ •‹ ൐ Ͳ nên

…‘•ଶ ൒ ͳ͸

ʹͷฺ •‹ଶ ൑ ͻ ͷ •‹

(124)

Mặt khác •‹ ʹ•‹ ʹ•‹ ʹ ൌ ͳ ʹ•‹ ʹ൬…‘• െ ʹ െ …‘• ൅ ʹ ൰ ൑ ͳ ʹ•‹ ʹ൬ͳ െ •‹ ʹ൰ Vì ͳ ʹ•‹ ʹ൬ͳ െ •‹ ʹ൰ ൌ ͳ ʹ൬•‹ ʹെ •‹ଶ ʹ൰ ൌ ͳ ʹቈെ ൬•‹ ʹെ ͳ ʹ൰ ଶ ൅ͳ Ͷ቉ ൌͳ ͺെ ͳ ʹ൬•‹ ʹെ ͳ ʹ൰ ଶ ൑ͳ ͺെ ͳ ʹቆ ξ͵ ʹ െ ͳ ʹቇ ଶ ൌʹξ͵ െ ͵ ͺ

Do đó,

•‹ ʹ•‹ ʹ•‹ ʹ൑ ʹξ͵ െ ͵ ͺ

Dấu ̶ ൌ ̶ xảy ൜ ෡ ൌ ͳʹͲ୭ ෡ ൌ ෠ ൌ ͵Ͳ୭

Giải:

Vậy ta có

ቀ െߨ Ͷ •‹଺ ൅ߨ Ͷ…‘•଺ ൒ Ͳ ฻ •‹଺ ൅ ቀߨ ʹ ߨ Ͷሺ•‹଺ ൅ …‘•଺ሻ ฻ •‹଺ ൅ …‘•଺ ൒ ߨ Ͷ൬ͳ െ ͵ Ͷ•‹ଶʹ൰ ൒ ߨ ͳ͸ ฻ …‘• ଺ ൅ …‘•଺ ൒ ߨ ͳ͸

Mặt khác

൑ ൬ ൅ ʹ ൰ ଶ ൌ ߨ ଶ ͳ͸ …‘•଺ ൅ ൒ ͳ ߨ

- Nếu Ͷͷ୭ ൑ ൏ ͻͲ୭ •‹ ൒ …‘• ฺ •‹଺ ൒ …‘•଺

- Nếu Ͳ ൏ ൏ Ͷͷ୭ •‹଺ ൏ …‘•଺

ቁ ሺ•‹଺ െ …‘•଺ሻ ൒ Ͳ ฻ •‹଺ െ …‘•଺ െߨ Ͷ െ ቁ …‘•଺ ൒ …‘•଺

(125)

Nên

…‘•଺

…‘•଺

൒ ͳ ߨ

Dấu ̶ ൌ ̶ xảy tam giác vuông cân

Giải: Ta xét hàm số

ฺ ݂ᇱሺݔሻ ൌ Ž ݔ ൅ ͳ ฺ ݂ᇱᇱሺݔሻ ൌͳ

ݔ

Theo bất đẳng thức Jensen, ta có :

Do đó,

͵ ൰ Ǥ Ž ൬

•‹ ൅ •‹ ൅ •‹ ͵ ൰ •‹ ൅ •‹ ൅ •‹ ൒͵ξ͵

ʹ

Mặt khác tam giác nhọn, suy

•‹ ൅ •‹ ൅ •‹ ൒ •‹ଶ ൅ •‹ଶ ൅ •‹ଶ ൌ ʹ ൅ ʹ …‘• …‘• …‘• ൐ ʹ

Và hàm số݃ሺݔሻ ൌ Ž ݔ ǡ ݔ ൐ Ͳ đồng biến nên ta có

ሺ•‹ ൅ •‹ ൅ •‹ ሻ Ž ൬•‹ ൅ •‹ ൅ •‹ ͵ ൰ ൐

͵ξ͵ ʹ Ž

ʹ ͵

Do đó,

ሺ•‹ ሻୱ୧୬ ୅ሺ•‹ ሻୱ୧୬ ୆ሺ•‹ ሻୱ୧୬ େ ൐ ൬ʹ൰ ଷξଷ

ሺ•‹ ሻୱ୧୬ ୅ሺ•‹ ሻୱ୧୬ ୆ሺ•‹ ሻୱ୧୬ େ ൐ ൬ʹ

͵൰

ଷξଷ ଶ

Bài 23: Cho tam giác nhọn Chứng minh

݂ሺݔሻ ൌ ݔ Ž ݔ ǡ ݔ ൐ Ͳ ൐ Ͳ

݂ሺ•‹ ሻ ൅ ݂ሺ•‹ ሻ ൅ ݂ሺ•‹ ሻ ͵ ൒ ݂ ൬

•‹ ൅ •‹ ൅ •‹ ͵

•‹ Ǥ Ž •‹ ൅ •‹ Ǥ Ž •‹ ൅ •‹ Ǥ Ž •‹ ͵

•‹ ൅ •‹ ൅ •‹ ൒ ൬

Mà theo bất đẳng thức bản, ta có :

(126)

Giải: Ta xét hàm số

݂ሺݔሻ ൌ ሺ–ƒ ݔሻଶξଶǡ ݔ א ቀͲǡߨ

ʹቁ ݂ᇱሺݔሻ ൌ ʹξʹሺͳ ൅ –ƒଶݔሻሺ–ƒ ݔሻଶξଶିଵ

݂ᇱᇱሺݔሻ ൌ Ͷξʹ –ƒ ݔ ሺͳ ൅ –ƒଶݔሻሺ–ƒ ݔሻଶξଶିଵ

݂ ൬

ʹ൰ ൅ ݂ ൬

ʹ൰ ൅ ݂ ൬

ʹ൰ ൒ ͵݂ ൬ ൰

Hay

൬–ƒ൰

ଶξଶ

൅ ൬–ƒ ʹ൰

ଶξଶ

൅ ൬–ƒ ൰ ൒ ൬–ƒ ൅ ൅ ͸ ൰

ଶξଶ

ൌ ͵ଵିξଶ

Giải: Ta xét hàm số

ߨ ʹቁ ݂ᇱሺݔሻ ൌ ͳ

…‘•ଶݔ൅ …‘• ݔ െ ʹ ൌ ൌ

ሺ…‘• ݔ െ ͳሻሺ…‘•ଶݔ െ …‘• ݔ െ ͳሻ

…‘•ଶݔ

…‘•ଶݔ ൐ Ͳ

Vậy hàm số݂ሺݔሻđồng biến Suy

݂ሺݔሻ ൐ ݂ሺͲሻ ൌ Ͳ ฺ –ƒ ݔ ൅ •‹ ݔ ൐ ʹݔ

Áp dụng bất đẳng thức trên, ta có :

൝•‹ ൅ –ƒ ൐ ʹ•‹ ൅ –ƒ ൐ ʹ •‹ ൅ –ƒ ൐ ʹ ൬–ƒ

ʹ൰

ଶξଶ

൅ ൬–ƒ ʹ൰

ଶξଶ

൅ ൬–ƒ ʹ൰

ଶξଶ

൒ ͵ଵିξଶ

Bài 24: Cho tam giác nhọn Chứng minh

൅ ʹξʹ൫ʹξʹ െ ͳ൯ሺͳ ൅ –ƒଶݔሻଶሺ–ƒ ݔሻଶξଶିଶ൐ Ͳ

Do đó, theo bất đẳng thức Jensen, ta có :

൅ ൅ ͸

ʹ ʹ

ଶξଶ

Dấu ̶ ൌ ̶ xảy tam giác

݂ሺݔሻ ൌ –ƒ ݔ ൅ •‹ ݔ െ ʹݔǡ ݔ א ቀͲǡ …‘•ଷݔ െ ʹ …‘•ଶݔ ൅ ͳ

…‘•ଶݔ

ሺͳ െ …‘• ݔሻሺ…‘• ݔ ൅ •‹ଶݔሻ

•‹ ൅ •‹ ൅ •‹ ൅ –ƒ ൅ –ƒ ൅ –ƒ ൐ ʹߨ

(127)

Do đó,

•‹ ൅ •‹ ൅ •‹ ൅ –ƒ ൅ –ƒ ൅ –ƒ ൐ ʹሺ ൅ ൅ ሻ ൌ ʹߨ

Giải:Theo đẳng thức bản, ta có :

–ƒ –ƒ –ƒ ൌ –ƒ ൅ –ƒ ൅ –ƒ

Bất đẳng thức tương đương với

ξ͵ሺ–ƒ ൅ –ƒ ൅ –ƒ ሻ ൅ ʹ ൬ ͳ …‘• ൅

ͳ …‘• ൅

Ta xét hàm số

݂ሺݔሻ ൌ ξ͵ –ƒ ݔ ൅ ʹ

…‘• ݔ ቁ ݂ᇱሺݔሻ ൌ ξ͵

…‘•ଶݔെ

ʹ •‹ ݔ …‘•ଶݔ ൌ

฻ ݔ ൌ ߨ ͵ ݐ Ͳ గ

ଶ ݂Ԣሺݐሻ Ͳ െ

͹ ξ͵ –ƒ ݔ ൅ ʹ

…‘• ݔ ൑ ͹

Áp dụng bất đẳng thức trên, ta suy

ە ۖ ۔ ۖ

ۓξ͵ –ƒ ൅ ʹ

…‘• ൑ ͹ ξ͵ –ƒ ൅ ʹ

…‘• ൑ ͹ ξ͵ –ƒ ൅ ʹ

…‘• ൑ ͹

Cộng bất đẳng thức trên, ta có điều phải chứng minh Dấu ̶ ൌ ̶ xảy tam giác

ξ͵ –ƒ –ƒ –ƒ ൅ ʹ ൬ ͳ …‘• ൅

ͳ …‘• ൅

ͳ

…‘• ൰ ൑ ʹͳ

Bài 26: Cho tam giác nhọn Chứng minh

ͳ

…‘• ൰ ൑ ʹͳ ǡ ݔ א ቀͲǡߨ

ʹ ξ͵ െ ʹ •‹ ݔ

…‘•ଶݔ

݂ᇱሺݔሻ ൌ Ͳ ฻ •‹ ݔ ൌξ͵

ʹ గ

ସ ൅

݂ሺݐሻ

(128)

Giải: Giả sử ൑ ൑ Ta suy

ߨ

Ͷ ൑ ൑ ߨ ͵

Ta có :

…‘– ൅ …‘– ൅ …‘– ൅ ͵ …‘– …‘– …‘–

ൌ …‘– ൅ …‘– ൅ …‘– ൅ ͵ …‘– ሾͳ െ …‘– ሺ…‘– ൅ …‘– ሻሿ ൌ Ͷ …‘– ൅ ሺͳ െ ͵ …‘–ଶሻሺ…‘– ൅ …‘– ሻ

൑ߨ

͵ ฺ ͳ െ ͵ …‘–ଶ ൑ ͳ െ ͵ ൬ ൰

…‘– ൅ …‘– ൒ ʹ …‘– ൅ ʹ

Suy

ʹ ൌ ͶǤͳ െ –ƒ

ʹ

ʹ –ƒʹ ቍ

൪ ʹǤ –ƒ ʹ ൌ

Ͷ െ ͵ ቀͳ െ –ƒଶ

ʹቁ

ʹ –ƒʹ

Ta xét hàm số

ǡ ݔ א ൤ξʹ െ ͳǡ ͳ ξ͵൨ ݂ᇱሺݔሻ ൌ െሺ͵ݔଶെ ͳሻଶ

ʹݔଶ

Suy hàm số݂ሺݔሻ nghịch biến Do đó,

݂ሺݔሻ ൑ ݂൫ξʹ െ ͳ൯ ൌ Ͷ൫ʹ െ ξʹ൯

Vậy ta có điều phải chứng minh

Dấu ̶ ൌ ̶ xảy tam giác cân có góc ෡ ൌ Ͷͷ୭

ሺʹܴ ൅ ܽሻሺʹܴ ൅ ܾሻሺʹܴ ൅ ܿሻ ൏ ͺܴଷ݁ଷξଷଶ

Bài 28: Cho tam giác Chứng minh

…‘– ൅ …‘– ൅ …‘– ൅ ͵ …‘– …‘– …‘– ൑ Ͷ൫ʹ െ ξʹ൯

Bài 27: Cho tam giác khơng tù góc khơng nhỏhơn Ͷͷ୭ Chứng minh rằng

ͳ

ξ͵ ൑ Ͳ ൌ ʹ –ƒ

ʹ ൐ Ͳ

Ͷ …‘– ൅ ሺͳ െ ͵ …‘–ଶሻሺ…‘– ൅ …‘– ሻ ൑ Ͷ …‘– ൅ ሺͳ െ ͵ …‘–ଶሻʹ –ƒ

൅ ൦ͳ െ ͵ ቌͳ െ –ƒ

ʹ ʹ –ƒʹ ݂ሺݔሻ ൌ Ͷ െ ͵ሺͳ െ ݔଶሻଶ

(129)

Giải:Ta có điều phải chứng minh tương đương với

ቀͳ ൅ ܽ

ʹܴቁ ൬ͳ ൅ ܾ

ʹܴ൰ ቀͳ ൅ ܿ

ʹܴቁ ൏ ݁

ଷξଷ ଶ

Theo định lý hàm sốsin, ta

ሺͳ ൅ •‹ ሻሺͳ ൅ •‹ ሻሺͳ ൅ •‹ ሻ ൏ ݁ଷξଷଶ

Ta xét hàm số :

݂ሺݔሻ ൌ Žሺͳ ൅ ݔሻ െ ݔǡ ݔ א ሺͲǡͳሻ ݂ᇱሺݔሻ ൌ ͳ

ͳ ൅ ݔെ ͳ ൏ Ͳ

Suy hàm số݂ሺݔሻ nghịch biến Do đó,

݂ሺݔሻ ൏ ݂ሺͲሻ ൌ Ͳ

Vậy Žሺͳ ൅ ݔሻ ൏ ݔ

Áp dụng bất đẳng thức trên, ta có :

͵ξ͵ ʹ

݂ሺݔሻ ൌ ݔ௫ǡ ݔ א ሺͲǢ ͳሻ

݂ᇱሺݔሻ ൌ ݔ௫ሺͳ ൅ Ž ݔሻ

݂ᇱሺݔሻ ൌ Ͳ ฻ ݔ ൌͳ

݁ ݔ Ͳ ଵ

௘ ͳ ݂Ԣሺݔሻ െ Ͳ ൅

݂ሺݔሻ

ͳ ͳ ଵ

ξ௘

Žሺͳ ൅ •‹ ሻ ൅ Žሺͳ ൅ •‹ ሻ ൅ Žሺͳ ൅ •‹ ሻ ൏ •‹ ൅ •‹ ൅ •‹

Hay

ሺͳ ൅ •‹ ሻሺͳ ൅ •‹ ሻሺͳ ൅ •‹ ሻ ൏ ݁ሺୱ୧୬ ୅ାୱ୧୬ ୆ାୱ୧୬େሻ

Mặt khác, theo bất đẳng thức bản, ta có :

•‹ ൅ •‹ ൅ •‹ ൑

Vậy ta có điều phải chứng minh

Giải: Ta xét hàm số

ሺ•‹ ሻୱ୧୬ ୆൅ ሺ•‹ ሻୱ୧୬ େ൅ ሺ•‹ ሻୱ୧୬ ୅ ൐ ͳǡͳͻ

(130)

Từ bảng biến thiên, ta có :

݂ሺݔሻ ൒ ͳ ξ݁

Theo bất đẳng thức Bernoulli, với ǡ ݒ א ሺͲǡͳሻ , ta có :

ݑ௩ ൌ ݑ

ሾͳ െ ሺͳ െ ݑሻሿଵି௩ ൐

ݑ

ͳ െ ሺͳ െ ݒሻሺͳ െ ݑሻൌ

ݑ

ݑ ൅ ݒ െ ݑݒ ൐ ݑ ݑ ൅ ݒ

Do đó,

ሺ•‹ ሻୱ୧୬ େ ൅ ሺ•‹ ሻୱ୧୬ ୅ ൐ •‹

•‹ ൅ •‹ ൅

•‹ •‹ ൅ •‹

Giả sử : •‹ ൒ •‹ ൒ •‹

Suy

ە ۔

ۓ ሺ•‹ ሻୱ୧୬ ୆ ൒ ሺ•‹ ሻୱ୧୬ ୅ ൒

•‹

ʹሺ•‹ ൅ •‹ ሻ൅

•‹

•‹ ൅ •‹ ൐

Khi

ሺ•‹ ሻୱ୧୬ ୆൅ ሺ•‹ ሻୱ୧୬ େ൅ ሺ•‹ ሻୱ୧୬ ୅ ൐ ൌ ͳ

ξ݁

೐ ൅

ͳ

ʹ൐ ͳǡͳͻ

Vậy ta có điều phải chứng minh

ξ͵

ʹ ൅ ǥ൅ ξ͵

ʹ ᇣᇧᇧᇧᇤᇧᇧᇧᇥ

ሺ௡ିଵሻୱố

൒ ݊ ඨቆξ͵ ʹ ቇ

௡ିଵ

•‹

Do đó,

݊ ඨቆξ͵ ʹ ቇ

௡ିଵ

൑ •‹ ൅ •‹ ൅ •‹ ൅ ͵ሺ݊ െ ͳሻξ͵ ʹ

Theo bất đẳng thức bản, ta có :

•‹ ൅ •‹ ൅ •‹ ൑͵ξ͵ ʹ

೙ ೙ ೙

൑ ͵ ඨ͵ Ͷ

మ೙

ͳ

ξ݁ •‹ ʹሺ•‹ ൅ •‹ ሻ ͳ

ξ݁൅

•‹ ൅ •‹ ʹሺ•‹ ൅ •‹ ሻ

Giải: Theo bất đẳng thức Cauchy, ta có :

•‹ ൅

ξ•‹ ൅ ξ•‹ ൅ ξ•‹

(131)

Suy

݊ ඨቆξ͵ ʹ ቇ

௡ିଵ

൑ ͵݊ξ͵ ʹ

Hay

ඨቆξ͵ ʹ ቇ

௡ିଵ

൑ ͵ξ͵ ʹ

Vậy

൑ ͵ ඨమ೙ ͵

Giải: Theo bất đẳng thức Cauchy, ta có :

ͳ

•‹௡൅ ൬

ʹ ξ͵൰

൅ ǥ൅ ൬ ʹ ξ͵൰

ᇣᇧᇧᇧᇧᇧᇧᇤᇧᇧᇧᇧᇧᇧᇥ

ሺ௡ିଵሻୱố

൒ ݊ ඨ ͳ •‹௡൬

ʹ ξ͵൰

௡ሺ௡ିଵሻ

ൌ ݊ ൬ ʹ ξ͵൰

௡ିଵ ͳ

•‹ ͳ

•‹௡൅

ͳ •‹௡൅

ͳ

•‹௡൒ ͵ ൬

ʹ ξ͵൰

Bài 31: Cho tam giác Chứng minh

ඨ•‹ ʹ

൅ ඨ•‹ ʹ

೙ ೙

ξ•‹ •‹

೙ ೙ ೙

൑ ͵ ඨ͵ Ͷ

ඨ•‹ ʹ

•‹ ʹ

൅ ඨ•‹ ʹ•‹

ʹ

൑ ͵ ξͶ

Ͷ

Dấu ̶ ൌ ̶ xảy tam giác

Chú ý: Chứng minh tương tự tốn trên, ta có bất đẳng thức sau :

൅ ඨ•‹ ʹ ൑

͵ ξʹ ൅ ξ•‹ •‹ ൅ ξ•‹ •‹ •‹

(132)

Do đó,

൅ ͵ሺ݊ െ ͳሻ ൬ ʹ ξ͵൰

൒ ݊ ൬ ʹ ξ͵൰

௡ିଵ

൬ ͳ •‹ ൅

ͳ •‹ ൅

ͳ •‹ ൰

Theo bất đẳng thức bản, ta có :

ͳ •‹ ൅

ͳ •‹ ൅

ͳ

•‹ ൒ ʹξ͵

Suy

൅ ͵ሺ݊ െ ͳሻ ൬ ʹ ξ͵൰

൒ ʹξ͵݊ ൬ ʹ ξ͵൰

௡ିଵ

ൌ ͵݊ ൬ ʹ ξ͵൰

Vậy

൒ ͵ ൬ ʹ ൰

Giải: Ta có :

ൌ ݊ଷ൅ ݊ଶቌ ͳ

•‹ʹ൅ ͳ •‹ʹ൅

ͳ

•‹ʹቍ ൅ ݊ ቌ ͳ

•‹ʹ •‹ʹ൅ ͳ

•‹ʹ •‹ʹ൅ ͳ •‹ʹ •‹ʹቍ ൅ ͳ

•‹ʹ •‹ʹ •‹ʹ ͳ

•‹ʹቍ ቌ݊ ൅ ͳ

•‹ʹቍ ቌ݊ ൅ ͳ

•‹ʹቍ ൒ ሺ݊ ൅ ʹሻ

ͳ •‹௡

ʹ

൅ ͳ •‹௡

ʹ

൅ ͳ ൒ ͵Ǥʹ௡

•‹௡

ʹ

ʹ൒

͵

ʹ௡ǡ ݊ ൒ ʹ

ቌ݊ ൅

Bài 32: Cho tam giác Chứng minh với ݊ א Գ, ta có :

•‹௡

ʹ ൅ •‹௡

ʹ൅ •‹௡ ξ͵

Dấu ̶ ൌ ̶ xảy tam giác

(133)

Theo bất đẳng thức bản, ta có : •‹ ʹ•‹ ʹ•‹ ʹ൑ ͳ ͺ

Do đó, theo bất đẳng thức Cauchy

ە ۖ ۖ ۔ ۖ ۖ ۓ ͳ •‹ʹ൅ ͳ •‹ʹ൅ ͳ •‹ʹ ൒ ͵ඨ ͳ •‹ʹ •‹ʹ •‹ʹ య ൒ ͸ ͳ •‹ʹ •‹ʹ൅ ͳ •‹ʹ •‹ʹ൅ ͳ •‹ʹ •‹ʹ ൒ ͵ඩቌ ͳ •‹ʹ •‹ʹ •‹ʹቍ ଶ య ൒ ͳʹ Suy

Giải: Ta có :

•‹௡ ʹ•‹ ʹ•‹ ʹൌ ͳ ʹ•‹௡ ʹ൬…‘• െ ʹ െ •‹ ʹ൰ ൑ ͳ ʹ•‹௡ ʹ൬ͳ െ •‹ ʹ൰

Mặt khác, theo bất đẳng thức Cauchy, ta có :

ͳ ʹ•‹௡ ʹ൬ͳ െ •‹ ʹ൰ ൌ ͳ ʹ݊•‹௡ ʹቌͳ ൅ǥ൅ ͳᇣᇧᇧᇤᇧᇧᇥ௡ୱố െ •‹ ʹെ ǥെ •‹ ʹ ᇣᇧᇧᇧᇧᇧᇤᇧᇧᇧᇧᇧᇥ ௡ୱố ቍ ൑ ͳ ʹ݊ቀ ݊ ݊ ൅ ͳቁ ௡ାଵ ൌ ݊ ௡ ʹሺ݊ ൅ ͳሻ௡ାଵ ʹ ඨ•‹ ʹ•‹ ʹ ೙ ൑ ݊ ሺ݊ ൅ ͳሻ ඥʹሺ݊ ൅ ͳሻ೙ ۉ ۇͳ ൅ ͳ ට•‹ ʹ ೙ ی ۊ ۉ ۇͳ ൅ ͳ ೙ ۊ ۉ ೙ ی ۊ ൒ ൫ͳ ൅ ξʹ೙ ൯ଷ ൒ ݊ଷ൅ ͸݊ଶ൅ ͳʹ݊ ൅ ͺ ൌ ሺ݊ ൅ ʹሻଷ

Dấu ̶ ൌ ̶ xảy tam giác

Chú ý: Chứng minh tương tự tốn trên, ta có bất đẳng thức sau :

•‹

Bài 33: Cho tam giác Chứng minh

ට•‹ ʹی

(134)

Do đó, •‹ ʹ ඨ•‹ ʹ•‹ ʹ ೙ ൑ ݊ ሺ݊ ൅ ͳሻ ඥʹሺ݊ ൅ ͳሻ೙

Dấu ̶ ൌ ̶ xảy tam giác cân có góc thỏa mãn :

•‹ ʹ ൌ

݊ ݊ ൅ ͳ

Giải: Theo bất đẳng thức Cauchy, ta có :

•‹ଶ ʹ൅ ͳ •‹ଶ ʹ ൌ •‹ଶ ʹ൅ ͳ ͳ ൒ ͳ͹ ඩ •‹ ଶ ʹ ͳ͸ଵ଺•‹ଷଶ ʹ భళ Suy ൒ ξͳ͹೙ ൮ ඩ ͳ͸ଵ଺•‹ଷଶ ʹ భళ೙ ൅ ඩ •‹ ଶ ʹ ͳ͸ଵ଺•‹ଷଶ ʹ భళ೙ ൅ ඩ •‹ ଶ ʹ ͳ͸ଵ଺•‹ଷଶ ʹ భళ೙ ൲ ඩ ͳ͸ଵ଺•‹ଷଶ ʹ భళ೙ ൅ ඩ •‹ ଶ ʹ ͳ͸ଵ଺•‹ଷଶ ʹ భళ೙ ൅ ඩ •‹ ଶ ʹ ͳ͸ଵ଺•‹ଷଶ ʹ భళ೙ ൒ ͵ ඩ •‹ ଶ ʹ ͳ͸ଵ଺•‹ଷଶ ʹ Ǥ •‹ ଶ ʹ ͳ͸ଵ଺•‹ଷଶ ʹ Ǥ •‹ ଶ ʹ ͳ͸ଵ଺•‹ଷଶ ʹ ఱభ೙ ൌ ͵ ඨ ͳ ͳ͸ସ଼•‹ଷଶ ʹ •‹ଷଶʹ •‹ଷଶʹ ఱభ೙ ඨ•‹ଶʹ൅ ͳ •‹ଶ ʹ ೙ ൅ ඨ•‹ଶ ʹ൅ ͳ •‹ଶ ʹ ೙ ൅ ඨ•‹ଶ ʹ൅ ͳ •‹ଶ ʹ ೙ ൒ ͵ ඨͳ͹ Ͷ ೙

Bài 34: Cho tam giác Chứng minh

൅ ǥ൅ ᇣ ͳ͸ᇧ•ᇧ‹ᇧଶ ᇧʹᇧᇧᇧᇤᇧᇧᇧͳ͸ᇧᇧ•‹ᇧᇧଶᇥʹ ଵ଺ୱố •‹ଶ ʹ

Theo bất đẳng thức Cauchy, ta :

•‹ଶ

(135)

Do đó,

൒ ͵ ξͳ͹೙ Ǥ ඨ ͳ ͳ͸ସ଼•‹ଷ଴

ʹ •‹ଷ଴ʹ •‹ଷ଴ʹ

ఱభ೙

Mặt khác, theo bất đẳng thức bản, ta có :

•‹ ʹ•‹ ʹ•‹ ʹ൑ ͳ ͺ

Vậy

ඨ•‹ଶʹ൅ ͳ •‹ଶ ʹ ೙ ൅ ඨ•‹ଶ ʹ൅ ͳ •‹ଶ ʹ ೙ ൅ ඨ•‹ଶ ʹ൅ ͳ •‹ଶ ʹ ೙ ൒ ͵ ඨͳ͹ Ͷ ೙

Dấu ̶ ൌ ̶ xảy tam giác

Giải: Theo bất đẳng thức Cauchy, ta có :

•‹௡ ʹ •‹ʹ ൅ •‹ʹ൅ •‹ʹ ൅ •‹ʹ ʹ௡ ൅ ͳ ʹ௡൅ ǥ൅ ͳ ʹ௡ ᇣᇧᇧᇧᇤᇧᇧᇧᇥ ቀ௡ଶିଶቁୱố ൒݊ ʹ ඩ •‹௡ ʹ ʹ௡Ǥ ʹ௡ቀ௡ଶିଶቁ ೙ మ ൌ݊ ʹ•‹ଶ ʹǤ ͳ ʹ௡ିଶ

Do đó, ta

൅ ͳ ʹ௡Ǥ ʹ ൬•‹ ʹ൅ •‹ ʹ൅ •‹ ʹ൰ ൅ ͵ ʹ௡ቀ ݊ ʹെ ʹቁ ൒ ݊ ʹ௡ିଵ൬•‹ଶ ʹ൅ •‹ଶ ʹ൅ •‹ଶ ʹ൰

Áp dụng bất đẳng thức

൞ •‹ ʹ൅ •‹ ʹ൅ •‹ ʹ൑ ͵ ʹ •‹ଶ ʹ൅ •‹ଶ ʹ൅ •‹ଶ ʹ൒ ͵ Ͷ

Ta suy

൅ ͵ ʹ௡ቀ ݊ ʹെ ʹቁ ൒ ͵݊ ʹ௡ାଵെ ͵ ʹ௡ Hay •‹௡ ʹ •‹ʹ ൅ •‹ʹ൅ •‹௡ ʹ •‹ʹ ൅ •‹ʹ൅ •‹௡ ʹ •‹ʹ ൅ •‹ʹ ൒ ͵ ʹ௡

(136)

•‹௡

ʹ •‹ʹ ൅ •‹ʹ൅

•‹௡

ʹ •‹ʹ ൅ •‹ʹ൅

•‹௡

ʹ •‹ʹ ൅ •‹ʹ ൒

͵ ʹ௡

Dấu ̶ ൌ ̶ xảy tam giác

Chú ý: Từ tốn trên, ta có kết sau :

Giải: Theo bất đẳng thức Cauchy, ta có :

–ƒ௡

•‹௠

ʹ

൅ ʹ௠ାଵ൫ξ͵൯௡•‹

ʹ൅ǥ൅ ʹ௠ାଵ൫ξ͵൯

•‹ ʹ ᇣᇧᇧᇧᇧᇧᇧᇧᇧᇧᇧᇧᇧᇤᇧᇧᇧᇧᇧᇧᇧᇧᇧᇧᇧᇧᇥ

௠ୱố

൒ ሺ݉ ൅ ͳሻ ට–ƒ೘శభ ௡ Ǥ ቂʹ௠ାଵ൫ξ͵൯௡ቃ௠

Do đó,

൅ ݉ʹ௠ାଵ൫ξ͵൯௡൬•‹

ʹ൅ •‹ ʹ൅ •‹

ʹ൰

൒ ሺ݉ ൅ ͳሻʹ௠൫ξ͵൯௠ାଵ௡௠൫ ξ–ƒ೘శభ ௡൅೘శభξ–ƒ௡

൅ ೘శభξ–ƒ௡൯

Ta xét hàm số

݂ሺݔሻ ൌ –ƒ஑ݔ ǡ ݔ א ቀͲǡߨ

ʹቁ ǡ ߙ ൒ ͳ ݂ᇱሺݔሻ ൌ ߙ –ƒ஑ାଵݔ ൅ ߙ –ƒ஑ିଵݔ

–ƒ௡

•‹௠

ʹ

൅–ƒ

•‹௠

ʹ

൅–ƒ

•‹௠

ʹ

൒ ʹ௠͵௡ାଶଶ

Bài 36: Cho tam giác nhọn Chứng minh với ݊ ൒ ݉ ൅ ͳǡ ݉ ൒ Ͳ

•‹௡

ʹ •‹ ൅ •‹ ൅

•‹௡

ʹ •‹ ൅ •‹ ൅

•‹௡

ʹ •‹ ൅ •‹ ൒

ξ͵ ʹ௡

–ƒ௡

–ƒ ൅ –ƒ ൅

–ƒ௡

–ƒ ൅ –ƒ ൅

–ƒ௡

–ƒ ൅ –ƒ ൒ ͵௡ାଵଶ

ʹ

(137)

݂ԢԢሺݔሻ ൌ ߙሺߙ ൅ ͳሻ –ƒ஑ݔ ሺ–ƒଶݔ ൅ ͳሻ ൅ ߙሺߙ െ ͳሻ –ƒ஑ିଶݔ ሺ–ƒଶݔ ൅ ͳሻ ൐ Ͳ

Theo bất đẳng thức Jensen, ta có :

݂ሺሻ ൅ ݂ሺሻ ൅ ݂ሺሻ ൒ ͵݂ ൬ ൅ ൅ ͵ ൰

Hay

–ƒ஑ ൅ –ƒ஑ ൅ –ƒ஑ ൒ ͵ –ƒ஑ ൅ ൅

͵ ൌ ͵Ǥ ൫ξ͵൯

Áp dụng bất đẳng thức trên, ta

ξ–ƒ௡

೘శభ

൅೘శభξ–ƒ௡

൅ ೘శభξ–ƒ௡൒ ͵Ǥ ൫ξ͵൯௠ାଵ௡

Theo bất đẳng thức bản, ta có

•‹ ʹ൅ •‹

ʹ൅ •‹

ʹ൑

͵ ʹ

Do đó,

൒ ሺ݉ ൅ ͳሻʹ௠൫ξ͵൯௠ାଵ௡௠ Ǥ ͵Ǥ ൫ξ͵൯

ݔǤ –ƒ௡ ൅ ݕǤ –ƒ௡ ൅ ݖǤ –ƒ௡ ൒ ͻǤ͵ ௡ ଶݔݕݖ

ݔݕ ൅ ݕݖ ൅ ݖݔ

Bài 37: Cho tam giác nhọn Chứng minh với ݊ ൒ ʹǡ ݔǡ ݕǡ ݖ ൐ Ͳ

൒ ʹ௠ඥ͵ଶା௡ି௠

൅–ƒ

…‘•௠൅

–ƒ௡

…‘•௠ ൒ ʹ௠͵ ௡ାଶ

൅ –ƒ

…‘•௠

ʹ

൅ –ƒ

…‘•௠

ʹ

൒ ʹ௠ඥ͵ଶା௡ି௠ ௡

௠ାଵെ ݉ʹ௠ାଵ൫ξ͵൯௡Ǥ͵

ʹ

ൌ ͵ሺ݉ ൅ ͳሻʹ௠൫ξ͵൯௡െ ͵݉ʹ௠൫ξ͵൯௡ ൌ ͵Ǥʹ௠൫ξ͵൯௡ ൌ ʹ௠͵௡ାଶଶ

Dấu ̶ ൌ ̶ xảy tam giác

Chú ý: Từ toán trên, ta có bất đẳng thức tổng quát sau :

–ƒ௡

•‹௠൅

–ƒ௡

•‹௠൅

–ƒ௡

•‹௠

–ƒ௡

…‘•௠

–ƒ௡

…‘•௠

(138)

Giải: Theo bất đẳng thức Bunyakovsky, ta có : ሺݔǤ –ƒ௡ ൅ ݕǤ –ƒ௡ ൅ ݖǤ –ƒ௡ሻ ൬ͳ ݔ൅ ͳ ݕ൅ ͳ ݖ൰ ൒ ቀ–ƒ ௡ ଶ ൅ –ƒ௡ଶ ൅ –ƒ௡ଶቁ ଶ

Áp dụng bất đẳng thức :

–ƒ஑ ൅ –ƒ஑ ൅ –ƒ஑ ൒ ͵Ǥ ൫ξ͵൯ఈ

Do đó,

ݔǤ –ƒ௡ ൅ ݕǤ –ƒ௡ ൅ ݖǤ –ƒ௡ ൒ ͻǤ͵ ௡ ଶ ͳ ݔ ൅ͳݕ ൅ͳݖ ൌ ͻǤ͵ ௡ ଶݔݕݖ ݔݕ ൅ ݕݖ ൅ ݖݔ

Dấu ̶ ൌ ̶ xảy

൝ݔǤ –ƒ ௡ ଶ ൌ ݕǤ –ƒ௡ଶ ൌ ݖǤ –ƒ௡ଶ –ƒ௡ଶ ൌ –ƒ௡ଶ ൌ –ƒ௡ଶ ฻ ቄ ඨͳ െ –ƒ ʹ–ƒ ʹ ೙ ʹ–ƒ ʹ ೙ ൅ ඨͳ െ –ƒ ʹ–ƒ ʹ ೙ ൑ ͵ ඨʹ ͵ ೙ ͳ െ –ƒ ʹ–ƒ ʹ൅ ʹ ͵൅ ڮ ൅ ʹ ͵ ᇣᇧᇧᇤᇧᇧᇥ ሺ௡ିଵሻୱố ൒ ݊ ඨ൬ͳ െ –ƒ ʹ–ƒ ʹ൰ ൬ ʹ ͵൰ ௡ିଵ ೙

Do đó,

͵ െ ൬–ƒ ʹ–ƒ ʹ൅ –ƒ ʹ–ƒ ʹ൅ –ƒ ʹ–ƒ ʹ൰ ൅ ͵Ǥ ʹ ͵ሺ݊ െ ͳሻ ൒ ݊ ൬ ʹ ͵൰ ଵିଵ௡ Ǥ

Mặt khác, theo đẳng thức bản, ta có :

–ƒ ʹ–ƒ ʹ൅ –ƒ ʹ–ƒ ʹ൅ –ƒ ʹ–ƒ ʹ ൌ ͳ ඩ •‹ ʹ …‘•ʹ …‘•ʹ ೙ ൅ ඩ •‹ ʹ ೙ …‘•ʹ …‘•ʹ ೙ ൑ ͵ ඨʹ ͵ ೙

Bài 38: Cho tam giác Chứng minh

ݔ ൌ ݕ ൌ ݖ ൌ ൌ ൌ ͸Ͳ୭

Giải:Điều cần chứng minh tương đương với

൅ ඨͳ െ –ƒ

Theo bất đẳng thức Cauchy, ta có :

…‘• ʹ…‘•ʹ൅ ඩ

(139)

Suy

൑ ͵ ඨʹ ͵

Dấu ̶ ൌ ̶ xảy tam giác

Chú ý: Từ toán trên, ta có bất đẳng thức tổng quát sau

ܽ ൅ ܾ ൅ ܾ ൅ ܿ ൅ „Ǥܽଶ൫ͳ െ ξ͵ …‘– ൯ ൅ ܾଶ൫ͳ െ ξ͵ …‘– ൯ ൅ ܿଶ൫ͳ െ ξ͵ …‘– ൯ ൒ Ͳ …Ǥ݈௔݈௕݈௖ ൑ ܾܽܿ …‘• ʹ…‘• ʹ…‘• ʹ †Ǥ …‘•ଷ ͵൅ …‘•ଷ ͵൅ …‘•ଷ ͵൑ ͵ ͺ൅ ͵ Ͷ൬…‘• ͵൅ …‘• ͵൅ …‘• ͵൰

(ĐHQG Hà Nội 1998)

‡Ǥ …‘• െ ʹ ൅ …‘• െ ʹ ൅ …‘• െ ʹ ൑ …‘•ͳ ͵ቀ െ ߨ ͵ቁ ൅ …‘• ͳ ͵ቀ െ ߨ ͵ቁ ൅ …‘• ͳ ͵ቀ െ ߨ ͵ቁ

(ĐHQG Hà Nội 1995)

ඩ …‘• െ ʹ …‘•ʹ …‘•ʹ ೙ ൅ ඩ…‘• െ ʹ …‘•ʹ …‘•ʹ ೙ ൅ ඩ…‘• െ ʹ …‘•ʹ …‘•ʹ ೙ ൑ ͵ ඨͶ ͵ ೙ •‹௡ ʹ …‘•௡ ʹ …‘•௡ʹ ൅ •‹ ௡ ʹ …‘•௡ ʹ …‘•௡ʹ ൅ •‹ ௡ ʹ ൒ ͵Ǥ ൬ʹ ͵൰ ௡ …‘•௡ െ ʹ …‘•௡ ʹ …‘•௡ʹ ൅ …‘• ௡ െ ʹ …‘•௡ ʹ …‘•௡ʹ ൅ ʹ …‘•௡ ʹ …‘•௡ʹ ൒ ͵Ǥ ൬Ͷ ͵൰ ௡

- BÀI TẬP TỰ LUYỆN

3.2.1 Chứng minh tam giác ta có :

(140)

ˆǤ ൬ͳ ൅ –ƒ

ʹ൰ ൬ͳ ൅ –ƒ

ʹ൰ ൬ͳ ൅ –ƒ

ʹ൰ ൒ ʹ ൅ ξ͵ ൅ ܵ ݌ଶ

(ĐH Ngoại Thương 1997)

‰Ǥ ͳ •‹ଶ

ʹ

൅ ͳ •‹ଶ

ʹ

൅ ͳ •‹ଶ

ʹ

൒ Ͷܾܽܿ ൬ͳ ܽଷ ൅

ͳ ܾଷ൅

ͳ ܿଷ൰

ŠǤܴሾʹሺܾ ൅ ܿሻ െ ܽሿ ൒ ξ͵ ‹Ǥ ͳ

݈௔൬

ͳ ܾ൅

ͳ ܿ൰ ൅

ͳ ݈௕൬

ͳ ܿ൅

ͳ ܽ൰ ൅

ͳ ݈௖൬

ͳ ܽ൅

ͳ

ܾ൰ ൑ ͵ξ͵

(ĐH Kỹ Thuật Quân Sự 1997)

ŒǤ…‘• െ

ʹ •‹ʹ ൅

…‘• െ ʹ •‹ʹ ൅

…‘• െ ʹ •‹ʹ ൒ ͸ Ǥͳ ൅ݔ

ʹ ൒ …‘• ൅ ݔ …‘• ൅ ݔ …‘• ǡ ׊ݔ א Թ ŽǤݕݖ •‹ଶ ൅ ݖݔ •‹ଶ ൅ ݔݕ •‹ଶ ൑ͳ

ͳ͹ Ͷ

ʹ൅ ඨ•‹ ʹ

(ĐH Bách Khoa Tp.HCM 1995)

ʹ൅ ඨ…‘–

ʹ൅ ඨ…‘–

ʹǡ ȟŠọ

(ĐH Ngoại Thương 1996)

’Ǥʹሺ•‹ ൅ •‹ ൅ •‹ ሻ ൐ …‘•ሺ െ ሻ ൅ …‘•ሺ െ ሻ ൅ …‘•ሺ െ ሻ െ͵ ʹ “Ǥ ʹ

ͳ ൅ •‹ •‹ ൒

ͳ

ͳ ൅ •‹ଶ൅

ͳ ͳ ൅ •‹ଶ

”Ǥ •‹ ൅ •‹ ൅ •‹ …‘•ʹ ൅ …‘•ʹ ൅ …‘•ʹ ൒

ʹݎ ܴ •Ǥඨͳͷ

Ͷ ൅ …‘•ሺ െ ሻ ൅ …‘•ሺ െ ሻ ൅ …‘•ሺ െ ሻ ൒ •‹ ൅ •‹ ൅ •‹

(Đề nghị Olympic 30-4, 2007)

ሺݔ ൅ ݕ ൅ ݖሻଶ

Ͷ

Ǥʹ …‘• •‹ •‹ ൅ ξ͵ሺ…‘• ൅ …‘• ൅ •‹ ሻ ൑ Ǥξ…‘• ൅ ξ…‘• ൅ ξ…‘• ൑ ඨ•‹

(141)

–Ǥ •‹ ʹ൅ •‹

ʹ൅ •‹

ʹ ൑

ͷ Ͷ൅

ͳ

͸ሺ…‘• ൅ …‘• ൅ …‘• ሻǡ ȟŠọ

(Đề nghị Olympic 30-4, 2010)

3.2.2 Cho tam giác nhọn, chứng minh với ݊ א Գ ξ–ƒ௡

మబబబ

൅మబబబξ–ƒ௡൅మబబబξ–ƒ௡൒݊൫͵ξ͵ െ ͳ൯ ൅ ͸ͲͲͲ

ʹͲͲͲ

3.2.3 Chứng minh

ܵ୅୆େ ൌ

ͳ Ͷฺ

͵ ߨ൬

–ƒ ൅

–ƒ ൅

–ƒ ʹ

ܴ ൅ ͵ ݎ

൏ͳ ʹ

ʹ൅ –ƒ ʹ ൌ ͳ ͵

Ͷ൑ –ƒ ʹ൏ ͳ

(ĐH Bách Khoa Hà Nội 1998)

3.2.7. Cho tam giác có góc thỏa mãn : …‘• ʹ ൅ …‘• ʹ ൅ …‘• ʹ ൒ െͳ

Chứng minh

•‹ ൅ •‹ ൅ •‹ ൑ ͳ ൅ ξʹ

൰ ൑ ܽଶ൅ ܾଶ൅ ܿଶ

3.2.4 Cho tam giác có diện tích ͳ Gọi ܴ ݎ bán kính đường trịn ngoại tiếp nội tiếp tam giác , chứng minh

൒ Ͷξʹ͹

3.2.5 Cho tam giác có ܽǡ ܾǡ ܿ thỏa ܽଶ ൅ ܾଶ ൑ ܿଶ Chứng minh rằng

ʹ ͷ൏

ݎ ݄௖

3.2.6 Cho tam giác có góc ǡ thỏa

–ƒ

(142)

3.2.8 Cho tam giác có Ͳ ൏ ൑ ൑ ൏ ͻͲ୭ Chứng minh rằng

ʹ …‘• ͵ െ Ͷ …‘• ʹ ൅ ͳ …‘• ൒ ʹ

3.2.9. Cho tam giác có độ dài đường phân giác nhỏhơn ͳ Chứng minh

ܵ୅୆େ ൏

ͳ ξ͵

3.2.10 Cho tam giác nhọn

ƒǤͳ

͵ሺ–ƒ ൅ –ƒ ൅ –ƒ ሻ ൅ ʹ

͵ሺ•‹ ൅ •‹ ൅ •‹ ሻ ൐ ߨ „Ǥ͵ ൬–ƒ

ʹ൅ –ƒ ʹ൅ –ƒ

ʹ൰ ൅ …‘– ʹ…‘–

ʹ…‘–

ʹ ൒ ͸ξ͵ …Ǥ …‘– ൅ …‘– ൅ …‘– ൅ ͵ξ͵ ൑ ʹ ൬ ͳ

•‹ ൅ ͳ •‹ ൅

ͳ •‹ ൰

൤ܽ ൅ ܾ ൅ െ

ͳ ʹ

ܾ ൅ ܿ ൅ െ

ͳ

ʹሺܾ ൅ ܿሻ൨ ൅ ൤

ܿ ൅ ܽ ൅ െ

ͳ

ʹሺܿ ൅ ܽሻ൨ ൒ Ͳ ฻ ൅ʹሺ െ ሻሺܿ െ ܽሻ

൅ ൒ Ͳ

b Theo định lý hàm số sin, ta có :

•‹ଶ ൅ •‹ଶ ൅ •‹ଶ ൒ ξ͵ሺ•‹ …‘• ൅ •‹ …‘• ൅ •‹ …‘• ሻ

฻ •‹ଶ ൅ •‹ଶ ൅ •‹ଶ ൒ ξ͵

ʹ ሺ•‹ ʹ ൅ •‹ ʹ ൅ •‹ ʹሻ ฻ •‹ଶ ൅ •‹ଶ ൅ •‹ଶ ൒ ʹξ͵ •‹ •‹ •‹

- GỢI Ý GIẢI BÀI TẬP TỰ LUYỆN 3.2.1

a Điều cần chứng minh tương đương với

ሺܽ ൅ ܾሻ൨ ൅ ൤ ʹሺ െ ሻሺܽ െ ܾሻ

൅ ൅

(143)

Theo bất đẳng thức Cauchy, ta có :

•‹ଶ ൅ •‹ଶ ൅ •‹ଶ ൒ ͵ඥሺ•‹ •‹ •‹ ሻయ ଶ

Mà theo bất đẳng thức

ξ•‹ •‹ •‹

൑ξ͵ ʹ

Suy

͵ඥሺ•‹ •‹ •‹ ሻయ ଶ ൒ ʹξ͵ •‹ •‹ •‹

c Áp dụng công thức độ dài trung tuyến bất đẳng thức Cauchy

݈௔ ൌ

ʹܾܿ ܾ ൅ ܿ…‘•

ʹ ൑ ξܾܿ …‘• ʹ

d Cần chứng minh :

ൌ ͳ

Ͷ ൬…‘•

͵൅ …‘• ͵൅ …‘•

͵൰

e Ta có :

…‘• െ ʹ ൅ …‘•

െ Ͷ …‘•

൅ െ ʹ

Ͷ ൑ ʹ …‘• ൬ ߨ Ͷെ

͵ Ͷ ൰

Để ý

ͳ Ͷቀ െ

ߨ ͵ቁฬ ൑ ฬ

ߨ Ͷെ

͵ Ͷ ฬ ൏

ߨ ʹ

Do đó,

…‘• ൬ߨ Ͷെ

͵

Ͷ ൰ ൑ …‘• ͳ ͵ቀ െ

ߨ ͵ቁ

f Ta đặt :

ݔ ൌ –ƒ

ʹǡݕ ൌ –ƒ

ʹǡݖ ൌ –ƒ ʹ

ฺ ൌ ͳ ൅ ݔ ൅ ݕ ൅ ݖ ൅ ݔݕ ൅ ݕݖ ൅ ݖݔ ൌ ʹ ൅ ݔ ൅ ݕ ൅ ݖ ൅ ݔݕݖ ሺ…‘• ൅ …‘• ൅ …‘• ሻ ൅͵

Ͷ െ

(144)

Để ý

ݔ ൌ ܵ

݌ሺ݌ െ ܽሻǡ ݕ ൌ ܵ

݌ሺ݌ െ ܾሻǡ ݖ ൌ ܵ ݌ሺ݌ െ ܿሻ ² ൌ ʹ ൅ ݔ ൅ ݕ ൅ ݖ ൅ ܵ

݌ଶ

Khi cần chứng minh ݔ ൅ ݕ ൅ ݖ ൒ ξ͵

g Áp dụng định lý hàm số cos, ta chứng minh

ܽଶ ൌ ሺܾ െ ܿሻଶ൅ Ͷܾܿ •‹ଶ

ʹ ฺ ܽଶ ൒ Ͷܾܿ •‹ଶ

ʹ ฺ ൒ Ͷܾܿ

ܽଶ

•‹ ൅ •‹ െͳ

ʹ•‹ ൒ ξ͵ •‹ •‹ ฻ •‹ ቆͳ െͳ

ʹ…‘• െ ξ͵

ʹ

ξ͵

ʹ •‹ ቇ ൒ Ͳ ߨ

͵

ߨ

͵ቁቃ ൒ Ͳ ݈௔൬ͳ

ܾ൅ ͳ

ܿ൰ ൌ ʹ …‘• ʹ ൒ ͵ඩ…‘•

ʹ …‘• െ ʹ …‘• െ ʹ •‹ʹ •‹ʹ •‹ʹ

Để ý

…‘• െ ʹ …‘• െ ʹ …‘• െ ʹ •‹ʹ •‹ʹ •‹ʹ ൌ

ሺ•‹ ൅ •‹ ሻሺ•‹ ൅ •‹ ሻሺ•‹ ൅ •‹ ሻ •‹ •‹ •‹

ͳ •‹ଶ

ʹ

h Áp dụng định lý hàm sốsin, điều cần chứng minh tương đương với

•‹ ቇ ൅ •‹ ቆͳ െͳ

ʹ…‘• െ ቁቃ ൅ •‹ ቂͳ െ …‘• ቀ െ

฻ •‹ ቂͳ െ …‘• ቀ െ

i Cần chứng minh

(145)

Theo bất đẳng thức Cauchy, ta có :

ቐ•‹ ൅ •‹ ൒ ʹξ•‹ •‹ •‹ ൅ •‹ ൒ ʹξ•‹ •‹ •‹ ൅ •‹ ൒ ʹξ•‹ •‹

Suy

ሺ•‹ ൅ •‹ ሻሺ•‹ ൅ •‹ ሻሺ•‹ ൅ •‹ ሻ •‹ •‹ •‹ ൒ ͺ

k Ta coi điều cần chứng minh toán xét dấu tam thức bậc hai

ܽݔଶ൅ ܾݔ ൅ ܿ ൒ Ͳǡ ׊ݔ א Թ ฻ ቄȟ ൑ Ͳ

ܽ ൐ Ͳ ቆ•‹ െξ͵

ʹ ቇ

൅ ቆ…‘• െξ͵ ʹ ቇ

ξ͵ ʹ ቇ

൒ Ͳ

n Để ý

ʹ

o Để ý

൒ ʹඨ…‘–ଶ

ʹ

ൌ ʹඨ…‘– ʹ

p Ta từ

ͳ ൅ •‹ •‹ െ

ͳ

ͳ ൅ •‹ଶ൅

ͳ

ͳ ൅ •‹ •‹ െ

ͳ ͳ ൅ •‹ଶ

ൌሺ•‹ െ •‹ ሻ

ͳ ൅ •‹ •‹ ൤

ͳ െ •‹ •‹ ሺͳ ൅ •‹ଶሻሺͳ ൅ •‹ଶሻ൨

r Theo định lý hàm số sin, ta có :

•‹ ൅ •‹ ൅ •‹ ൌ ݌ ܴ

Bất đẳng thức tương đương với

݌ଶ൒ ʹܵ ൬…‘•

ʹ൅ …‘• ʹ൅ …‘•

ʹ൰

l Tương tựnhư câu k

m Ta có đẳng thức

•‹ଶ ൅ •‹ଶ െ •‹ଶ ൌ ʹ …‘• •‹ •‹

Biến đổi điều phải chứng minh thành

൅ ቆ…‘• െ ξ…‘• ൅ ξ…‘• ൑ ඥʹሺ…‘• ൅ …‘• ሻ ൑ ʹඨ•‹ ξ–ƒ ൅ ξ–ƒ ൒ ʹξ–ƒ –ƒ

ሺͳ െ •‹ ሻሺ•‹ ൅ •‹ ሻ ൐ Ͳ

q Ta sử dụng phép biến đổi tương đương để ý

(146)

฻ ݌ଷ ൒ Ͷሺ݌ െ ܽሻሺ݌ െ ܾሻሺ݌ െ ܿሻ ൬…‘• ʹ൅ …‘• ʹ൅ …‘• ʹ൰ ଶ

Ta có :

ሺ݌ െ ܽሻሺ݌ െ ܾሻሺ݌ െ ܿሻ ൑ ሺ݌ െ ܽ ൅ ݌ െ ܾ ൅ ݌ െ ܿሻଷ ʹ͹ ൌ ݌ଷ ʹ͹ ൬…‘• ʹ൅ …‘• ʹ൅ …‘• ʹ൰ ଶ ൑ʹ͹ Ͷ

s Ta có bất đẳng thức cho tương đương với

ሺ•‹ ൅ •‹ ൅ •‹ ሻଶ ൑ͳͷ Ͷ ൅ …‘•ሺ െ ሻ ൅ …‘•ሺ െ ሻ ൅ …‘•ሺ െ ሻ ൅ ሺ…‘• …‘• െ •‹ •‹ ሻ ൅͵ Ͷ൒ Ͳ ͵ Ͷ൒ Ͳ ͵ Ͷ൒ Ͳ ֞ ൬…‘• െͳ ʹ൰ ଶ ൅ ൬…‘• െͳ ʹ൰ ଶ ͳ ʹ൰ ଶ ൒ Ͳ

t Ta có :

(147)

Mặt khác

•‹ ʹ൅ •‹

ʹ൅ •‹

ʹ൑

͵

ʹ֜ ൤ʹ ൬•‹ ʹ൅ •‹

ʹ൅ •‹

ʹ൰ െ ͵൨

൒ Ͳ ฺ Ͷ ൬•‹

ʹ൅ •‹ ʹ൅ •‹

ʹ൰

൅ ͻ ൒ ͳʹ ൬•‹ ʹ൅ •‹

ʹ൅ •‹

ʹ൰ ฺ Ͷ ൬…‘•ଶ

ʹ൅ …‘•ଶ

ʹ൅ …‘•ଶ

ʹ൰ ൅ ͻ ൒ ͳʹ ൬•‹ ʹ൅ •‹

ʹ൅ •‹

ʹ൰ ฺ ʹሺ͵ ൅ …‘• ൅ …‘• ൅ …‘• ሻ ൅ ͻ ൒ ͳʹ ൬•‹

ʹ൅ •‹ ʹ൅ •‹

ʹ൰ ฺ •‹

ʹ൅ •‹ ʹ൅ •‹

ʹ൑

ͷ Ͷ൅

ͳ

͸ሺ…‘• ൅ …‘• ൅ …‘• ሻ

3.2.2. Chứng minh tương tựBài 9câu b, để ý

͵Ǥ ൫͵ξ͵൯

଺଴଴଴ ൌ ͵ൣͳ ൅ ൫͵ξ͵ െ ͳ൯൧଺଴଴଴௡ ൒ ͵ ቂͳ ൅ ݊

͸ͲͲͲ൫͵ξ͵ െ ͳ൯ቃ

3.2.3 Ta có :

ܽଶ൅ ܾଶ൅ ܿଶ

Ͷܵ ʹ

ܴ൅ ͵ ݎ ൌ

ͺܵ ܾܽܿ൅

͵݌ ܵ ൌ

ͺ

ܾܽܿ൅ ͵݌

Theo bất đẳng thức Cauchy, ta có :

ܾܽܿ ൑ ൬ܽ ൅ ܾ ൅ ܿ ͵ ൰

ൌ ൬ʹ݌ ͵ ൰

Suy

ሺ െ ሻሺ…‘– െ …‘– ሻ ൑ Ͳ ֜ …‘– ൅ …‘– ൑ …‘– ൅ …‘–

Để ý

ʹሺ …‘– ൅ …‘– ൅ …‘– ሻ ൑ ሺ ൅ ሻ …‘– ൅ ሺ ൅ ሻ …‘– ൅ ሺ ൅ ሻ …‘– ฺ ͵ሺ …‘– ൅ …‘– ൅ …‘– ሻ ൑ ሺ ൅ ൅ ሻሺ…‘– ൅ …‘– ൅ …‘– ሻ

ฺ ͵ሺ …‘– ൅ …‘– ൅ …‘– ሻ ൑ ߨ

(148)

൒ ʹ͹

݌ଷ ൅ ͵݌ ൌ

ʹ͹

݌ଷ൅ ݌ ൅ ݌ ൅ ݌ ൒ Ͷඨ

ʹ͹ ݌ଷ݌ଷ

ൌ Ͷξʹ͹ర

3.2.5 Ta có :

ݎ ݄௖ ൌ

ܿ ʹ݌ ൌ

ܿ

ܽ ൅ ܾ ൅ ܿ൏ ܿ ʹܿ ൌ

ͳ ʹ

Mặt khác theo bất đẳng thức Bunyakovsky, ta có :

Ͷሺܽ ൅ ܾሻଶ ൑ ͺሺܽଶ൅ ܾଶሻ ൑ ͺܿଶ ൏ ͻܿଶ

Do

ݎ ݄௖ ൌ

ܿ

ܽ ൅ ܾ ൅ ܿ ൐ ʹ ͷ

3.2.6 Để ý

–ƒ ʹൌ

ͳ െ –ƒʹ –ƒʹ

–ƒʹ ൅ –ƒʹ ൌ ͳ െ –ƒ ʹ–ƒ

ʹ ൏ ͳ

ͳ ൌ –ƒ

ʹ൅ –ƒ

ʹ ൒ ʹඨ–ƒ ʹ–ƒ

ʹ ฺ ͳ െ –ƒ ʹ–ƒ

ʹ ൒

͵ Ͷ ฺ Ͳ ൏ ൅

ʹ ൑ ߨ Ͷ

Khi

൅ ʹ …‘•

ʹ ൑ ͳ ൅ ʹ •‹ ߨ

Ͷ ൌ ͳ ൅ ξʹ ͺ …‘•ଷ െ ͺ …‘•ଶ െ ͺ …‘• ൅ ͷ ൒ Ͳǡ …‘• א ൬ͲǢͳ

ʹ൨

Khi ta cần khảo sát hàm số

݂ሺݔሻ ൌ ͺݔଷെ ͺݔଶെ ͺݔ ൅ ͷǡ ݔ א ൬ͲǢͳ

ʹ൨

3.2.9 Để ý :

݄௔ ൑ ݈௔ ൏ ͳ

Giả sửܽ ൑ ܾ ൑ ܿ Ta suy

3.2.7 Để ý

…‘• ʹ ൅ …‘• ʹ ൅ …‘• ʹ ൒ െͳ ฻ …‘• …‘• …‘• ൑ Ͳ

Do đó, chọn góc cho

൒ ߨ ʹ ൑ ͳ ൅ ʹ •‹

(149)

൝ ͳ ܽ ൒

ͳ ܾ ൒

ͳ ܿ ෡ ൑ ෡ ൑ ෠

Mặt khác

݈௔ ൌ

ʹܾܿ …‘•ʹ

ܾ ൅ ܿ ฺ …‘• ʹ ൏

ͳ ʹ൬

ͳ ܾ ൅

ͳ ܿ൰ ൑

ͳ ܽ

Do

෡ ൑ ෡ ൑ ෠ ฺ ෡ ൑ ߨ

͵ ฺ …‘• ʹ ൒

ξ͵

ʹ ฺ ܽ ൑ ʹ ξ͵

3.2.10

a Ta xét hàm số

݂ሺݔሻ ൌ ͳ

͵–ƒ ݔ ൅ ʹ

͵•‹ ݔ െ ݔǡ ݔ א ቀͲǡ ߨ ʹቁ

b Ta xét hàm số

݂ሺݔሻ ൌ ͵ݔ ൅ͳ

ݔǡ ݔ א ቀͲǡ ߨ ʹቁ

c Ta xét hàm số

݂ሺݔሻ ൌ …‘– ݔ െ ʹ

•‹ ݔǡ ݔ א ቀͲǡ ߨ ʹቁ

3. NHẬN DẠNG TAM GIÁC VÀ TÍNH CÁC GĨC TRONG TAM GIÁC - Đây loại toán tổng kết loại toán từ phương pháp

Khi tam giác thỏa hay đẳng thức bất đẳng thức cạnh hàm sốlượng giác góc, ta phải tìm tính chất tam giác đó, chẳng hạn :

tìm sốđo góc, chứng tỏ giá trị hàm lượng giác góc, chứng minh

tam giác cân, vuông, đều…

(150)

Giải:

a Giả thuyết tương đương với

•‹ሺ െ ሻ •‹ሺ ൅ ሻ ൅ •‹ ൅ …‘• ൌ͵ ʹ ฻ ͳ

ʹ

͵ ฻ ͳ

ʹ

͵ ʹ ฻ ൬•‹ଶ െ •‹ ൅ͳ

Ͷ

ͳ Ͷ൰ ൌ Ͳ ฻ ൬•‹ െͳ

ʹ൰

ଶ ͳ

ʹ൰

ൌ Ͳ ฻ ൞•‹ ൌ

ͳ …‘• ൌ ͳ ʹ

b Giả thuyết tương đương với

ͳ ൅ …‘• ൌ ʹ •‹ ൅ ʹ …‘•

െ ʹ െ

ͳ ʹ ฻ ʹ …‘•ଶ

ʹ ൌ ʹ …‘• ʹ…‘•

െ ʹ െ

ͳ ʹ ฻ …‘•ଶ

ʹെ …‘• ʹ…‘•

െ ʹ ൅

ͳ Ͷൌ Ͳ ƒǤ •‹ሺ െ ሻ •‹ ൅ •‹ ൅ …‘• ൌ͵

ʹ „Ǥ …‘• ൌ •‹ ൅ •‹ െ͵

ʹ

…Ǥ …‘• ʹ ൅ ξ͵ሺ…‘• ʹ ൅ …‘• ʹሻ ൅ͷ ʹൌ Ͳ

Bài 1: Tính góc tam giác , biết

(ĐH Mở Hà Nội 2000)

(ĐH Sư Phạm Kỹ Thuật Tp.HCM 2001)

ሺ…‘• ʹ െ …‘• ʹሻ ൅ •‹ ൅ …‘• ൌ ʹ

ሺͳ െ ʹ •‹ଶ ൅ ͳ െ ʹ …‘•ଶሻ ൅ •‹ ൅ …‘• ൌ

൰ ൅ ൬…‘•ଶ െ …‘• ൅

൅ ൬…‘• െ

ʹ ฻ ቄ ൌ ͵Ͳ ൌ ͸Ͳ୭୭ ฺ ൌ ͻͲ୭

(151)

฻ ൬…‘•ଶ

ʹെ …‘• ʹ…‘•

െ ʹ ൅

ͳ Ͷ…‘•ଶ

െ ʹ ൰ ൅

ͳ Ͷെ

ͳ Ͷ…‘•ଶ

െ ʹ ൌ Ͳ ฻ ൬…‘•

ʹെ ͳ ʹ…‘•

െ ʹ ൰

൅ͳ Ͷ•‹ଶ

െ ʹ ൌ Ͳ ฻ ൞…‘•

ʹെ

ͳ ʹ…‘•

െ ʹ ൌ Ͳ •‹ െ

ʹ ൌ Ͳ

฻ ቄ ൌ ͳʹͲ ൌ ൌ ͵Ͳ୭୭

c Giả thuyết tương đương với

ʹ …‘•ଶ െ ͳ ൅ ʹξ͵ …‘•ሺ ൅ ሻ …‘•ሺ െ ሻ ൅ͷ

ʹൌ Ͳ ฻ …‘•ଶ െ ξ͵ …‘•ሺ െ ሻ …‘• ൅͵

Ͷൌ Ͳ

Ta thấy phương trình bậc có nghiệm …‘• Khi

ȟ ൌ ͵ …‘•ଶ െ

ʹ െ ͵ ൒ Ͳ ฻ •‹ଶ െ

ʹ ൑ Ͳ

Suy ൌ

Như

…‘•ଶ െ ξ͵ …‘• ൅͵

Ͷൌ Ͳ ฻ …‘• ൌ ξ͵

ʹ

Do : ൌ ͵Ͳ୭ǡ ൌ ൌ ͹ͷ୭

Giải:

a Theo định lý hàm số sin, ta có :

•‹ ͳ ൌ

•‹ ξ͵ ൌ

•‹ ʹ ฻

ܽ ͳൌ

ܾ ξ͵ ൌ

ܿ

ʹ฻ ൜ܾ ൌ ܽξ͵ܿ ൌ ʹܽ

Theo định lý hàm sốcos, ta :

ƒǤ•‹ ͳ ൌ

•‹ ξ͵ ൌ

•‹ ʹ

„Ǥ …‘• ʹ ൅ ʹξʹ …‘• ൅ ʹξʹ …‘• ൌ ͵ሺȟŠØ‰–ỵሻ

Bài 2: Tính góc tam giác biết

(ĐH An Ninh 1998)

(152)

…‘• ൌ ܾ

ଶ൅ ܿଶെ ܽଶ

ʹܾܿ ൌ

͵ܽଶ൅ Ͷܽଶെ ܽଶ

ʹܽξ͵Ǥ ʹܽ ൌ ξ͵

ʹ ฺ ൌ ͵Ͳ୭ …‘• ൌ Ͷܽଶ൅ ܽଶ െ ͵ܽଶ

ʹܽǤ ʹܽ ൌ ͳ

ʹ ฺ ൌ ͸Ͳ୭

Do đó, ൌ ͻͲ୭

b Giả thuyết tương đương với

ʹ …‘•ଶ െ ͳ ൅ Ͷξʹ …‘• ൅

ʹ …‘• െ

ʹ ൌ ͵ ฻ …‘•ଶ ൅ ʹξʹ •‹

ʹ…‘• െ

ʹ ൌ ʹ

ʹ ൌ …‘•ଶ ൅ ʹξʹ •‹

ʹ…‘•

ʹ…‘•

െ ʹ ฻ ʹ ൑ ͳ െ ʹ •‹ଶ

ʹ൅ ʹξʹ •‹ ʹ…‘• ฻ •‹ଶ

ʹെ ξʹ •‹ ʹ ฻ ቆ•‹

ʹെ ξʹ

ʹ ቇ ൅ ͳ ʹ•‹ଶ

െ ʹ ൑ Ͳ ฻

ە ۔ ۓ•‹

ʹെ ξʹ

ʹ …‘• െ

ʹ ൌ Ͳ •‹ െ

ʹ ൌ Ͳ

฻ ቄ ൌ ͻͲ୭ ൌ ൌ Ͷͷ୭

Mặt khác :

Do tam giác không tù nên …‘• א ሾͲǢ ͳሻ ฺ …‘•ଶ ൑ …‘•

Nên

ʹ ൑ …‘• ൅ ʹξʹ •‹ െ

ʹ …‘• െ

ʹ ൅ ͳ ʹ൑ Ͳ …‘• െ

ʹ

(153)

Giải:

a Giả sử ൒ ൒ Do đó, ta có :

൅ ൌ ʹ

Mặt khác : ൅ ൅ ൌ ͳͺͲ୭ ฺ ቄ ൅ ൌ ͳʹͲ୭

ൌ ͸Ͳ୭

Khi :

•‹ ൅ •‹ ൅ •‹ ൌ ʹ •‹ ൅

ʹ …‘• ʹ

ʹ ൅ •‹ ͸Ͳ୭ ൌ ξ͵ …‘• െ

ʹ ൅ ξ͵

ʹ

Như

ξ͵ …‘• െ

ʹ ൅ ൌ ฺ …‘• െ

ʹ ൌ ξ͵

ʹ ฺ െ ൌ ͸Ͳ୭

ە ۔

ۓ ൌ •‹ ͸Ͳ୭ ൌξ͵

ʹ ൌ •‹ ͵Ͳ୭ ൌ

ʹ

Mặt khác

݌ ൌ ͳ

ʹሺ ൅ ൅ ሻ ൌ ͷͲ •‹ ൅ •‹ ൅ •‹ ൌ ͵ ൅ ξ͵

ʹ

Bài 3: Sốđo góc tam giác lập thành cấp số cộng thỏa mãn đẳng thức a Tính góc ǡ ǡ

b Biết nửa chu vi tam giác 50 Tính cạnh tam giác

(ĐH Sư Phạm Kỹ Thuật Tp.HCM 1993)

൅ •‹ ൌ ʹ •‹ ͸Ͳ୭…‘•

ξ͵ ʹ

͵ ൅ ξ͵ ʹ

Vậy ൌ ͻͲ୭ǡ ൌ ͸Ͳ୭ǡ ൌ ͵Ͳ୭

(154)

Suy

ʹ ൅ ൅ ξ͵

ʹ ൌ ͳͲͲ ฻ ە ۖ ۖ ۔ ۖ ۖ

ۓ ൌ ʹͲͲ ͵ ൅ ξ͵ ൌ ͳͲͲ

͵ ൅ ξ͵ ൌ ͳͲͲξ͵ ͵ ൅ ξ͵

฻ ͳ െ ʹ •‹ଶ

ʹ൅ ʹ …‘• ൅

ʹ ൌ ξʹ ฻ ͳ െ ξʹ ൌ ʹ •‹ଶ

ʹ

ʹ ൒ ʹ •‹ଶ

ʹെ ʹ •‹ ʹ ฻ •‹ଶ

ʹെ •‹ ʹ

Ta xét hàm số

݂ሺݔሻ ൌ ݔଶെ ݔǡ ݔ א ቈξʹ

ʹ Ǣ ͳቇ ݂ᇱሺݔሻ ൌ ʹݐ െ ͳ ൐ Ͳ

Do đó, hàm số݂ሺݔሻđồng biến Suy

൜ …‘• ൅ …‘• ൅ …‘• ൌ ξʹ …‘•ଶ ൅ …‘•ଶ ൅ …‘•ଶ ൒ ͳ

Bài 4:Xác định góc tam giác thỏa điều kiện sau :

Giải:Theo đẳng thức bản, ta có :

ͳ െ ʹ …‘• …‘• …‘• ൌ …‘•ଶ ൅ …‘•ଶ ൅ …‘•ଶ ൒ ͳ ฺ …‘• …‘• …‘• ൑ Ͳ

Do đó, tam giác tồn góc tù vuông Chọn ൒ ͻͲ୭ Ta xét : …‘• ൅ …‘• ൅ …‘• ൌ ξʹ

…‘• െ ʹ െ ʹ •‹

ʹ…‘•

ʹ ൑

ͳ െ ξʹ

(155)

݂ሺݔሻ ൒ ݂ ቆξʹ ʹ ቇ ൌ

ͳ െ ξʹ ʹ

Như vậy, ta :

ە ۔

ۓ •‹ ʹ ൌ

ξʹ ʹ …‘• െ

ʹ ൌ ͳ

฻ ቄ ൌ ͻͲ୭ ൌ ൌ Ͷͷ୭

Giải:Ta xét trường hợp sau :

- ൐ ͻͲ୭ …‘• ൏ Ͳ Mà theo định lý hàm số cos, ta có

…‘• ൌ ܽ

ଶ൅ ܾଶെ ܿଶ

ʹܾܽ ൏ Ͳ ฺ ܽଶ൅ ܾଶ ൏ ܿଶ

Mặt khác, theo định lý hàm số sin, ta suy :

మ೙శభ

- మ೙శభ ൏ ͳ

•‹ଶ ൅ •‹ଶ ൌͳ െ …‘• ʹ

ʹ ൅

ͳ െ …‘• ʹ

ʹ ൌ ͳ െ …‘•ሺ ൅ ሻ …‘•ሺ െ ሻ ൌ ͳ ൅ …‘• …‘•ሺ െ ሻ

Vì góc nhọn nên …‘• ൐ Ͳ …‘•ሺ െ ሻ ൐ Ͳ Do đó,

ඥ•‹ଶ

మ೙శభ

ൌ •‹ଶ ൅ •‹ଶ ൌ ͳ ൅ …‘• …‘•ሺ െ ሻ ൐ ͳ

Điều mâu thuẫn

•‹ଶ ൅ •‹ଶ ൌ మ೙శభඥ•‹ଶ

Bài 5: Cho góc tam giác thỏa mãn hệ thức Biết góc ǡ nhọn Hãy tính giá trị góc

•‹ଶ ൅ •‹ଶ ൏ •‹ଶ ൏ ඥ•‹ଶ

Điều mâu thuẫn với giả thuyết

(156)

- ൌ ͻͲ୭, dễ thấy giá trị thỏa đẳng thức đã cho.

Giải: Theo công thức biến đổi định lý hàm số sin, giả thuyết tương đương với

…‘• ൅ ʹ ൅ …‘•

െ ʹ ൌ

ͳ ʹ൅

•‹ ൅ •‹ •‹ Ǥ •‹

ʹ ฻ •‹

ʹ൅ …‘• െ

ʹ ൌ ͳ ʹ൅

ʹ •‹ ฻ •‹

ʹ൅ …‘• െ

ʹ ൌ ͳ ʹ ฻ •‹

ʹൌ ͳ

ʹ฻ ൌ ͸Ͳ୭

Giải: Giả thuyết tương đương với

…‘• Ǥ ʹ •‹ ൅ ʹ …‘•

ʹ ൌ ʹ •‹ ʹ…‘•

ʹ…‘•ሺ െ ሻ ฻ …‘• …‘• െ

ʹ ൌ •‹

ʹ…‘•ሺ െ ሻ ฻ ൬ͳ െ ʹ •‹ଶ

ʹ൰ …‘• െ

ʹ ൌ •‹

ʹ൬ʹ …‘•ଶ

െ ʹ െ ͳ൰ ฻ …‘• െ

ʹ ൅ •‹

ʹൌ ʹ •‹ ʹ…‘•

െ ʹ ൬•‹

ʹ൅ …‘• െ

ʹ ൰ ʹ …‘•

ʹ…‘• ʹ ൌ

ͳ ʹ൅

ܾ ൅ ܿ ܽ •‹

ʹ

Bài 6: Cho tam giác có góc thỏa mãn điều kiện Hãy tính góc

(ĐH Mỏ-Địa Chất Hà Nội 1997)

ʹ ൅

…‘• ʹ െ ʹ •‹ʹ …‘•ʹ •‹

ʹ ൅ …‘• െ

ʹ

Bài 7: Cho tam giác có góc thỏa mãn hệ thức

…‘• ሺ•‹ ൅ •‹ ሻ ൌ •‹ …‘•ሺ െ ሻ

(157)

Mặt khác,

•‹ ʹ൅ …‘•

െ ʹ ് Ͳ

Nên

ʹ •‹ ʹ…‘•

ʹ ൌ ͳ ฻ …‘• ൅ …‘• ൌ ͳ

Giải:

ʹ ൌ

ܽଶ

Ͷܾܿ ͳ

ʹሺͳ െ …‘• ሻ ൌ ฻ ͳ െ

ܾଶ൅ ܿଶെ ܽଶ

ʹܾܿ ൌ ܽଶ

ʹܾܿ ฻ ሺܾ െ ܿሻଶ ൌ Ͳ ฻ ܾ ൌ ܿ

Vậy tam giác cân b Giả thuyết tương đương với

ʹሺ…‘•ଶ ൅ …‘•ଶሻ ൌ …‘•ଶ ൅ •‹ଶ …‘–ଶ ൅ •‹ଶ …‘–ଶ ൅ …‘•ଶ

฻ …‘•ଶ െ •‹ଶ …‘•ଶ ൌ •‹ଶ …‘–ଶ െ …‘•ଶ

฻ •‹ଶ ሺ…‘–ଶ െ …‘–ଶሻ ൌ •‹ଶ ሺ…‘–ଶ െ …‘–ଶሻ

฻ ሺ…‘–ଶ െ …‘–ଶሻሺ•‹ଶ െ •‹ଶሻ ൌ Ͳ

ƒǤ •‹ ʹ ൌ

ܽ ʹξܾܿ „Ǥ…‘•

ଶ ൅ …‘•ଶ

•‹ଶ ൅ •‹ଶ ൌ

ͳ

ʹሺ…‘–ଶ ൅ …‘–ଶሻ …Ǥ •‹

ʹ…‘•ଷ ʹ ൌ •‹

ʹ

Bài 8: Chứng minh tam giác cân có góc thỏa mãn hệ thức

(ĐH Thương Mại 1999)

a Giả thuyết tương đương với

•‹ଶ

ܽଶ

Ͷܾܿ

…‘•ଷ

ʹ

(ĐH Khoa Học Tự Nhiên Hà Nội 1994)

(158)

฻ ቂ…‘–ଶ ൌ …‘–ଶ

•‹ଶ ൌ •‹ଶ ฻ ቂ…‘– ൌ േ …‘– •‹ ൌ •‹ ฻ ൌ

Vậy tam giác cân c Giả thuyết tương đương với

•‹ʹ …‘•ʹǤ

ͳ …‘•ଶ

ʹ

ൌ •‹ ʹ …‘•ʹǤ

ͳ …‘•ଶ

ʹ ฻ –ƒ

ʹ൬ͳ ൅ –ƒଶ

ʹ൰ ൌ –ƒ

ʹ൬ͳ ൅ –ƒଶ ʹ൰ ฻ –ƒଷ

ʹെ –ƒଷ ʹ൅ –ƒ

ʹെ –ƒ

ʹ ൌ Ͳ ฻ ൬–ƒ

ʹെ –ƒ

ʹ൰ ቌ–ƒଶ

ʹ൅ –ƒଶ ʹ൅ –ƒ

ʹ–ƒ

ʹ൅ ͳ ᇣᇧᇧᇧᇧᇧᇧᇧᇧᇧᇧᇤᇧᇧᇧᇧᇧᇧᇧᇧᇧᇧᇥ

வ଴

฻ –ƒ

ʹ ൌ –ƒ

ʹ ฻ ൌ

Vậy tam giác cân

Giải:

a Ta lấy hệ thức trừ cho hệ thức :

•‹ െ •‹ ൌ •‹ ሺ…‘• െ …‘• ሻ ƒǤ ቊ

„Ǥͳ ൅ …‘• •‹ ൌ …Ǥʹሺͳ ൅ …‘• ሻ

•‹ ൌ –ƒ ൅ –ƒ

(ĐH Thủy Lợi Hà Nội 2000)

(ĐH Giao Thông Vận Tải 2001)

(ĐH Kiến Trúc Tp.HCM 2001)

ቍ ൌ Ͳ

•‹ ൌ ൫ξʹ െ …‘• ൯ •‹ •‹ ൌ ൫ξʹ െ …‘• ൯ •‹

ʹܽ ൅ ܿ ξͶܽଶ െ ܿଶ

(159)

฻ ʹ …‘• ൅ ʹ •‹

ʹ ൌ െʹ •‹ •‹ ൅

ʹ •‹ െ

ʹ ฻ •‹ െ

ʹ ቌ…‘• ൅

ʹ ൅ •‹ •‹ ൅

ʹ ᇣᇧᇧᇧᇧᇧᇧᇧᇤᇧᇧᇧᇧᇧᇧᇧᇥ

வ଴

ቍ ൌ Ͳ ฻ •‹ െ

ʹ ൌ Ͳ ฻ ൌ

Vậy tam giác cân b Giả thuyết tương đương với

ሺͳ ൅ …‘• ሻଶ

•‹ଶ ൌ

ሺʹܽ ൅ ܿሻଶ

Ͷܽଶെ ܿଶ

฻ ͳ ൅ …‘• ͳ െ …‘• ൌ

ʹܽ ൅ ܿ ʹܽ െ ܿ ฻ ͳ ൅ …‘•

ͳ െ …‘• ൅ ͳ ൌ

ʹܽ ൅ ܿ ʹܽ െ ܿ൅ ͳ ฻ ͳ

ͳ െ …‘• ൌ ʹܽ

ʹܽ െ ܿ ฻ ܿ ൌ ʹܽ …‘• ฻ ܿ ൌ ʹܽ ቆܿ

ଶ൅ ܽଶെ ܾଶ

ʹܽܿ ቇ ฻ ܽ ൌ ܾ

Vậy tam giác cân c Giả thuyết tương đương với

Ͷ …‘•ଶ

ʹ ʹ •‹ʹ …‘•ʹ ൌ

•‹ሺ ൅ ሻ …‘• …‘• ฻ ʹ …‘•

ʹ •‹ʹ ൌ

•‹ …‘• …‘• ฻ ʹ …‘• …‘• ൌ ʹ •‹ଶ

ʹ

฻ …‘•ሺ ൅ ሻ ൅ …‘•ሺ െ ሻ ൌ ͳ െ …‘• ฻ …‘•ሺ െ ሻ ൌ ͳ ฻ ൌ

(160)

Giải:

a Giả thuyết tương đương với

ܽ ൬–ƒ ൅

ʹ െ –ƒ ൰ ൌ ܾ ൬–ƒ െ –ƒ

൅ ʹ ൰ ฻ ܽǤ•‹ ቀ

൅ ʹ െ ቁ …‘• ൅ ʹ …‘• ൌ ܾǤ

•‹ ቀ െ ൅ ʹ ቁ …‘• …‘• ൅ ʹ ฻ ܽǤ•‹

െ ʹ

…‘• ൌ ܾǤ

•‹ െ ʹ …‘• ฻ •‹ െ

ʹ ൬

ʹܴ •‹ …‘• െ

ʹܴ •‹ …‘• ൰ ൌ Ͳ ฻ ൥•‹ െ ʹ ൌ Ͳ

–ƒ ൌ –ƒ

฻ ൌ

Vậy tam giác cân b Giả thuyết tương đương với

•‹ –ƒ ൅ •‹ –ƒ ൌ ሺ•‹ ൅ •‹ ሻ –ƒ ൅ ʹ ฻ •‹ ൬–ƒ െ –ƒ ൅

ʹ ൰ ൅ •‹ ൬–ƒ െ –ƒ

൅ ʹ ൰ ൌ Ͳ ƒǤܽ ൬…‘–

ʹെ –ƒ ൰ ൌ ܾ ൬–ƒ െ …‘– ʹ൰ „Ǥ•‹

…‘• ൅

•‹ଶ

…‘• ൌ

•‹ ൅ •‹ –ƒʹ …Ǥܽଶ•‹ ʹ ൅ ܾଶ•‹ ʹ ൌ ܿଶ

–ƒʹ †Ǥݎ ൌ Ͷܴ …‘• െ ݎ௔

(161)

฻ •‹ Ǥ•‹ ቀ െ

൅ ʹ ቁ

…‘• …‘• ൅ ʹ ൅ •‹ Ǥ

•‹ ቀ െ ൅ ʹ ቁ …‘• …‘• ൅ ʹ ൌ Ͳ ฻ •‹ Ǥ•‹

െ ʹ

…‘• ൅ •‹ Ǥ

•‹ െ ʹ …‘• ൌ Ͳ ฻ •‹ െ

ʹ ሺ–ƒ െ –ƒ ሻ ൌ Ͳ ฻ ൥•‹ െ ʹ ൌ Ͳ

–ƒ ൌ –ƒ

฻ ൌ

Vậy tam giác cân

c Theo định lý hàm số sin, ta có :

Do đó,

ͶܾܽǤ •‹ ʹ…‘•

ʹൌ

ܿଶ

–ƒʹ ฻ Ͷܾܽ •‹ଶ

ʹ ൌ ܿଶ

d Ta có :

൞ ʹ

ฺ ݎ ൅ ݎ௔ ൌ ሺʹ݌ െ ܽሻ –ƒ

ʹ ൌ ሺܾ ൅ ܿሻ –ƒ ʹ

Theo định lý hàm sốsin, ta

ݎ ൅ ݎ௔ ൌ ʹܴሺ•‹ ൅ •‹ ሻǤ

•‹ʹ

…‘•ʹ ൌ ͶܴǤ …‘• ʹ…‘•

െ ʹ Ǥ

•‹ʹ …‘•ʹ ൌ ൌ Ͷܴ …‘• ൅

ʹ …‘• െ

ʹ ൌ ʹܴሺ…‘• ൅ …‘• ሻ

Do đó,

ʹܴሺ…‘• ൅ …‘• ሻ ൌ Ͷܴ …‘• ฻ …‘• ൌ …‘• ฻ ൌ

Vậy tam giác cân

ൌ Ͷܴଶ•‹ଶ Ǥ ʹ •‹ …‘• ൅ Ͷܴଶ•‹ଶ Ǥ ʹ •‹ …‘• ൌ ͺܴଶ•‹ •‹ •‹ሺ ൅ ሻ

ൌ ሺʹܴ •‹ ሻሺʹܴ •‹ ሻʹ •‹ ൌ ͶܾܽǤ •‹ ʹ…‘•

ʹ ฻ ʹܾܽሺͳ െ …‘• ሻ ൌ ܽଶ൅ ܾଶെ ʹܾܽ …‘•

฻ ʹܾܽ ൌ ܽଶ൅ ܾଶ ฻ ܽ ൌ ܾ

Vậy tam giác cân

(162)

Giải:

a Theo bất đẳng thức Cauchy, ta có :

ͳ

Ͷሺܽ ൅ ܾሻଶ ൒ ͳ ʹܾܽ ൒

ͳ ʹ ൞

ʹܾܿ

ܾ ൅ ܿ ʹ ൒

ʹܾܿ ܾ ൅ ܿ…‘•

ʹ

Suy

ξܾܿ …‘•

ʹ ൒ ݈௔ ൒ ݄௔

c Ta có :

݉௔ଶ ൌͳ

Ͷሺʹܾଶ൅ ʹܿଶെ ܽଶሻ ൌ ͳ

Ͷሾܾଶ൅ ܿଶ൅ ሺܾଶ൅ ܿଶെ ܽଶሻሿ ൒ ͳ

Ͷሺܾଶ൅ ܿଶ൅ ʹܾܿ …‘• ሻ

Mặt khác, theo bất đẳng thức Cauchy, ta có :

݉௔ଶ ൌͳ

Ͷሺܾଶ൅ ܿଶ൅ ʹܾܿ …‘• ሻ ൒ ͳ

Ͷሺʹܾܿ ൅ ʹܾܿ …‘• ሻ ൌ ͳ

ʹܾܿሺͳ ൅ …‘• ሻ ൌ ܾܿ …‘•ଶ ʹ

Suy

݉௔ ൒ ξܾܿ …‘•

ʹ൬˜¿ …‘• ʹ ൒ Ͳ൰

Dấu ̶ ൌ ̶ xảy ܾ ൌ ܿ Do đó, tam giác cân

ƒǤܵ ൌͳ

Ͷሺܽଶ ൅ ܾଶሻ „Ǥ݄௔ ൌ ξܾܿ …‘•

ʹ …Ǥ݉௔ ൌ ξܾܿ …‘•

ʹ

†Ǥ •‹ସ ൅ ʹ •‹ସ ൅ ʹ •‹ସ ൌ ʹ •‹ଶ ሺ•‹ଶ ൅ •‹ଶሻ

Bài 11: Chứng minh tam giác cân thỏa mãn hệ thức

ܾܽ •‹ ൌ ܵ

Dấu ̶ ൌ ̶ xảy ܽ ൌ ܾ Do đó, tam giác cân b Ta ln có :

݄௔൑ ݈௔

Mặt khác, theo bất đẳng thức Cauchy, ta có :

൑ ξܾܿ …‘•

ʹ൒ Ͳ

ฺ ξܾܿ …‘•

(163)

d Theo định lý hàm số sin, giả thuyết tương đương với :

ͳ͸ܴସሺܿସ൅ ʹܽସ൅ ʹܾସሻ ൌ ͳ͸ܴସǤ ʹܿଶሺܽଶ൅ ܾଶሻ

฻ ܿସെ ʹሺܽଶ ൅ ܾଶሻܿଶ൅ ʹܽସ൅ ʹܾସ ൌ Ͳ

ƒ–Šấy phương trình bậc hai theo ݔ ൌ ܿଶ, ta xét :

ȟᇱ ൌ ሺܽଶ൅ ܾଶሻଶെ ሺʹܽସ൅ ʹܾସሻ ൌ െሺܽସെ ʹܽଶܾଶ൅ ܾସሻ ൌ െሺܽଶെ ܾଶሻଶ ൑ Ͳ

Do đó, phương trình có nghiệm

ቄ ȟᇱ ൌ Ͳ

ݔ ൌ ܽଶ൅ ܾଶ ฻ ൜ ܽ

ଶെ ܾଶ ൌ Ͳ

ܿଶ ൌ ܽଶ൅ ܾଶ

Vậy tam giác vuông cân

Giải:

a Giả thuyết tương đương với

ʹ •‹ሺ ൅ ሻ …‘•ሺ െ ሻ ൌ ʹሾ…‘•ሺ െ ሻ െ …‘•ሺ ൅ ሻሿ ฻ •‹ …‘•ሺ െ ሻ ൌ …‘•ሺ െ ሻ ൅ …‘•

฻ ሺͳ െ •‹ ሻ …‘•ሺ െ ሻ ൅ …‘• ൌ Ͳ

฻ …‘•ଶ …‘•ሺ െ ሻ ൅ ሺͳ ൅ •‹ ሻ …‘• ൌ Ͳ

฻ …‘• ቈ…‘• …‘•ሺ െ ሻ ൅ ͳ ൅ •‹ ᇣᇧᇧᇧᇧᇧᇧᇧᇧᇤᇧᇧᇧᇧᇧᇧᇧᇧᇥ

வ଴

቉ ൌ Ͳ ฻ ൌ ͻͲ୭

Vậy tam giác vuông

ƒǤ •‹ ʹ ൅ •‹ ʹ ൌ Ͷ •‹ •‹ „Ǥ …‘• ൅ …‘• ൌܾ ൅ ܿ

ܽ …Ǥ ܾ

…‘• ൅ ܿ …‘• ൌ †Ǥ …‘•

ʹ…‘• ʹ…‘•

ʹ •‹ ʹ ൌ

ͳ ʹ

Bài 12: Chứng tỏ tam giác vuông thỏa mãn hệ thức

(ĐH Đà Nẵng 1997)

(ĐH Ngoại Thương 2001)

ܽ •‹ •‹

െ •‹ ʹ•‹

ʹ

(ĐH Kinh Tế Tp.HCM 1990)

(164)

b Theo định lý hàm số sin, giả thuyết tương đương với …‘• ൅ …‘• ൌ •‹ ൅ •‹ •‹ ฻ ʹ …‘• ൅ ʹ …‘• െ ʹ ൌ ʹ •‹ ൅ ʹ …‘• െ ʹ ʹ •‹ʹ …‘•ʹ ฻ •‹ ʹ…‘• െ ʹ ൌ …‘•ʹ …‘• െ ʹ ʹ •‹ʹ …‘•ʹ ฻ ʹ •‹ଶ ʹ ൌ ͳ ฻ ൌ ͻͲ୭

Vậy tam giác vuông

c Theo định lý hình chiếu, ta có :

ൌ ܾ …‘• ൅ ܿ …‘• …‘• …‘• ൌ

Nên từ giả thuyết, ta :

ܽ

…‘• …‘• ൌ

ܽ •‹ •‹ ฻ …‘•ሺ ൅ ሻ ൌ Ͳ ฻ ൌ ͻͲ୭

Vậy tam giác vuông d Giả thuyết tương đương với

൬•‹ ʹ൅ …‘• െ ʹ ൰ …‘• ʹെ ൬…‘• െ •‹ ʹ൰ •‹ ʹ ൌ ͳ ฻ ൬•‹ ʹെ …‘• ʹ൰ …‘• ʹ ʹ…‘• ʹ൅ ͳ െ •‹ଶ ʹ ൌ Ͳ ฻ ൬•‹ ʹെ …‘• ʹ ʹ െ •‹ ʹ…‘• ʹ൅ …‘•ଶ ʹൌ Ͳ ฻ ൬•‹ ʹ ʹ൰ …‘• ʹ െ ൬•‹ ʹെ …‘• ʹ൰ …‘• ʹൌ Ͳ ฻ ൬•‹ ʹെ …‘• ʹ൰ ൬…‘• െ ʹ െ …‘• ʹ൰ ൌ Ͳ ฻ ൦ •‹ ʹ ൌ …‘• ʹ …‘• െ ʹ ൌ …‘• ʹ ฻ ൦ –ƒ ʹ ൌ ͳ െ ʹ ൌ േ ʹ ฻ ቈ ൌ ͻͲ ୭ ൌ ͻͲ୭ ൌ ͻͲ୭

Vậy tam giác vuông

(165)

Giải:

ʹሺ͵ •‹ ൅ Ͷ …‘• ሻ ൑ ͳͲ

Dấu ̶ ൌ ̶ xảy

൞ …‘•

͵ ൌ Ͷ •‹

ൌ…‘• Ͷ

ݐ ൌ –ƒ ʹ ൐ Ͳ

Ta

ʹݐ

ͳ ൅ ݐଶǤ …‘•ሺ െ ሻ ൌ …‘•ሺ െ ሻ ൅

ͳ െ ݐଶ

ͳ ൅ ݐଶ

฻ ʹݐǤ …‘•ሺ െ ሻ ൌ ሺͳ ൅ ݐଶሻ …‘•ሺ െ ሻ ൅ ͳ െ ݐଶ

฻ ሺݐ െ ͳሻଶ…‘•ሺ െ ሻ ൅ ͳ െ ݐଶ ൌ Ͳ

฻ ሺͳ െ ݐሻሾሺͳ െ ݐሻ …‘•ሺ െ ሻ ൅ ͳ ൅ ݐሿ ൌ Ͳ ฻ ൥ݐሾͳ െ …‘•ሺ െ ሻሿᇣᇧᇧᇧᇧᇧᇤᇧᇧᇧᇧᇧᇥݐ ൌ ͳ

வ଴

൅ ሾͳ ൅ …‘•ሺ െ ሻሿᇣᇧᇧᇧᇧᇤᇧᇧᇧᇧᇥ

வ଴

ൌ Ͳ ƒǤ͵ሺ…‘• ൅ ʹ •‹ ሻ ൅ Ͷሺ•‹ ൅ ʹ …‘• ሻ ൌ ͳͷ „Ǥ •‹ሺ ൅ ሻ …‘•ሺ െ ሻ ൌ ʹ •‹ •‹

…Ǥ –ƒ ʹ ൌ ʹܾܿ ܾଶെ ܿଶ

Bài 13: Chứng minh tam giác vng thỏa mãn hệ thức

(ĐH Cần Thơ 1996)

(ĐH Sư Phạm Vinh 2001)

a Từ giả thuyết, ta viết lại thành

͵ …‘• ൅ Ͷ •‹ ൅ ʹሺ͵ •‹ ൅ Ͷ …‘• ሻ ൌ ͳͷ

Theo bất đẳng thức Bunyakovsky, ta có :

൜ ͵ …‘• ൅ Ͷ •‹ ൑ ͷ •‹

ฺ –ƒ ൌ …‘– ฺ ൅ ൌ ͻͲ୭

͵

Vậy tam giác vuông b Giả thuyết tương đương với

•‹ሺ ൅ ሻ …‘•ሺ െ ሻ ൌ …‘•ሺ െ ሻ െ …‘•ሺ ൅ ሻ ฻ •‹ …‘•ሺ െ ሻ ൌ …‘•ሺ െ ሻ ൅ …‘•

(166)

฻ ݐ ൌ –ƒ

ʹ ൌ ͳ ฻ ൌ ͻͲ୭

Vậy tam giác vuông c Theo định lý hàm số sin, ta có

ൌ ͺܴ

ଶ•‹ •‹

Ͷܴଶሺ•‹ଶ െ •‹ଶሻ ൌ

ʹ •‹ •‹ •‹ଶ െ •‹ଶ

Do đó, giả thuyết tương đương với

ʹ •‹ …‘• …‘• ʹ ൌ

ʹ •‹ •‹ •‹ଶ െ •‹ଶ

ۏ ێ ێ ێ

ۍ…‘• ൌͳ ൅ ξͷ ʹ …‘• ൌͳ െ ξͷ

ʹ

Vậy tam giác vuông

฻ …‘• ሺ•‹ଶ െ •‹ଶሻ ൌ •‹ ሺ…‘•ଶ െ •‹ଶሻ

฻ •‹ …‘• ሺ•‹ െ …‘• ሻ ൅ •‹ଶ ሺ•‹ െ •‹ ሻ ൌ Ͳ

฻ ሺ•‹ െ …‘• ሻሺ•‹ …‘• ൅ •‹ଶሻ ൌ Ͳ

•‹ ൌ …‘• ሺͳሻ ฻ ൤

•‹ …‘• ൅ •‹ଶ ൌ Ͳሺʹሻ

Ta xét :

ሺͳሻ ฻ •‹ ൌ •‹ሺͻͲ୭െ ሻ ฻ ൅ ൌ ͻͲ୭

Vậy tam giác vuông

ሺʹሻ:

- Nếu tam giác vng •‹ ൌ …‘• ฺ …‘•ଶ ൅ •‹ଶ ൌ Ͳ Điều này

vô lý

- Nếu tam giác vuông ǡ phải nhọn

•‹ ൌ ͳ ฺ …‘• ൅ ͳ െ …‘•ଶ ൌ Ͳ ฻

Điều vơ lý

(167)

Giải: Ta có :

ܵ ൌ ݌ሺ݌ െ ܽሻ –ƒ

ʹ ൌ ݌ሺ݌ െ ܾሻ –ƒ ʹ ฺ –ƒ

ʹ–ƒ ʹ ൌ

ܵଶ

݌ଶሺ݌ െ ܽሻሺ݌ െ ܾሻ ൌ ൌ

ܽ ൅ ܾ െ ܿ ܽ ൅ ܾ ൅ ܿ

Do đó, từ giả thuyết ta có :

ͳ ʹ ൌ

Mặt khác :

ܵ ൌ ݌ݎ ൌ ݎ

Ͷܴ ൌ ܵଶൌ ݌ሺ݌ െ ܽሻሺ݌ െ ܾሻሺ݌ െ ܿሻሺʹሻ ܾܽǤ ൌ ݌ଶെ ሺܽ ൅ ܾሻ݌ ൅ ܾܽ ฺ ቀͳ െ ݎ

Ͷܴቁ ܾܽ ൌ ʹܿଶ •‹

ʹ•‹ ʹ•‹

ʹ ൌ

ͳ ͳͲฺ

ݎ Ͷܴ ൌ

ͳ ͳͲ

Do đó,

ܾܽ ൌ ʹͲ

ͻ ܿଶ ฺ ʹͲܽଶ൅ ʹͲܾଶെ Ͷͳܾܽ ൌ Ͳ –ƒ

ʹ–ƒ ʹ ൌ

ͳ ʹ •‹

ʹ•‹ ʹ•‹

ʹൌ

ͳ ͳͲ

Bài 14: Cho tam giác thỏa mãn hệ thức :

Chứng minh điều kiện cần đủđể tam giác vuông

(Đề nghị Olympic 30-4, 2006)

ൌ ඥ݌ሺ݌ െ ܽሻሺ݌ െ ܾሻሺ݌ െ ܿሻ ݌ െ ܿ

݌ ܽ ൅ ܾ െ ܿ

ܽ ൅ ܾ ൅ ܿ ฻ ܽ ൅ ܾ ൌ ͵ܿ ฻ ݌ ൌ ʹܿሺͳሻ ܾܽܿ

ฺ ݌Ǥ ܾܽܿǤ Ͷܴ

Thay ሺͳሻ vào ሺʹሻ, ta :

ݎ Ͷܴ

(168)

ฺ ൦ ܽ ܾ ൌ

ͷ Ͷ ܽ ܾ ൌ

Ͷ ͷ

Xét ܽ ൒ ܾ, ta có :

൞ ܽ ܾ ൌ

ͷ Ͷ ܽ ܿ ൌ

ͷ ͵

ฺ ܾଶ൅ ܿଶ ൌ ܽଶ

Vậy tam giác vuông

Chiều nghịch: Giả sử tam giác vuông , ta có :

ܽ ൌ ʹܴ ܽଶ ൌ ܾଶ൅ ܿଶ

ܵ ൌ ͳ

ʹܾܿ ൌ ݌ݎ

ሺଵሻ

ሳሰ ൝

ฺ •‹ ʹ•‹

ʹ•‹

ʹൌ

ͳ ͳͲ

„Ǥ ە ۔ ۓݎ

ݎ௔ ൌ

ͳ ͵ ݎ ܴ ൌ

ʹ ͷ

…Ǥݎሺ•‹ ൅ •‹ ሻ ൌ ξʹܿǤ •‹ ʹ…‘•

െ ʹ

ܽ ൌ ʹܴ

ሺ͵ܿ െ ܾሻଶ ൌ ܾଶ൅ ܿଶ

ܾ ൌ Ͷݎ

ฺ ൝ܽ ൌ ʹܴ ܿ ൌ ͵ݎ ܾ ൌ Ͷݎ

ʹ•‹ ʹ•‹

ʹൌ

ʹ ͷ

Từሺͳሻta

ͷݎ ൌ ʹܴ ฺ Ͷ •‹

Vậy ta có điều phải chứng minh

ƒǤξʹ •‹ሺ ൅ Ͷͷ୭ሻ ൌܽଶሺܾ ൅ ܿ െ ܽሻ ൅ ܾଶሺܽ ൅ ܿ െ ܾሻ ൅ ܿଶሺܽ ൅ ܾ െ ܿሻ

ʹܾܽܿ

(169)

Giải:

a Ta có :

ൌܽ

ଶܾ ൅ ܽଶܿ െ ܽଷ൅ ܾଶܽ ൅ ܾଶܿ െ ܾଷ൅ ܿଶܽ ൅ ܿଶܾ െ ܿଷ

ʹܾܽܿ ൌܽሺܾ

ଶ൅ ܿଶെ ܽଶሻ ൅ ܾሺܽଶ ൅ ܿଶെ ܾଶሻ ൅ ܿሺܽଶ൅ ܾଶെ ܿଶሻ

ʹܾܽܿ ൌܾ

ଶ൅ ܿଶെ ܽଶ

ʹܾܿ ൅

ܽଶ൅ ܿଶെ ܾଶ

ʹܽܿ ൅

ܽଶ൅ ܾଶെ ܿଶ

ʹܾܽ ൌ …‘• ൅ …‘• ൅ …‘•

Do đó, hệ thức tương đương với

ξʹ •‹ሺ ൅ Ͷͷ୭ሻ ൌ …‘• ൅ …‘• ൅ …‘•

฻ •‹ ൅ …‘• ൌ …‘• ൅ …‘• ൅ …‘• ฻ •‹ ൌ …‘• ൅ …‘•

฻ ʹ •‹ ʹ…‘•

ʹ ൌ ʹ •‹ ʹ…‘•

െ ʹ ฻ …‘•

ʹ ൌ …‘• െ

ʹ ฻ ൦

ʹ ൌ

െ ʹ

ʹ ൌ

െ ʹ

฻ ቂ ൅ ൌ ൅ ൌ

ܵ ݌ ݎ௔ ൌ ܵ

݌ െ ܽ ฺ ͳ

͵ൌ ݎ ݎ௔ ൌ

݌ െ ܽ ݌ ͵ܽ

ʹ ฻ ܾ ൅ ܿ ൌ ʹܽሺͳሻ ൞ܵ ൌ

ܾܽܿ Ͷܴ ൌ ݌ݎ ݎ

ܴ ൌ ʹ ͷ

ฺ Ͷܵ

ܾܽܿǤ ݌ ൌ ʹ ͷ

Theo công thức Heron, ta suy

݌ሺ݌ െ ܽሻሺ݌ െ ܾሻሺ݌ െ ܿሻ ܾܽܿǤ ݌ ൌ

ͳ ͳͲ ฻ ͳͲ ൬͵ܽെ ܽ൰ ൬ܽ ൅ ܿ െ ܾ൰ ൬ܽ ൅ ܾ െ ܿ൰ ൌ ܾܽܿ

฻ ቂ ൌ ͻͲ୭ ൌ ͻͲ୭

Vậy tam giác vuông b Ta áp dụng công thức :

۔ ە ۓ ݎ ൌ ฻ ݌ ൌ ͵ሺ݌ െ ܽሻ ฻ ݌ ൌ

(170)

฻ ͷܽሾܽ െ ሺܾ െ ܿሻሿሾܽ ൅ ሺܾ െ ܿሻሿ ൌ Ͷܾܽܿ ฻ ܽଶെ ሺܾ െ ܿሻଶ ൌͶ

ͷܾܿ

฻ ͷܽଶെ ͷሺܾଶ൅ ܿଶሻ ൅ ͸ܾܿ ൌ Ͳ

Từሺͳሻ ta suy : ܾଶ൅ ܿଶ൅ ʹܾܿ ൌ Ͷܽଶ ฻ ܾଶ൅ ܿଶൌ Ͷܽଶെ ʹܾܿ Do đó,

ͷܽଶെ ͷሺܾଶ൅ ܿଶሻ ൅ ͸ܾܿ ൌ Ͳ

฻ ͷܽଶെ ͷሺͶܽଶ െ ʹܾܿሻ ൅ ͸ܾܿ ൌ Ͳ ฻ ܾܿ ൌͳͷ

ͳ͸ܽଶሺʹሻ

Từሺͳሻǡ ሺʹሻ; theo định lý Viète, ta có ܾǡ ܿ nghiệm phương trình

ݔଶെ ʹܽݔ ൅ͳͷ

ͳ͸ܽଶ ൌ Ͳ ฺ ൦

ݔ ൌ ͵ܽ Ͷ ݔ ൌ ͷܽ

Ͷ

Giả sử ܾ ൒ ܿ, suy

൞ܾ ൌ ͷܽ

Ͷ

ܿ ൌ͵ܽ ฺ ܽ

ଶ൅ͻܽଶ

ͳ͸ ൌ ʹͷܽଶ

ͳ͸ ʹ൰ ʹݎ …‘•

ʹ…‘•

െ ʹ

ʹ…‘•

െ ʹ ሺכሻ ฻ ʹ …‘•

ʹ ൌ ξʹǤ Ǥ •‹ ʹ ฻ ʹ …‘•

ʹ ൌ ξʹ …‘•ʹ •‹ʹ ฻ •‹

ʹൌ ξʹ

ʹ ฻ ൌ ͻͲ୭

Vậy tam giác vuông

฻ ܽଶ൅ ܿଶ ൌ ܾଶ

Ͷ

Vậy tam giác vng

c Ở tốn này, ta sử dụng công thức

ܿ ൌ ݎ ൬…‘– ʹ൅ …‘–

Do đó, giả thuyết tương đương với

ൌ ξʹݎ ൬…‘– ʹ൅ …‘–

ʹ൰ •‹ •‹ ൅ ʹ

(171)

Giải: Ta có

ܾଶ൅ ܿଶ ൌ ʹ݉ ௔ ଶ ൅ͳ

ʹܽଶ ൒ ʹ݉௔ܽ

Suy :

ͳ ʹ݉௔ ൒

ܽ

ܾଶ൅ ܿଶ ฻

ܽ ʹ݉௔ ൒

ܽଶ

ܾଶ൅ ܿଶ

Theo định lý hàm số sin, ta có :

ʹܴ •‹ ൒

ൌ ͳ െʹܾܿ …‘•

ܾଶ൅ ܿଶ ൒ ͳ െ …‘•

Do đó,

൒ ͳ െ …‘• ฻ʹܴ •‹ ʹ …‘•ʹ

݉௔ ൒ ʹ •‹ଶ ʹ ¿

ʹא ቀͲǢ ߨ

ʹቁ ² •‹

ʹ˜ …‘•

ʹ൐ ͲǤƒ¯ượ… ܴ

݉௔ ൒ –ƒ

ʹ

Dấu ̶ ൌ ̶ xảy tam giác vuông cân

ܴ

݉௔ ൌ –ƒ ʹ

Bài 16: Chứng minh tam giác không tù thỏa mãn hệ thức vng cân

(Đề nghị Olympic 30-4, 2007)

ܾଶ൅ ܿଶെ ʹܾܿ …‘•

ܾଶ൅ ܿଶ

ʹ݉௔

Theo bất đẳng thức Cauchy, ta lại có :

ܾଶ൅ ܿଶെ ʹܾܿ …‘•

ܾଶ൅ ܿଶ

(172)

Giải:

͵ܾܽܿ

Ͷܴ ൌ ʹܴଶቈቀ ܽ ʹܴቁ

൅ ൬ ܾ ʹܴ൰

൅ ቀ ܿ ʹܴቁ

Nên từ giả thuyết ta có :

൅ ξ͵ሺܿ •‹ ൅ ܾ •‹ ሻ

൜…‘•ሺ͸Ͳ୭ െ ሻ Ǣ…‘•ሺ͸Ͳ•‹ Ǣ •‹ ൐ Ͳ୭ െ ሻ ൑ ͳ

Do

ʹ •‹ ሾͳ െ …‘•ሺ͸Ͳ୭െ ሻሿ ൅ ʹ •‹ ሾͳ െ …‘•ሺ͸Ͳ୭െ ሻሿ ൒ Ͳ

Dấu ̶ ൌ ̶ xảy

ƒǤ͵ܵ ൌ ʹܴଶሺ•‹ଷ ൅ •‹ଷ ൅ •‹ଷሻ

„Ǥܾ ൅ ܿ ൌܽ

ʹ൅ ݄௔ξ͵

…Ǥʹሺܽ …‘• ൅ ܾ …‘• ൅ ܿ …‘• ሻ ൌ ܽ ൅ ܾ ൅ ܿ †Ǥ •‹ ൅ •‹

൅ ൅

•‹ ൅ •‹ ൅ ൅

•‹ ൅ •‹

൅ ൌ •‹ ൅ •‹ ൅ •‹

Bài 17: Chứng minh tam giác thỏa mãn hệ thức

a Theo cơng thức tính diện tích định lý hàm số sin, giả thuyết tương đương với

฻ ܽଷ൅ ܾଷ൅ ܿଷ ൌ ͵ܾܽܿ

Mặt khác, theo bất đẳng thức Cauchy, ta có :

ܽଷ൅ ܾଷ൅ ܿଷ ൒ ͵ܾܽܿ

Dấu ̶ ൌ ̶ xảy ܽ ൌ ܾ ൌ ܿ Vậy tam giác

b Trong tam giác ta ln có :

݄௔ ൌ ܿ •‹ ൌ ܾ •‹ ܾ ൅ ܿ ൌ ܽ

ʹ

Theo định lý hàm số sin từđẳng thức trên, ta có :

ʹ •‹ ൅ ʹ •‹ ൌ •‹ ൅ ξ͵ሺ•‹ •‹ ൅ •‹ •‹ ሻ

Ta viết lại đẳng thức thành

ʹ •‹ ൅ ʹ •‹ ൌ ൫ξ͵ ൅ ͳ൯ሺ•‹ •‹ ൅ •‹ •‹ ሻ ฻ ʹ •‹ ሾͳ െ …‘•ሺ͸Ͳ୭െ ሻሿ ൅ ʹ •‹ ሾͳ െ …‘•ሺ͸Ͳ୭െ ሻሿ ൌ Ͳ

(173)

൜…‘•ሺ͸Ͳ୭െ ሻ ൌ ͳ

…‘•ሺ͸Ͳ୭െ ሻ ൌ ͳ ฻ ൌ ൌ ͸Ͳ୭

Vậy tam giác

c Theo định lý hình chiếu, giả thuyết tương đương với

ʹሺܽ …‘• ൅ ܾ …‘• ൅ ܿ …‘• ሻ

ൌ ሺܾ …‘• ൅ ܿ …‘• ሻ ൅ ሺܿ …‘• ൅ ܽ …‘• ሻ ൅ ሺܽ …‘• ൅ ܾ …‘• ሻ ൌ

Hệ thức viết lại thành

ሺܽ െ ܾሻሺ…‘• െ …‘• ሻ ൅ ሺܾ െ ܿሻሺ…‘• െ …‘• ሻ ൅ ሺܿ െ ܽሻሺ…‘• െ …‘• ሻ ൌ Ͳ

Mà tam giác ta ln có :

൅ܿ ൅ ܽ

൅ ൌ ܽ ൅ ܾ ൅ ܿ ሺ െ ሻሺܾ െ ܿሻ

ሺ ൅ ሻሺܾ ൅ ܿሻ൅

ሺ െ ሻሺܿ െ ܽሻ ሺ ൅ ሻሺܿ ൅ ܽሻ቉ ൌ Ͳ ቐ

ሺ െ ሻሺܽ െ ܾሻ ൒ Ͳ ሺ െ ሻሺܾ െ ܿሻ ൒ Ͳ ሺ െ ሻሺܿ െ ܽሻ ൒ Ͳ ฺ ሺ െ ሻሺܽ െ ܾሻ

ሺ ൅ ሻሺܽ ൅ ܾሻ൅

ሺ െ ሻሺܾ െ ܿሻ ሺ ൅ ሻሺܾ ൅ ܿሻ൅

ሺ െ ሻሺܿ െ ܽሻ ሺ ൅ ሻሺܿ ൅ ܽሻ൒ Ͳ

Dấu ̶ ൌ ̶ xảy ቄܽ ൌ ܾ ൌ ܿ ൌ ൌ

Vậy tam giác

ሺܽ െ ܾሻሺ…‘• െ …‘• ሻ ൑ Ͳ ቐሺܾ െ ܿሻሺ…‘• െ …‘• ሻ ൑ Ͳ ሺܿ െ ܽሻሺ…‘• െ …‘• ሻ ൑ Ͳ

ฺ ሺܽ െ ܾሻሺ…‘• െ …‘• ሻ ൅ ሺܾ െ ܿሻሺ…‘• െ …‘• ሻ ൅ ሺܿ െ ܽሻሺ…‘• െ …‘• ሻ ൑ Ͳ

Dấu ̶ ൌ ̶ xảy ቄܽ ൌ ܾ ൌ ܿ ൌ ൌ

Vậy tam giác

d Theo định lý hàm số sin, giả thuyết tương đương với

ܽ ൅ ܾ

൅ܾ ൅ ܿ ൅ ൅

Hệ thức viết lại thành

ͳ ሺ െ ሻሺܽ െ ܾሻ ቈ ൅ ʹ ሺ ൅ ሻሺܽ ൅ ܾሻ

(174)

Giải:

a Ta kí hiệu

ฺ ʹ •‹ ൐ •‹ ൅ •‹ ൜ሺ•‹ ൅ •‹ ሻଶ ൒ Ͷ •‹ଶ

ሺ…‘• ൅ …‘• ሻଶ ൒ Ͷ …‘•ଶ

ฺ ሺ•‹ ൅ •‹ ሻଶ൅ ሺ…‘• ൅ …‘• ሻଶ ൒ Ͷሺ•‹ଶ ൅ …‘•ଶሻ

฻ ʹ ൅ ʹ …‘•ሺ െ ሻ ൒ Ͷ ฻ …‘•ሺ െ ሻ ൒ ͳ

Vì …‘•ሺ െ ሻ ൑ ͳ nên …‘•ሺ െ ሻ ൌ ͳ Vậy ൌ

Từሺͳሻ ta có •‹ ൒ •‹ ฻ ܽ ൒ ܿ ฻ ൒ ƒǤ ቄ •‹ ൅ •‹ ൒ ʹ •‹ …‘• ൅ …‘• ൒ ʹ …‘•

„Ǥ ൝•‹ ͵ ൅ •‹ ͵ ൅ •‹ ͵ ൌ Ͳ…‘• …‘• ൌ •‹ଶ ʹ …Ǥ ൞ …‘• …‘• ൌ

ͳ Ͷ ܽଶ ൌܽଷെ ܾଷെ ܿଷ

ܽ െ ܾ െ ܿ †Ǥ ቄ •‹ ൅ •‹ ൌ ʹ •‹

–ƒ ൅ –ƒ ൌ ʹ –ƒ

Bài 18: Chứng minh tam giác thỏa mãn hệ thức

(ĐH Kiến Trúc Hà Nội 1997)

•‹ ൅ •‹ ൒ ʹ •‹ ሺͳሻ ൜…‘• ൅ …‘• ൒ ʹ …‘• ሺʹሻ

Từሺͳሻ ta nhận xét khơng góc lớn lớn cạnh đối diện ܿ lớn theo định lý hàm số sin, ta có

ቄ•‹ ൐ •‹ •‹ ൐ •‹

Điều mâu thuẫn với giả thuyết

Vậy phải góc nhọn Ta : …‘• ൐ Ͳ nên vế bất đẳng thức ሺͳሻ ሺʹሻđều

dương Do

(ĐH Ngoại Ngữ Hà Nội 1997)

(175)

…‘• ൒ …‘• ฺ ൑

Như vậy, ൌ

Tóm lại, ta chứng minh tam giác b Ta có :

…‘• …‘• ൌ •‹ଶ

ʹ ฻ ͳ

ʹሾ…‘•ሺ ൅ ሻ ൅ …‘•ሺ െ ሻሿ ൌ ͳ

ʹሺͳ െ …‘• ሻ ฻ െ …‘• ൅ …‘•ሺ െ ሻ ൌ ͳ െ …‘•

฻ …‘•ሺ െ ሻ ൌ ͳ ฻ ൌ

Khi đó,

•‹ ͵ ൅ •‹ ͵ ൅ •‹ ͵ ൌ Ͳ ฻ ʹ •‹ ͵ ൅ •‹ሺͷͶͲ୭െ ͸ሻ ൌ Ͳ

฻ ʹ •‹ ͵ ൅ •‹ ͸ ൌ Ͳ ฻ •‹ ͵ ሺͳ ൅ …‘• ͵ሻ ൌ Ͳ ฻ ቂ •‹ ͵ ൌ Ͳ

…‘• ͵ ൌ െͳ฻ ൌ ͸Ͳ୭

Vậy tam giác c Ta có :

ܽଶ ൌ ܽଷെ ܾଷെ ܿଷ

ܽ െ ܾ െ ܿ ฻ ܽଷെ ܽଶሺܾ ൅ ܿሻ ൌ ܽଷെ ܾଷെ ܿଷ ฻ ܾଷ൅ ܿଷ ൌ ܽଶሺܾ ൅ ܿሻ

฻ ܾଶ൅ ܿଶെ ܾܿ ൌ ܽଶ

฻ ܾ

ଶ൅ ܿଶെ ܽଶ

ʹܾܿ ൌ ͳ ʹ ฻ …‘• ൌ ͳ

ʹ฻ ൌ ͸Ͳ୭

Mặt khác,

…‘• …‘• ൌ ͳ

Ͷ฻ െ …‘• ൅ …‘•ሺ െ ሻ ൌ ͳ ʹ ฻ …‘•ሺ െ ሻ ൌ ͳ ฻ ൌ

Vậy tam giác d Ta có :

•‹ ൅ •‹ ൌ ʹ •‹ ฻ …‘• ʹ…‘•

ʹ ൌ ʹ •‹ ʹ…‘•

ʹ ฻ …‘• െ

(176)

Mặt khác,

–ƒ ൅ –ƒ ൌ ʹ –ƒ ฻ •‹ሺ ൅ ሻ

…‘• …‘• ൌ ʹ –ƒ ฻ •‹

…‘• …‘• ൌ ʹǤ •‹ …‘• ฻ …‘• ൌ ʹ …‘• …‘•

฻ …‘• ൌ െ …‘• ൅ …‘•ሺ െ ሻ ฻ ʹ ൬ͳ െ ʹ •‹ଶ

ʹ൰ ൌ ʹ …‘•ଶ െ

ʹ െ ͳ

Do ta có

…‘• െ

ʹ ൌ ʹ •‹ ʹ

Nên

ʹ ൬ͳ െ ʹ •‹ଶ

ʹ൰ ൌ ͺ •‹ଶ ʹ

Do đó,

…‘• െ ʹ

Vậy tam giác

Giải:

a Theo đẳng thức bản, ta có :

•‹ ൅ •‹ ൅ •‹ ൌ Ͷ …‘• ʹ…‘•

ʹ…‘•

ʹ

Giả thuyết tương đương với

„Ǥ ቐ

…Ǥ ൝ ǡ ǡ א ቀͲǡ ߨ ʹቁ

݌ଶ ൌ ܾܽ •‹ଶ ൅ ܾܿ •‹ଶ ൅ ܿܽ •‹ଶ

(ĐH Y Thái Bình 2000)

െ ͳ ฻ •‹ଶ

ʹൌ ͳ

Ͷ ฻ ൌ ͸Ͳ୭ ൌ ͳ ฻ ൌ

Bài 19: Chứng minh tam giác thỏa mãn hệ thức

ƒǤ •‹ ൅ •‹ ൅ •‹ ൌ Ͷ •‹ •‹ •‹ ͳ ൅ …‘• ʹܽ ൅ ܾ

•‹ ξͶܽଶെ ܾଶ

ܽଶሺܾ ൅ ܿ െ ܽሻ ൌ ܾଷ൅ ܿଷെ ܽଷ

(177)

…‘• ʹ…‘•

ʹ…‘•

ʹൌ •‹ •‹ •‹ ฻ …‘•

ʹ…‘• ʹ…‘•

ʹൌ ͺ •‹ ʹ…‘•

ʹ•‹

ʹ…‘•

ʹ•‹

ʹ…‘•

ʹ ฻ •‹

ʹ•‹ ʹ•‹

ʹൌ

ͳ ͺ

Theo bất đẳng thức bản, ta lại có

•‹ ʹ•‹

ʹ•‹

ʹ൑

ͳ ͺ

Dấu ̶ ൌ ̶ xảy ൌ ൌ Vậy tam giác

b Từܽଶሺܾ ൅ ܿ െ ܽሻ ൌ ܾଷ൅ ܿଷെ ܽଷ, ta suy được

ܽଶ ൌ ܾଶ൅ ܿଶെ ܾܿ

Theo định lý hàm số cos

Mặt khác

ͳ ൅ …‘• •‹ ൌ

ʹܽ ൅ ܾ ξͶܽଶെ ܾଶ

฻ ൬ͳ ൅ …‘• •‹ ൰

ൌ ሺʹܽ ൅ ܾሻ

Ͷܽଶ െ ܾଶ ൌ

฻ ቌ ʹ …‘•

ʹ ʹ •‹ʹ …‘•ʹቍ

ൌ ൌ ʹܽ ൅ ܾ ʹܽ െ ܾ ฻ ͳ ൅ …‘•

ͳ െ …‘• ൌ ൌ …‘• ฻ ܾ

ଶ൅ ܽଶെ ܿଶ

ʹܾܽ ൌ ฻ ܽ ൌ ܿ

Vậy tam giác

c Giả thuyết tương đương với

ሺܽ ൅ ܾ ൅ ܿሻଶ ൌ ʹܾܽሺͳ െ …‘• ʹሻ ൅ ʹܾܿሺͳ െ …‘• ʹሻ ൅ ʹܿܽሺͳ െ …‘• ʹሻ

฻ ܽଶ൅ ʹሺܾ …‘• ʹ ൅ ܿ …‘• ʹሻܽ ൅ ܾଶ൅ ܿଶ൅ ʹܾܿ …‘• ʹ ൌ Ͳሺͳሻ

Ta xét :

ȟᇱ ൌ ሺܾ …‘• ʹ ൅ ܿ …‘• ʹሻଶെ ܾଶെ ܿଶെ ʹܾܿ …‘• ʹ

ൌ െܾଶ•‹ଶʹ െ ܿଶ•‹ଶʹ ൅ ʹܾܿሾ…‘• ʹ …‘• ʹ െ …‘•ሺʹሻ ൅ ʹሿ

ൌ െܾଶ•‹ଶʹ െ ܿଶ•‹ଶʹ ൅ ʹܾܿ •‹ ʹ •‹ ʹ

ܾଶ൅ ܿଶെ ʹܾܿ …‘• ൌ ܾଶ൅ ܿଶെ ܾܿ ฻ …‘• ൌͳ

ʹ฻ ൌ ͸Ͳ୭ ʹܽ ൅ ܾ

ʹܽ െ ܾ ʹܽ ൅ ܾ

ʹܽ െ ܾ ฻ …‘•ଶ

ʹ •‹ଶ

ʹ ʹܽ ൅ ܾ

ʹܽ െ ܾ ฻ ܾ ʹܽ ܾ

(178)

Do đó,

ሺͳሻ ฻ ቄܽ ൌ െܾ …‘• ʹ െ ܿ …‘• ʹܾ •‹ ʹ ൌ ܿ •‹ ʹ

ฺ ൜ܽଶ ൌ ܾଶ…‘•ଶʹ ൅ ܿଶ…‘•ଶʹ ൅ ʹܾܿ …‘• ʹ …‘• ʹ Ͳ ൌ ܾଶ•‹ଶʹ ൅ ܿଶ•‹ଶʹ െ ʹܾܿ •‹ ʹ •‹ ʹܥ

ฺ ܽଶ ൌ ܾଶ൅ ܿଶ൅ ʹܾܿ …‘•ሺʹ ൅ ʹሻ

฻ …‘• ʹ ൌ െ …‘• ൌ …‘•ሺߨ െ ሻ ฺ ൤ʹ ൌ ߨ െ ൅ ݇ʹߨሺʹሻ

ʹ ൌ െ ߨ ൅ ݇ʹߨሺ͵ሻሺ݇ א Ժሻ

฻ …‘• ʹ ൌ െͳ ʹ ฻ …‘• ൌ ͳ

ʹ฻ ൌ ͸Ͳ୭

Vậy tam giác

Giải:

a Theo định lý hàm số sin, ta có :

ܽ …‘• ൅ ܾ …‘• ൅ ܿ …‘• ൌ ܴሺʹ …‘• •‹ ൅ ʹ …‘• •‹ ൅ ʹ …‘• •‹ ሻ ൌ ܴሺ•‹ ʹ ൅ •‹ ʹ ൅ •‹ ʹሻ

ƒǤ

ͻܴ „Ǥ …‘•ଶ

ʹ൅ …‘•ଶ

ʹ൅ …‘•ଶ

ʹെ ʹ ൌ ͳ Ͷ…‘•

െ ʹ …‘•

െ ʹ …‘•

െ ʹ …Ǥ ൝ ൑ ൑ ൑ ͻͲ

•‹ ʹ ൅ •‹ ʹ ൅ •‹ ʹ ൌ •‹ ൅ •‹ ൅ •‹ ൅ Ͷ •‹ െ ʹ •‹

െ ʹ •‹

െ ʹ

(ĐH Y Dược Tp.HCM 2001) Ta thấy :

ሺʹሻ ฻ െ ൌ ݇ʹߨ

Do ݇ א Ժ ȁ െ ȁ ൏ ߨ ฻ ȁ݇ʹߨȁ ൏ ߨ ฻ ݇ ൌ Ͳ Suy : ൌ ሺ͵ሻ ฻ ʹ െ ൌ ሺʹ݇ െ ͳሻߨ

Do ݇ א Ժ Ͳ ൏ ǡ ൏ ߨ nên ݇ ൌ ͳ Suy : ʹ ൌ ൅ ߨ Vậy tù, điều mâu thuẫn giả thuyết

Do đó, từ hệ thức ܾ •‹ ʹ ൌ ܿ •‹ ʹ, ta :

െ •‹ •‹ Ͷ ൌ •‹ •‹ ʹ

ܽ …‘• ൅ ܾ …‘• ൅ ܿ …‘• ܽ •‹ ൅ ܾ •‹ ൅ ܿ •‹ ൌ

ʹ݌

(179)

Theo đẳng thức bản, ta có :

•‹ ʹ ൅ •‹ ʹ ൅ •‹ ʹ ൌ Ͷ •‹ •‹ •‹

Do đó,

ܽ …‘• ൅ ܾ …‘• ൅ ܿ …‘• ൌ Ͷܴ •‹ •‹ •‹

Mặt khác, theo bất đẳng thức Cauchy :

ܽ •‹ ൅ ܾ •‹ ൅ ܿ •‹ ൌ ܴሺʹ •‹ •‹ ൅ ʹ •‹ •‹ ൅ ʹ •‹ •‹ ሻ ൒ ͵ܴ ඥͺ •‹య ଶ •‹ଶ •‹ଶ Suy ܽ …‘• ൅ ܾ …‘• ൅ ܿ …‘• ܽ •‹ ൅ ܾ •‹ ൅ ܿ •‹ ൑ Ͷܴ •‹ •‹ •‹ ͸ܴ ξ•‹య ଶ •‹ଶ •‹ଶൌ ʹ ͵ξ•‹ •‹ •‹ య

Trong :

ʹ݌ ͻܴ ൌ ܽ ൅ ܾ ൅ ܿ ͻܴ ൌ ʹܴሺ•‹ ൅ •‹ ൅ •‹ ሻ ͻܴ ൒ ʹ ͵

Dấu ̶ ൌ ̶ xảy •‹ ൌ •‹ ൌ •‹ Vậy tam giác

b Theo đẳng thức bản, ta có :

ൌ ʹ ൅ ʹ •‹ ʹ •‹ ʹ•‹ ʹ

Do đó, giả thuyết tương đương với

ͺ •‹ ʹ•‹ ʹ …‘• െ ʹ …‘• െ ʹ ฻ ͸Ͷ •‹ ʹ•‹ ʹ•‹ ʹ…‘• ʹ •‹ ൅ ʹ …‘• െ ʹ •‹ ൅ ʹ …‘• െ ʹ ቐܽ ൅ ܾ ൒ ʹξܾܾܽ ൅ ܿ ൒ ʹξܾܿ ܿ ൅ ܽ ൒ ʹξܿܽ ฺ ሺܽ ൅ ܾሻሺܾ ൅ ܿሻሺܿ ൅ ܽሻ ൒ ͺܾܽܿ

Dấu ̶ ൌ ̶ xảy ܽ ൌ ܾ ൌ ܿ Vậy tam giác

c Ta có :

Ͷ •‹ െ ʹ •‹ െ ʹ •‹ െ ʹ ൌ ʹ •‹ െ ʹ ൬…‘• ൅ െ ʹ ʹ െ …‘• െ ʹ ൰ ൌ ʹ •‹ െ …‘• ൅ െ ʹെ ʹ •‹ െ …‘• െ య ξ•‹ •‹ •‹ •‹ ʹ•‹ ʹെ ʹ ൌ ʹ •‹ ʹ •‹ ʹൌ …‘• െ ʹ …‘• ʹ…‘• ʹ ൌ ͺ •‹ ൅ ʹ …‘• െ ʹ ฻ ͺ •‹ •‹ •‹ ൌ ሺ•‹ ൅ •‹ ሻሺ•‹ ൅ •‹ ሻሺ•‹ ൅ •‹ ሻ ฻ ͺܾܽܿ ൌ ሺܽ ൅ ܾሻሺܾ ൅ ܿሻሺܿ ൅ ܽሻ

(180)

ൌ •‹ሺ െ ሻ ൅ •‹ሺ െ ሻ ൅ •‹ሺ െ ሻ

Do đó, giả thuyết tương đương với

•‹ ʹ ൅ •‹ ʹ ൅ •‹ ʹ ൌ •‹ሺ ൅ ሻ ൅ •‹ሺ ൅ ሻ ൅ •‹ሺ ൅ ሻ ൅ •‹ሺ െ ሻ ൅ •‹ሺ െ ሻ ൅ •‹ሺ െ ሻ ฻ •‹ …‘• ൅ •‹ …‘• ൅ •‹ …‘• ൌ •‹ …‘• ൅ •‹ …‘• ൅ •‹ …‘• ฻ •‹ ሺ…‘• െ …‘• ሻ ൅ •‹ ሺ…‘• െ …‘• ሻ ൅ •‹ ሺ…‘• െ …‘• ሻ ൌ Ͳ ฻ െ •‹ ሾሺ…‘• െ …‘• ሻ ൅ ሺ…‘• െ …‘• ሻሿ ൅ •‹ ሺ…‘• െ …‘• ሻ ൅ •‹ ሺ…‘• െ …‘• ሻ ൌ Ͳ ฻ ሺ…‘• െ …‘• ሻᇣᇧᇧᇧᇧᇤᇧᇧᇧᇧᇥ ஹ଴ ሺ•‹ െ •‹ ሻ ᇣᇧᇧᇧᇤᇧᇧᇧᇥ ஸ଴ ൅ ሺ…‘• െ …‘• ሻᇣᇧᇧᇧᇧᇤᇧᇧᇧᇧᇥ ஹ଴ ሺ•‹ െ •‹ ሻ ᇣᇧᇧᇧᇤᇧᇧᇧᇥ ஸ଴ ൌ Ͳ

Dấu ̶ ൌ ̶ xảy ൌ ൌ Vậy tam giác

Giải:

a Ta có :

•‹ ʹ…‘• ʹ…‘• ʹ ൌ ͳ ʹ•‹ ʹ൬…‘• ൅ ʹ ൅ …‘• െ ʹ ൰ ൌ ͳ ʹ•‹ଶ ʹ൅ ͳ ʹ…‘• ൅ ʹ Ǥ …‘• െ ʹ ൌͳ Ͷሺͳ െ …‘• ሻ ൅ ͳ Ͷሺ…‘• ൅ …‘• ሻ

Tương tự vậy, ta có :

ƒǤ •‹ ʹ…‘• ʹ…‘• ʹ൅ •‹ ʹ…‘• ʹ…‘• ʹ ͻ ͺ „Ǥܽ ൅ ܾ ൅ ܿ ൌ ʹට݉௔ଶ൅ ݉ ௕ ଶ൅ ݉ ௖ ଶ …Ǥ •‹͵ ʹ ൅ •‹ ͵ ʹ ൌ ʹ …‘• †Ǥ ͳ •‹ଶ ʹ ൅ ൌ ʹ ቌ ͳ •‹ʹ൅ ͳ •‹ʹ൅ ͳ •‹ʹቍ

(Đề nghị Olympic 30-4, 2006)

(Olympic 30-4, 2007)

(Đề nghị Olympic 30-4, 2008)

൅ •‹ ʹ…‘• ʹ…‘• ʹൌ െ ʹ ͳ •‹ଶ ʹ ൅ ͳ •‹ଶ ʹ

(181)

•‹ ʹ…‘• ʹ…‘• ʹൌ ͳ Ͷሺͳ െ …‘• ሻ ൅ ͳ Ͷሺ…‘• ൅ …‘• ሻ •‹ ʹ…‘• ʹ…‘• ʹ ൌ ͳ Ͷሺͳ െ …‘• ሻ ൅ ͳ Ͷሺ…‘• ൅ …‘• ሻ

Do đó,

•‹ ʹ…‘• ʹ…‘• ʹ൅ •‹ ʹ…‘• ʹ…‘• ʹ൅ •‹ ʹ…‘• ʹ…‘• ʹ ൌ ͳ Ͷሺ͵ ൅ …‘• ൅ …‘• ൅ …‘• ሻ ൑ ͳ Ͷ൬͵ ൅ ͵ ʹ൰ ൌ ͻ ͺ

Dấu ̶ ൌ ̶ xảy ൌ ൌ Vậy tam giác

b Ta có :

ە ۖ ۖ ۔ ۖ ۖ ۓ݉௔ଶ ൌʹሺܾଶ൅ ܿଶሻ െ ܽଶ Ͷ ݉௕ଶ ൌʹሺܿଶ൅ ܽଶሻ െ ܾଶ Ͷ ݉௖ଶ ൌ ʹሺܽଶ൅ ܾଶሻ െ ܿଶ Ͷ ฻ ە ۖ ۖ ۔ ۖ ۖ ۓ͵ ʹ ଶ ͵ ʹ ଶ ͵ ʹ ଶ Suy ͵ ʹ ଶ ଶ ଶ ଶ ଶ ଶ ଶ ଶ •‹͵ ʹ ൅ •‹ ͵ ʹ ൌ ʹ •‹ ͵ሺ ൅ ሻ Ͷ …‘• ͵ሺ െ ሻ Ͷ

Ta dựđoán :

•‹͵൅ •‹͵൑ ʹ …‘• െ ܽ ൌ ටʹሺ݉௕ଶ ൅ ݉ ௖ ଶሻ െ ݉ ௔ ܿ ൌ ටʹሺ݉௔൅ ݉௕ଶሻ െ ݉ ௖ ଶ ܾ ൌ ටʹሺ݉௖ଶ൅ ݉௔ሻ െ ݉௕ଶ ሺܽ ൅ ܾ ൅ ܿሻ ൌ ටʹሺ݉௕ଶ൅ ݉ ௖ଶሻ െ ݉௔൅ ටʹሺ݉௖ଶ൅ ݉௔ሻ െ ݉௕ଶ൅ ටʹሺ݉௔൅ ݉௕ଶሻ െ ݉௖ଶ

Khi đó, theo bất đẳng thức Bunyakovsky, ta :

ටʹሺ݉௕ଶ൅ ݉

௖ଶሻ െ ݉௔൅ ටʹሺ݉௖ଶ൅ ݉௔ሻ െ ݉௕ଶ൅ ටʹሺ݉௔൅ ݉௕ଶሻ െ ݉௖ଶ

൑ ξ͵Ǥ ට͵ሺ݉௔൅ ݉௕ଶ൅ ݉௖ଶሻ

Hay

ܽ ൅ ܾ ൅ ܿ ൑ ʹට݉௔൅ ݉௕ଶ൅ ݉௖ଶ

Dấu ̶ ൌ ̶ xảy ݉௔ ൌ ݉௕ ൌ ݉௖ Vậy tam giác

(182)

Vậy ta cần chứng minh

Ͳ ൑ …‘•͵ሺ െ ሻ

Ͷ ൑ …‘•

െ ʹ ሺכሻ

Ta có :

…‘• െ

ʹ ൒ Ͳ ֜ ʹ •‹

͵ሺ ൅ ሻ Ͷ …‘•

͵ሺ െ ሻ

Ͷ ൒ Ͳ ֜ …‘•

͵ሺ െ ሻ

Ͷ ൒ Ͳሺככሻ ቆ†‘ •‹͵ሺ ൅ ሻ

Ͷ ൒ Ͳቇ

Lại có :

Ͳ ൑ ȁ െ ȁ ʹ ൑

͵ȁ െ ȁ Ͷ ฺ …‘•ȁ െ ȁ

ʹ ൒ …‘•

͵ȁ െ ȁ Ͷ

͵ሺ െ ሻ

Ͷ ሺכככሻ ừሺכሻǡ ሺככሻ ሺכככሻ, ta có :

െ ʹ ൅ ൅ ͳ

•‹ଶ

ʹ ൒ͳ

͵ቌ ͳ •‹ʹ൅

ͳ •‹ʹ൅

ͳ •‹ʹቍ

ͳ •‹ʹ൅

ͳ •‹ʹ൅

ͳ •‹ʹ ൒

͵

ට•‹ ʹ•‹ʹ •‹ʹ

Theo bất đẳng thức bản, ta :

•‹ ʹ•‹

ʹ•‹

ʹ൑

ͳ ͺ

Do đó,

ͳ •‹ʹ൅

ͳ •‹ʹ൅

ͳ •‹ʹ൒ ͸

൑ ߨ ฺ …‘•ሺ െ ሻ

ʹ ൒ …‘• •‹͵

ʹ ൅ •‹ ͵

ʹ ൑ ʹ …‘•

Dấu ̶ ൌ ̶ xảy tam giác

d Theo bất đẳng thức Bunyakovsky, ta có :

ͳ •‹ଶ

ʹ

ͳ •‹ଶ

ʹ

(183)

Suy

ͳ •‹ଶ

ʹ

൅ ͳ •‹ଶ

ʹ

൅ ͳ •‹ଶ

ʹ

൒ ʹ ቌ ͳ •‹ʹ൅

ͳ •‹ʹ൅

ͳ •‹ʹቍ

Dấu ̶ ൌ ̶ xảy ൌ ൌ Vậy tam giác

Giải:

a Theo bất đẳng thức Bunyakovsky, ta có :

Mặt khác, ta có :

ە ۖ ۖ ۔ ۖ ۖ ۓ ଶ

ฺ ݉௔ଶ ൅ ݉௕ଶ ൅ ݉௖ଶ ൌ

͵

Ͷሺܽଶ൅ ܾଶ൅ ܿଶሻ

Do đó,

ሺ݉௔൅ ݉௕ ൅ ݉௖ሻଶ ൑

ͻ

Ͷሺܽଶ ൅ ܾଶ൅ ܿଶሻ

Theo định lý hàm số sin bất đẳng thức bản, ta có :

ܽଶ൅ ܾଶ൅ ܿଶ ൌ Ͷܴଶሺ•‹ଶ ൅ •‹ଶ ൅ •‹ଶሻ ൑ ͶܴଶǤͻ

Ͷ ൌ ͻܴଶ

Khi

ሺ݉௔൅ ݉௕ ൅ ݉௖ሻଶ ൑

ͺͳ Ͷ ܴଶ ƒǤ݉௔൅ ݉௕ ൅ ݉௖ ൌͻܴ

ʹ „Ǥ ܽ

݉௔ ൌ

ܾ ݉௕ ൌ

ܿ ݉௖

…Ǥͳ ܽ…‘•ଶ

ʹ൅

ͳ ܾ…‘•ଶ

ʹ൅

ͳ ܿ…‘•ଶ

ʹ ൌ

ʹ͹ ͺ݌ †Ǥ ͳ

ݎ௔൅

ͳ ݎ௕൅

ͳ ݎ௖ ൌ

ͳͺܴ ܾܽ ൅ ܾܿ ൅ ܿܽ

Bài 22: Chứng tỏ tam giác

ሺ݉௔൅ ݉௕ ൅ ݉௖ሻଶ ൑ ͵ሺ݉

௔൅ ݉௕ଶ ൅ ݉௖ଶሻ

݉௔ ൌʹሺܾଶ൅ ܿଶሻ െ ܽଶ ݉௕ଶ ൌ

Ͷ

ʹሺܿଶ൅ ܽଶሻ െ ܾଶ

݉௖ଶ ൌ

Ͷ

ʹሺܽଶ൅ ܾଶሻ െ ܿଶ

(184)

Hay

݉௔൅ ݉௕ ൅ ݉௖ ൑

ͻܴ ʹ

Dấu ̶ ൌ ̶ xảy ൜݉ ൌ ൌ

௔ ൌ ݉௕ ൌ ݉௖

Vậy tam giác b Ta có :

ܽ ݉௔ ൌ ܾ ݉௕ ฻ ܽଶ ܾଶ ൌ ݉௔ଶ ݉௕ଶ ൌ ʹܾଶ൅ ʹܿଶെ ܽଶ ʹܽଶ൅ ʹܿଶെ ܾଶ ฻ ܽଶሺʹܽଶ൅ ʹܿଶെ ܾଶሻ ൌ ܾଶሺʹܾଶ൅ ʹܿଶെ ܽଶሻ ฻ ʹܽସ൅ ʹܽଶܿଶെ ܽଶܾଶ ൌ ʹܾସ൅ ʹܾଶܿଶെ ܾଶܽଶ ฻ ሺܽଶെ ܾଶሻሺܽଶ൅ ܾଶ൅ ʹܿଶሻ ൌ Ͳ ฻ ܽ ൌ ܾ

Tương tự vậy, ta có :

ܾ ݉௕ ൌ

ܿ

݉௖ ฻ ܾ ൌ ܿ

Vậy tam giác

c Theo định lý hàm số cos, ta có :

…‘• ൌܾ ଶ൅ ܿଶെ ܽଶ ʹܾܿ ฺ ʹ …‘•ଶ ʹെ ͳ ൌ ܾଶ൅ ܿଶെ ܽଶ ʹܾܿ ฺ …‘•ଶ ൌܾ ଶ൅ ܿଶ൅ ʹܾܿ െ ܽଶ Ͷܾܿ ฺ …‘•ଶ ʹ ൌ ሺܾ ൅ ܿሻଶെ ܽଶ Ͷܾܿ ൌ Ͷܾܿ ൌ ݌ሺ݌ െ ܽሻ ܾܿ

Tương tự, ta :

൞…‘• ଶ ʹ ൌ ݌ሺ݌ െ ܾሻ ܿܽ …‘•ଶ ʹൌ ݌ሺ݌ െ ܿሻ ܾܽ

Do đó,

ͳ ܽ…‘•ଶ ʹ൅ ͳ ܾ…‘•ଶ ʹ൅ ͳ ܿ…‘•ଶ ʹൌ ݌ሺ݌ െ ܽሻ ܾܽܿ ൅ ݌ሺ݌ െ ܾሻ ܾܽܿ ൅ ݌ሺ݌ െ ܿሻ ܾܽܿ ൌ ݌ଶ ܾܽܿ

Mặt khác, theo bất đẳng thức Cauchy, ta có :

(185)

Dấu ̶ ൌ ̶ xảy ܽ ൌ ܾ ൌ ܿ Vậy tam giác

d Ta có :

ە ۖ ۔ ۖ ۓͳ

ݎ௔൅

ͳ ݎ௕ ൅

ͳ ݎ௖ ൌ

݌ െ ܽ ܵ ൅

݌ െ ܾ ܵ ൅

݌ െ ܿ ܵ ൌ

݌ ܵ ൌ

ܽ ൅ ܾ ൅ ܿ ʹܵ ͳͺܴ

ܾܽ ൅ ܾܿ ൅ ܿܽ ൌ

ͳͺܾܽܿ

Ͷܵሺܾܽ ൅ ܾܿ ൅ ܿܽሻ ൌ ͻ ʹܵǤ

ͳ ͳ

ܽ ൅ͳܾ ൅ͳܿ

Do đó, giả thuyết tương đương với

ሺܽ ൅ ܾ ൅ ܿሻ ൬ͳ ܽ൅

ͳ ܾ൅

ͳ ܿ൰ ൌ ͻ

Mặt khác, theo bất đẳng thức Cauchy, ta lại có :

ቐܽ ൅ ܾ ൅ ܿ ൒ ͵ξܾܽܿ

ͳ ܽ൅

ͳ ܾ൅

ͳ ܿ ൒

͵ ξܾܽܿ

ฺ ሺܽ ൅ ܾ ൅ ܿሻ ൬

Dấu ̶ ൌ ̶ xảy ܽ ൌ ܾ ൌ ܿ Vậy tam giác

Giải:

a Ta có :

൜•‹ ͸ ൅ •‹ ͸ ൌ ʹ •‹ ͵ሺ ൅ ሻ …‘• ͵ሺ െ ሻ•‹ ͸ ൌ െʹ •‹ ͵ሺ ൅ ሻ …‘• ͵ሺ ൅ ሻ

ฺ •‹ ͸ ൅ •‹ ͸ ൅ •‹ ͸ ൌ െʹ •‹ ͵ሺ ൅ ሻ ሾ…‘• ͵ሺ െ ሻ െ …‘• ͵ሺ ൅ ሻሿ ൌ െʹ •‹ ͵ ሺെʹ •‹ ͵ •‹ ͵ሻ ൌ Ͷ •‹ ͵ •‹ ͵ •‹ ͵

Do đó, giả thuyết tương đương với

Ͷ •‹ ͵ •‹ ͵ •‹ ͵ ൌ Ͳ ฻ ൥•‹ ͵ ൌ Ͳ•‹ ͵ ൌ Ͳ •‹ ͵ ൌ Ͳ ͸Ͳ୭ ͳʹͲ୭

„Ǥ •‹ ൅ •‹ ൅ •‹ …‘• ൅ …‘• ൅ …‘• …Ǥ ൜

(ĐH Luật Hà Nội 1995)

ͳ ܽ൅

ͳ ܾ൅

ͳ ܿ൰ ൒ ͻ

Bài 23:Xác định đặc điểm tam giác thỏa mãn hệ thức

ƒǤ •‹ ͸ ൅ •‹ ͸ ൅ •‹ ͸ ൌ Ͳ ൌ ξ͵ ܵ ൌ ͳ

(186)

b Giả thuyết tương đương với

൫•‹ െ ξ͵ …‘• ൯ ൅ ൫•‹ െ ξ͵ …‘• ൯ ൅ ൫•‹ െ ξ͵ …‘• ൯ ൌ Ͳ ฻ •‹ ቀ െߨ

͵ቁ ൅ •‹ ቀ െ ߨ

͵ቁ ൅ •‹ ቀ െ ߨ ͵ቁ ൌ Ͳ ฻ ʹ •‹ ൬ ൅

ʹ െ ߨ ͵൰ …‘•

ʹ െ ʹ •‹ ൬ ൅

ʹ െ ߨ

͵൰ …‘• ൬ ൅

ʹ െ ߨ ͵൰ ൌ Ͳ ฻ ʹ •‹ ൬ ൅

ʹ െ ߨ ͵൰ ൤…‘•

ʹ െ …‘• ൬ ൅

ʹ െ ߨ

͵൰൨ ൌ Ͳ ฻ •‹ ൬ ൅

ʹ െ ߨ

͵൰ •‹ ൬ ʹെ

ߨ

͸൰ •‹ ൬ ʹെ

ߨ ͸൰ ൌ Ͳ ฻ െ •‹ ൬

ʹെ ߨ

͸൰ •‹ ൬ ʹെ

ߨ

͸൰ •‹ ൬ ʹെ

ߨ ͸൰ ൌ Ͳ ฻

ۏ ێ ێ ێ ێ ۍ•‹ ൬

ʹെ ߨ ͸൰ ൌ Ͳ •‹ ൬

ʹെ ߨ ͸൰ ൌ Ͳ •‹ ൬

ʹെ ߨ ͸൰ ൌ Ͳ

Ta xét :

•‹ ൬ ʹെ

ߨ ͸

݌ െ ܽ ൅ ݌ െ ܾ ൅ ݌ െ ܿ ͵ ൰

Do đó,

݌ସ

ʹ͹൒ ͳ

Suy

݌ଶ ൒ ͵ξ͵

Hay

ξ͵ሺܽ ൅ ܾ ൅ ܿሻଶ ൒ ͵͸

Dấu ̶ ൌ ̶ xảy ܽ ൌ ܾ ൌ ܿ Vậy tam giác

൰ ൌ Ͳ ฺ ൌ ͸Ͳ୭

Vậy tam giác có góc ͸Ͳ୭

c Theo công thức Heron bất đẳng thức Cauchy, ta có :

(187)

Giải: Từ giả thuyết, ta có : ە ۖ ۔ ۖ ۓݔଶ൅ ܽ ଵݔ ൅ ܾଵ ൌ ൬ݔ െ –ƒ Ͷ൰ ൬ݔ െ –ƒ Ͷ൰ ݔଶ൅ ܽ ଶݔ ൅ ܾଶ ൌ ൬ݔ െ –ƒ Ͷ൰ ൬ݔ െ –ƒ Ͷ൰ ݔଶ൅ ܽ ଷݔ ൅ ܾଷ ൌ ൬ݔ െ –ƒ Ͷ൰ ൬ݔ െ –ƒ Ͷ൰

Lấy ݔ ൌ െͳ, ta có :

ሺͳ െ ܽଵ൅ ܾଵሻሺͳ െ ܽଶ൅ ܾଶሻሺͳ െ ܽଷ൅ ܾଷሻ ൌ ൬ͳ ൅ –ƒ Ͷ൰ ଶ ൬ͳ ൅ –ƒ Ͷ൰ ଶ ൬ͳ ൅ –ƒ Ͷ൰ ଶ ൌ ൤ͳ ൅ ൬–ƒ Ͷ൅ –ƒ Ͷ൅ –ƒ Ͷ൰ ൅ ൬–ƒ Ͷ–ƒ Ͷ൅ –ƒ Ͷ–ƒ Ͷ൅ –ƒ Ͷ–ƒ Ͷ൰ ൅ –ƒ Ͷ–ƒ Ͷ–ƒ Ͷ൨ ଶ

Mặt khác :

–ƒ ൬ ൅ Ͷ ൰ ൌ –ƒ ൬ ߨ Ͷെ Ͷ൰ ฺ –ƒͶ ൅ –ƒͶ ͳ െ –ƒͶ –ƒͶൌ –ƒߨͶ െ –ƒͶ ͳ ൅ –ƒߨͶ –ƒͶ ฺ ൬–ƒ Ͷ൅ –ƒ Ͷ൰ ൬ͳ ൅ –ƒ Ͷ൰ ൌ ൬ͳ െ –ƒ Ͷ൰ ൬ͳ െ –ƒ Ͷ–ƒ Ͷ൰ ฺ –ƒ Ͷ൅ –ƒ Ͷ൅ –ƒ Ͷ൅ –ƒ Ͷ–ƒ Ͷ൅ –ƒ Ͷ–ƒ Ͷ൅ –ƒ Ͷ–ƒ Ͷ ൌ ͳ ൅ –ƒ Ͷ–ƒ Ͷ–ƒ Ͷ ฺ ሺͳ െ ܽଵ൅ ܾଵሻሺͳ െ ܽଶ൅ ܾଶሻሺͳ െ ܽଷ൅ ܾଷሻ ൌ Ͷ ൬ͳ ൅ –ƒ Ͷ–ƒ Ͷ–ƒ Ͷ൰ ଶ ‘ǢǢ א ቀͲǢߨቁ ฺ –ƒǢ –ƒǢ –ƒא ሺͲǢ ͳሻ ฺ –ƒ–ƒ–ƒ א ሺͲǢ ͳሻ –ƒ ͶǢ –ƒ ͶŽ‰Š‹ệ…ủƒ’Šươ‰–”¿Šݔଶ൅ ܽଵݔ ൅ ܾଵ ൌ Ͳ –ƒ ͶǢ –ƒ ͶŽ‰Š‹ệ…ủƒ’Šươ‰–”¿Šݔଶ൅ ܽଶݔ ൅ ܾଶ ൌ Ͳ –ƒ ͶǢ –ƒ ͶŽ‰Š‹ệ…ủƒ’Šươ‰–”¿Šݔଶ൅ ܽଷݔ ൅ ܾଷ ൌ Ͳ

Bài 24: Nhận dạng tam giác biết

(188)

Theo bất đẳng thức Cauchy, ta có :

ͳ ൅ ݔଷ ൌ ͳ ൅ –ƒ

Ͷ–ƒ Ͷ–ƒ

Ͷ ൌ –ƒ

Ͷ൅ –ƒ Ͷ൅ –ƒ

Ͷ൅ –ƒ

Ͷ–ƒ

Ͷ൅ –ƒ

Ͷ–ƒ

Ͷ൅ –ƒ

Ͷ–ƒ

Ͷ ൒ ͵ඨ–ƒ

Ͷ–ƒ Ͷ–ƒ

Ͷ

൅ ͵ඨ൬–ƒ Ͷ–ƒ

Ͷ–ƒ

Ͷ൰

ൌ ͵ݔ ൅ ͵ݔଶ

฻ ݔଷെ ͵ݔଶെ ͵ݔ ൅ ͳ ൒ Ͳ

฻ ሺݔ ൅ ͳሻ൫ݔ െ ʹ ൅ ξ͵൯൫ݔ െ ʹ െ ξ͵൯ ൒ Ͳ ฻ Ͳ ൏ ݔ ൑ ʹ െ ξ͵

Do đó,

ሺͳ െ ܽଵ൅ ܾଵሻሺͳ െ ܽଶ൅ ܾଶሻሺͳ െ ܽଷ൅ ܾଷሻ ൌ Ͷሺͳ ൅ ݔଷሻଶ ൑ Ͷ ቂͳ ൅ ൫ʹ െ ξ͵൯ଷቃଶ

ൌ ͷ͸ͳ͸ െ ͵ʹͶͲξ͵

Dấu ̶ ൌ ̶ xảy tam giác

Giải:

a Theo định lý hàm số cot, ta có :

…‘– ൅ …‘– ൅ …‘– ൌܽ

ଶ൅ ܾଶ൅ ܿଶ

Ͷܵ

Theo định lý hàm sốsin, ta :

…‘– ൅ …‘– ൅ …‘– ൌͶܴଶ

Ͷܵ ሺ•‹ଶ ൅ •‹ଶ ൅ •‹ଶሻ

Do •‹ଶ ൅ •‹ଶ ൅ •‹ଶ ൌ …‘– ൅ …‘– ൅ …‘– nên ܵ ൌ ܴଶ

ƒǤ ൜ ܵ ൌ ܽଶ െ ሺܾ െ ܿሻଶ

•‹ଶ ൅ •‹ଶ ൅ •‹ଶ ൌ …‘– ൅ …‘– ൅ …‘–

„Ǥሺͳ ൅ ܾ ൅ ܿ െ ܾܿሻ …‘• ൅ ሺͳ ൅ ܿ ൅ ܽ െ ܿܽሻ …‘• ൅ ሺͳ ൅ ܽ ൅ ܾ െ ܾܽሻ …‘• ൌ ͵ …Ǥ

ە ۔ ۓ͵ୱ୧୬ ୅

͵ୱ୧୬ ୆

͵ୱ୧୬ ୅

͵ୱ୧୬ େ

†Ǥ –ƒ ൅ –ƒ ൅ –ƒ ൌ͵ሺ•‹ ൅ •‹ ൅ •‹ ሻ …‘• ൅ …‘• ൅ …‘•

Bài 25: Tìm tất cảcác đặc điểm tam giác đồng thời thỏa điều kiện

(189)

Theo định lý hàm số sin, ta lại có :

ܴଶ ൌ ܵ ൌ ܽଶെ ሺܾ െ ܿሻଶ ൌ Ͷܴଶሾ•‹ଶ െ ሺ•‹ െ •‹ ሻଶሿ

฻ ͳ

Ͷൌ ሺ•‹ ൅ •‹ െ •‹ ሻሺ•‹ െ •‹ ൅ •‹ ሻ ൌ ൬ʹ •‹ ൅

ʹ …‘•

ʹ െ ʹ •‹ ʹ…‘•

ʹ൰ ൬ʹ •‹ ൅

ʹ …‘• െ

ʹ െ ʹ •‹ ʹ…‘•

ʹ൰ ൌ ʹ …‘•

ʹ൬…‘• െ

ʹ െ …‘• ൅

ʹ ൰ ʹ …‘• ʹ൬…‘•

ʹ െ …‘• ൅

ʹ ൰ ൌ ͳ͸ …‘•

ʹ…‘• ʹ•‹

ʹ•‹

ʹ•‹

ʹ•‹

ʹ

Do đó,

Ͷ •‹ •‹ •‹ଶ

ʹ ൌ ͳ Ͷ

Mặt khác, theo định lý hàm số sin, ta lại có :

ͺܴଷ ͺܴଷ

Nên

ͳ ͳ͸ൌ

•‹ଶ

ʹ

ʹ •‹ ൌ ͳ Ͷ

Ta biết

ͳ

ʹ •‹ •‹ ൌ ͺ

ͳ͹ฺ •‹ •‹ ൌ ͳ͹ ͳ͸൐ ͳ

ܾܿ …‘• ൅ ܿܽ …‘• ൅ ܾܽ …‘• ൌ ͳ

ʹሺܽଶ൅ ܾଶ൅ ܿଶሻ

Do đó, giả thuyết tương đương với

…‘• ൅ …‘• ൅ …‘• ൌ ͳ

ʹሺܽଶ ൅ ܾଶ൅ ܿଶሻ െ ሺܽ ൅ ܾ ൅ ܿሻ ൅ ͵

Mặt khác, theo bất đẳng thức Bunyakovsky, ta có :

͵ሺܽଶ൅ ܾଶ൅ ܿଶሻ ൒ ሺܽ ൅ ܾ ൅ ܿሻଶ

•‹ •‹ •‹ ൌܾܽܿ ൌܵǤ Ͷܴ ൌͳ ʹ ൌͳ

Ͷ–ƒ

ʹ ฻ –ƒ ʹ ൌ •‹ ൌ –ƒ

ʹ ͳ ൅ –ƒଶ

ʹ

Điều xảy

Vậy không tồn tam giác thỏa mãn hai hệ thức cho b Từ giả thuyết, ta viết lại thành

…‘• ൅ …‘• ൅ …‘• ൅ ሺܽ …‘• ൅ ܾ …‘• ሻ ൅ ሺܾ …‘• ൅ ܿ …‘• ሻ ൅ ሺܽ …‘• ൅ ܿ …‘• ሻ െ ሺܾܿ …‘• ൅ ܿܽ …‘• ൅ ܾܽ …‘• ሻ ൌ Ͳ

Theo định lý hình chiếu định lý hàm số cos, ta có :

(190)

Nên

ͳ

ʹሺܽଶ൅ ܾଶ൅ ܿଶሻ െ ሺܽ ൅ ܾ ൅ ܿሻ ൅ ͵ ൒ ͳ

͸ሺܽ ൅ ܾ ൅ ܿሻଶെ ሺܽ ൅ ܾ ൅ ܿሻ ൅ ͵ ൌͳ

͸ሺܽ ൅ ܾ ൅ ܿ െ ͵ሻଶ൅ ͵ ʹ൒

͵ ʹ

Theo bất đẳng thức bản, ta có :

…‘• ൅ …‘• ൅ …‘• ൑͵ ʹ

Do đó, dấu ̶ ൌ ̶ xảy ܽ ൌ ܾ ൌ ܿ ൌ ͳ Vậy tam giác đều, có độ dài cạnh ͳ c Hệđã cho viết lại thành

Ta xét hàm số

݂ᇱᇱሺݒሻ ൌ ͵௩Žଶ͵ ൐ Ͳ

Suy ݒ ൌ Ͳ ݒ ൌ ʹ hai nghiệm phương trình

Với ݒ ൌ Ͳ •‹ ൌ •‹ ฻ ൌ Với ݒ ൌ ʹ •‹ ൌ •‹ ൅ ʹ ൐ ʹ (vô lý) Vậy tam giác

d Theo đẳng thức bản, ta có :

•‹ ൅ •‹ ൅ •‹ ൐ Ͳ …‘• ൅ …‘• ൅ …‘• ൌ ͳ ൅ Ͷ •‹

ʹ•‹ ʹ•‹

ʹ൐ ͳ –ƒ ൅ –ƒ ൅ –ƒ ൌ –ƒ –ƒ –ƒ

Kết hợp với giả thuyết, ta suy

͵ୱ୧୬ ୅ିୱ୧୬ ୆൅ Ͷሺ•‹ െ •‹ ሻ ൌ ͳሺͳሻ

͵ୱ୧୬ ୅ିୱ୧୬ େെ Ͷሺ•‹ െ •‹ ሻ ൌ ͳሺʹሻ

Xét ሺͳሻ, ta đặt ݑ ൌ •‹ െ •‹ Khi :

͵௨൅ Ͷݑ ൌ ͳ

݂ሺݑሻ ൌ ͵௨ ൅ Ͷݑǡ ݑ א Թ

݂ᇱሺݑሻ ൌ ͵௨Ž ͵ ൅ Ͷ ൐ Ͳ

Do đó, hàm sốđồng biến

Ta thấy ݑ ൌ Ͳ nghiệm phương trình hàm nên ݑ ൌ Ͳ nghiệm phương trình

Suy : •‹ ൌ •‹ ฻ ൌ

Xét ሺʹሻ, ta đặt ݒ ൌ •‹ െ •‹ Khi :

Ta xét hàm số

͵௩െ Ͷݒ ൌ ͳ

݂ሺݒሻ ൌ ͵௩െ Ͷݒǡ ݒ א Թ

(191)

–ƒ –ƒ –ƒ ൐ Ͳ

Tương đương tam giác nhọn Giả sử :

Ͳ ൏ ൑ ൑ ൏ ߨ

ʹ ฺ ቄ–ƒ ൒ –ƒ ൒ –ƒ ൐ ͲͲ ൏ …‘• ൑ …‘• ൑ …‘•

Theo bất đẳng thức Chebyshev, ta có :

–ƒ ൅ –ƒ ൅ –ƒ ͵ Ǥ

…‘• ൅ …‘• ൅ …‘• ͵ ൒

–ƒ …‘• ൅ –ƒ …‘• ൅ –ƒ …‘• ͵

Ta viết lại bất đẳng thức thành

–ƒ ൅ –ƒ ൅ –ƒ ൒ ͵ሺ•‹ ൅ •‹ ൅ •‹ ሻ …‘• ൅ …‘• ൅ …‘•

Dấu ̶ ൌ ̶ xảy ൌ ൌ Vậy tam giác

Giải:

͸Ͷܴଶ

ʹ͹ …‘• …‘• •‹ଶ ൌ ͳ͸

ʹ͹

Theo bất đẳng thức Cauchy, ta có :

…‘• …‘• •‹ଶ ൌ ͳ

ʹሺͳ െ …‘•ଶሻሾ…‘•ሺ െ ሻ െ …‘• ሿ ൑ͳ

ʹሺͳ െ …‘•ଶሻሺͳ െ …‘• ሻ ൌ ͳ

Ͷሺʹ ൅ ʹ …‘• ሻሺͳ െ …‘• ሻሺͳ െ …‘• ሻ …Ǥ

ଶ଴ଵଶ൅ ଶ଴ଵଶ൅ ଶ଴ଵଶ

͵ ൌ ቀ ߨ ͵ቁ

ଶ଴ଵଶ

†ǤͶ •‹ •‹ •‹ ܾܽ ൅ ܾܿ ൅ ܿܽ ൌ

a Theo định lý hàm số cos, ta có :

൜ܽଶ൅ ܿଶെ ܾଶ ൌ ʹܽܿ …‘• ܽଶ൅ ܾଶെ ܿଶ ൌ ʹܾܽ …‘•

Do đó, giả thuyết tương đương với

ʹ͹Ǥʹܽܿ …‘• Ǥ ʹܾܽ …‘• ൌ ʹͷ͸ܾܴܿଶ ฻ ܽଶ…‘• …‘• ൌ

Theo định lý hàm số sin, ta viết hệ thức thành

Bài 26:Tìm đặc điểm tam giác thỏa mãn điều kiện

ƒǤʹ͹ሺܽଶ൅ ܿଶെ ܾଶሻሺܽଶ൅ ܾଶെ ܿଶሻ ൌ ʹͷ͸ܾܴܿଶ

„Ǥݎ௔݈௔൅ ݎ௕݈௕൅ ݎ௖݈௖ ൌ ݌ଶ

(192)

൑ͳ Ͷ൬

ʹ ൅ ʹ …‘• ൅ ͳ െ …‘• ൅ ͳ െ …‘• ͵ ൰

ൌͳ͸ ʹ͹

Do đó, dấu ̶ ൌ ̶ xảy ቄ …‘•ሺ െ ሻ ൌ ͳ ʹ ൅ ʹ …‘• ൌ ͳ െ …‘• ฻ ൝…‘• ൌ െ ൌ ͳ

͵ ậ›–ƒ‰‹ž……Ÿ–ạ‹˜–Šỏƒ  …‘• ൌ െͳ

͵

b Ta có :

ە ۖ ۔ ۖ ۓ

ݎ௔ ൌ ܵ

݌ െ ܽ ൌ ඨ

݌ሺ݌ െ ܾሻሺ݌ െ ܿሻ ݌ െ ܽ ݈௔ ൌ ʹ

ܾ ൅ ܿඥܾܿ݌ሺ݌ െ ܽሻ

Theo bất đẳng thức Cauchy, ta có :

ݎ௔݈௔ൌ

ʹξܾܿ

ܾ ൅ ܿ݌ඥሺ݌ െ ܾሻሺ݌ െ ܿሻ ൑ ܾ ൅ ܿ ܾ ൅ ܿǤ ݌

݌ െ ܾ ൅ ݌ െ ܿ ʹ ൌ

ܽ݌ ʹ

Tương tự, ta :

൞ݎ௕݈௕ ൑ ܾ݌

ʹ ݎ௖݈௖ ൑

ܿ݌ ʹ

Do đó,

݂ሺݔሻ ൌ ݔଶ଴ଵଶǡ ݔ א ሺͲǢ ߨሻ

݂ᇱሺݔሻ ൌ ʹͲͳʹݔଶ଴ଵଵ

݂ᇱᇱሺݔሻ ൌ ʹͲͳʹǤʹͲͳͳǤ ݔଶ଴ଵ଴ ൐ Ͳ

Theo bất đẳng thức Jensen, ta có :

݂ሺሻ ൅ ݂ሺሻ ൅ ݂ሺሻ ͵ ൒ ݂ ൬

൅ ൅ ͵ ൰ ൌ ቀ

ߨ ͵ቁ

ଶ଴ଵଶ

Dấu ̶ ൌ ̶ xảy ൌ ൌ Vậy tam giác

ݎ௔݈௔൅ ݎ௕݈௕ ൅ ݎ௖݈௖ ൑ ݌ଶ

Dấu ̶ ൌ ̶ xảy ܽ ൌ ܾ ൌ ܿ Vậy tam giác

(193)

d Theo định lý hàm sốsin đẳng thức bản, ta có :

ܽ ൅ ܾ ൅ ܿ ͳͺܴଷ ൌ

•‹ ൅ •‹ ൅ •‹ ͻܴଶ ൌ

Ͷ …‘•ʹ …‘•ʹ …‘•ʹ ͻܴଶ

Do đó, giả thuyết tương đương với

͹ʹܴଶ•‹

ʹ•‹ ʹ•‹

ʹ ൌ ܾܽ ൅ ܾܿ ൅ ܿܽ

Mặt khác, ta lại có kết sau :

ە ۔

ۓ ݎ ൌ Ͷܴ •‹ ʹ•‹

ʹ•‹

ʹ ܵ ൌ ݌ݎ ൌ ܾܽܿ

Ͷܴ ฺ ܴݎ ൌ ܾܽܿ

Ͷ݌

Nên hệ thức viết lại thành

ͳͺܴݎ ൌ ܾܽ ൅ ܾܿ ൅ ܿܽ ฻ ͳͺǤܾܽܿ

Ͷ݌ ൌ ܾܽ ൅ ܾܿ ൅ ܿܽ

฻ ͻܾܽܿ ൌ ሺܾܽ ൅ ܾܿ ൅ ܿܽሻሺܽ ൅ ܾ ൅ ܿሻ

Theo bất đẳng thức Cauchy, ta có :

Giải:

a Ta có :

•‹ ͷ ൅ •‹ ͷ ൅ •‹ ͷ ൌ ʹ •‹ͷ ൅ ͷ ʹ …‘•

ͷ െ ͷ

ʹ ൅ •‹ ͷሺ ൅ ሻ ൌ ʹ •‹ͷ ൅ ͷ

ʹ ൬…‘•

ͷ െ ͷ ʹ ൅ …‘•

ͷ ൅ ͷ

ʹ ൰ ൌ ʹ …‘• ʹ…‘•

ʹ…‘•

ʹ

Do đó,

„Ǥ ൜ܽଶ•‹ ʹ ൅ ܾଶ•‹ ʹ ൌ Ͷܾܽ …‘• •‹ •‹ ʹ ൅ •‹ ʹ ൌ Ͷ •‹ •‹ …Ǥ ൝•‹

ଶ ൅ •‹ଶ ൌ •‹௧

ǡ א ቀͲǡߨ

ʹቁ Ǣ ݐ א ሺͲǢ ʹሻ

ቊܾܽ ൅ ܾܿ ൅ ܿܽ ൒ ͵

ඥܽଶܾଶܿଶ

ܽ ൅ ܾ ൅ ܿ ൒ ͵ξܾܽܿ ฺ ሺܾܽ ൅ ܾܿ ൅ ܿܽሻሺܽ ൅ ܾ ൅ ܿሻ ൒ ͻܾܽܿ

Dấu ̶ ൌ ̶ xảy ܽ ൌ ܾ ൌ ܿ Vậy tam giác

Bài 27:Tìm đặc điểm tam giác thỏa mãn đẳng thức

(194)

•‹ ͷ ൅ •‹ ͷ ൅ •‹ ͷ ൌ Ͳ ฻ ۏ ێ ێ ێ ێ ۍ…‘•

ʹ ൌ Ͳ …‘•

ʹ ൌ Ͳ …‘•

ʹൌ Ͳ ฻

ۏ ێ ێ ێ ێ ۍ ൌ ߨ

ͷש ൌ ͵ߨ

ͷ ൌߨ

ͷש ൌ ͵ߨ

ͷ ൌ ߨ

ͷש ൌ ͵ߨ

ͷ

Mặt khác, theo đẳng thức ta có :

ͳ െ ʹ …‘• …‘• …‘• ൌ …‘•ଶ ൅ …‘•ଶ ൅ …‘•ଶ ൏ ͳ ฻ …‘• …‘• …‘• ൐ Ͳ

Suy ra, ta chọn

ۏ ێ ێ ێ ێ ۍ ൌߨͷ

ൌߨ ͷ ൌߨ ͷ

Vậy tam giác có góc ͵͸୭

b Theo định lý hàm số sin, ta có :

ܽଶ•‹ ʹ ൅ ܾଶ•‹ ʹ ൌ Ͷܾܽ …‘• •‹

฻ •‹ଶ Ǥ ʹ •‹ …‘• ൅ •‹ଶ Ǥ ʹ •‹ …‘• ൌ Ͷ •‹ …‘• •‹ …‘•

฻ •‹ …‘• െ •‹ …‘• ൌ Ͳ ฻ •‹ሺ െ ሻ ൌ Ͳ ฻ ൌ

Khi đó, thay ൌ vào hệ thức •‹ ʹ ൅ •‹ ʹ ൌ Ͷ •‹ •‹ Ta :

ʹ •‹ ʹ ൌ Ͷ •‹ଶ

฻ …‘• ൌ •‹ ฻ ൌ Ͷͷ୭

Vậy tam giác vuông cân c Từđẳng thức :

•‹ଶ ൅ •‹ଶ ൅ •‹ଶ ൌ ʹ ൅ ʹ …‘• …‘• …‘•

Ta suy : •‹௧ ൅ •‹ଶ ൌ ʹ ൅ ʹ …‘• …‘• …‘•

Mà Ͳ ൏ •‹ ൑ ͳ ฺ •‹௧ ൅ •‹ଶ ൑ ʹ

ฺ ʹ ൅ ʹ …‘• …‘• …‘• ൑ ʹ ฺ …‘• …‘• …‘• ൑ Ͳ ฺ …‘• ൑ Ͳ

Mặt khác từ : Ͳ ൏ •‹ ൑ ͳ ฺ •‹௧ ൒ •‹ଶ

ฺ •‹ଶ ൅ •‹ଶ ൒ •‹ଶ

ฺ ܽଶ൅ ܾଶ ൒ ܿଶ ൌ ܽଶ൅ ܾଶെ ʹܾܽ …‘•

ฺ …‘• ൒ Ͳ

Do đó, …‘• ൌ Ͳ ฻ ൌ ͻͲ୭

(195)

Giải:

Ta có cơng thức :

ݎ ൌ ሺ݌ െ ܽሻ –ƒ ʹ ൌ ሺ݌ െ ܾሻ –ƒ ʹ ൌ ሺ݌ െ ܿሻ –ƒ ʹ ฺ ݎଷ…‘– ʹ…‘– ʹ…‘– ʹൌ ሺ݌ െ ܽሻሺ݌ െ ܾሻሺ݌ െ ܿሻ

Mà theo công thức Heron, ta lại có :

ሺ݌ െ ܽሻሺ݌ െ ܾሻሺ݌ െ ܿሻ ൌܵଶ ݌ ൌ

݌ଶݎଶ

݌ ൌ ݌ݎଶ

Do đó,

݌ ݎ ൌ …‘– ʹ…‘– ʹ…‘– ʹ

Theo đẳng thức bản, ta có :

…‘– ʹ…‘– ʹ…‘– ʹ ൌ …‘– ʹ൅ …‘– ʹ൅ …‘– ʹሺכሻ

Kết hợp với giả thuyết, ta

Ͷͻ ൬…‘–ଶ ʹ൅ Ͷ …‘–ଶ ʹ൅ ͻ …‘–ଶ ʹ൰ ൌ ͵͸ ൬…‘– ʹ൅ …‘– ʹ൅ …‘– ʹ൰ ଶ

Theo bất đẳng thức Bunyakovsky, ta có :

ʹ ʹ ʹ൰ ൑ Ͷͻ ൬…‘–ଶ ʹ൅ Ͷ …‘–ଶ ʹ൅ ͻ …‘–ଶ ʹ൰

Dấu ̶ ൌ ̶ xảy

…‘–ʹ ͸ ൌ ʹ …‘–ʹ ͵ ൌ ͵ …‘–ʹ ʹ

Kết hợp với ሺכሻ, ta có :

ە ۖ ۔ ۖ ۓ…‘– ʹ ൌ ͹ …‘– ʹ ൌ ͹ Ͷ …‘–ൌ͹ ฻ ە ۖ ۔ ۖ ۓ•‹ ൌ ͹ ʹͷ •‹ ൌͷ͸ ͸ͷ •‹ ൌ͸͵ …‘–ଶ ʹ൅ Ͷ …‘–ଶ ʹ൅ ͻ …‘–ଶ ʹൌ ൬ ͸݌ ͹ݎ൰ ଶ

Bài 28: Tìm tất tam giác có độ dài cạnh sốnguyên dương, khơng có ước chung thỏa mãn đẳng thức

(Đề nghị Olympic 30-4, 2006)

͵͸ ൬…‘– ൅ …‘– ൅ …‘–

(196)

Chú ý:Đến đây, tốn hồn thành, ta có thểcó kết quảđẹp

hơn việc áp dụng định lý hàm sốsin, :

ʹͷܽ ͹ ൌ ͸ͷܾ ͷ͸ ൌ ͸ͷܿ ͸͵

Ta chọn : ܽ ൌ ͳ͵ǡ ܾ ൌ ͶͲǡ ܿ ൌ Ͷͷ

Vậy tam giác có cạnh thỏa mãn hệ thức :

ܽǣ ܾǣ ܿ ൌ ͳ͵ǣ ͶͲǣ Ͷͷ

Giải:

‘ ʹǢ ʹǢ ʹ א ቀͲǢ ߨ ʹቁ ² –ƒ –ƒ –ƒ ൌ Ͷ ൬•‹ ʹ …‘• ʹ൅ •‹ ʹ…‘• ʹ൰ ൐ Ͳ

Ta giả sử :

ߨ ʹ ฺ ൬•‹ ʹെ •‹ ʹ൰ ൬…‘• ʹെ …‘• ʹ൰ ൑ Ͳ ʹ…‘• ʹ൑ •‹ ʹ…‘• ʹ൅ •‹ ʹ…‘• ʹ Mà •‹ ʹ…‘• ʹൌ ͳ ʹ൤…‘• ʹ൅ •‹ ൬ െ ʹ ൰൨ ൑ ͳ ʹ…‘• ʹ

Suy :

Ͷ ൬•‹ ʹ…‘• ʹ൅ •‹ ʹ…‘• ʹ൅ •‹ ʹ…‘• ʹ൰ ൑ Ͷ …‘• ʹ൅ ʹ •‹

Ta xét hàm số

݂ሺݔሻ ൌ Ͷ …‘•ݔ ʹ൅ ʹ •‹ ݔ ǡ ݔ א ቀͲǡ ߨ ʹቁ ݂ᇱሺݔሻ ൌ ʹ ቀ…‘• ݔ െ •‹ݔ ʹቁ Ͷ ൬•‹ ʹ…‘• ʹ൅ •‹ ʹ…‘• ʹ൅ •‹ ʹ…‘• ʹ൰ ൌ –ƒ –ƒ –ƒ

Bài 29:Xác định hình dạng tam giác có góc ǡ ǡ thỏa mãn

…‘• ʹ൅ •‹

ʹ

Do đó, tam giác nhọn Theo bất đẳng thức bản, ta có :

–ƒ –ƒ –ƒ ൒ ͵ξ͵ Ͳ ൏ ൑ ൑ ൏ ʹ ฺ •‹ …‘• ʹ൅ •‹

(197)

݂ᇱሺݔሻ ൌ Ͳ ฻ ݔ ൌ ߨ

͵ ݔ Ͳ గ

గ ଶ ݂Ԣሺݔሻ ൅ Ͳ െ

݂ሺݔሻ ͵ξ͵

Từ bảng biến thiên, ta

݂ሺݔሻ ൑ ͵ξ͵

Do đó,

Ͷ ൬•‹ ʹ…‘•

ʹ൅ •‹

ʹ…‘•

ʹ൅ •‹

ʹ…‘•

ʹ

Dấu ̶ ൌ ̶ xảy

Vậy tam giác

൅ …‘•ଶ

ʹ൅ …‘•ଶ ʹ

(ĐH Cơng Đồn 2001) (ĐH Vinh 2000)

…Ǥ •‹ ൅ •‹ ൅ •‹ െ ʹ •‹ ʹ•‹

ʹ ൌ ʹ •‹ ʹ

(ĐH An Ninh 2000)

†Ǥ ൜ ܾଶ൅ ܿଶ ൑ ܽଶ

•‹ ൅ •‹ ൅ •‹ ൌ ͳ ൅ ξʹ

(ĐH Ngoại Thương Tp.HCM 1998)

൰ ൑ Ͷ …‘•

ʹ൅ ʹ •‹ ൑ ͵ξ͵ ൌ

ൌ ͵ ߨ

- BÀI TẬP TỰ LUYỆN

3.3.1 Tính góc tam giác thỏa mãn

ƒǤ •‹ଶ ൅ •‹ଶ ൅ •‹ଶ ൌ …‘•ଶ

(198)

‡Ǥ ൝ ൌ ‹ሼǡ ǡ ሽ ൒ ߨ Ͷ …‘• ൅ …‘• ൒ …‘•ሺ െ ሻ

3.3.2 Hãy xác định góc tam giác , biết

…‘•ͷ

ʹ ൅ …‘• ͷ

ʹ ൅ …‘• ͷ

ʹ ൌ ͵ξ͵

ʹ

(Đề nghị Olympic 30-4, 2006)

3.3.3 Tính góc tam giác nhọn biết

Ͷ ͵

͵ Ͷ

െ ʹ ൌ Ͳ

Tính •‹ ൅ •‹

3.3.7 Chứng minh tam giác cân góc thỏa mãn hệ thức

ƒǤ•‹

…‘• ൌ ʹ •‹ „Ǥ•‹ଶ

•‹ଶ ൌ

–ƒ –ƒ

…Ǥ •‹ ൅ •‹ ൌͳሺ–ƒ ൅ –ƒ ሻ

ሺ…‘•ଷ ൅ …‘•ଷ ൅ …‘•ଷሻ ൅ ʹ …‘• …‘• …‘• ൌ

(Đề nghị Olympic 30-4, 2007)

3.3.4 Tính sốđo góc tam giác có diện tích ܵ cạnh ܽǡ ܾǡ ܿ thỏa mãn hệ thức :

൫ξʹ െ ͳ൯ሺܽଶ൅ ܾଶሻ ൅ ܿଶ ൌ Ͷܵ

(Đề nghị Olympic 30-4, 2008)

3.3.5. Tính diện tích tam giác , biết

ܾ •‹ ሺܾ …‘• ൅ ܿ …‘• ሻ ൌ ʹͲ

3.3.6 Cho tam giác có góc thỏa mãn

…‘•

(199)

†Ǥ –ƒ ൅ ʹ –ƒ ൌ –ƒ –ƒଶ

‡Ǥʹ •‹ •‹

•‹ ൌ …‘– ʹ

ƒǤ …‘– ʹ ൌ

ܽ ൅ ܿ ܾ †Ǥܵ ൌ ͳ

Ͷ ‡Ǥ ͳ

•‹ ൅ …‘– ൌ ‰Ǥ –ƒ െ

ʹ ൌ ŠǤ ൬ܾ െ ܿ

ܾ ൰

ൌͶ •‹

ଶ െ

ʹ ͳ െ …‘• ʹ ‹Ǥ݄௔ ൌ ʹξʹ݌Ǥ •‹

ʹ•‹ ʹ

3.3.9 Chứng minh tam giác thỏa mãn hệ thức

ƒǤ •‹ ൅ •‹ ൅ •‹ ൌ …‘•

ʹ൅ …‘• ʹ൅ …‘•

ʹ ˆǤܽ •‹ሺ െ ሻ ൅ ܾ •‹ሺ െ ሻ ൌ Ͳ

‰Ǥܽ ൌ ܽ …‘• ʹ ൅ ʹܾ •‹ ʹ •‹

ŠǤሺܽଶ൅ ܾଶሻ •‹ሺ െ ሻ ൌ ሺܽଶെ ܾଶሻ •‹

‹Ǥ ൜ܽଶ•‹ ʹ ൅ ܾଶ•‹ ʹ ൌ Ͷܾܽ …‘• …‘• •‹ ʹ ൅ •‹ ʹ ൌ Ͷ •‹ •‹

3.3.8. Chứng minh tam giác vng thỏa mãn hệ thức

„Ǥ •‹ ൅ •‹ ൅ •‹ ൌ ͳ ൅ …‘• ൅ …‘• ൅ …‘• …Ǥ •‹ ൅ •‹ ൅ •‹ ൌ ͳ െ …‘• ൅ …‘• ൅ …‘•

ሺܽ ൅ ܾ െ ܿሻሺܽ ൅ ܾ ൅ ܿሻ ܿ ൅ ܾ

ܽ

ˆǤ •‹ଶ ൅ •‹ଶ ൌ ͳ ൅ …‘•ଶ

(200)

„Ǥ …‘• ൅ …‘• ൅ …‘• ൌ •‹ ʹ൅ •‹

ʹ൅ •‹

ʹ …Ǥξ•‹ ൅ ξ•‹ ൅ ξ•‹ ൌ ඨ…‘•

ʹ൅ ඨ…‘•

ʹ൅ ඨ…‘• ʹ †Ǥξ…‘• ൅ ξ…‘• ൅ ξ…‘• ൌ ඨ•‹

ʹ൅ ඨ•‹

ʹ൅ ඨ•‹ ʹ ‡Ǥ …‘• …‘• …‘• ൌ •‹

ʹ•‹ ʹ•‹

ʹ ˆǤ ͳ

…‘• ൅ ͳ …‘• ൅

ͳ …‘• ൌ

ͳ •‹ʹ൅

ͳ •‹ʹ൅

ͳ •‹ʹ

ŒǤ –ƒ Ͷ–ƒ

Ͷ–ƒ

Ͷ Ǥ ൝

ܽଶ ൌ

Ǥ ൞ ܽ ܾ൅

ܾ ܽെ

ܿଶ

ܾܽ ൌ ͳ …‘• …‘• ൌ ͳ Ͷ

3.3.10 Cho tam giác nhọn thỏa điều kiện

ͳ ൅ …‘• …‘• ൅ …‘• …‘• ൅ …‘• …‘• െ ሺ…‘• ൅ …‘• ൅ …‘• ሻ ൌ ʹ …‘• …‘• …‘•

‰Ǥʹξ͵ •‹ •‹ ൅ •‹ െ ʹሺ•‹ ൅ •‹ ሻ ൌ Ͳ

ŠǤ …‘• ൅ …‘• ൅ …‘• ൅ …‘• ʹ ൅ …‘• ʹ ൅ …‘• ʹ ൌ Ͳ ‹Ǥ͵ሺ–ƒଶ ൅ –ƒଶ ൅ –ƒଶሻ ൌ –ƒଶ –ƒଶ –ƒଶ

ൌ ൫͹ െ Ͷξ͵൯൫ʹ െ ξ͵൯

Ǥܽሺͳ െ ʹ …‘• ሻ ൅ ܾሺͳ െ ʹ …‘• ሻ ൅ ܿሺͳ െ ʹ …‘• ሻ ൌ Ͳ ܽ ൌ ʹܾ …‘•

Ngày đăng: 23/02/2021, 18:06

TÀI LIỆU CÙNG NGƯỜI DÙNG

TÀI LIỆU LIÊN QUAN

w